Você está na página 1de 293

00-PS Rheumatology-prelims-ccpp:prelims 27/3/08 13:06 Page iii

Problem Solving in
Rheumatology
KEVIN PILE MB ChB, MD, FRACP
Conjoint Professor of Medicine, University of Western Sydney,
New South Wales, Australia

LEE KENNEDY BSc, MB ChB, MD, PhD, FRCP, FRCPE, FRACP


Professor of Medicine, School of Medicine, Department of Medicine,
James Cook University, Queensland, Australia

CLINICAL PUBLISHING
OXFORD
00-PS Rheumatology-prelims-ccpp:prelims 27/3/08 13:06 Page iv
00-PS Rheumatology-prelims-ccpp:prelims 27/3/08 13:06 Page v

Contents

Abbreviations vii

SECTION 1 General Rheumatology and Soft Tissue


Rheumatism
1. New Onset Painful Joints 1
2. An Acutely Swollen/Hot Joint 6
3. Painful Shoulders Rotator Cuff and Frozen Shoulder 11
4. Tennis Elbow and Golfers Elbow 18
5. Carpal Tunnel Syndrome and Other Entrapment Neuropathies 21
6. Fibromyalgia Syndrome 27
7. Plantar Fasciitis 33

SECTION 2 Osteoarthritis
8. Causes and Prevention 39
9. Non-Pharmacological Treatment 45
10. Drug Treatment 50
11. NSAIDs Gastric Side Effects and Protection 54
12. NSAIDs Cardiac Complications 60
13. Joint Replacement Surgery 65

SECTION 3 Rheumatoid Arthritis


14. Causes 71
15. Laboratory and Imaging Investigations 77
16. Managing Rheumatoid Arthritis at Onset 82
17. Evaluating the Response to Treatment 87
18. Pregnancy and Rheumatic Diseases 92
19. Diet and Arthritis 97
20. Polyarthritis in the Elderly 103

SECTION 4 Systemic Lupus Erythematosus, Sjgrens


Syndrome and Scleroderma
21. Antinuclear Factor 109
22. SLE Risk Factors and Diagnosis 116
23. Monitoring and Managing SLE 122
00-PS Rheumatology-prelims-ccpp:prelims 27/3/08 13:06 Page vi

vi Contents

24. Sjgrens Syndrome 129


25. Raynauds Phenomenon 134
26. Assessing and Treating Scleroderma 139
27. Immunosuppressive Drugs 147

SECTION 5 Vasculitic Syndromes


28. Vasculitic Disease 153
29. Giant Cell Arteritis and Polymyalgia Rheumatica 159
30. Behets Syndrome 165

SECTION 6 Back and Specific Joint Problems


31. Acute Back Pain 169
32. Chronic Back Pain 175
33. Psoriatic Arthritis 178
34. Asymptomatic Hyperuricaemia 184
35. Gout Acute Attack and Beyond 189
36. Pseudogout Investigation and Management 195
37. Joint and Bone Infections 199
38. Viral Arthritis 205
39. Rheumatological Complications of Diabetes 211

SECTION 7 Bone Diseases


40. Osteoporosis Prevention and Lifestyle Management 217
41. Bisphosphonates for Osteoporosis Which Agent and When? 222
42. Osteoporosis Drugs Other Than Bisphosphonates 227
43. Male Osteoporosis 233
44. Glucocorticoid-Induced Osteoporosis 237
45. Pagets Disease of Bone 241
46. Bone Complications of Renal Disease 246

SECTION 8 Muscle Diseases


47. Steroid myopathy 253
48. Inflammatory Myopathies 260
49. Muscle Complications of Statin Therapy 265

General index 271


00-PS Rheumatology-prelims-ccpp:prelims 27/3/08 13:06 Page vii

Abbreviations

ABD adynamic bone disease CIM critical iIlness myopathy


ACE angiotensin-converting enzyme CK creatine kinase
ACR American College of Rheumatology CKD chronic kidney disease
ADAMTS a disintegrin and metalloproteinase CKD-MBD CKD-mineral and bone disorder
with thrombospondin motif CLASS Celecoxib Long-term Arthritis
ADFR Activate, Decrease osteoclast Safety Study
activity, Free of treatment and Clc-l chloride channel
Repeat CMC carpometacarpophalangeal
ADP adenosine diphosphate CNS central nervous system
ADR adverse drug reaction CORE Continuing Outcomes Relevant to
AMP adenosine monophosphate Evista
ANA antinuclear antibody COX cyclooxygenase
ANCA anti-neutrophil cytoplasmic COX-1 cyclooxygenase-1
antibodies COX-2 cyclooxygenase-2
ANF antinuclear factor CPEO Chronic Progressive External
AP alkaline phosphatase Ophthalmoplegia
AP-1 activator protein-1 CPPD calcium pyrophosphate dihydrate
APPROVe Adenomatous Polyp Prevention on CREST Calcinosis; Raynauds phenomenon;
Vioxx study Esophageal dysmotility;
APS antiphospholipid syndrome Sclerodactyly, Telangiectasia
AS ankylosing spondylitis CRP C-reactive protein
ASC apoptosis-associated speck-like CSS ChurgStrauss syndrome
protein CT computed tomography
ATP adenosine triphosphate CTG cytosine-thymine-guanine
B19 parvovirus B19 CTGF connective tissue growth factor
BASMI British Ankylosing Spondylitis CTS carpal tunnel syndrome
Metrology Index CTLA4-Ig cytotoxic lymphocyte-associated
BMD bone mineral density antigen linked to immunoglobulin
BMI body mass index CVD cardiovascular disease
BP blood pressure CXR chest X-ray
BPs bisphosphonates D3 1,25-dihydroxy-vitamin D3
C5 fifth cervical segment DC dendritic cell
c-ANCA cytoplasmic anti-neutrophil DD Dupuytrens disease
cytoplasmic antibody DEXA dual-energy X-ray absorptiometry
CCB calcium channel blocker DHA docosahexaenoic acid
CCTG cytosine-cytosine-thymine-guanine DHEA dehydroepiandrosterone
CCL2 monocyte chemoattractant protein- DIL drug-induced lupus
1 (see also MCP-1) DIP distal interphalangeal
CCP cyclic citrullinated peptide DISH diffuse idiopathic skeletal
CDSN corneodesmin hyperostosis
CEP circulating endothelial precursor DLCO diffusing capacity for carbon
cGMP cyclic guanosine monophosphate monoxide
CHB congenital heart block DM dermatomyositis
CI confidence interval DM1 myotonic dystrophy type 1
00-PS Rheumatology-prelims-ccpp:prelims 27/3/08 13:06 Page viii

viii Abbreviations

DM2 myotonic dystrophy type 2 hnRNP heterogeneous nuclear


DMARD disease-modifying antirheumatic ribonucleoprotein
drug HPRT hypoxanthine
DMOAD disease-modifying osteoarthritis phosphoribosyltransferase
drug HRCT high-resolution computed
DMPK myotonic dystrophy protein kinase tomography
dsDNA double-stranded DNA HRT hormone replacement therapy
EBV EpsteinBarr virus HSP Henoch-Schnlein purpura
EDTA ethylenediaminetetraacetic acid HTLV-1 human T-lymphotropic virus type 1
EEG electroencephalogram IBD inflammatory bowel disease
EGF epidermal growth factor IBM inclusion body myositis
eGFR estimated glomerular filtration rate IFN interferon
ELISA enzyme-linked immunosorbent Ig immunoglobulin
assay IGF-1 insulin-like growth factor-1
EMG electromyography I inhibitor of kappa-beta
ENA extractable nuclear antigen IL interleukin
eNOS endothelial nitric oxide synthase IL-1ra interleukin-1 receptor antagonist
EPA eicosapentaenoic acid IMPDH inosine monophosphate
ESR erythrocyte sedimentation rate dehydrogenase
ET endothelin IMT intima-media thickness
FA fatty acid INR International Normalized Ratio
FBC full blood count IP inflammatory polyarthritis
FDG-PET (18)-F-fluorodeoxyglucose-positron IU International Units
emission tomography JSN joint space narrowing
FGF fibroblast growth factor LBP low back pain
FKBP-12 12 kDa FK506-binding protein LDL low-density lipoprotein
FMS fibromyalgia syndrome LFA-1 lymphocyte function-associated
FVC forced vital capacity antigen-1
FSH follicle-stimulating hormone LFT liver function test
GAIT Glucosamine/chondroitin Arthritis LIFE Losartan Intervention for Endpoint
Intervention Trial reduction
GCA giant cell arteritis LJM limited joint mobility
GDM gestational diabetes LORA late-onset RA
GFR glomerular filtration rate LRP-5 LDL receptor-related protein-5
GI gastrointestinal LUMINA Lupus in minorities: nature versus
GMP guanosine monophosphate nurture
GSD glycogen storage disease LH luteinizing hormone
GTP guanosine triphosphate MCP metacarpophalangeal
GVHD graft-versus-host disease MCP-1 monocyte chemoattractant protein-
H2RA histamine H2 receptor antagonist 1 (see also CCL2)
HBA1C glycosylated haemoglobin MCTD mixed connective tissue disease
HBO2 hyperbaric oxygen MELAS Myopathy, Encephalopathy, Lactic
HDL high-density lipoprotein Acidosis and Stroke
HELLP Haemolytic anaemia, Elevated Liver MERRF Myoclonic Epilepsy with Ragged
enzymes, Low Platelets Red Fibres
HIV human immunodeficiency virus MI myocardial infarction
HLA human leukocyte antigen (genetic MMF mycophenolate mofetil
designation for human major MMP matrix metalloproteinase
histocompatibility complex) MORE Multiple Outcome of Raloxifene
HNPP hereditary neuropathy with liability Evaluation
to pressure palsies MPA microscopic polyangiitis
00-PS Rheumatology-prelims-ccpp:prelims 27/3/08 13:07 Page ix

Abbreviations ix

MRI magnetic resonance imaging PPI proton pump inhibitor


MRSA methicillin-resistant Staphylococcus PPRP 5phosphoribosyl 1-pyrophosphate
aureus PRIMO Prediction of Muscular Risk in
MSA myositis-specific antibodies Observational conditions
MTOR mammalian target of rapamycin PsA psoriatic arthritis
MTP metatarsophalangeal PTH parathyroid hormone
MUA manipulation under anaesthesia PTNP22 protein tyrosine phosphate non-
NALP pyrin domain-containing proteins receptor type 22
sharing structural homology with PUFAs polyunsaturated fatty acids
NODs QALY quality-adjusted life year
NCS nerve conduction studies RA rheumatoid arthritis
NFAT nuclear factor of activated T RANK receptor activator of NF-B
lymphocytes RANKL receptor activator of NF-B ligand
NF-B nuclear factor--beta RCT randomized controlled trial
NHANES National Health and Nutrition REM rapid eye movement
Examination Survey RF rheumatoid factor
NIH National Institutes of Health RISC RNA-induced silencing complex
NO nitric oxide RNA ribonucleic acid
NOD nucleotide-binding and RNP ribonucleoprotein
oligomerization domain proteins ROD renal osteodystrophy
NOS nitric oxide synthase ROS reactive oxygen species
NOS-2 inducible nitric oxide synthase RR relative risk
NOS-3 endothelial nitric oxide synthase RS3PE remitting seronegative symmetric
(eNOS) synovitis with pitting oedema
NSAID non-steroidal anti-inflammatory RUTH Raloxifene Use for The Heart
drug SAPHO Synovitis, Acne, Pustulosis,
OA osteoarthritis Hyperostosis and Osteitis
OCP oral contraceptive pill SE shared epitope
25(OH)D 25-hydroxy-vitamin D SELENA Safety of Estrogens in Lupus
OPG osteoprotegerin Erythematosus National Assessment
OR odds ratio SERM selective oestrogen receptor
PADAM partial androgen deficiency in aging modulator
men SHBG sex hormone binding globulin
PADI peptidylarginine deaminase SI sacroiliac
PAH pulmonary artery hypertension sIL-6R soluble receptor for IL-6
PAN polyarteritis nodosa SJC swollen joint count
p-ANCA perinuclear anti-neutrophil SLC22A4 solute carrier family 22 A4
cytoplasmic antibody SLE systemic lupus erythematosus
PCR polymerase chain reaction Sm Smith antigen
PCT plasma procalcitonin SOBOE shortness of breath on exertion
PDGF platelet-derived growth factor SOTI Spinal Osteoporosis Therapeutic
PET positron emission tomography Intervention
PG prostaglandin SPARC secreted protein acidic and rich in
PGI2 prostacyclin cysteine
PIP proximal interphalangeal SPECT single photon emission computed
PM polymyositis tomography
PM/DM polymyositis/dermatomyositis SRP signal recognition particle
PMR polymyalgia rheumatica SRRR sibling recurrence risk ratio
PP pyrophosphate SS Sjgrens syndrome
PPAR peroxisomal proliferator-activated SSc systemic sclerosis
receptor ssDNA single-stranded DNA
00-PS Rheumatology-prelims-ccpp:prelims 27/3/08 13:07 Page x

x Abbreviations

STAT1 signal transducer and activator of TROPOS Treatment Of Peripheral


transcription-1 Osteoporosis Study
sTNFR soluble receptor for TNF TSH thyroid-stimulating hormone
SSRI selective serotonin reuptake TxA2 thromboxane A2
inhibitor U1RNP uracil-rich 1 ribonucleoprotein
TB tuberculosis UA uric acid
TBF thermal biofeedback U/E urea and electrolytes
TGF- transforming growth factor- UDP uridine diphosphate
Th1 T helper 1 cells UK United Kingdom
Th2 T helper 2 cells US United States
TIMP tissue inhibitor of UV ultraviolet light
metalloproteinase VDR vitamin D receptor
TJC tender joint count VEGF vascular endothelial growth factor
TLR Toll-like receptor VIGOR Vioxx Gastrointestinal Outcomes
TKA total knee arthroplasty Research study
TMV turnover, mineralization and WBC white blood cell
volume WHO World Health Organization
TNF tumour necrosis factor WOMAC Western Ontario and McMaster
TNFR2 TNF- receptor type 2 Universities
TRAP tartrate-resistant acid phosphatase XO xanthine oxidase
01-PS Rheumatology-cpp:01-PS Rheumatology-ppp.QXD 18/3/08 14:28 Page 1

S E C T I O N O N E 01
General Rheumatology and Soft
Tissue Rheumatism
01 New onset painful joints
02 An acutely swollen/hot joint
03 Painful shoulders rotator cuff and frozen shoulder
04 Tennis elbow and golfers elbow
05 Carpal tunnel syndrome and other entrapment neuropathies
06 Fibromyalgia syndrome
07 Plantar fasciitis

P R O B L E M

01 New Onset Painful Joints

Case History
June is a 32-year-old tour guide with an eight-week history of painful stiff hands and
difficulty walking in the mornings. The symptoms usually last for 90 minutes. For the last
six weeks she has been using diclofenac 50 mg bd with moderate benefit. Her mother has
rheumatoid arthritis treated with methotrexate.
What additional history will help to determine a diagnosis?
What is the relevance of her family history?
What aspects of the examination will be particularly relevant?
Which investigations should be performed?

Atlas Medical Publishing Ltd


01-PS Rheumatology-cpp:01-PS Rheumatology-ppp.QXD 18/3/08 14:28 Page 2

2 01 General Rheumatology and Soft Tissue Rheumatism

Background
History
Obtaining a clear history of Junes symptoms will assist greatly in narrowing your initial
differential diagnosis as a prelude to examination and investigations. Open questions
that encourage the person to start with their initial symptoms provide chronology and
the pattern of progression. Gentle prompting can, towards the end of consultation, be
supplemented with specific questions. As you listen to the story, you will be assessing the
impact of the symptoms on the individuals life and its components of family, work and
leisure. Specifically:
Are symptoms related to a musculoskeletal problem?
Was there an identified trigger or precipitant?
What has been the pattern or progression of symptoms?
Are there features of systemic illness or inflammatory disease?
Has anything helped the problem?
Pain and loss of function are primary presenting symptoms, but do not always coexist.
Individuals differ in their descriptors of pain, its intensity and its impact. You will be told
when the problem began and where. Is the pain in a joint; in a related joint structure such
as tendon, ligament or bursa; or in a bone? What is the nature of the pain; when does it
occur; and what is the effect of movement? Malignant pain is usually a dull, deep ache
within a bone, occurring at night or when resting. Similar symptoms may occur with
Pagets disease or with a fracture. Differentiators of inflammatory from non-inflamma-
tory/mechanical joint pain are summarized in Table 1.1.

Table 1.1 Differentiators of joint pain

Inflammatory pain Non-inflammatory/mechanical pain

Pain and stiffness predominant in morning and at end of day Short-lived joint stiffness
Stiffness greater than 30 minutes Pain worsens with activity
Symptoms lessen with activity Pain improves with rest
Pain does not improve with rest
Localized erythema, swelling, tenderness
Systemic features fatigue, weight loss

Localization of pain requires clarification as to whether symptoms are recreated by


contact or movement in the area, or whether the pain is referred from another site.
Referred pain occurs when sensory perception externalizes nociceptive input from the
sclerotome or myotome of an affected structure to the relevant dermatome. Table 1.2
shows common referred pain patterns.
Onset of symptoms following trauma supports mechanical disruption of a joint, dis-
ruption of a joints surrounding capsule and ligaments, or fracture. Less obvious triggers
to explore are infections (Table 1.3), vaccinations (Rubella) and recent travel. A tactful
approach is required when soliciting information on genitourinary symptoms or a
01-PS Rheumatology-cpp:01-PS Rheumatology-ppp.QXD 18/3/08 14:28 Page 3

01 New onset painful joints 3

Table 1.2 Common presentations of referred pain

Area pain experienced Origin of pain

Shoulder Cervical spine


Biceps and lateral upper arm Shoulder and rotator cuff
Groin, inner knee Hip
Lateral thigh, buttock Trochanteric bursa

Table 1.3 Common infections associated with arthritis

Viral Gastrointestinal Genitourinary

Hepatitis B, C Salmonella typhimurium Chlamydia trachomatis


Rubella Shigella flexneri
Parvovirus Yersinia enterocolitica
Arbovirus * Campylobacter jejuni
* Serology should be tested according to exposure.

history of a new sexual partner, as it is not obvious to a patient with arthritis as to why
you would be asking such questions.
A comprehensive family history is a key part of every clinical history. A familial pat-
tern of a specific diagnosis such as rheumatoid arthritis (RA), ankylosing spondylitis or
systemic lupus erythematosus (SLE) highlights that diagnosis, and may also raise related
diagnoses that are particularly relevant for seronegative spondyloarthritides such as pso-
riasis or inflammatory bowel disease.

Examination
Examination identifies the pattern and number of joints involved and extra-articular fea-
tures (Table 1.4). Features of inflammation are sought: temperature, pulse and blood
pressure are measured, and an assessment is made of localized erythema and warmth,
tenderness, inflammation obscuring the joint margins, and reduced function. You
should distinguish monoarthritis from oligoarthritis (4 joints) and polyarthritis (>4
joints), whether these joints are large or small, and whether there is spinal (particularly
sacroiliac) involvement. Distal to the wrist and ankle there are at least 56 joints, so that as
the number of joints increases, the greater the probability is of involvement of both
hands and feet and of the pattern becoming increasingly symmetrical. Fingernails are
assessed for pitting or onycholysis suggestive of psoriasis. The scalp, umbilicus, natal cleft
and extensor surfaces of knee and elbow should be inspected. The presence of a malar
rash or photosensitive rash in a young woman suggests SLE.

Investigations
Investigations serve to:
Confirm or refute a diagnostic possibility
01-PS Rheumatology-cpp:01-PS Rheumatology-ppp.QXD 18/3/08 14:28 Page 4

4 01 General Rheumatology and Soft Tissue Rheumatism

Table 1.4 Patterns of arthritis

Pattern Monoarthritis Inflammatory Asymmetrical Symmetrical small DIP hands


spinal disease large joint joint arthritis
Sacroiliitis arthritis (MCP, PIP, MTP)
Differential Trauma Ankylosing Psoriatic arthritis RA Inflammatory OA
diagnosis spondylitis (if involves PIP and
1st CMC)
Haemophilia Psoriatic arthritis Reactive arthritis SLE Psoriatic arthritis
Septic IBD IBD Psoriatic arthritis
Gout
Pseudogout
Further X-ray Review personal Review personal Examine X-ray hands
investigations and family history and family history rheumatoid nodules
Aspirate for HLA-B27 Examine for Skin rashes,
crystals and conjunctivitis and serositis or
culture urethritis, and scalp mucositis
and buttocks for
psoriasis
X-ray lumbar Infection screen Urinalysis
spine and SI joints RF, CCP antibodies,
ANA
X-ray hands
and feet
ANA, antinuclear antibodies; CCP, cyclic citrullinated peptides; CMC, carpometacarpophalangeal; DIP, distal interphalangeal; IBD, inflammatory bowel
disease; MCP, metacarpophalangeal; MTP, metatarsophalangeal; OA, osteoarthritis; PIP, proximal interphalangeal; RA, rheumatoid arthritis; RF,
rheumatoid factor; SI, sacroiliac; SLE, systemic lupus erythematosus.

Monitor for known complications of the disease process or proposed treatment


Document a parameter that changes with disease activity or treatment
The latter includes the inflammatory markers erythrocyte sedimentation rate (ESR)
and C-reactive protein (CRP), which are non-specific markers. Whenever the possibility
of a septic joint is considered, obtaining aspirate and culture from the joint is mandatory.
Aspirated fluid is collected into a sterile container and an ethylenediaminetetraacetic acid
(EDTA)-containing tube to enable a cell count, and is sent with a request for Gram stain-
ing, polarized light microscopy, culture and sensitivity, and cell count and differential
cell count. If there will be a significant delay in the sample reaching the laboratory, fluid
can be inoculated into a blood culture system.
The early signs and symptoms of RA are not always typical. RA is characterized as
autoimmune partly on the basis of the presence of rheumatoid factor (RF), an autoanti-
body (usually immunoglobulin M [IgM]) targeting the Fc portion of IgG. Its sensitivity is
low, ranging from 60%80%, and specificity is lower, the antibody being frequently pre-
sent in other connective tissue diseases, which limits the diagnostic utility.

Recent Developments
1 RF is present in 70% of RA cases but is not specific, occurring in 5% of healthy
individuals, and globally is more associated with chronic infection than rheumatic
diseases. Non-RF antibodies were first described in the 1960s, with the target
01-PS Rheumatology-cpp:01-PS Rheumatology-ppp.QXD 18/3/08 14:28 Page 5

01 New onset painful joints 5

epitopes now identified as citrulline residues, which are arginine residues


modified by peptidylarginine deaminase (PADI). Assays are now available for the
detection of antibodies to cyclic citrullinated peptides (anti-CCP antibodies),
which are highly sensitive and specific for RA and are a poor prognostic marker
of joint erosion, vasculitis and rheumatoid nodules.1 The specificity of anti-CCP
in RA is >90% with sensitivity of 33%87%. When combined with IgM-RF, anti-
CCP has positive predictive value of >90% for RA.2 A study of undifferentiated
polyarthritis found that 93% of subjects positive for anti-CCP at first clinic visit
progressed to RA compared to 25% who were anti-CCP negative.3
2 Smoking increases the risk of RA 24 fold and also influences the manifestations
of the disease with increased RF positivity and erosive disease, nodularity and
vasculitis similar to the findings noted with anti-CCP antibodies. Smoking may
break immune tolerance by creating neo-epitopes on IgG and thus leading to RF
development. Recent work has shown that smoking is associated with increased
citrullination. The subsequent citrullinated antigens bind with more affinity to
the HLA-DR4 shared epitope subtypes, leading to increased risk of RA.4

Conclusion
Persistent arthropathy in a younger patient necessitates both accurate diagnosis and
effective management. A working knowledge of local infectious triggers is required, with
supplemental knowledge of the likely pathologies based on age and gender. History and
examination need to include potential exposure to infectious triggers, along with per-
sonal and family history. Examination will confirm or exclude significant joint inflam-
mation, and provide information on its pattern and severity (number of joints and
functional impact). Targeted investigations will narrow the diagnosis, with the urgent
investigation being exclusion of septic arthritis if there is clinical suspicion.

Further Reading
1 Mimori T. Clinical significance of anti-CCP antibodies in rheumatoid arthritis. Intern Med
2005; 44: 11226.
2 Schellekens GA, Visser H, De Jong BAW et al. The diagnostic properties of rheumatoid arthri-
tis antibodies recognizing a cyclic citrullinated peptide. Arthritis Rheum 2000; 43: 15563.
3 van Gaalen FA, Linn-Rasker SP, van Venrooij WJ et al. Autoantibodies to cyclic citrullinated
peptides predict progression to rheumatoid arthritis in patients with undifferentiated arthritis:
a prospective cohort study. Arthritis Rheum 2004; 50: 70915.
4 Gorman JD. Smoking and rheumatoid arthritis: another reason just to say no. Arthritis Rheum
2006; 54: 1013.
01-PS Rheumatology-cpp:01-PS Rheumatology-ppp.QXD 18/3/08 14:28 Page 6

6 01 General Rheumatology and Soft Tissue Rheumatism

P R O B L E M

02 An Acutely Swollen/Hot Joint

Case History
You have been asked to see a 28-year-old man who presents with a 36-hour history of a
red and very swollen right knee, upon which he is unable to weight bear. He has been
previously well and has no relevant family history. The clinic nurse has recorded his
temperature as 37.9C and a random blood glucose is 7.3 mmol/l.
What is your preliminary differential diagnosis?
What additional history and examination is relevant?
What are the key investigations?
How should he be managed?

Background
Differential diagnosis
The knee is one of the most common joints affected by monoarthritis, which is fortunate
since it is so easy to aspirate. The differential diagnosis of monoarthritis is listed in Table
2.1.

Table 2.1 Differential diagnosis of monoarthritis

Trauma Meniscal or ligamentous tears haemarthrosis


Sepsis Gonococcal arthritis, Staphylococcus aureus, penetrating injury, foreign body
Reactive arthritis Following gastrointestinal or genitourinary infection
Haemophilia
Crystal arthritis Gout, pseudogout
Inflammatory e.g. Rheumatoid, psoriatic

Trauma conjures images of motor vehicle accidents or dramatic tackles in rugby; how-
ever, much more mundane twisting injuries or valgus/varus strains when under load are
common. A rapidly developing joint swelling within minutes of the injury is suspicious
of an anterior cruciate ligament tear with involvement of the blood vessel running along
its surface. If internal mechanical derangement is considered possible, then imaging or

Atlas Medical Publishing Ltd


01-PS Rheumatology-cpp:01-PS Rheumatology-ppp.QXD 18/3/08 14:28 Page 7

02 An acutely swollen/hot joint 7

Table 2.2 Common errors in diagnosing acute monarthritis

Error Reality

The problem is the joint because the patient Surrounding soft tissues, including bursitis, may be the
has joint pain source of pain
The presence of intra-articular crystals excludes Crystals can be present in a septic joint
infection
Fever distinguishes infectious causes from Fever may be absent in septic monoarthritis, and in the
other causes immunocompromised patient. Acute crystal arthritis may cause
fever
A normal serum urate makes gout unlikely, and a Serum urate is normal for 30% of acute gout attacks, and only
high level confirms gout 5% of those with hyperuricaemia develop gout each year
Gram staining and culture of synovial fluid are Fastidious, slow-growing organisms, or fragile organisms, may
sufficient to exclude infection not be identified in early infection. Liaison with the laboratory is
required for specialist media and prolonged incubation

orthopaedic review is warranted. Table 2.2 highlights some common errors in diagnosing
acute monoarthritis.
The presence of fever suggests infection, and the patient should be questioned and
examined to determine the likely source. Septic arthritis is usually exquisitely tender with
resistance to joint movement. Staphylococci are the most common cause of muscu-
loskeletal sepsis, with the prevalence of both streptococcal and mycobacterial infection
increasing. For infections with staphylococci, streptococci, Gram-negative bacteria and
anaerobes, only one joint is usually involved. Polyarticular involvement is more likely in
the elderly or immunosuppressed, with infection by Haemophilus influenza, meningo-
cocci and Neisseria gonorrhoeae. Lyme disease can present with knee involvement,
although the diagnosis can be quickly excluded if there has been no exposure to the tick
vector of Borrelia burgdorferi.
In young patients, gonococcal arthritis is the most common non-traumatic acute
monoarthritis, and questioning regarding sexual partners and genitourinary symptoms
is necessary. In addition to arthritis (often polyarticular), tenosynovitis and a pustular
rash of the extremities should be sought. Gonococcal arthritis is 34 times more com-
mon in women, who often develop arthritis in the perimenstrual period. Men often
experience a urethritis as dysuria, and may notice a morning discharge, whereas women
may be asymptomatic.
Reactive arthritis is a sterile arthritis, occurring distant in both time and place from an
inciting infection (usually gastrointestinal or genitourinary). Lower limb asymmetric
oligoarthritis is most common, with associated enthesitis such as Achilles tendinitis, and
mucocutaneous features of conjunctivitis, pustular rash on the hands and feet and sterile
urethritis. Common triggers are genitourinary infection with Chlamydia trachomatis and
gastrointestinal infection with Salmonella typhimurium, Shigella flexneri, Campylobacter
jejuni and Yersinia enterocolitica. Stool culture and collection of early morning urine for
detection of chlamydia DNA by polymerase chain reaction (PCR) should be considered.
Crystal arthritis is both dramatic and rapid in onset, with the most commonly impli-
cated crystals being uric acid, calcium pyrophosphate and hydroxyapatite. Gout is
01-PS Rheumatology-cpp:01-PS Rheumatology-ppp.QXD 18/3/08 14:28 Page 8

8 01 General Rheumatology and Soft Tissue Rheumatism

unusual in the young and is usually preceded by more distal joint involvement, classically
the first metatarsophalangeal joint (podagra). Pseudogout or calcium pyrophosphate
dihydrate (CPPD) deposition disease is uncommon below the age of 50 years and
the knee is most often involved, followed by wrist and shoulder. Basic calcium phosphate
(hydroxyapatite) results in a calcific periarthritis, which most commonly affects the
shoulder.

Aspirating a knee joint


Every medical graduate should feel confident to undertake knee aspiration (Figure 2.1).
The knee is exposed with the patient lying so that you can obtain access to either the
medial or lateral aspect. The knee is generously cleaned with antiseptic and allowed to dry
whilst you are preparing the aspiration syringes. The patella is pinched between thumb
and index finger at its midpoint, which allows you to detect tension in the quadriceps
muscles and also allows you to distract the patella upwards to increase the infrapatellar
space. Local anaesthetic (510 ml) is infiltrated via a 21G or 23G needle at a point proxi-
mal and inferior to where you are holding the patella, noting that the pain-sensitive
structures are the dermis and the thickened synovium as you enter the joint. When the
anaesthetic has been given time to work, the joint is aspirated along the same needle track
with a fresh 1020 ml syringe and 18G needle. Afterwards, a dressing is applied firmly for
several minutes to ensure haemostasis and to prevent synovial fluid leakage.

Figure 2.1 Arthrocentesis of the left knee medial approach.

Only 12 ml of fluid is sufficient to complete all investigations; however, the joint


should be aspirated of as much fluid as possible without increasing the trauma of the pro-
cedure. Substantial pain relief is achieved by aspirating a tense effusion, and while reac-
cumulation will occur, it buys some time while the preliminary investigation results are
received. As the aspirate is removed, you should note its colour, viscosity and turbidity.
Normal synovial fluid is similar to egg white (syn = resembling, ovium = egg) and is both
viscous and acellular. As the degree of inflammation increases from the negligible
amount found in osteoarthritis to the mid-range of rheumatoid arthritis and the extreme
of septic arthritis the viscosity decreases and the cellularity and turbidity increase.
01-PS Rheumatology-cpp:01-PS Rheumatology-ppp.QXD 18/3/08 14:28 Page 9

02 An acutely swollen/hot joint 9

Table 2.3 Synovial fluid characteristics

Normal Non-inflammatory Inflammatory Septic

Colour Clear Straw yellow Yellow Variable


Clarity Transparent Transparent Hazy opaque Opaque
Viscosity High High Low LowThick
WBC ( 106/l) 0200 2002000 200075 000 >75 000
Neutrophils <25% <25% 25%50% >75%
WBC, white blood cell.

Blood-coloured effusions suggest either trauma or CPPD deposition disease. Synovial


fluid characteristics are shown in Table 2.3.
It is suggested that approximately 2 ml of fluid is collected into a container plus anti-
coagulant, and the remaining fluid collected in a large-volume sterile container. Tests
requested should include an urgent Gram stain, cell count and differential count, crystal
examination using polarized light microscopy and culture. If gonococcal or fungal infec-
tions are suspected, this needs to be highlighted as it influences the culture medium and
length of culture required.
Analgesics, antipyretics and rest should be employed in the first instance, with the
aspiration itself often affording a considerable pain relief. If septic arthritis is suspected,
then intravenous antibiotics covering Staphylococcus aureus and N. gonorrhoeae should
be commenced after the synovial fluid aspiration. The presence of bacteria on Gram
staining or subsequent bacterial growth requires specialist medical and orthopaedic
review to combine antibiotic therapy with joint lavage.
Gout is confirmed by the presence of intracellular, negatively birefringent urate crys-
tals, with intracellular pyrophosphate crystals confirming pseudogout. Both of these con-
ditions are self-limited and spontaneously improve over a few days. Adequate hydration
combined with analgesia and the introduction of a non-steroidal anti-inflammatory
drug will generally suffice. Colchicine at a dose sufficient to impact on acute gout invari-
ably causes diarrhoea. If you have confirmed the joint is sterile, then intra-articular corti-
costeroid injection provides excellent resolution.

Recent Developments
1 A prospective study of children presenting for investigation of possible septic
arthritis of the hip concluded that oral temperature >38.5C was the best
predictor, followed by an elevated serum C-reactive protein (CRP), an elevated
erythrocyte sedimentation rate, refusal to weight bear and an elevated white cell
count.2 CRP >20 mg/l was a strong independent risk factor and a valuable tool
for assessing and diagnosing septic arthritis of the hip. As the number of risk
factors increases so does the predicted probability of septic arthritis, such that
three to five factors present is associated with 83%98% predictive probability.
01-PS Rheumatology-cpp:01-PS Rheumatology-ppp.QXD 18/3/08 14:28 Page 10

10 01 General Rheumatology and Soft Tissue Rheumatism

2 Increased plasma procalcitonin (PCT) may be a useful marker for osteomyelitis


but not septic arthritis. Procalcitonin is cleaved in neuroendocrine tissues such
as thyroid C cells, lung and pancreatic tissue to calcitonin. During infection,
large amounts of PCT are released. The source is probably monocytes stimulated
by endotoxin, and hepatocytes stimulated by tumour necrosis factor or
interleukin-6. The role of PCT measurement with a rapid immunoassay was
investigated in children admitted with suspected osteomyelitis or septic arthritis.3
The authors reported specificity of 100% and sensitivity of 58% for osteomyelitis
and the same specificity, but lower 27% sensitivity, in septic arthritis.
3 High-resolution magnetic resonance imaging (MRI) of soft tissues and joints is
increasingly used prior to interventions such as arthroscopy. In a cohort of
children, Luhmann and colleagues4 compared radiological interpretation of knee
MRI with that of the surgeon who integrated the history, clinical examination,
plain radiographs, MRI scans and radiologist report. The pre-operative diagnosis
by the surgeon was better (P <0.05) than the formal radiology interpretation with
respect to anterior cruciate ligament tear, lateral meniscal tear, osteochondritis
dissecans and discoid lateral meniscus.

Conclusion
An acutely hot, swollen joint is an urgent presentation. Exclusion of sepsis is mandatory,
particularly in children and immunocompromised patients. Joint aspiration remains the
investigation of choice. Subsequently, treatment will often include antibiotics, pending
laboratory results, combined with judicious use of analgesia and anti-inflammatory
medications. Analysis of synovial fluid is valuable in establishing the diagnosis of gout,
particularly in joints other than the classical podagra of the great toe. Patients often inter-
pret the doctors it could be gout comment about their sore joint as either a definitive
diagnosis or as a slur on an indulgent lifestyle, when neither may be intended.

Further Reading
1 Siva C, Velazquez C, Mody A, Brasington R. Diagnosing acute monoarthritis in adults: a
practical approach for the family physician. Am Fam Physician 2003; 68: 8390.
2 Caird MS, Flynn JM, Leung YL, Millman JE, DItalia JG, Dormans JP. Factors distinguishing
septic arthritis from transient synovitis of the hip in children. A prospective study. J Bone Joint
Surg Am 2006; 88: 12517.
3 Butbul-Aviel Y, Koren A, Halevy R, Sakran W. Procalcitonin as a diagnostic aid in
osteomyelitis and septic arthritis. Pediatr Emerg Care 2005; 21: 82832.
4 Luhmann SJ, Schootman M, Gordon JE, Wright RW. Magnetic resonance imaging of the knee
in children and adolescents. Its role in clinical decision-making. J Bone Joint Surg Am 2005; 87:
497502.
01-PS Rheumatology-cpp:01-PS Rheumatology-ppp.QXD 18/3/08 14:28 Page 11

03 Painful shoulders rotator cuff and frozen shoulder 11

P R O B L E M

03 Painful Shoulders Rotator Cuff and


Frozen Shoulder

Case History
Mr Lawrence, a 76-year-old retired driver, is having difficulty living independently after
returning home following a recent myocardial infarction. On the day of discharge he fell
heavily, landing on his left upper arm. His concern is a very painful left shoulder,
especially at night and when he tries to move his left arm during the day.
How would you determine whether he has adhesive capsulitis (frozen shoulder)?
Is there a role for medical imaging, and if so, what modality?
What treatment should be initiated?

Background
Shoulder pain is an almost unavoidable life experience; in one study, 7% of an adult pop-
ulation aged 2575 years reported at least one months shoulder pain in the previous
year. The peak annual incidence of shoulder disorders is in the fourth and fifth decades,
at a rate of 0.25%. A Dutch study found that 25% of all 85-year-olds in Leiden suffered
from chronic shoulder pain and restriction. Community-based surveys concur with this
high incidence of soft tissue lesions about the shoulder, with roughly equal sex incidence.
Up to 20% of patients with chronic symptoms and 65% of all diagnoses relate to lesions
of the rotator cuff. Rotator cuff disease is the most common cause of shoulder pain found
in these studies. An ultrasound study found rotator cuff tears in 13% of 5059-year-olds,
20% of 6069-year-olds, 31% of 7079-year-olds and 51% of subjects aged over 80 years,
even when they were asymptomatic.
Table 3.1 summarizes causes of shoulder pain. The pain-sensitive structures of the
shoulder are mainly innervated by the fifth cervical segment (C5), so that pain from these
structures is referred to the C5 dermatome creating the sensation of pain over the anter-
ior arm, especially the deltoid insertion. The acromioclavicular joint is innervated by the
C4 segment pain arising here is felt at the joint itself and radiates over the top of the
shoulder into the trapezius muscle and to the side of the neck.

Clinical assessment
A history of trauma, marked night pain and weakness on resisted abduction strongly sug-
gests a rotator cuff tear. The sleeping position that induces night pain is an important clue:

Atlas Medical Publishing Ltd


01-PS Rheumatology-cpp:01-PS Rheumatology-ppp.QXD 18/3/08 14:28 Page 12

12 01 General Rheumatology and Soft Tissue Rheumatism

Table 3.1 Causes and clinical characteristics of shoulder pain

Category Cause Clinical features

Extracapsular Rotator cuff and subacromial bursa (e.g. impingement Painful arc of abduction
lesions syndromes, calcific tendinitis, cuff tears, bursitis) Pain on resisted cuff muscle movements, with intact
passive movement (allowing for pain and guarding)
Pain on impingement manoeuvres as the inflamed
rotator cuff tendons impinge on the inferior surface of
the acromion and coracoacromial arch
Intracapsular Glenohumeral joint (inflammatory arthritis RA, Loss of both active and passive movement
lesions spondyloarthritis, pseudogout) Reduced glenohumeral range
Joint capsule (adhesive capsulitis) Night pain
Bone disease (Pagets disease, metastases) Muscle strength, allowing for pain, is intact
Referred pain Cervical spine (facet joint root impingement, discitis) Arm and hand pain with paraesthesia
Brachial plexus (brachial amyotrophy) Marked muscle weakness and wasting
Thorax (Pancoasts tumour) Neck pain and stiffness
Thoracic outlet syndrome Herpes zoster rash
Suprascapular nerve entrapment Systemic features with weight loss
Subdiaphragmatic (abscess, blood, hepatic lesions)

shoulder pain that results in awakening when not lying on that shoulder is found in adhe-
sive capsulitis and inflammatory arthritis; pain when lying on the affected shoulder is seen
in acromioclavicular joint disease and rotator cuff disease. Prior shoulder problems sug-
gest rotator cuff disease with chronic impingement, or calcific tendinitis. A history of
marked shoulder joint swelling suggests inflammatory arthropathy with the presence of
an anterior bulge in the shoulder usually secondary to a subacromial bursa effusion.
Glenohumeral osteoarthritis (OA) is characterized by morning stiffness, pain with use and
chronicity of symptoms. OA, however, is less common than rotator cuff dysfunction.
Examination of the shoulder is best undertaken with the patient wearing the mini-
mum of upper body clothing. The contours of the shoulder are examined for wasting,
asymmetry and muscle fasciculation. Palpation should proceed from the sternoclavicular
joint along the clavicle to the acromioclavicular joint, to the tip of the acromion and the
humeral head beneath the acromion. The shoulder range of movement should be exam-
ined both actively and passively, with muscle strength and pain on resistance assessed.
There are essentially three movements to test in the shoulder: abduction due to
supraspinatus contraction; external rotation as a result of infraspinatus and teres minor
movement; and internal rotation due to subscapularis movement (Box 3.1).

Box 3.1 Practice Point


Three positive clinical tests (supraspinatus weakness, weakness of external rotation and
impingement) or two positive results for a patient older than 60 years are highly pre-
dictive of a rotator cuff tear.1

Complete rotator cuff tears will show no active abduction but near full-range movement
when passively moved. During examination ask about a painful arc during abduction
(Figure 3.1). When examining active and passive abduction you should stand behind the
patient and place one hand over the shoulder and scapula. The scapula should not begin to
01-PS Rheumatology-cpp:01-PS Rheumatology-ppp.QXD 18/3/08 14:28 Page 13

03 Painful shoulders rotator cuff and frozen shoulder 13

Painful arc of abduction


acromioclavicular joint

180
Painful arc of
abduction in
rotator cuff

120

70

Figure 3.1 Painful arc: the patient slowly abducts the arm as high as possible, describing symptoms as the
arm rises.

elevate or rotate until at least 90 degrees of abduction has been reached. Early scapulotho-
racic movement localizes the abnormality to the glenohumeral joint or capsule, as seen in
frozen shoulder. You should examine external rotation at 0 degrees abduction, with the
elbow beside the chest, and if external rotation is absent then a frozen shoulder is likely.
Next re-examine both internal and external rotation at 90 degrees abduction; if both are
restricted, a frozen shoulder is again likely. Bicipital tendinitis is examined by testing
resisted flexion at 30 degrees external rotation, and feeling for tenderness in the bicipital
groove. Shoulder impingement can be reproduced by internally rotating the arm held
flexed at 90 degrees and bringing the inflamed rotator cuff against the anterior acromion.
The empty can test is suggestive of a rotator cuff tear: it shows pain on resisted elevation of
the inverted arm held extended at 90 degrees, as if emptying a can of drink.

Rotator cuff disease


The glenohumeral joint is, by virtue of its anatomical shape, inherently unstable, relying
on the joint capsule as well as the rotator cuff muscles (supraspinatus, infraspinatus and
subscapularis) for additional stability. Impingement of the rotator cuff between the prox-
imal humerus and the acromioclavicular arch may occur from anomalies of the arch
(structural impingement) and from instability due to joint hyperlaxity or weak rotator
cuff muscles (functional impingement). Coracoacromial arch anomalies may be congen-
ital, dependent on acromial shape. Three shapes have been described flat, curved and
01-PS Rheumatology-cpp:01-PS Rheumatology-ppp.QXD 18/3/08 14:28 Page 14

14 01 General Rheumatology and Soft Tissue Rheumatism

hooked although there is poor inter-observer agreement on identifying the shape.


Acquired impingement occurs secondary to osteophytes growing from the acromioclav-
icular joint or calcification of the acromioclavicular ligament. Impingement occurs when
the cuff becomes compressed in the subacromial space as the arm is elevated. As the
humeral head rotates, the rotator cuff tendons are compressed between the greater
tuberosity of the humerus and the anterior edge of the acromion, the coracoacromial lig-
ament, the under-surface of the acromioclavicular joint and with the reactive inflamma-
tory subacromial bursa.
In addition to the impingement theory, a vascular theory has been proposed. With the
arm at the side, it has been suggested that the supraspinatus tendon has a relative avascu-
lar area 1 cm proximal to its insertion at the greater tuberosity, directly beneath the
impingement zone. This may be affected by the position of the shoulder and increases
with age. However, the infraspinatus tendon has a similar vascular watershed area, sug-
gesting that factors other than vascularity are important. Chronic irritation in the avascu-
lar region produces tendinitis, leading to local inflammation and further compression.
Other causes of tendinitis include trauma, instability and possibly infarction of the cuff in
patients with vascular disease. The vascular and impingement theories are not mutually
exclusive and it is possible that the high incidence of supraspinatus pathology is the result
of impingement in and around a critical zone of vascular supply.
With time, wearing and attrition of the cuff leads to impaired action or rupture of the
short rotators stabilizing the humeral head into the glenoid fossa, so that the deltoid pulls
the humerus against the under-surface of the acromion and a vicious impingement cycle
is established. Impingement-caused tears are usually incomplete in the supraspinatus
and infraspinatus tendons and complete in the long head of biceps. Complications of
impingement include a frozen shoulder, rupture of the rotator cuff tendons or long head
of biceps and, in elderly patients with a long-standing tear, a feared end-stage lesion,
recurrent haemorrhagic shoulder of the elderly.
Treatment depends on the mechanism of impingement. Patients with functional
impingement are treated with a resting sling for 2436 hours, pendular exercises and full-
dose non-steroidal anti-inflammatory drug (NSAID). Structural impingement is treated
similarly but the surgical options of arthroscopic surgery to remove osteophytes or trim
the acromion are available. Corticosteroid injection to the subacromial space can be
combined with an initial 47 days of pendulum exercises and avoidance of abduction
prior to a programme of shoulder-strengthening exercises. Infraspinatus strengthening
may be important to provide stabilization of the humeral head to prevent superior sub-
luxation on abduction.
Studies of eccentric loading exercises have shown promising results, particularly in
lesion of the Achilles tendon. Eccentric loading exercises involve a load being applied to a
muscle in its contracted position and the muscle is lengthened under the load. In the
shoulder, the supraspinatus would be contracted with the arm abducted and under load
the arm would slowly return to the side. Exercise programmes require highly motivated
people and there is concern that exercises can increase symptoms initially.

Frozen shoulder/adhesive capsulitis


Initially described in 1872, this condition remains as difficult to treat and difficult to
explain from the point of pathology as Codman observed in 1934. This common disor-
01-PS Rheumatology-cpp:01-PS Rheumatology-ppp.QXD 18/3/08 14:28 Page 15

03 Painful shoulders rotator cuff and frozen shoulder 15

der (2% cumulative risk in an at-risk population annually) is frequently misdiagnosed


and is characterized by painful restriction of all shoulder movements, both active and
passive, with characteristic restriction in the glenohumeral range. There is marked reduc-
tion or absence of shoulder external rotation at 0 degrees abduction, reduction of both
internal and external rotation at 90 degrees abduction, as well as prominent restriction of
placing the hand behind the back on internal rotation. Frozen shoulder is characterized
pathologically by fibrosis and retraction affecting predominantly the anterior and infer-
ior structures of the glenohumeral joint capsule. Patients usually present in the sixth
decade and onset before the age of 40 years is uncommon. Table 3.2 lists the diseases
associated with frozen shoulder, diabetes being the most significant. Diabetes, particu-
larly long-standing insulin-dependent diabetes, is associated with glycosylation of subcu-
taneous collagen and the development of soft tissue contraction so called diabetic
cheiroarthropathy.

Table 3.2 Common disorders associated with frozen shoulder

Acute shoulder trauma and shoulder immobilization


Diabetes mellitus
Thyroid disease (both hyper- and hypothyroidism)
Cardiac disease, particularly after cardiac surgery
Neurological disease with loss of consciousness or hemiplegia
Pulmonary disease tuberculosis and carcinoma
Rotator cuff disease, especially cuff tear
Acute glenohumeral joint inflammation

Three phases of frozen shoulder are recognized:


1 Painful inflammatory phase. Beginning insidiously, with often only a minor injury
being recalled, nocturnal awakening pain develops. The pain may be constant and
prevents the patient lying on the shoulder. Physiotherapy often aggravates symptoms
at this stage and corticosteroid injections are of limited benefit. This phase lasts
29 months.
2 Frozen shoulder. With time, the night and rest pain eases, but the shoulder remains
frozen. Mean duration is 412 months.
3 Recovery phase. After a mean delay of 526 months, shoulder limitation slowly
recovers in the majority of patients towards normal range (usually a 10%30% loss
of motion, which is often undetected by the patient). The total duration of
symptoms lasts 1242 months, with mean disease duration of 30 months.
In 10%20% of patients a contralateral frozen shoulder develops, usually milder than
the first, while the original shoulder is thawing. It is important to educate patients that
the condition will spontaneously resolve and the stiffness will greatly reduce. NSAIDs
and analgesics are used but there are no randomized controlled trials studying efficacy. A
prospective study in frozen shoulder compared exercise within the limits of pain with
intensive physiotherapy. Those who performed exercises within the limits of pain had
better results, recorded as near-normal painless shoulder movement (64% of patients at
12 months, 89% at 24 months), compared to intensive physiotherapy (63% of patients at
01-PS Rheumatology-cpp:01-PS Rheumatology-ppp.QXD 18/3/08 14:28 Page 16

16 01 General Rheumatology and Soft Tissue Rheumatism

24 months).2 An early meta-analysis by Hazleman on the use of intra-articular steroids


reported that the outcome depended on the duration of symptoms and hence possible
stage of disease. Patients who receive the injection earlier in the course of the disease
recover more quickly.3 An extensive meta-analysis by Buchbinder et al. found a benefit
for glenohumeral intra-articular corticosteroid injection for frozen shoulder compared
with placebo.4
For those unable to tolerate the pain and disability of a frozen shoulder, manipulation
under anaesthesia (MUA) is a reliable way to improve the range of movement. It is par-
ticularly indicated when disability persists after six months of non-operative therapy.
More recently, arthroscopic release of the capsule has been advocated as a more con-
trolled release of the capsule than MUA. Arthroscopic release also avoids the complica-
tions of MUA such as fracture of the humerus and iatrogenic intra-articular shoulder
lesions.5

Imaging
Imaging is undertaken primarily when considering referred pain or a malignant process.
In the assessment of rotator cuff disease, no imaging may be required initially, and may
only be undertaken subsequently if the clinical progression is not as expected. A plain X-
ray should then be the initial imaging modality, because if there is marked superior
migration of the humeral head, there must be complete rotator cuff disruption. Either
magnetic resonance imaging or ultrasound can confirm a possible full-thickness rotator
cuff tear, although ultrasound is significantly cheaper and is preferred by patients.
Suspected partial-thickness tears are best verified with an ultrasound scan.1

Recent Developments
1 Oral steroids may be useful in frozen shoulder, particularly during the early
inflammatory phase. Buchbinder et al.6 undertook a randomized controlled trial
on a series of 50 patients and found that oral steroid therapy initially improved
the frozen shoulder but the effect did not last beyond six weeks. Their subsequent
analysis of five small trials, in which all subjects received physiotherapy or an
exercise programme, confirmed that oral prednisolone or cortisone when given
for 34 weeks had a modest benefit on pain and disability and ability to move
the shoulder.7
2 Recently, a neural aetiology for tendinopathy has been considered.8 Tendinopathy
was proposed as an appropriate term for a symptomatic primary tendon disorder,
as it made no assumption as to the underlying pathological process. Underlying
the neural theory are four basic observations: tendons are innervated; substance P
has been found in rotator cuff tendinopathy and is a pro-inflammatory mediator;
the neurotransmitter glutamate is also present in tendinopathy; and tendon nerve
cell endings are closely associated with mast cells. It has been tentatively
postulated that neural stimuli secondary to overuse or mechanical irritation lead
to mast cell degranulation and release of mediators that begin an inflammatory
cascade.
01-PS Rheumatology-cpp:01-PS Rheumatology-ppp.QXD 18/3/08 14:28 Page 17

03 Painful shoulders rotator cuff and frozen shoulder 17

Conclusion
Frozen shoulder is a common and painful condition that impacts adversely on an indi-
viduals activities of daily living. Despite being self-limited, recovery is protracted and a
high proportion of patients do not regain full function. As a condition, it is largely man-
aged in the community by primary physicians, physiotherapists and occupational thera-
pists. Treatments that aim to mechanically stretch or disrupt the joint capsule (MUA,
arthroscopic release or hydrodilation of the capsule) are reserved for those with severe
symptoms who have failed to progress with conservative therapy.

Further Reading
1 Diehr S, Ison D, Jamieson B, Oh R. Clinical inquiries. What is the best way to diagnose a
suspected rotator cuff tear? J Fam Pract 2006; 55: 6214.
2 Diercks RL, Stevens M. Gentle thawing of the frozen shoulder: a prospective study of
supervised neglect versus intensive physical therapy in seventy-seven patients with frozen
shoulder syndrome followed up for two years. J Shoulder Elbow Surg 2004; 13: 499502.
3 Hazleman BD. The painful stiff shoulder. Rheumatol Phys Med 1972: 11: 41321.
4 Buchbinder R, Green S, Youd JM. Corticosteroid injections for shoulder pain. Cochrane
Database Syst Rev 2003; CD004016.
5 Dias R, Cutts S, Massoud S. Frozen shoulder. BMJ 2005; 331: 14536.
6 Buchbinder R, Hoving JL, Green S, Hall S, Forbes A, Nash P. Short course prednisolone
for adhesive capsulitis (frozen shoulder or stiff painful shoulder): a randomised, double
blind, placebo controlled trial. Ann Rheum Dis 2004; 63: 14609.
7 Buchbinder R, Green S, Youd JM, Johnston RV. Oral steroids for adhesive capsulitis.
Cochrane Database Syst Rev 2006; CD006189.
8 Rees JD, Wilson AM, Wolman RL. Current concepts in the management of tendon
disorders. Rheumatology 2006; 45: 50821.
01-PS Rheumatology-cpp:01-PS Rheumatology-ppp.QXD 18/3/08 14:28 Page 18

18 01 General Rheumatology and Soft Tissue Rheumatism

P R O B L E M

04 Tennis Elbow and Golfers Elbow

Case History
Simon is a 48-year-old labourer. His work has required a large amount of manual
screwdriver use, and he presents with a three-month history of an increasingly painful
elbow. He now has trouble grasping objects such as a cup.
What is the difference between tennis elbow and golfers elbow?
What are the characteristics of each condition, and what treatment is indicated?

Background
Tennis elbow
Tennis elbow or lateral epicondylitis is an overload injury, which occurs after minor or
unrecognized microtrauma to the proximal insertion of the extensor muscles of the fore-
arm particularly extensor carpi radialis brevis. Tennis elbow is the most frequently
diagnosed elbow condition (Box 4.1); it occurs commonly in middle life (age 3555
years) and has an incidence in general practice of 47 cases per 1000. Despite its common
name, most cases occur in nontennis players and it is frequently a work-related enthe-
sopathy affecting up to 15% of workers in at-risk industries. Operative specimens reveal
tendon glycosaminoglycan infiltration and microtears, as well as new bone formation at
the attachment site. Both traction injury and ischaemia play a role in its development.
Flexion deformity is unusual, occurs late and is minimal. Loss of 20 degrees of exten-
sion cannot be attributed to tennis elbow and warrants investigation for arthritis,
impingement at the olecranon fossa or a soft tissue mass in the posterior aspect of the
elbow. Tennis elbow is usually self-limiting, having an average duration of six months to
two years, with 90% of subjects recovering within one year. Various conservative inter-
ventions exist including pain-relieving medications, corticosteroid injections, physio-
therapy, elbow supports, acupuncture, surgery and shockwave therapy (Box 4.2).

Box 4.1 Diagnosis of tennis elbow


Lateral elbow pain with tenderness on palpation just distal to the lateral
epicondyle
Worsening pain localizing to the lateral epicondyle on resisted wrist dorsiflexion
X-rays excluding calcific tendinitis, exostoses and osteoarthritis of the radio-ulnar
joint

Atlas Medical Publishing Ltd


01-PS Rheumatology-cpp:01-PS Rheumatology-ppp.QXD 18/3/08 14:28 Page 19

04 Tennis elbow and golfers elbow 19

Box 4.2 Treatment of tennis elbow


Structured physiotherapy consisting of elbow manipulation and exercise, supple-
mented with home exercises and self-manipulation
Practical advice booklet on self-management and ergonomics
Recommend avoidance of corticosteroid injections, as short-term benefit is offset
by a poorer longer-term outcome

Most important in treatment is activities modification both frequency and method


of performance. In tennis players, common errors are inadequate conditioning, incorrect
grip size, faulty backhand style and problems with the racquet and its stringing. In the
work setting, a review by an occupational therapist is recommended, particularly focus-
ing on pronation/supination movements and grip size. A physiotherapy programme that
includes strengthening exercises for the entire upper limb and a graded resistive pro-
gramme for wrist dorsiflexors is recommended.
In the setting of localized tenderness it is tempting to inject the lesion. As noted in Box
4.2, the short-term gain may be offset by a poorer long-term outcome. The injection
technique is a small volume of corticosteroid and local anaesthetic injected into the
tendinous insertion of extensor carpi radialis brevis into the lateral epicondyle. As the
injection is not into a potential space but into an already tender, dense area, the injection
is against resistance and is both uncomfortable and has the risk of steroid tracking super-
ficially to the subcutaneous tissues, leading to depigmentation and atrophy. In contrast
to other painful overuse syndromes in which total tendon ruptures have been reported
(Achilles, biceps, patella), the tendon of the extensor carpi radialis brevis is strongly con-
nected and supported by other extensors of the wrist.
A small number of patients have recalcitrant lateral epicondylitis and are considered
for operative intervention open, arthroscopic and percutaneous. Operative interven-
tions followed for a minimum of two years demonstrate an improvement compared to
pre-operative status, with no difference in outcome according to procedure technique.1

Golfers elbow
Golfers elbow or medial epicondylitis is the mirror image of tennis elbow, and is thought
also to relate to repetitive traction stress and microtears at the insertion of the forearm
flexors (flexor carpi radialis) and pronator teres into the medial epicondyle. It occurs in
both professional and amateur sports players, as well as manual workers such as brick-
layers. It is much less common than tennis elbow, with approximately one-twentieth the
incidence. Similar to tennis elbow, the diagnosis is clinical, with localized tenderness that
worsens on resisted wrist flexion and forearm pronation (Box 4.3).

Box 4.3 Golfers elbow


Elbow pain at the medial epicondyle
Increasing symptoms on resisted wrist flexion and resisted forearm pronation
Treatment includes modification of activities, upper limb exercises and analgesics
01-PS Rheumatology-cpp:01-PS Rheumatology-ppp.QXD 18/3/08 14:28 Page 20

20 01 General Rheumatology and Soft Tissue Rheumatism

Recent Developments
A randomized controlled trial compared the effectiveness of physiotherapy, cortico-
steroid injections and a wait and see approach in 198 patients with tennis elbow who
were randomized to the three treatment arms.2 Physiotherapy was eight sessions of
mobilization with movement and exercises plus home exercises and self-manipulation.
Injection therapy with triaminolone acetonide (10 mg) and 1% lidocaine was the second
study arm. The wait and see approach consisted of ergonomic instruction and use of
analgesics, heat, cold and braces if needed. At six weeks the main outcome measures
(global improvement, pain-free grip strength, assessors rating of complaints, severity of
elbow pain and elbow disability) were significantly better in the corticosteroid-treated
group than in the other groups. However, all groups were improving and the benefit of
the steroid injection was short-lived, such that a crossover occurred around twelve weeks,
with the one-year results showing physiotherapy superior to corticosteroid injections for
all outcome measures. Importantly, at one year, the injection-treated group was signifi-
cantly worse on all outcomes compared with the physiotherapy group, and on two out of
three measures compared with the wait and see group. The corticosteroid injection
group also had the most reported recurrences. A similar study with only seven weeks of
follow-up confirmed the benefits of steroid injections in the short term.3

Conclusion
Tennis elbow is a common problem in general practice and is best treated with the
knowledge that it is a self-limiting condition, with the majority of patients improving in
the medium term. Whilst corticosteroid injections offer short-term benefit, there is the
potential for both short-term adverse effects and the possibility of a worse outcome at
one year. Physiotherapy provides benefit that is slower in onset but is more sustained and
allows patients to become self-reliant in their own management.

Further Reading
1 Szabo SJ, Savoie FH, Field LD, Ramsey JR, Hosemann CD. Tendinosis of the extensor carpi
radialis brevis: an evaluation of three methods of operative treatment. J Shoulder Elbow Surg
2006; 15: 7217.
2 Bisset L, Beller E, Jull G, Brooks P, Darnell R, Vicenzino B. Mobilisation with movement and
exercise, corticosteroid injection, or wait and see for tennis elbow: randomised trial. BMJ
2007; 333; 93945.
3 Tonks JH, Pai SK, Murali SR. Steroid injection therapy is the best conservative treatment for
lateral epicondylitis: a prospective randomised controlled trial. Int J Clin Pract 2007; 61:
2406.
01-PS Rheumatology-cpp:01-PS Rheumatology-ppp.QXD 18/3/08 14:28 Page 21

05 Carpal tunnel syndrome and other entrapment neuropathies 21

P R O B L E M

05 Carpal Tunnel Syndrome and Other


Entrapment Neuropathies

Case History
Beatrix is a 33-year-old production-line worker. For the last four weeks she has been
awakening with painful pins and needles in her left hand and a dull pain that radiates
from her wrist to her elbow. Shaking the arm improves the symptoms and she sometimes
sleeps with her arm hanging out of the bed.
What are the clinical features of the carpal tunnel syndrome?
What is the role for imaging and nerve conduction studies?
What investigations are appropriate to determine the cause?
How would you manage this problem?

Background
Entrapment neuropathies are disorders where peripheral nerves are damaged by com-
pression as they pass through a bony or fibrous canal. The disorders may be precipitated
by repetitive motion or strain, and carpal tunnel syndrome (CTS) is by far the common-
est entrapment neuropathy and the most common focal peripheral neuropathy. The
median nerve, along with the flexor tendons, passes through the carpal tunnel, which is
bridged by the transverse carpal ligament (Figure 5.1). CTS affects 3% of the population
although there is an imperfect correlation between reported symptoms and electrophysi-
ological findings. Women are three times more likely than men to be affected with CTS,
and a number of predisposing conditions are recognized (Box 5.1).
CTS causes pain, numbness and tingling in the distribution of the median nerve: i.e.
anteriorly, in the lateral half of the ring finger to the median half of the thumb; and poste-
riorly, in the distal halves of the ring and middle fingers. If severe, the symptoms may
radiate up the arm and they can often occur at night, thus disturbing sleep. In severe cases
there is a loss of small muscle function, which impairs manual dexterity and can lead to
wasting of muscles of the thenar eminence. The symptoms of CTS are common and clin-
ical signs (Box 5.2) are not always present. Accurate diagnosis is one of the major deter-
minants of successful treatment. The diagnosis should be confirmed wherever possible
by nerve conduction studies.

Atlas Medical Publishing Ltd


01-PS Rheumatology-cpp:01-PS Rheumatology-ppp.QXD 18/3/08 14:28 Page 22

22 01 General Rheumatology and Soft Tissue Rheumatism

Median nerve

Tendon sheath

Carpal ligament

Bundle of tendons

Figure 5.1 Anatomy of the carpal tunnel.

Other common nerve entrapment syndromes


Thoracic outlet syndromes
These are due to compression of the brachial plexus and brachial vessels in the neck.
Costoclavicular syndrome, due to a narrowing of the space between the clavicle and first
rib, may arise from congenital abnormality or because of poor posture. Cervical rib syn-
drome is due either to an extra rib or to a fibrous band between the seventh cervical ver-
tebra and the sternum. Compression of nerves and vessels occurs as they pass over the
additional structures. Adsons test may be positive: the patient looks to the affected side
and takes a deep breath while the examiner lifts the arm to 90 degrees. If compression is
present, the radial pulse may disappear.
Suprascapular neuritis
The suprascapular nerve (cervical segments C5/C6) supplies sensation to the shoulder joint
and motor supply to the infraspinatus and supraspinatus muscles. It can become com-
pressed as it passes through the suprascapular notch and under the transverse ligament.
Ulnar neuritis
Compression of the ulnar nerve usually occurs in the canal, where it is covered by the arcu-
ate ligament. It may also occur between the two heads of flexor carpi ulnaris just distal to
the elbow joint. The syndrome may occur as a result of direct trauma or fracture, repetitive
01-PS Rheumatology-cpp:01-PS Rheumatology-ppp.QXD 18/3/08 14:28 Page 23

05 Carpal tunnel syndrome and other entrapment neuropathies 23

Box 5.1 Causes of CTS


Overuse Repetitive flexion or extension of the wrist, particularly while
gripping objects firmly
Use of walking stick in patients with mobility disorders
Occupational use of power tools, assembly-line work
Injury Colles fracture
Subluxation of the lunate bone
Arthritis Rheumatoid tendon sheath inflammation
Osteoarthritis
Gout or pseudogout
Wrist ganglion Outpouching of the wrist joint capsule
Increased canal Pregnancy
volume Obesity
Congestive cardiac failure
Lipoma
Infections Septic arthritis
Lyme disease
Tuberculosis
Metabolic Diabetes
Hypothyroidism
Acromegaly
Amyloidosis

Box 5.2 Clinical signs of CTS


Tinels sign Tapping over the median nerve elicits symptoms in the distribution
of the nerve
Phalens sign Place both hands together palm to palm, with the wrists extended to
90 degrees, and forearms horizontal and close to the chest. The
affected hand will begin to tingle within 12 minutes
Reverse Phalens As above, but with the hands placed back to back

movements or rheumatoid arthritis affecting the elbow joint. It causes pain and tingling
down the inside of the forearm to the little finger and medial aspect of the ring finger. The
nerve gives rise to a sensory supply to the skin of the hypothenar eminence and a motor
supply to muscles of the hypothenar eminence and other small muscles in the hand.
Median neuritis
This is a much less common syndrome and is usually due to entrapment of the nerve at
the elbow. Symptoms are similar to those of the carpal tunnel syndrome and may be
exacerbated by pronation of the forearm.
Radial neuritis
Again, this is relatively uncommon. Compression usually occurs at the elbow and causes
sensory symptoms in the forearm bone to the base of the thumb.
01-PS Rheumatology-cpp:01-PS Rheumatology-ppp.QXD 18/3/08 14:28 Page 24

24 01 General Rheumatology and Soft Tissue Rheumatism

Meralgia paraesthetica
The lateral cutaneous nerve of the thigh passes through the femoral canal, where it is
sharply angulated and liable to compression. The syndrome leads to sensory symptoms
in the middle and lower part of the lateral aspect of the thigh. It is caused by obesity,
direct trauma or by repetitive flexion of the thigh.
Anterior compartment syndrome
This part of the lower leg contains the tibialis anterior and extensor digitorum muscles
and the deep peroneal nerve (supplies skin between the first and second toes). The nerve
may be injured by unaccustomed running, as a result of tibial or fibular fractures or
through direct trauma.
Medial compartment syndrome
This is the most common lower-leg nerve entrapment syndrome. The symptoms include
pain and tenderness on the medial aspect of the shin (shin splints). It is often precipi-
tated by unaccustomed running on a hard surface.
Posterior compartment syndrome
This compartment contains the soleus and gastrocnemius muscles, which join together to
form the Achilles tendon and are responsible for plantar flexion. The syndrome is associated
with calf pain precipitated by exercise and with altered sensation on the sole of the foot.
The management of all of these nerve entrapment syndromes is somewhat similar: the
patient should rest wherever possible and avoid movements or actions that exacerbate
the symptoms; local injection with anaesthetic or steroid is indicated in some cases, and a
minority of patients require surgical decompression of the affected nerve.

Management of CTS1,2
General measures include trying to relax the grip, using grip-adapted implements
such as large pens, taking frequent breaks, keeping the hands warm and considering
posture and position (e.g. if using a keyboard, this should be at elbow height).
Conservative management with ultrasound has been advocated but there are limited
trial data to support this therapy.
Splinting the wrist in neutral position may alleviate symptoms related to soft tissue
swelling and is most effective when used soon after the onset of symptoms. Night-
time splinting is often sufficient.
Non-steroidal anti-inflammatory drugs are effective in some cases, although
improvement may be short-lived. Oral corticosteroids are more effective (e.g.
prednisolone 20 mg/day for 23 weeks, followed by reducing doses). Diuretics are
widely used but are often disappointing in their effect.
The use of local injection of anaesthetic and steroid into the proximal carpal tunnel
is supported by trial data. The outcome is probably comparable to that of systemic
steroids, but the patient is not exposed to the risk of side effects associated with high-
dose steroid therapy. The injection may be directly into the carpal tunnel or
proximal to the carpal tunnel. Benefit from local injection may last for up to three
months and is increased by concurrent splinting.
For patients who have either severe symptoms or do not respond to conservative
measures, surgery is required. This has traditionally been carried out by an open
01-PS Rheumatology-cpp:01-PS Rheumatology-ppp.QXD 18/3/08 14:28 Page 25

05 Carpal tunnel syndrome and other entrapment neuropathies 25

Confirm symptoms are in median nerve distribution

History and examination to search for underlying causes


Enquire about occupation and repetitive strain

Mild symptoms Moderate symptoms with signs Severe symptoms

Rest
Remove precipitating cause NCS imaging
Trial of splinting

No further action Confirmed diagnosis

Conservative measures

Local injection
Systemic steroids

Repeat treatment Consider surgery


at 3 months Open
Endoscopic

Figure 5.2 Investigation and management of CTS. Imaging is with high-resolution ultrasound or with
MRI. NCS, nerve conduction studies.

procedure, which can be performed without admission to hospital. More recently,


endoscopic carpal tunnel release through two small incisions has been used by many
surgeons. This has the advantage of causing less scarring.

Recent Developments
1 Not all patients have ready access to nerve conduction studies. Several studies have
shown that high-resolution ultrasound and magnetic resonance imaging (MRI) may
be very accurate in diagnosing CTS.3,4 These methods can demonstrate altered
01-PS Rheumatology-cpp:01-PS Rheumatology-ppp.QXD 18/3/08 14:28 Page 26

26 01 General Rheumatology and Soft Tissue Rheumatism

anatomy and decreased volume of the carpal tunnel, and in patients with CTS show
the median nerve is swollen distal to the compression.
2 Some familial cases of nerve entrapment are due to inherited anatomical
abnormalities. Recently, the condition of hereditary neuropathy with liability to the
pressure palsies (HNPP) has been described.5,6 This condition is inherited in an auto-
somal dominant manner and is due to a deletion at locus 17p11.2. HNPP is a slowly
progressive condition, punctuated by episodes of acute peripheral neuropathy at sites
that are liable to nerve entrapment.
3 Endoscopic surgery has revolutionized treatment of CTS. The two-portal endoscopic
approach to managing CTS has been adopted in many centres. Although this
approach is attractive, recent trials7,8 suggest that it has very little to offer over tradi-
tional open surgery. In general, surgical treatment is more successful than medical or
conservative treatment in patients with proven CTS.9

Conclusion
CTS is the most common form of entrapment neuropathy. Definitive diagnosis is by
nerve conduction studies, but ultrasound and MRI are increasingly being used to confirm
the diagnosis. It is worth routinely excluding hypothyroidism and diabetes as predispos-
ing causes but there is not usually a treatable or identifiable underlying cause. A limited
trial of conservative or medical measures is justified in mild cases but surgery is generally
required for severe, progressive or unresponsive cases.

Further Reading
1 Viera AJ. Management of carpal tunnel syndrome. Am Family Physician 2003; 68: 26572.
2 Ashworth N. Carpal tunnel syndrome. Clin Evid 2006; 15: 118.
3 Wiesler ER, Chloros GD, Cartwright MS, Smith BP, Rushing J, Walker FO. Use of diagnostic
ultrasound in carpal tunnel syndrome. J Hand Surg 2006; 31: 72632.
4 de Noordhout AM. Diagnosing entrapment neuropathies: probes and magnets instead of
electrodes and needles? Clin Neurophysiol 2006; 117: 4845.
5 Sander MD, Abbasi D, Ferguson AL, Steyers CM, Wang K, Morcuende JA. The prevalence of
hereditary neuropathy with liability to pressure palsies in patients with multiple surgically
treated entrapment neuropathies. J Hand Surg 2005; 30: 123641.
6 Koc F, Guzel R, Benlidayi IC, Yerdelen D, Guzel I, Sarca Y. A rare genetic disorder in the
differential diagnosis of the entrapment neuropathies: hereditary neuropathy with liability to
pressure palsies. J Clin Rheumatol 2006; 12: 7882.
7 Rab M, Grunbeck M, Beck H et al. Intra-individual comparison between open and 2-portal
endoscopic release in clinically matched bilateral carpal tunnel syndrome. J Plast Reconstr
Aesthet Surg 2006; 59: 7306.
8 Atroshi I, Larsson G-U, Ornstein E, Hofer M, Johnsson R, Ranstam J. Outcomes of endoscopic
surgery compared with open surgery for carpal tunnel syndrome among employed patients:
randomised controlled trial. BMJ 2006; 332: 14736.
9 Hui ACF, Wong S, Leung CH et al. A randomized controlled trial of surgery vs steroid
injection for carpal tunnel syndrome. Neurology 2005; 64: 20748.
01-PS Rheumatology-cpp:01-PS Rheumatology-ppp.QXD 18/3/08 14:28 Page 27

06 Fibromyalgia syndrome 27

P R O B L E M

06 Fibromyalgia Syndrome

Case History
Sandra is in her early 40s and is seeing you because she hurts from her scalp to her toes.
This has been present for at least eight years and is ruining her life. She tires easily and
aches with any activity. Her sleeping is restless, she awakes tired and she has an irritable
bowel. There are no abnormalities on physical examination.
What is fibromyalgia and how would you support this diagnosis?
Are there any investigations that might help?
What treatment, if any, would you suggest to Sandra?

Background
Fibromyalgia syndrome (FMS) is a soft tissue musculoskeletal condition with many
features in common with chronic fatigue syndrome, the major difference being the pre-
dominance of musculoskeletal features in FMS. Diagnosis of FMS is based on the
American College of Rheumatology (ACR) criteria (1990):
Pain on both sides of the body
Pain above and below the waist
Pain in an axial distribution
Local tenderness in at least 11 out of 18 defined trigger points (Figure 6.1)
The pain is often defined as aching or burning and varies in intensity and location
from day to day. Other features of FMS are shown in Table 6.1.

Table 6.1 Frequency of FMS symptoms

Symptom % Symptom %

Muscular pain 100 Paraesthesiae 52


Fatigue 96 Memory impairment 46
Insomnia 86 Leg cramps 42
Joint pains 72 Poor concentration 41
Headaches 60 Anxiety 32
Restless legs 56 Major depression 20

Atlas Medical Publishing Ltd


01-PS Rheumatology-cpp:01-PS Rheumatology-ppp.QXD 18/3/08 14:28 Page 28

28 01 General Rheumatology and Soft Tissue Rheumatism

Figure 6.1 Trigger points for the diagnosis of FMS. There are 18 points in total (nine identical locations on
each side). Anterior: anterior aspects of C5, C6 and C7; second rib; lateral epicondyle; knee (medial fat pad).
Posterior: suboccipital muscle insertions; supraspinatus muscle origin; trapezius (midpoint upper border);
gluteal (upper outer quadrants); greater trochanter. Adapted with permission from Borg-Stein 2006.1

Musculoskeletal pain is the most consistent feature of FMS. Fatigue can be almost as
debilitating. Disordered sleep is also a very frequent feature and contributes to fatigue
and to the mood disturbances. Sleep abnormalities are strongly correlated with the
alpha-electroencephalogram (EEG) abnormality and movement disorders including the
periodic jerking of arms and legs, teeth grinding (bruxism) and restless legs. Gastro-
oesophageal reflux disease occurs with high frequency, as does irritable bowel syndrome.
Headaches may be of the migraine or tension type. Facial pain is also relatively common,
including discomfort related to temporomandibular joint dysfunction. Psychological
and psychiatric morbidity are increased. There is high prevalence of anxiety disorders
including obsessive-compulsive disorder and post-traumatic stress disorder.2
01-PS Rheumatology-cpp:01-PS Rheumatology-ppp.QXD 18/3/08 14:28 Page 29

06 Fibromyalgia syndrome 29

Epidemiology and aetiology


A number of recent studies24 have examined incidence and prevalence of FMS. The esti-
mated prevalence is between 1% and 4%. FMS is between two and six times more likely
to occur in women. Incidence in the female population has been estimated at 11.3 per
1000 person-years. It can occur at any age but becomes more common with advancing
years. FMS has been associated with other rheumatic disorders including rheumatoid
arthritis (RA) and systemic lupus erythematosus (SLE).
There is clearly strong interplay between physical and psychological factors in FMS.
The onset of illness may be triggered by physical illness (including viral diseases) or by
trauma (including surgery). There is some suggestion that heredity may play a part, with
components of the serotonergic and dopaminergic systems being potential candidates
for involvement. Some of the symptomatology around the trigger points may be due to
increased acetylcholine at the motor endplate causing contraction and shortening of the
sarcomere. This may lead to increased energy consumption and increased local blood
supply, with resulting local tenderness. A number of local and systemic mediators have
been implicated. These include bradykinin, calcitonin gene-related peptide, substance P,
tumour necrosis factor-a, interleukin-1, noradrenaline and serotonin.

Investigations
Routine investigations including full blood count and biochemistry, plus erythrocyte
sedimentation rate (ESR), C-reactive protein (CRP) and other inflammatory markers
are within the normal range. Because thyroid disease is common, it is useful to include
thyroid function tests. There are no specific endocrine abnormalities. X-ray, computed
tomography (CT) and magnetic resonance imaging (MRI) scans are generally normal.
There are no specific abnormalities on muscle biopsy, electromyography or nerve con-
duction studies. EEG or more formal sleep studies may be requested in patients who have
marked sleep disturbance. This may reveal abnormalities including periodic limb move-
ment disorder, rapid eye movement (REM) sleep disorder or sleep apnoea. The diagnosis
of FMS is one of exclusion and is made clinically.

Prognosis, differential diagnosis and treatment


The outlook for FMS is variable and the condition tends to become chronic. However,
more widespread understanding and clearer definition, along with a more highly devel-
oped treatment flow, are beginning to streamline management and improve the outlook.
There is a danger that over-enthusiastic investigation might contribute to making the
condition more chronic. However, this should not deter the clinician from making a full
investigation and the clinical picture warrants it. The major differential diagnosis is other
connective tissue disorders, including rheumatoid disease, SLE and scleroderma. Major
differential diagnoses of FMS and investigation of the condition are summarized in
Figure 6.2.
There is no specific treatment for FMS. Therapeutic measures include the following:
General: investigation and clear diagnosis; educating the patient as to the nature of
the diagnosis and reassuring them; attention to psychological and social factors, and
encouraging the patient to have a normal sleep pattern as well as to engage in
physical activity consistent with their state of health and preferences.
01-PS Rheumatology-cpp:01-PS Rheumatology-ppp.QXD 18/3/08 14:28 Page 30

30 01 General Rheumatology and Soft Tissue Rheumatism

Affecting the axial skeleton


Musculoskeletal pain
Both sides of the body
Above and below the waist
At least 11/18 trigger points tender

Associated symptoms Joint swelling or ? Referred pain


deformity Visceral
Skeletal

Limited Ix Further Ix Imaging


FBC 1ESR 1ve RF X-rays
CRP ANF CT scan
FT4 2ve
1TSH Viral serology MRI scan

2ve
1ve
2ve
Classic FMS
(clinical diagnosis)

Consider:
RA
Scleroderma
SLE
Lyme disease

Figure 6.2 Investigation of suspected FMS. - ve, negative; + ve, positive; ANF, antinuclear factor ; CRP,
C-reactive protein; CT, computed tomography; ESR, erythrocyte sedimentation rate; FBC, full blood count;
FMS, fibromyalgia syndrome; FT4, thyroxine; Ix, investigation; MRI, magnetic resonance imaging; RA,
rheumatoid arthritis; RF, rheumatoid factor; SLE, systemic lupus erythematosus; TSH, thyroid stimulating
hormone. Adapted from Schneider et al. 2006.5

Pain relief. This may range from simple analgesics such as paracetamol to more
powerful agents. Inappropriate use of powerful opioid analgesics should be avoided as
this may lead to dependence and seldom alleviates the symptoms in the long term.
Non-steroidal anti-inflammatory drugs have marginal benefits over simple analgesics.
Tramadol is a weak opioid with some action to inhibit the uptake of serotonin.
Antidepressants. Either tricyclic antidepressants or selective serotonin reuptake
inhibitors are of benefit in many cases, even when there is not major evidence of
depression. Dual inhibitors of both noradrenaline and serotonin uptake, such as
duloxetine, may be of particular benefit.
a2-adrenergic agonists, such as clonidine or tizanidine, are of benefit to some
patients and may act by preventing the action of neurotransmitters such as
glutamate or substance P within the central nervous system.
Anticonvulsants may contribute to pain relief. There is increasing experience with
newer agents such as pregabalin or gabapentin, which have proved to be of use in
other painful neurological conditions.
01-PS Rheumatology-cpp:01-PS Rheumatology-ppp.QXD 18/3/08 14:28 Page 31

06 Fibromyalgia syndrome 31

Trigger point injection may be undertaken with local anaesthetic, steroid or


botulinum toxin. The latter may be more effective than steroid injection; it may act
by diminishing acetylcholine release, thus decreasing muscle activity and local
ischaemia.
Postural training, exercise and ergonomic adjustments (particularly in the
workplace) may help a patient to adapt to disability associated with FMS.
Stress reduction may be achieved by a variety of means including cognitive
behavioural therapy, relaxation techniques and biofeedback methods.
Physical therapies including acupuncture, massage, transcutaneous electrical nerve
stimulation and ultrasound may be helpful. These should always be performed by a
suitably qualified and reputable practitioner.

Recent Developments
1 There is some discordance between the clinical diagnosis of FMS and the diagnosis
using ACR criteria.6 In a survey of 206 patients, FMS was diagnosed in 49.0% while
only 29.1% met ACR criteria. The authors of the report proposed a survey method
for diagnosis of FMS that did not require clinical examination.
2 Improved understanding of the pathophysiological basis for the syndrome is not only
lending credence to the diagnosis, but is also giving rise to more logical management.7
Patients are often mislabelled as having a purely psychological diagnosis, leading to
the undertreatment or mismanagement of the physical symptoms. Increased activity
of the hypothalamicpituitaryadrenal axis and the sympathetic nervous system has
been proposed to be responsible for some of the clinical features.8
3 More than 90% of patients have tried alternative or complementary treatments.
Acupuncture has been widely used but not intensively studied. A recent randomized
controlled trial of acupuncture in FMS9 reported greater improvements in the
treatment group compared with the control group. Symptoms of fatigue and anxiety
were particularly improved and the treatment is very well tolerated. These patients
also experienced better pain relief.
4 Being overweight or obese is associated with a variety of physical and psychological
symptoms, including many of the symptoms that form part of the FMS complex.
Results of the recently published behavioural weight loss programme10 confirm that
modest weight loss is associated with an improvement in FMS symptoms.

Conclusion
Attempts should be made to establish the diagnosis in the above patient. This should
include the exclusion of thyroid disease and connective tissue disorders. Thorough med-
ical assessment should be undertaken, with care not to increase the patients anxiety.
Once the diagnosis is established, this should be carefully explained to the patient and a
realistic management plan should be agreed. This should include trying to manage pain
and sleep disturbance with the minimum of pharmacological intervention. Physical
activity should be encouraged within the limits imposed by her condition. Weight loss is
01-PS Rheumatology-cpp:01-PS Rheumatology-ppp.QXD 18/3/08 14:28 Page 32

32 01 General Rheumatology and Soft Tissue Rheumatism

important in the overweight patient for future health and may also help to alleviate the
symptoms of FMS.

Further Reading
1 Borg-Stein J. Treatment of fibromyalgia, myofascial pain, and related disorders. Phys Med
Rehabil Clin N Am 2006; 17: 491510.
2 Raphael KG, Janal MN, Nayak S, Schwartz JE, Gallagher RM. Psychiatric comorbidities in a
community sample of women with fibromyalgia. Pain 2006; 124: 11725.
3 Weir PT, Harlan GA, Nkoy FL et al. The incidence of fibromyalgia and its associated comor-
bidities: a population-based retrospective cohort study based on International Classification
of Diseases 9th Revision codes. J Clin Rheumatol 2006; 12: 1248.
4 McNally JD, Matheson DA, Bakowsky VS. The epidemiology of self-reported fibromyalgia in
Canada. Chronic Dis Can 2006; 27: 916.
5 Schneider MJ, Brady DM, Perle SM. Commentary: differential diagnosis of fibromyalgia
syndrome: proposal of a model and algorithm for patients presenting with the primary
symptom of chronic widespread pain. J Manipulative Physiol Ther 2006; 29: 493501.
6 Katz RS, Wolfe F, Michaud K. Fibromyalgia diagnosis: a comparison of clinical, survey, and
American College of Rheumatology criteria. Arthritis Rheum 2006; 54: 16976.
7 Dadabhoy D, Clauw DJ. Fibromyalgia: progress in diagnosis and treatment. Curr Pain
Headache Rep 2005; 9: 399404.
8 Sarzi-Puttini P, Atzeni F, Diana A, Doria A, Furlan R. Increased neural sympathetic activation
in fibromyalgia syndrome. Ann NY Acad Sci 2006; 1069: 10917.
9 Martin DP, Sletten CD, Williams BA, Berger IH. Improvement in fibromyalgia symptoms
with acupuncture: results of a randomized controlled trial. Mayo Clin Proc 2006; 81: 74957.
10 Shapiro JR, Anderson DA, Danoff-Burg S. A pilot study of the effects of behavioural weight
loss treatment on fibromyalgia symptoms. J Psychsom Res 2005; 59: 27582.
01-PS Rheumatology-cpp:01-PS Rheumatology-ppp.QXD 18/3/08 14:28 Page 33

07 Plantar fasciitis 33

P R O B L E M

07 Plantar Fasciitis

Case History
Shirley is aged 43 years and presents with exquisite pain beneath her left heel when
walking. She is moderately obese (body mass index 32 kg/m2) and finds it hard to walk
first thing in the morning. She prefers to wear open sandals.
Should Shirley have X-rays to determine whether she has a plantar spur?
What effect, if any, does Shirleys weight have on her condition?
What are the current supported therapies?

Background
Plantar fasciitis commonly causes inferior heel pain and occurs in up to 10% of the
United States population. It affects both active and sedentary adults of all ages, but is
more likely to occur in persons who are obese, who spend most of the day on their feet or
who have limited ankle dorsiflexion. Plantar fasciitis is a musculoskeletal disorder pri-
marily affecting the fascial enthesis. Although poorly understood, development of plan-
tar fasciitis is thought to have a mechanical origin. In particular, pes planus foot types
and lower-limb biomechanics that result in a lowered medial longitudinal arch create
excessive tensile strain within the fascia, producing microscopic tears. The roles of both
chronic inflammation and arch mechanics in the aetiology of plantar fasciitis are
controversial.1
Diagnosis is based on the history and physical examination (Table 7.1); the differential
diagnosis is listed in Table 7.2. Unaccustomed walking in the sedentary or prolonged
running in the athlete may induce fatigue tears of the plantar fascia and avulsion fracture
may cause pain at the medial calcaneal tuberosity. The same area is involved in spondy-
loarthropathy (ankylosing spondylitis, psoriatic arthritis and reactive arthritis) as plantar

Table 7.1 Diagnosis of plantar fasciitis2

Chronic inferior heel pain on weight bearing: throbbing, searing, piercing in character
Pain worst with the first steps in the morning or after rest
Pain reduces after mobilization, to recur with continued activity
Walking barefoot, on toes or up stairs exacerbates pain
Tenderness around medial calcaneal tuberosity at the plantar aponeurosis
Bilateral plantar fasciitis is highly suggestive of spondyloarthropathy

Atlas Medical Publishing Ltd


01-PS Rheumatology-cpp:01-PS Rheumatology-ppp.QXD 18/3/08 14:28 Page 34

34 01 General Rheumatology and Soft Tissue Rheumatism

Table 7.2 Common causes of heel pain2,3

Posterior calcaneal pain


Superficial bursitis
Achilles tendonitis
Retrocalcaneal bursitis
Calcaneal epiphysitis (Severs disease) in adolescents

Plantar calcaneal pain


Central: fat pad atrophy (primary, or secondary to corticosteroid infiltration), obesity
Medial: compression neuropathy of nerve to abductor digiti minimi and medial calcaneal branch of the
posterior tibial nerve; posterior tibialis tendonitis
Medial central: plantar fasciitis, avulsion fractures, fascial microtears
Lateral: calcaneal stress fracture, calcaneal cysts

fascia enthesitis. Insertional plantar fascia pain is typically increased by passive dorsiflex-
ion of the big toe and is located centromedially at the medial calcaneal tuberosity. Heel
pain may be caused by an entrapment neuropathy of the calcaneal branches of the
posterior tibial nerve or the first branch of the lateral plantar nerve. Since encroachment
occurs between the abductor hallucis fascia and the quadratus plantae muscle, maximal
tenderness is in the medial border of the heel. Diagnostic imaging does not contribute
unless another diagnosis is strongly suspected, such as calcaneal stress fracture. The latter
may be diagnosed by plain X-ray or by isotope bone scanning for confirmation.
Radiography may show calcifications in the soft tissues around the heel, or osteophytes
on the anterior calcaneus (i.e. heel spurs). Fifty per cent of patients with plantar fasciitis
and 20% of persons without plantar fasciitis have heel spurs.
Plantar fasciitis is five times more likely to occur in obese individuals. Although the
link between obesity and plantar heel pain is poorly understood, research to date has
focused on the impact of adiposity of the subcalcaneal fat pad and the function of the
medial longitudinal arch.4 The plantar fascia is the primary structure stabilizing the
medial longitudinal arch of the foot. Both abnormal arch structure and movement have
been implicated in the development of plantar fasciitis. In particular, pes planus or

Table 7.3 Therapeutic recommendations2

Clinical recommendation Evidence rating

Off-the-shelf (non-magnetic) insoles B


Custom-made insoles are not more effective than fabricated insoles B
Plantar fascia stretching more effective than calf stretching and should be recommended B
to all patients
Corticosteroid iontophoresis should be considered for short-term relief if initial therapy fails B
Custom-made night splints B
Extracorporeal shock therapy is not effective B
Walking casts in those who have failed conservative therapy C
Open or endoscopic surgery for those who have failed all conservative measures B
B, inconsistent or limited quality evidence; C, consensus, usual practice or expert opinion.
01-PS Rheumatology-cpp:01-PS Rheumatology-ppp.QXD 18/3/08 14:28 Page 35

07 Plantar fasciitis 35

biomechanics that result in a lower longitudinal arch (i.e. foot pronation) increase ten-
sion within the plantar fascia and thereby increase the risk of fascial injury. The plantar
fat pad is a specially organized and richly innervated adipose tissue that provides cush-
ioning to the underlying foot structures of the heel, dampening impulses associated with
heel strike. The heel pad is particularly receptive to detecting vibration, suggesting a role
for detection of gait-induced shock waves. Consequently, changes leading to increased
stiffness and reduced compressibility of the subcalcaneal fat pad have been linked to the
development of plantar fasciitis, presumably by lowering the attenuation of impulse aris-
ing from heel strike.
Most patients with plantar fasciitis eventually improve, albeit slowly; 80% of patients
treated conservatively are in complete remission at four years. A variety of therapies are
used in the treatment of plantar fasciitis and key recommendations, including exercises
(Figure 7.1), are summarized in Table 7.3. Limited evidence supports the use of cortico-
steroid injections; the benefit is short-lived and may be associated with adverse effects
such as fascial rupture. Corticosteroid-induced atrophy of the subcalcaneal fat pad may
also exacerbate the underlying biomechanical predisposition.

Recent Developments
1 Foot orthoses are commonly used for plantar fasciitis but, if custom-made, require a
period of weeks between initial consultation and use. Off-the-shelf orthoses are
quicker and cheaper, but cost and availability may still be a barrier to use. Short-term
treatments, such as supportive taping, are frequently used to alleviate symptoms
initially. Low-Dye taping is one of the most frequently applied methods, and
improves symptoms by reducing strain in the plantar fascia during standing and
walking. Utilizing taping around both the midfoot and the level of the Achilles
insertion into the calcaneum, comparison was made between individuals who
received taping plus sham ultrasound and individuals receiving sham ultrasound
only.5 The primary outcome was first-step pain one week after tape application, and
there was a statistically significant though clinically small reduction in pain with
taping. The 12.3 mm reduction in a visual analogue scale for pain was greater than
the 910 mm difference thought to be clinically meaningful.
2 The sensory nerve supplying the skin of the heel and the medial side of the sole is the
medial calcaneal branch of the posterior tibial nerve; researchers have suggested that
entrapment of this nerve might present as an important factor causing heel pain.
Nerve conduction tests have confirmed latency in the medial calcaneal nerve in
subjects with plantar fasciitis, but these subjects have normal conduction in their
sural, medial and lateral plantar nerves and posterior tibial nerves.6

Conclusion
Plantar fasciitis, if not treated, tends to follow a chronic course. Being overweight, the
presence of bilateral symptoms and duration greater than six months are associated with
poorer outcome. Treatment should be directed along biomechanical lines weight
01-PS Rheumatology-cpp:01-PS Rheumatology-ppp.QXD 18/3/08 14:28 Page 36

36 01 General Rheumatology and Soft Tissue Rheumatism

Figure 7.1 Recommended exercises for plantar fasciitis. (1) Before stepping down, especially after
sleeping or resting, stretch the arch of the foot by stretching your legs out in front of you (do not bend the
knee). Place a towel around the ball of the foot. Slowly pull on the ends of the towel, pulling the toes and ball
of the foot back as far as is comfortable. Hold the foot in this position for ten seconds. Repeat at least ten
times. You should feel a pull on the bottom of the foot, especially in the arch. This stretches the plantar
fascia and reduces its pull on the heel. (2) Stand about two to three feet from a wall. Lean forward with your
hands against the wall. With the painful foot behind, place the other foot forward. Press against the wall,
shifting weight over the front foot, while straightening the back leg. Keep the heel of the back foot on the
floor and feel the stretch in the heel, Achilles tendon and calf. Hold this position for ten seconds. Repeat at
least ten times, and try to do this three times a day.
01-PS Rheumatology-cpp:01-PS Rheumatology-ppp.QXD 18/3/08 14:28 Page 37

07 Plantar fasciitis 37

reduction, orthoses and plantar stretching. Since there is limited evidence regarding the
value of treatments, a reasonable approach is to start with patient-directed, low-risk,
minimal-cost interventions, such as regularly stretching the calf muscles and the plantar
fascia, avoiding flat shoes and walking barefoot, using over-the-counter arch supports
and heel cushions, and limiting extended physical activities. A trial of non-steroidal anti-
inflammatory drugs is reasonable. More costly treatments, such as custom-made
orthotics, night splints and immobilization with casts, may be options when the condi-
tion does not improve, although the value of these treatments is uncertain.

Further Reading
1 Wearing SC, Smeathers JE, Urry SR, Hennig EM, Hills AP. The pathomechanics of plantar
fasciitis. Sports Med 2006; 36: 585611.
2 Cole C, Seto C, Gazewood J. Plantar fasciitis: evidence-based review of diagnosis and therapy.
Am Fam Physician 2005; 72: 223742.
3 Canoso JJ. Foot pain. In: Rheumatology in Primary Care. WB Saunders, Philadelphia, 1997;
Chapter 29.
4 Wearing SC, Hennig EM, Byrne NM, Steele JR, Hills AP. Musculoskeletal disorders associated
with obesity: a biomechanical perspective. Obes Rev 2006; 7: 23950.
5 Radford JA, Landorf KB, Buchbinder R, Cook C. Effectiveness of low-Dye taping for the
short-term treatment of plantar heel pain: a randomised trial. BMC Musculoskelet Disord 2006;
7: 64.
6 Chang CW, Wang YC, Hou WH, Lee XX, Chang KF. Medial calcaneal neuropathy is
associated with plantar fasciitis. Clin Neurophysiol 2007; 118: 11923.
02-PS Rheumatology-cpp:02-PS Rheumatology-ppp.QXD 18/3/08 14:29 Page 39

S E C T I O N T W O 02
Osteoarthritis
08 Causes and prevention
09 Non-pharmacological treatment
10 Drug treatment
11 NSAIDs gastric side effects and protection
12 NSAIDs cardiac complications
13 Joint replacement surgery

P R O B L E M

08 Causes and Prevention

Case History
John is a 48-year-old accountant who is overweight (body mass index 32 kg/m2). His
general health has been good, but he has become concerned recently about twinges of
pain in his knees. He was a keen road runner in his teens and 20s, but no longer takes
regular exercise. He smokes and takes no regular medications. Both of his parents
suffered from osteoarthritis, his father requiring knee replacements in his early 60s.
What is the current thinking about the inheritance of osteoarthritis?
Which environmental factors play a part in its progression?
Can the disease be prevented in a predisposed individual?

Background
Osteoarthritis (OA) is an increasing public health problem because of the rising preva-
lence of obesity and because of the increasing number of older people in the population.1
Currently, in the United States, 350 000 hip and knee replacements are performed each
year. OA is markedly age related, affecting around 12% of those aged 55 years, 50% of
Atlas Medical Publishing Ltd
02-PS Rheumatology-cpp:02-PS Rheumatology-ppp.QXD 18/3/08 14:29 Page 40

40 02 Osteoarthritis

those aged 65 years and as many as 80% of those aged 75 years and over. Prevalence esti-
mates have largely come from population-based X-ray surveys. However, there is a vari-
able relationship between radiological features and symptoms. In clinical practice, OA is
only diagnosed when clinical symptoms (pain and stiffness) coexist with evidence of
structural damage to the joint. With age, the higher prevalence of OA in females becomes
even more exaggerated. The hips, knees and hands are the most commonly affected
joints, although almost any joint can be affected. Joint failure arises from a combination
of systemic and mechanical factors, including weakness of supporting muscles and liga-
ments, growth of new bone, erosion of articular cartilage and loss of joint space. A simple
grading system is presented in Box 8.1. OA may be primary or secondary; secondary
causes are summarized in Box 8.2.

Box 8.1 Radiological severity scoring of OA


Grade
0 None No features
1 Possible Minimal osteophytes
2 Minimal Osteophytes, normal joint space
3 Moderate Diminution of joint space
4 Severe Decreased joint space, bony sclerosis

Box 8.2 Secondary causes of OA


Anatomical
Leg length inequality, joint malalignment
Slipped femoral epiphyses, epiphyseal dysplasias
Perthe disease
Congenital dislocation of the hip
Hypermobility syndromes

Traumatic
Occupational arthropathies
Fracture through or near a joint
Joint surgery
Major external trauma

Inflammatory
Inflammatory arthropathies
Septic arthritis

Metabolic
Crystal arthropathies
Haemochromatosis
Acromegaly
Ochronosis, chondrodysplasias
02-PS Rheumatology-cpp:02-PS Rheumatology-ppp.QXD 18/3/08 14:29 Page 41

08 Causes and prevention 41

The traditional view is that OA is a simple mechanical condition, in which repeated or


sustained stresses on the joint lead to articular cartilage damage and loss. The notion that
the cartilage is the tissue primarily affected in patients prone to OA has been challenged.
In fact, cartilage is a relatively inert tissue with no direct blood supply, nerve input or
capacity to become inflamed. Severe OA can occur in rare genetic conditions ochrono-
sis and chondrodysplasias but generally in OA, predisposed individuals do not appear
to have an alteration in their cartilage structure or composition. In the early stages,
changes in modelling of adjacent bone and in the collateral ligaments of the joint may
precede the development of changes in cartilage and symptoms. Aging and post-
menopausal changes in bone turnover are almost certainly important in the disease.
From post-mortem and other studies, it seems clear that changes in the joint capsule and
collateral ligaments can also precede the classic structural findings of OA. These changes
may affect joint stability and alignment leading to increased mechanical forces on bone
and cartilage. Furthermore, weakening of periarticular muscles further compromises
joint stability and predisposes to malalignment. This is a particular factor in knee OA
where quadriceps wasting and weakness leads to decreased joint stability. Loss of coordin-
ation, vision and proprioception with age contribute to the individual responding less
well to unexpected stresses on the joint and thus increasing the impact of minor, poten-
tially traumatic events.
A genetic predisposition to OA was first suspected in the 1940s when prevalence of
Heberdens nodes was noted to be three times higher in first-degree relatives of OA
patients. Several segments of the human genome have been identified as susceptibility
loci, and at least twelve candidate genes have been studied. Genetic factors account for
around 50% of the variance in susceptibility to OA. Twin studies suggest a concordance
rate of around 0.65 for monozygotic twins and 0.4 for dizygotic twins.2 In a United
Kingdom study,2 first-degree relatives of patients who had total hip replacement carried a
relative risk of 1.9 for also requiring the procedure. The comparable figure for total knee
replacement was 4.8. From genome-wide scans, susceptibility loci have been hypothe-
sized at chromosomes 2q, 4q, 6p, 11q and 16p. The only candidate genes identified so far
in these regions of the genome are the collagen genes COL29A1 and COL11A2. Other
genes are COL2A1, which codes for collagen II (the major protein of cartilage), CRTL1
(coding for a cartilage link protein) and CRTM (coding for a cartilage matrix protein).
Polymorphisms of the genes for the vitamin D receptor (VDR) and the oestrogen-a
receptor have also been related to risk both are major determinants of bone density.
The COL2A1 and VDR genes are located closely together on chromosome 12q. Cartilage-
remodelling protein gene expression may also play a role; expression of the gene for
matrix metalloproteinase-3 (MMP-3), which degrades collagen, and the gene for a disin-
tegrin and metalloproteinase with thrombospondin motif-5 (ADAMTS-5) both increase
cartilage breakdown in vitro. Levels of expression of these enzymes may be both geneti-
cally determined and increase in response to mechanical stress of cartilage.
Obesity is by far the best-studied environmental factor, particularly with respect to
increasing risk of knee arthritis. Several factors may underlie the association between obe-
sity and OA. Mechanical stresses predispose to joint damage, although obesity also
increases risk of OA in non-weight-bearing joints, and alterations in gait and predisposi-
tion to injury contribute to joint alignment. Hormonal changes are factors, including low
local levels of androgens because of increased aromatase activity. Hyperglycaemia appears
to be a risk factor partly because of its association with obesity, but also because of other
02-PS Rheumatology-cpp:02-PS Rheumatology-ppp.QXD 18/3/08 14:29 Page 42

42 02 Osteoarthritis

metabolic factors including the inhibitory effect of advanced glycation end products on
proteoglycan synthesis. The low-grade inflammation found in obese subjects and other
pro-inflammatory states also predispose. Glucosamine blocks the effect of cytokines on
nitric oxide synthesis and also increases glycosaminoglycan production. Other nutritional
factors have been identified: low vitamin C intake and vitamin E as an antioxidant and
determinant of collagen turnover have been implicated, as has low vitamin D status.
Numerous studies have confirmed the increased risk of OA with obesity, particularly
when obesity develops at a younger age.35 In the study by Dawson et al.,3 women who
smoked heavily were also at increased risk. Even a modest increase in weight within the
normal healthy range increases risk of developing OA.4 The latter observation was also
confirmed in a national Swedish study,5 although a decreased risk of OA of the hip was
reported in smokers. The effect of smoking on OA is therefore uncertain but appears
unlikely to be marked.
Occupational risk factors are well documented,6 with those who undertake manual
labour being at increased risk. Repeated stress on the knee, particularly with the knee

Genetic Strong family history


Joint replacement in first-degree relative
Premature onset
Secondary OA

Anatomical Traumatic Metabolic Inflammatory

Primary OA

Overweight or obese Lose weight

Mechanical loading Avoid repeated strain


( occupational) Support knee

Female gender

Age, post-menopausal

Others Low vitamin C, D or E; neuromuscular weakness; low BMD

Figure 8.1 Risk factors for OA.


02-PS Rheumatology-cpp:02-PS Rheumatology-ppp.QXD 18/3/08 14:29 Page 43

08 Causes and prevention 43

bent, increases mechanical stress on the patello-femoral compartment of the joint.


Moderate sporting activity does not seem to pose a risk. Elite athletes are at increased
risk. Having high bone mineral density (BMD) may predispose to development of osteo-
phytes, but high bone turnover as in post-menopausal osteoporosis leads to micro-
architectural deterioration around the joint. The bone in this area consequently becomes
stiff and less able to absorb shock, placing the patient at higher risk of OA. Hormone
replacement therapy (HRT) has been said to be protective. This may relate to its protect-
ive effect on BMD, but it should be noted that use of HRT is associated with a variety of
other lifestyle factors. Risk factors for OA are summarized in Figure 8.1. The risk factors
for progression are similar and appear to operate more markedly on the knee. These
include obesity, injury, crystal arthropathy, neuromuscular dysfunction and poor phys-
ical alignment of the joint.

Recent Developments
1 Levels of adiponectin, a hormone product of fat cells, are decreased in obesity. Low
levels increase the risk of developing diabetes and vascular disease. High levels of
adiponectin have recently been reported in synovial fluid from patients with OA.7
The hormone is thought to have a protective role by upregulating the tissue inhibitor
of metalloproteinase-2 (TIMP-2) and by inhibiting interleukin(IL)-1induced
increases in matrix metalloproteinase-13 (MMP-13).
2 The rate of development of OA in women increases markedly after the menopause,
suggesting that oestrogen may have a protective effect. In a large study,8 low plasma
oestrogen and low urinary levels of its metabolites were associated with increased
risk of OA. Measures to preserve exposure to oestrogen may protect the joints.
3 Biomarkers of disease activity could prove to be a considerable asset.9 Candidates
include the urinary C-terminal peptide of collagen type II, serum hyaluronan and
cartilage oligomeric matrix protein. Measurement of these markers could be
combined with other clinical measures (e.g. C-reactive protein), symptom scores
and genetic/molecular markers of risk to help focus programmes of treatment on
patients who are either at high risk or have rapidly progressive disease.

Conclusion
OA is a disorder with a strong genetic component. Patients with a first-degree relative
who has a joint replacement, Heberdens nodes or who has developed premature OA
should be aware that they are at risk of developing OA. The major, modifiable risk factor
is being overweight or obese. There is evidence that weight loss in those who are over-
weight improves OA symptoms and retards progression. It is important to identify fac-
tors that place increased strain on the joints, including occupational risks, repetitive
injuries and joint malalignment. Simple measures may decrease the risk from physical
factors. OA is one of the major degenerative diseases of aging. While we can delay the
impact of the disorder on individual patients, we are not at a stage where OA can be rou-
tinely prevented.
02-PS Rheumatology-cpp:02-PS Rheumatology-ppp.QXD 18/3/08 14:29 Page 44

44 02 Osteoarthritis

Further Reading
1 Arden N, Nevitt MC. Osteoarthritis: epidemiology. Best Pract Res Clin Rheumatol 2006; 20:
325.
2 Loughlin J. Genetic epidemiology of primary osteoarthritis. Curr Opin Rheumatol 2001; 13:
11116.
3 Dawson J, Juszczak E, Thorogood M, Marks SA, Dodd C, Fitzpatrick R. An investigation of
risk factors for symptomatic osteoarthritis of the knee in women using a life course approach.
J Epidemiol Community Health 2003; 57: 82330.
4 Holmberg S, Thelin A, Thelin N. Knee osteoarthritis and body mass index: a population-based
case-control study. Scand J Rheumatol 2005; 34: 5964.
5 Jrvholm B, Lewold S, Malchau H, Vingrd E. Age, bodyweight, smoking habits and the risk of
severe osteoarthritis in the hip and knee in men. Eur J Epidemiol 2005; 20: 53742.
6 Rossignol M. Primary osteoarthritis and occupation in the Quebec national health and social
survey. Occup Environ Med 2004; 61: 72935.
7 Chen TH, Chen L, Hsieh MS, Chang CP, Chou DT, Tsai SH. Evidence for a protective role for
adiponectin in osteoarthritis. Biochim Biophys Acta 2006; 1762: 71118.
8 Sowers MR, McConnell D, Jannausch M, Buyuktur AG, Hochberg M, Jamadar DA. Estradiol
and its metabolites and their association with knee osteoarthritis. Arthritis Rheum 2006; 54:
24817.
9 Kraus VB. Do biochemical markers have a role in osteoarthritis diagnosis and treatment? Best
Pract Res Clin Rheumatol 2006; 20: 6980.
02-PS Rheumatology-cpp:02-PS Rheumatology-ppp.QXD 18/3/08 14:29 Page 45

09 Non-pharmacological treatment 45

P R O B L E M

09 Non-Pharmacological Treatment

Case History
AP is a 58-year-old woman whose symptoms of osteoarthritis (OA) are beginning to limit
her activity. She complains that she feels stiff. She takes paracetamol regularly but does
not want to take other drugs. One of her friends recently suffered gastrointestinal
haemorrhage while taking a non-steroidal anti-inflammatory drug (NSAID).
What non-pharmacological interventions are available?
Are natural treatments effective at decreasing symptoms and protecting joints?
Is cartilage protection a valid treatment goal?

Background
Effective analgesia and use of anti-inflammatory drugs can transform the lives of patients
with OA. However, pharmaceuticals are not the sole mode of treatment for OA. Non-
pharmacological approaches are often underused. Also many patients resort to natural
products. Health practitioners need to understand these products and whether they work.
Drug treatment should only be considered when lifestyle and non-pharmacological
management has failed.

Body weight
Being overweight or obese increases the risk of developing OA and accelerates progres-
sion of the disease.1 In the Framingham study, an increase in body mass index (BMI) of
only 2 kg/m2 was associated with a 50% increase in the risk of developing OA in women.2
Studies documenting the benefit of weight loss have generally been short term, lasting no
more than two years. Maintenance of a normal or near-normal body weight is also asso-
ciated with other health benefits including prevention of diabetes and cardiovascular dis-
ease. While the cornerstone of weight management is diet and exercise, the role of drugs
(including orlistat, sibutramine and rimonabant) should be considered.
Weight reduction reduces mechanical stresses on the joints of the axial skeleton. Being
overweight also increases the risk of joint malalignment. Some of the benefits of a weight-
management programme may relate directly to the effects of exercise and to improved
metabolic factors and lower levels of chronic inflammation. It is important that treat-
ment goals are realistic and that the patient receives adequate support. The practitioner
should always be alert to the possibility of psychological problems, including eating
disorders.

Atlas Medical Publishing Ltd


02-PS Rheumatology-cpp:02-PS Rheumatology-ppp.QXD 18/3/08 14:29 Page 46

46 02 Osteoarthritis

Exercise
Physical exercise contributes to weight loss or at least to weight maintenance.
Independently of this, it increases overall function and well-being, and decreases risk of
falling. As well as protection from OA,1 exercise also lowers risk of osteoporosis, diabetes
and cardiovascular disease. Exercise is of particular benefit in knee protection. Isokinetic
exercises that utilize either flexion against resistance or extension against resistance are
effective. Both aerobic exercise (low impact) and specific strengthening exercises for peri-
articular muscles have been found to be effective. Whichever form of exercise is used,
there is benefit from the input of a qualified therapist; undue pain or discomfort should
be avoided and the approach should be based on the patients ability and motivation,
with only gradual increase in the effort required.

Glucosamine and chondroitin


Glucosamine, available as sulphate or hydrochloride, is a hexosamine that is a natural
constituent of keratin sulphate, the glycosaminoglycan of hyaline cartilage. It is fre-
quently administered with chondroitin sulphate, another naturally occurring constituent
of the cartilage matrix (being a constituent of aggrecan, which is the major proteoglycan
of cartilage). Although studies have reported effects comparable with NSAIDs, results
have been extremely variable. This may be partly because of differences between glu-
cosamine hydrochloride and glucosamine sulphate. Studies using the former preparation
have tended to show negative results whereas use of the latter compound tends to be
more effective. This has been attributed by some to the fact that it is a source of sulphate,
rather than because of the glucosamine per se. Glucosamine sulphate is given as a single,
daily oral dose of 1500 mg and is well absorbed. Some of its effect is mediated through
inhibition of cytokine-induced nitric oxide production and through inhibition of prote-
olytic enzymes.
A Cochrane review in 2000 included 16 studies of patients (total 2029) who used glu-
cosamine sulphate 1500 mg each day for six weeks.3 Sixty per cent of patients reported
improved pain and 33% reported improved function. A more recent Cochrane review3
included 20 trials with 2570 patients and, overall, did not confirm improved pain and
function with glucosamine. However, ten trials using one preparation (Rotta
Pharmaceuticals) did show improvements. A recent study4 randomized 1583 patients
with knee arthritis to glucosamine hydrochloride 1500 mg/day, chondroitin sulphate
1200 mg/day, glucosamine and chondroitin together, 200 mg celecoxib or placebo. Up to
4 g/day of paracetamol was allowed as rescue analgesia. Neither glucosamine, chon-
droitin, nor the combination was successful overall. The combination of the two was
somewhat effective in patients with moderate to severe knee pain. It is noteworthy that
glucosamine hydrochloride was used in this study.

Other nutritional components and supplements


There is evidence that low vitamin C levels or status is related to the risk of developing
OA and to the progression of the disease. The two major essential fatty acids are linoleic
acid (18:2 n-6) and a-linolenic acid (18:3 n-3). These two fatty acids form eicosanoids
through the action of cyclooxygenase-1 (COX-1), cyclooxygenase-2 (COX-2) and
5-lipoxygenase. When cell membranes comprise predominantly n-6 fatty acids (leading
to prostaglandin E2 and leukotriene B4 production) a pro-inflammatory state exists,
02-PS Rheumatology-cpp:02-PS Rheumatology-ppp.QXD 18/3/08 14:29 Page 47

09 Non-pharmacological treatment 47

while the incorporation of predominantly n-3 fatty acids results in an anti-inflammatory


state. There is considerable evidence for the benefit of n-3 fatty acid supplementation in
management of OA.
Patients with OA frequently use complementary and alternative medicines up to
40% of patients were using such products in a recent survey.5 Fish oils, garlic extracts and
celery extract are among the most commonly used products. This high level of usage
reflects, to an extent, the fact that patients get incomplete relief from medical approaches,
and also that many patients do not want to be reliant on pharmaceuticals.

Viscosupplementation
Hyaluronic acid is a high-molecular-weight polysaccharide that contributes to the phys-
ical properties of synovial fluid and cartilage. Synovial fluid supplementation, or visco-
supplementation, was developed in the 1980s as a treatment proposed to relieve pain in
the short term and to restore the viscoelastic properties associated with hyaluronan in
synovial joints. The products used are derivatives of hyaluronan and hylan and are given
by intra-articular injection sometimes weekly for the first three weeks, but then at inter-
vals of at least four weeks for up to six months. A recent Cochrane review6 considered
76 controlled trials comparing viscosupplementation products, or examining their use
with intra-articular steroids or NSAIDs. The conclusion was that viscosupplementation
treatment was effective, especially at the 513-week period. Firm conclusions about the
relative value of different products could not be made.
Two recent studies have confirmed the potential benefits of viscosupplementation.
Sun et al.7 administered hyaluronic acid to 75 patients with ankle OA. Patients reported
increased pain relief and improved function. The effects were apparent both during
treatment and for up to six months afterwards. In another study,8 intra-articular
hyaluronic acid (three injections) was compared with an exercise regimen for patients
with knee OA. Both treatments were effective and there was no statistical difference
between the two.

Other measures
Use of a walking stick (cane) may help. The stick should be used on the opposite side to
the worst affected joints, with the hand holding the stick at the level of the greater
trochanter, and the foot of the affected leg and the stick striking the ground more or less
simultaneously. The importance of the joint being as well aligned as possible cannot be
overemphasized. Shoe inserts, braces or taping of the affected joint may minimize the
effect of malalignment including valgus (knees closer together than ankles) and varus
(ankles closer together than knees) deformities. Local application of capsaicin cream
(0.025%0.075%), which acts as a counter-irritant, can be useful. Footwear should be
chosen according to the following advice:
Thick, soft soles with maximum shock absorption
Minimum heel raise
Broad forefoot, allowing the toes to splay (thus improving balance and decreasing
the likelihood of falls and injuries)
Soft uppers
Adequate depth of the shoe
02-PS Rheumatology-cpp:02-PS Rheumatology-ppp.QXD 18/3/08 14:29 Page 48

48 02 Osteoarthritis

Recent Developments
1 Glucosamine has recently been studied in chondral and synovial cultures in vitro.9 In
this study, glucosamine suppressed the secretion of matrix metalloproteinases
(MMP-2 and MMP-9), key enzymes involved in cartilage degradation. Urokinase
plasminogen activator has also been implicated in the development of OA, particularly
where there are joint effusions. Again, levels of this enzyme were suppressed by
glucosamine.
2 The hexosamine biosynthetic pathway may be a major cellular nutrient sensor.10 The
first step in this pathway is the formation of glucosamine-6-phosphate from
fructose-6-phosphate. The pathway leads to formation of uridine diphosphate
(UDP)-N-acetyl-glucosamine, which becomes incorporated into the side chains of

Weight loss
(if overweight/obese)

Weight maintenance

Exercise Diet
Aerobic (general fitness) Suitable calorie intake
Resistance (muscle bulk and function) Reasonable protein content
Strengthening (periarticular muscles) ? Vitamin C
? Essential FA (n-3)

General measures
Accident/injury prevention
Sensible footwear
Taping or bracing affected joint
Orthotics (joint alignment)
Walking aids (stick etc.)

Capsaicin for pain relief

Glucosamine sulphate chondroitin sulphate

Viscosupplementation
(hyaluronic acid)

Figure 9.1 Non-pharmacological management of OA. FA, fatty acid.


02-PS Rheumatology-cpp:02-PS Rheumatology-ppp.QXD 18/3/08 14:29 Page 49

09 Non-pharmacological treatment 49

proteins and lipids. There is concern that increased flux through this pathway
(including by glucosamine supplementation) may worsen insulin resistance and
increase susceptibility to diabetes complications.
3 Some of the properties of mature cartilage depend on the charge of sulphate moieties
of chondroitin sulphate. Dietary sulphate largely comes from high-protein foods,
and low sulphate intake or low serum levels of sulphate have been associated with
increased risk of OA. Blinn et al.11 have shown that prolonging the overnight fast or
ingestion of calories without protein leads to lower serum levels of sulphate.

Conclusion
Non-pharmacological management should be considered at the outset before drug treat-
ments are initiated (Figure 9.1). The most important factors are weight management,
exercises to increase general fitness and to increase strength around affected joints,
measures to decrease stresses on joints, and improvement of the alignment of joints if
need be. Current evidence favours the use of glucosamine sulphate (usually with
chondroitin sulphate) and viscosupplementation with hyaluronic acid products. A
number of nutritional factors, including antioxidant vitamins and essential fatty acids,
are almost certainly important, but routine supplementation is not currently recommended.

Further Reading
1 Roddy E, Doherty M. Changing life-styles and osteoarthritis: what is the evidence? Best Pract
Res Clin Rheumatol 2006; 20: 8197.
2 Felson DT, Zhang Y, Hannan MT et al. Risk factors for incident radiographic knee
osteoarthritis in the elderly: the Framingham Study. Arthritis Rheum 1997; 40: 72833.
3 Towheed TE, Maxwell L, Anastassiades TP et al. Glucosamine therapy for treating osteoarthri-
tis. Cochrane Database Syst Rev 2005; CD002946.
4 Clegg DO, Reda DJ, Harris CL et al. Glucosamine, chondroitin sulphate, and the two in com-
bination for painful knee osteoarthritis. New Engl J Med 2006; 354: 795808.
5 Zochling J, March LM, Lapsley H, Cross M, Tribe K, Brooks P. Use of complementary
medicines for osteoarthritis a prospective study. Ann Rheum Dis 2004; 63: 54954.
6 Bellamy N, Campbell J, Robinson V, Gee T, Bourne R, Wells G. Viscosupplementation for the
treatment of osteoarthritis of the knee. Cochrane Database Syst Rev 2006; CD005321.
7 Sun SF, Chou YJ, Hsu CW et al. Efficacy of intra-articular hyaluronic acid in patients with
osteoarthritis of the ankle: a prospective study. Osteoarthritis Cartilage 2006; 14: 86774.
8 Karatosun V, Unver B, Gocen Z, Sen A, Gunal I. Intra-articular hyaluranic acid compared
with progressive knee exercises in osteoarthritis of the knee: a prospective randomized trial
with long-term follow-up. Rheumatol Int 2006; 26: 27784.
9 Chu SC, Yang SF, Lue KH et al. Glucosamine sulfate suppresses the expressions of urokinase
plasminogen activator and inhibitor and gelatinases during the early stage of osteoarthritis.
Clin Chim Acta 2006; 372 : 16772.
02-PS Rheumatology-cpp:02-PS Rheumatology-ppp.QXD 18/3/08 14:29 Page 50

50 02 Osteoarthritis

10 Buse MG. Hexosamines, insulin resistance, and the complications of diabetes: current status.
Am J Physiol Endocrinol Metab 2006; 290: E18.
11 Blinn CM, Biggee BA, McAlindon TE, Nuite M, Sibert JE. Sulphate and osteoarthritis:
decrease of serum sulphate levels in an additional 3-h fast and a 3-h glucose tolerance test after
an overnight fast. Ann Rheum Dis 2006; 65: 12235.

P R O B L E M

10 Drug Treatment

Case History
Mrs GT is 67 years old. Since she developed osteoarthritis (OA) five years ago she has tried
hard to control symptoms with diet, exercise and nutritional supplements. She is
increasingly frequently requiring analgesics. She wants to discuss drug treatments for her
OA. Her general health is good and she has no history of dyspepsia.
Do non-steroidal anti-inflammatory drugs (NSAIDs) have an advantage over simple
analgesia?
How should drug therapy be approached?
What are the prospects for disease-modifying drugs?

Background
The first line of treatment for OA should always be non-pharmacological. When drug
therapy is required, paracetamol in doses of up to 4 g/day is the usual first-line treatment.
Zhang et al.1 conducted a meta-analysis of published controlled trials of simple analgesia
for OA. The pain response to paracetamol was assessed along with the Western Ontario
and McMaster Universities (WOMAC) OA index score, a score that focuses largely on
functional symptoms. Combining data from the four trials identified, the pain response
was classified as weak, while there was minimal functional improvement. Further trial
data confirm that the effect of paracetamol is modest, while there is clear evidence from
numerous trials that NSAIDs have at least a modest benefit in alleviating pain and

Atlas Medical Publishing Ltd


02-PS Rheumatology-cpp:02-PS Rheumatology-ppp.QXD 18/3/08 14:29 Page 51

10 Drug treatment 51

improving function.2 Paracetamol should be used only with caution in patients with liver
disorders, but it seldom (except in overdose) causes hepatic dysfunction. Compared with
aspirin and NSAIDs, paracetamol is relatively safe in patients with renal impairment.
If paracetamol is of limited efficacy, do more potent analgesics have greater effect? The
best-studied agent is tramadol. In addition to its opioid action, it is a weak central
inhibitor of noradrenaline and serotonin uptake. There is no question from available tri-
als that tramadol is moderately effective at decreasing pain and improving function (by
WOMAC score) in patients with OA.3 A total daily dose of 200300 mg is usual (given in
divided doses), and the drug may be combined with paracetamol either to increase effect
or to minimize the dose of each agent. Side effects include dizziness, somnolence,
headache and constipation, but the drug is generally very safe, as well as being cheap.
There is a range of alternatives including dextropropoxyphene and pentazocine. More
potent opiates do not offer any advantage in terms of symptom relief and are more likely
to cause side effects including dependence. The efficacy of tramadol is comparable to that
of lower doses of NSAIDs or cyclooxygenase-2 (COX-2) inhibitors.
Topical NSAIDs may be as effective as systemic administration.2 Since only up to 5%
of the topical dose is absorbed systemically, risk of severe adverse drug reactions is sub-
stantially lower than with oral dosing. Side effects are usually limited to the site of admin-
istration localized itching, burning or skin rash. The only concern with topical NSAIDs
from trials is that their benefit may be relatively short-lived.
OA is not primarily an inflammatory disorder. However, inflammation does con-
tribute to symptoms and to disease progression. For knee OA in particular, intra-
articular corticosteroid injection can be useful. The agents used are prednisolone,
methylprednisolone, betamethasone and triamcinolone. The treatment leads to short-
term symptom relief but does not modify the long-term progression of the disease. From
assessment of nearly 30 trials,4 the treatment is clearly superior to placebo for pain relief.
It is comparable in effect to other pharmacological treatments for pain, but varying
effects on functional improvement have been reported. There is a risk of introducing
infection into the joint, and the treatment is not useful where there is substantial effusion
in the joint.
The focus of treatment in OA has been on symptom relief, but there is increasing
interest in development of drugs that may modify the progress of the disease, and in
approaches to reconstructing the cartilage matrix.5,6 Diacerein is an antagonist to inter-
leukin (IL)-1b that has already been demonstrated to have some efficacy in controlled
clinical trials. Other anti-inflammatory approaches include antagonists to tumour necro-
sis factor (TNF)-a. It appears most likely that local administration of disease-modifying
agents may be preferred to avoid the systemic effects of immunosuppression. Local
inhibitors of nitric oxide synthase (NOS) have the potential to decrease activation of
cytokines and the COX-2 enzyme system. Colchicine has been proposed as a means of
decreasing local inflammation within the joint. Both bisphosphonates and oestrogen
treatment (in women) may impact favourably on the increased bone turnover that
occurs around affected joints. Decreasing the turnover of the cartilaginous matrix is one
of the goals of therapy. Twenty-eight matrix metalloproteinase (MMP) enzymes have
now been identified and are key targets for disease-modifying drugs. Of these drugs, the
tetracyclines including doxycycline and minocycline are the most widely studied
agents. The joint space is a relatively easy target for locally administered gene therapy.
02-PS Rheumatology-cpp:02-PS Rheumatology-ppp.QXD 18/3/08 14:29 Page 52

52 02 Osteoarthritis

Amongst gene targets studied have been the insulin-like growth factor-1 (IGF-1) gene
and the gene for the IL-1 receptor antagonist. Experimental tissue-engineering
approaches include the use of mesenchymal stem cells, autologous cartilage plugs,
implantation of autologous chondrocytes and use of artificial cartilage matrix.
Although unlikely to be as potent as pharmaceuticals and tissue-engineering
approaches, there is considerable interest in the potential role of diets and supplements.
These approaches are cheap, safe and popular with patients. Evidence relating to their
effects and modes of action is accumulating. Various polyphenols, including the catechin
group, have been studied because of their anti-inflammatory and chondroproliferative
properties. The best-known supplement is green tea (epigallocatechin gallate), which is
widely used as a herbal remedy. There is also considerable interest in ginger extracts and
avocado/soybean non-saponifiables.

Diet
Exercise
Joint support and alignment

(Glucosamine chondroitin)

Paracetamol Local measures


Capsaicin cream
NSAIDs
Methylsalicylate
Maximum dose
4 g/day

Tramadol Low GI risk High GI risk


NSAID Add PPI
Misoprostol
COX-2
More potent opioids or
analgesia 1NSAID Intra-articular treatment
Steroids
Hyaluronan

Consider surgery

Figure 10.1 Drug treatments for OA. GI, gastrointestinal; PPI, proton pump inhibitor.
02-PS Rheumatology-cpp:02-PS Rheumatology-ppp.QXD 18/3/08 14:29 Page 53

10 Drug treatment 53

Recent Developments
1 Doxycycline has been proposed as a disease-modifying drug for OA. Amongst its
actions in the joint, the drug inhibits breakdown of type XI collagen and decreases
activity of collagenase and NOS. A 30-month trial of doxycycline in women with
radiologically proven knee OA7 showed 40% less reduction in the knee joint space in
treated patients compared with placebo. While doxycycline appeared to slow the
progression of joint disease in this study, its effects on symptom control were disap-
pointing.
2 Nuclear factor-k-beta (NF-kB) is a transcription factor that plays an important part
in modulating immunological and inflammatory responses. When the NF-kB
pathway is activated, there is increased expression of inflammatory mediators and
proteins involved in the regulation of apoptosis.8 The NF-kB pathway is inhibited by
corticosteroids and a number of other immunomodulatory agents. There is a search
for more specific agents. The pathway is involved in the pathogenesis of OA not only
through regulation of inflammation but also through its involvement in regulation
of cartilage turnover.

Conclusion
A scheme for managing drug therapy in OA is proposed in Figure 10.1. Paracetamol is
the most widely used analgesic and is often effective in alleviating pain without a major
risk of side effects. The potential benefits of local treatments and more potent analgesics
(including tramadol) are often forgotten before proceeding to NSAIDs. Progress is being
made with disease-modifying drugs on a number of fronts. The most promising
approaches are those that may downregulate the local increases in inflammation and car-
tilage turnover.

Further Reading
1 Zhang W, Jones A, Doherty M. Does paracetamol (acetaminophen) reduce the pain of
osteoarthritis? A meta-analysis of randomised controlled trials. Ann Rheum Dis 2004; 63:
9017.
2 Bannwarth B. Acetaminophen or NSAIDs for the treatment of osteoarthritis. Best Pract Res
Clin Rheumatol 2006; 20: 11729.
3 Cepeda MS, Camargo F, Zea C, Valencia L. Tramadol for osteoarthritis. Cochrane Database
Syst Rev 2006; 3: CD005522.
4 Bellamy N, Campbell J, Robinson V, Gee T, Bourne R, Wells G. Intraarticular corticosteroid
for treatment of osteoarthritis of the knee. Cochrane Database Syst Rev 2006; CD005328.
5 Fajardo M, Di Cesare PE. Disease-modifying therapies for osteoarthritis: current status. Drugs
Aging 2005; 22: 14161.
6 Baker CL, Ferguson CM. Future treatment of osteoarthritis. Orthopedics 2005; 28 (4 Suppl):
s22734.
02-PS Rheumatology-cpp:02-PS Rheumatology-ppp.QXD 18/3/08 14:29 Page 54

54 02 Osteoarthritis

7 Brandt KD, Mazzuca SA, Katz BP et al. Effects of doxycycline on progression of osteoarthritis:
results of a randomized, placebo-
controlled, double-blind trial. Arthritis Rheum 2005; 52: 201525.
8 Roman-Blas JA, Jimenez SA. NF-kb as a potential therapeutic target in osteoarthritis and
rheumatoid arthritis. Osteoarthritis Cartilage 2006; 14: 83948.

P R O B L E M

11 NSAIDs Gastric Side Effects and


Protection

Case History
Mr JS is a 58-year-old carpenter. He was diagnosed as having rheumatoid arthritis at the
age of 40 years. He is a smoker and is prone to dyspepsia. He does not currently require
disease-modifying drugs. However, he does have pain and stiffness and is not able to work
comfortably without symptomatic relief.
To what extent do non-steroidal anti-inflammatory drugs (NSAIDs) cause
gastrointestinal symptoms?
Does the presence of dyspeptic symptoms influence the choice of agent?
What strategies are available for gastric protection?

Background
The NSAIDs are an important class of drug with analgesic, antipyretic and anti-inflam-
matory activities. They are among the most common drugs prescribed. NSAIDs account
for around 3% of the total United States (US) drugs market, and it is estimated that
5%10% of the population use NSAIDs on a regular basis (Box 11.1). In the US during
2001, 70 million prescriptions were issued for NSAIDs and 30 billion doses were bought
over the counter. NSAIDs are weak acids (pKa 35) that are highly lipid soluble and read-
ily absorbed from the stomach. In the plasma they are highly protein bound (>95%,
mainly to albumin). They are metabolized and conjugated in the liver and mainly
excreted in the urine.
Atlas Medical Publishing Ltd
02-PS Rheumatology-cpp:02-PS Rheumatology-ppp.QXD 18/3/08 14:29 Page 55

11 NSAIDs gastric side effects & protection 55

Box 11.1 Uses of NSAIDs


Osteoarthritis
Rheumatoid arthritis
Inflammatory spondyloarthropathies (ankylosing spondylitis, reactive arthritis,
psoriatic arthritis)
Post-operative pain
Acute gout
Soft tissue injuries (including sports injuries)
Dysmenorrhoea
Renal colic

NSAIDs reversibly inhibit the cyclooxygenase (COX) enzyme, which catalyses the first
step in the pathway that converts arachidonic acid to key regulatory prostaglandins. The
enzyme exists in two isoforms: COX-1 is constitutively expressed in most tissues while
COX-2 is an inducible enzyme, the expression of which is upregulated at sites of inflam-
mation (Figure 11.1). The gastric side effects of NSAIDs relate to inhibition of COX-1 in
the gastric mucosa, where prostaglandins increase gastric mucous and bicarbonate secre-
tion, decrease gastric acid secretion and stimulate mucosal blood flow. COX-2 is not

Membrane phospholipids

Phospholipase A2

Arachidonic acid

COX-1 COX-2
constitutive/inducible constitutive/inducible

PGs PGs
GI cytoprotection Inflammation
Platelet aggregation Fever
Renal function Pain
(blood flow) Headache
Carcinogenesis
Kidney, stomach, uterus
CNS and spine
Endothelium

Figure 11.1 Prostaglandin generation by COX-1 and COX-2. CNS, central nervous system; GI,
gastrointestinal; PG, prostaglandin.
02-PS Rheumatology-cpp:02-PS Rheumatology-ppp.QXD 18/3/08 14:29 Page 56

56 02 Osteoarthritis

Box 11.2 Risk factors for gastric side effects


High dose of NSAID Use of multiple agents
Use of non-selective NSAID Concurrent use of aspirin, warfarin or steroids
First three months of treatment Previous ulcer history
Age >50 years Helicobacter pylori infection
Previous ulcer history General disability

normally expressed in the gastric mucosa. The beneficial, anti-inflammatory actions of


NSAIDs are mediated through inhibition of COX-2. NSAIDs vary greatly in their relative
COX-1 and COX-2 inhibitory activity. Aspirin, sulindac and piroxicam have much
greater selectivity for COX-1. The mechanism of aspirin action differs from that of other
NSAIDs in that it acts by irreversibly acetylating the active site of COX-1. Meloxicam, the
first of the selective COX-2 inhibitors, is three times more potent on COX-2 compared
with COX-1. COX-2 is not inhibited by aspirin but is downregulated by glucocorticoids.
NSAIDs are generally very safe and well tolerated. However, because of the number of
patients using them, adverse reactions are frequently encountered. By far the most com-
mon side effects relate to gastric mucosal injury (Box 11.2). Up to 20% of patients taking
NSAIDs suffer gastrointestinal symptoms including heartburn, dyspepsia, nausea,
vomiting and abdominal pain. In endoscopic studies, haemorrhagic reactions to the
presence of the tablets in the stomach are almost universal, superficial erosions occur in
50% of subjects and 20% of regular users have chronic ulceration (i.e. ulcers that pene-
trate the muscularis mucosa). There is a variable relationship between symptoms and the
changes in the mucosa visible on endoscopy.
The gastrointestinal side effects of NSAIDs are a major cause for concern.1 Serious
gastric side effects occur in less than 1% of patients. However, this is sufficient to account
for 100 000 hospital admissions, contribute to 16 000 deaths and cost $1.5 billion in the
US each year. Similarly in the United Kingdom, where there are around four million
users with osteoarthritis, use of NSAIDs contributes to 2000 deaths each year.
Hooper et al.2 conducted a review comparing gastric protection strategies in subjects
receiving NSAIDs. In all, 112 trials with 74 666 participants were included; 248 serious
gastrointestinal adverse drug reactions (ADRs) were reported and there were 138 deaths.
Histamine H2 receptor antagonists proved to be ineffective at preventing any of the pri-
mary outcomes. Proton pump inhibitors significantly reduced symptomatic ulcers (rela-
tive risk [RR] 0.09; 95% confidence interval [CI] 0.020.47) and serious complications
(RR 0.57; 95% CI 0.360.91). Misoprostol is a prostaglandin analogue that increases
mucous protection and decreases acid-induced damage. It may be prescribed alone
(Cytotec) or in combination with diclofenac (Arthrotec). A dose of 200 mg/day is usually
sufficient, while higher doses may cause diarrhoea, abdominal pain and bloating.
Misoprostol decreased the rate of symptomatic ulcers (RR 0.36; 95% CI 0.200.67) and
serious gastrointestinal side effects (RR 0.57; 95% CI 0.360.91). Use of COX-2-selective
agents was associated with lower rates of symptoms (RR 0.49; 95% CI 0.380.62) and
serious gastrointestinal ADRs (RR 0.55; 95% CI 0.380.80). H2 receptor antagonists are
effective at preventing symptoms and duodenal ulcers with NSAIDs but probably do not
prevent gastric ulcers. They are probably more cost-effective than COX-2-selective drugs
and their routine prescription with non-selective COX inhibitors may be justified.3
02-PS Rheumatology-cpp:02-PS Rheumatology-ppp.QXD 18/3/08 14:29 Page 57

11 NSAIDs gastric side effects & protection 57

Helicobacter pylori infection is implicated in the pathogenesis of gastric and duodenal


ulcers. It is not clear whether there should be routine screening for and eradication of this
agent in patients starting NSAIDs. Screening can be undertaken using breath tests, sero-
logical testing for antibodies or endoscopy with gastric biopsy. Serology is cheap and
convenient but a positive result does not necessarily indicate current infection and does
not change after eradication.
The Celecoxib Long-term Arthritis Safety Study (CLASS) and the Vioxx
Gastrointestinal Outcomes Research (VIGOR) study each enrolled over 8000 patients
and confirmed that COX-2 inhibitors were as effective as non-selective agents for
patients with arthritis.1 Use of COX-2 inhibitors was associated with lower risk of symp-
tomatic ulcers and serious gastrointestinal side effects. A number of studies have con-
firmed that COX-2-selective agents reduce the incidence of serious gastrointestinal side
effects by 40%50%. Their role in preventing dyspeptic symptoms has been less certain.
Spiegel et al.4 recently reviewed 26 published studies comparing COX-2-selective with
non-selective agents; the relative risk of dyspeptic symptoms was decreased by 12% and
the absolute risk was decreased by 3.7% with a COX-2-selective agent. By contrast, com-
pared with non-selective NSAIDs alone, NSAID plus concurrent proton pump inhibitor
led to a relative risk reduction of dyspeptic symptoms of 66% and an absolute risk reduc-
tion of 9%. Aspirin and NSAIDs may be associated with decreased risk of cancers, includ-
ing those of the gastrointestinal tract. Studies in vitro have shown that COX-2 inhibition
may decrease the growth of gastric cancers, and COX-2 inhibitors have been suggested
for the treatment of pre-cancerous gastric lesions in humans.5
Increased risk of myocardial infarction has been associated with use of high-dose,
non-selective NSAIDs and with use of COX-2-selective agents (chapter 12). Renal side
effects include salt and water retention, hypertension, acute renal failure, interstitial
nephritis, nephrotic syndrome and acute tubular necrosis.6 Renal side effects are more
likely when the patient is also taking angiotensin-converting enzyme inhibitors or diuret-
ics. COX-2 is constitutively expressed in the kidney, and patients taking COX-2
inhibitors are as likely to experience renal side effects as patients taking non-selective
agents. Other side effects are headache, dizziness, tinnitus and pruritus. Photosensitivity
may occur with piroxicam or diclofenac. NSAIDs may cause increased liver enzymes and,
occasionally, severe hepatitis.

Recent Developments
1 The role of COX isoenzymes in ulcer healing has recently been investigated in
COX-1 and COX-2 knockout mice with induced gastric ulcers.7 The COX-1
enzyme apparently had no part to play in ulcer healing, while ulcer healing was
impaired in COX-2 knockout mice or in wild-type mice treated with COX-2
inhibitors. Inducible nitric oxide synthase (NOS-2) and endothelial nitric oxide
synthase (NOS-3) were upregulated in healing ulcers, suggesting that these
enzymes might play a part in the healing process. Some of the potential benefit of
selective COX-2 inhibitors in protecting against ulcer formation may be offset by
their inhibitory effect on ulcer healing.
2 The proton pump inhibitors are widely used in prevention and treatment of
gastric injury induced by NSAIDs and their use is well supported by trial
02-PS Rheumatology-cpp:02-PS Rheumatology-ppp.QXD 18/3/08 14:29 Page 58

58 02 Osteoarthritis

Pain (e.g. from arthritis)

Simple analgesia (paracetamol)

NSAID indicated

Correct risk factors if possible


Smoking
Alcohol excess
Stress
Use of steroid, warfarin or aspirin

Previous ulcer No ulcer

H. Pylori (screen and eradicate)


Consider endoscopy/barium

Estimate risk of GI side effects


Warn patient of risks

Low risk Medium risk High risk

Lowest possible COX-2-selective NSAID 1 PPI


dose NSAID NSAID 1H2RA NSAID 1 misoprostol

Careful follow-up

Figure 11.2 NSAIDs and risk of gastrointestinal side effects. The considerations shown take account of the
relative costs of different drugs and combinations as well as their efficacy. COX-2, COX-2-selective agent;
H2RA, histamine H2 receptor antagonist (e.g. cimetidine, ranitidine); NSAID, non-steroidal anti-
inflammatory drug (non-selective agent); PPI, proton pump inhibitor.
02-PS Rheumatology-cpp:02-PS Rheumatology-ppp.QXD 18/3/08 14:29 Page 59

11 NSAIDs gastric side effects & protection 59

evidence. Several non-acid-dependent mechanisms are also thought to be


important in promotion of gastric ulcers. In a recent animal study,8 lansoprazole
decreased oxidative stress within the gastric mucosa and this was thought to
contribute to its effect in protecting from mucosal damage induced by NSAIDs.
3 The interaction between NSAIDs and H. pylori infection in the causation of
gastric and duodenal ulcers is controversial. A recent study compared rates of
gastrointestinal symptoms in patients positive and negative for H. pylori and
treated with NSAID.9 Furthermore, the positive group was divided, with half
being treated with H. pylori eradication therapy and half with placebo. Overall,
there was no relationship between dyspeptic symptoms induced by NSAID and
H. pylori status. These data do not favour routine screening for and eradication
of H. pylori in patients who are being started on NSAIDs.

Conclusion
NSAIDs represent one of the most commonly prescribed groups of drugs. Their use is
frequently associated with gastrointestinal symptoms, and serious drug reactions occur
in up to 1% of patients annually. The anti-inflammatory action of NSAIDs relates to
inhibition of the COX-2 isoenzyme, while gastrointestinal side effects are due to inhib-
ition of COX-1. Individual agents vary widely in their relative inhibitory activity for the
two isoenzymes. COX-2-selective agents are much less likely than non-selective agents to
cause dyspeptic symptoms. However, combining non-selective NSAIDs with proton
pump inhibitors or misoprostol probably provides even greater protection. Recent data
showing that the COX-2-specific agents in particular are associated with increased risk of
myocardial infarction have limited the use of this class of drug and led to the withdrawal
of some agents. Management of risk of gastric problems with NSAIDs is summarized in
Figure 11.2.

Further Reading
1 Fennerty MB. NSAID-related gastrointestinal injury. Evidence-based approach to a
preventable complication. Postgrad Med 2001; 110: 8794.
2 Hooper L, Brown TJ, Elliott R, Payne K, Roberts C, Symmons D. The effectiveness of five
strategies for the prevention of gastrointestinal toxicity induced by non-steroidal anti-
inflammatory drugs: systematic review. BMJ 2004; 329: 94858.
3 Elliott RA, Hooper L, Payne K, Brown TJ, Roberts C, Symmons D. Preventing non-
steroidal anti-inflammatory drug-induced gastrointestinal toxicity: are older strategies more
cost-effective in the general population? Rheumatology 2006; 45: 60613.
4 Spiegel BMR, Farid M, Dulai GS, Gralnek IM, Kanwal F. Comparing rates of dyspepsia
with Coxibs vs NSAID+PPI: a meta-analysis. Am J Med 2006; 119: 448.e2736.
5 Jiang XH, Wong BCY. Cyclooxygenase-2 inhibition and gastric cancer. Curr Pharm Des
2003; 9: 22818.
02-PS Rheumatology-cpp:02-PS Rheumatology-ppp.QXD 18/3/08 14:29 Page 60

60 02 Osteoarthritis

6 Cheng HF, Harris RC. Renal effects of non-steroidal anti-inflammatory drugs and selective
cyclooxygenase-2 inhibitors. Curr Pharm Des 2005; 11: 1795804.
7 Schmassmann A, Zoidl G, Peskar BM et al. Role of the different isoforms of cyclooxygenase
and nitric oxide synthase during gastric ulcer healing in cyclooxygenase-1 and -2 knockout
mice. Am J Physiol Gastrointest Liver Physiol 2006; 290: G74756.
8 Blandizzi C, Fornai M, Colucci R et al. Lansoprazole prevents experimental gastric injury
induced by non-steroidal anti-inflammatory drugs through a reduction of mucosal oxidative
damage. World J Gastroenterol 2005; 11: 405260.
9 Schaeverbeke T, Broutet N, Zerbib F et al. Should we eradicate Helicobacter pylori before pre-
scribing an NSAID? Result of a placebo-controlled study. Am J Gastroenterol 2005; 100:
263743.

P R O B L E M

12 NSAIDs Cardiac Complications

Case History
Mrs JA is a 64-year-old lady who finds that her osteoarthritis symptoms are no longer
controlled with simple analgesia. She has suffered from angina since the age of 57. She
has never had a myocardial infarction (MI) and her angina is stable. Her exercise tolerance
is not governed by angina and would be better if her arthritic symptoms were improved.
Would you prescribe her a non-steroidal anti-inflammatory drug (NSAID)?
Which agent would you choose?
What is the current thinking about NSAIDs and the risk of cardiac events?

Background
The cyclooxygenase (COX) enzymes catalyse the first stage in the formation of
prostaglandins and thromboxanes from arachidonic acid. The COX-1 isoenzyme is con-
stitutively expressed in most tissues, while COX-2 expression is induced at sites of
inflammation. The major side effects of the NSAID group of drugs, including the effects
on the stomach, are caused by COX-1 inhibition, while the beneficial effects are almost
entirely due to COX-2 inhibition. A classification of drugs with NSAID action is shown

Atlas Medical Publishing Ltd


02-PS Rheumatology-cpp:02-PS Rheumatology-ppp.QXD 18/3/08 14:29 Page 61

12 NSAIDs cardiac complications 61

Box 12.1 The non-steroidal anti-inflammatory drugs (NSAIDs)


Salicylates Arylalkanoic acids
Aspirin Diclofenac
Benorilate Etodolac
Diflunisal Indomethacin
Sulindac
2-arylpropionic acids (profens) Tolmetin
Ibuprofen
Fenbufen Oxicams
Fenoprofen Piroxicam
Naproxen Meloxicam
Ketoprofen Tenoxicam
Flurbiprofen
Pyrazolidine derivatives
Coxibs Phenylbutazone
Celecoxib (Celebrex) Oxyphenbutazone
Parecoxib (Dynastat) Azapropazone
Lumiracoxib (Prexige)
Rofecoxib * N-arylanthranilic acids
Valdecoxib * Mefenamic acid
Tolfenamic acid
* Drug withdrawn from the market.

in Box 12.1. Paracetamol has analgesic but no anti-inflammatory properties.


Unquestionably, endoscopic studies have shown that COX-2 inhibitors are less likely
than traditional NSAIDs to produce gastric ulceration. In late 2004, emerging data from
the Vioxx Gastrointestinal Outcomes Research (VIGOR) and Adenomatous Polyp
Prevention on Vioxx (APPROVe) studies demonstrated that risk of MI was increased up
to two-fold in patients treated with rofecoxib (Vioxx). This led to withdrawal of the drug
and valdecoxib was withdrawn because of similar considerations.
Activation of COX-1 in platelets leads to increased thromboxane A2 (TxA2) and con-
sequent platelet aggregation, vasoconstriction and smooth muscle proliferation.1 This
pathway is irreversibly blocked by aspirin. Recent studies suggest that non-selective
NSAIDs including naproxen and ibuprofen, when given concurrently with aspirin, may
interfere with this beneficial effect of aspirin and increase coronary events. COX-2 is
expressed only at very low levels in healthy blood vessels but is abundantly expressed in
atherosclerotic vessels. The enzyme is expressed in smooth muscle cells,
macrophages/foam cells and in the endothelium. Secretion of prostacyclin (PGI2) by
endothelial cells counteracts the effects of TxA2 released from platelets. This intrinsic
protection mechanism is lost when the COX-2 enzyme is inhibited. Figure 12.1 depicts
the proposed model of prothrombotic and antithrombotic balance for low-dose aspirin,
traditional NSAIDs and selective COX-2 inhibitors.
Recent data suggest that the adverse effects noted with rofecoxib are a class effect. A
trial with celecoxib2 revealed a dose-dependent increase in fatal and non-fatal cardiovas-
cular events. Deaths from cardiovascular events were three times more likely on
400 mg/day and six times more likely on 800 mg/day. Hazard ratios for any cardiovascular
02-PS Rheumatology-cpp:02-PS Rheumatology-ppp.QXD 18/3/08 14:29 Page 62

62 02 Osteoarthritis

LOW-DOSE
NORMAL ASPIRIN
PGI2
PGI2 TxA2
TxA2

Pro- Anti- Pro- Anti-


thrombotic thrombotic thrombotic thrombotic

SELECTIVE COX-2
NSAID ASPIRIN

TxA2

PGI2 TxA2 PGI2

Pro- Anti- Pro- Anti-


thrombotic thrombotic thrombotic thrombotic

Figure 12.1 Dynamic balance of prostacyclin (PGI2) and thromboxane A2 (TxA2) under the influence of
NSAIDs.

event were 1.5 and 1.9, respectively, for the two doses. Other studies with parecoxib (a
pro-drug for valdecoxib) have confirmed an increased cardiovascular risk, even with
short-term usage. Non-selective NSAIDs also inhibit COX-2 to a variable extent.
Increased cardiovascular risk has been noted in patients taking diclofenac, ibuprofen or
naproxen. Comparing COX-2 inhibitors with non-selective NSAIDs is difficult as the lat-
ter also have some COX-2-inhibitory activity. It is agreed that naproxen is the preferred
drug to compare with selective COX-2 inhibitors. Some of the conditions for which
NSAIDs are used are themselves associated with increased cardiovascular risk (e.g.
rheumatoid arthritis and systemic lupus erythematosus). The low-grade inflammation
associated with arthritic conditions may contribute to cardiovascular risk through
increasing endothelial dysfunction, decreasing availability of nitric oxide within vessels
and increasing the generation of reactive oxygen species.
Since the withdrawal of the two coxib drugs at the end of 2004, there has been consid-
erable activity to clarify whether safety concerns should include all selective COX-2
inhibitors and even extend to some non-selective agents. A large Canadian study3
included 113 927 elderly subjects starting COX-2 inhibitors and who did not have a his-
tory of MI. Rofecoxib was associated with increased risk of MI. The increased risk with
low-dose rofecoxib was offset by concurrent use of aspirin. There was no increased risk
associated with celecoxib or meloxicam use. Kearney et al.4 conducted a meta-analysis of
trials published between 1966 and April 2005. Patients taking COX-2 inhibitors had
increased risk of vascular events. The annual rates of serious cardiovascular events were
1.2% per year for patients taking COX-2 inhibitors and 0.9% per year for patients receiv-
ing placebo. There was no difference in event rate between COX-2-treated patients and
02-PS Rheumatology-cpp:02-PS Rheumatology-ppp.QXD 18/3/08 14:29 Page 63

12 NSAIDs cardiac complications 63

those treated with non-selective COX inhibitors. There was, however, considerable het-
erogeneity amongst the latter agents: relative risk of an event (compared with placebo)
was 0.92 (95% confidence interval 0.671.26) for naproxen, 1.51 (0.962.37) for ibupro-
fen and 1.63 (1.122.37) for diclofenac. Another meta-analysis of 114 controlled clinical
trials5 has reported increased risk of renal toxicity and arrhythmias with rofecoxib but no
increase with other coxib drugs.

Recent Developments
1 A Danish study6 has investigated rates of death and re-admission with MI in
58 432 patients discharged following acute MI. There was a trend towards
increased hospitalization for use of both COX-2-selective and non-selective
agents. The hazard ratios for death were 2.80 (95% confidence interval 2.413.25)
for rofecoxib, 2.57 (2.153.08) for celecoxib, 1.50 (1.361.67) for ibuprofen, 2.40

Non-pharmacological treatments been fully explored?

Analgesia (paracetamol up to 4 g/day)

Low cardiac risk* Angina Previous MI or stroke

NSAID NSAID 1 Stronger analgesia


aspirin (e.g. tramadol)
High gastric risk High gastric risk
or symptoms or symptoms
Naproxen 1
Or
aspirin
Add PPI or
misoprostol

Add PPI or
misoprostol
COX-2

Review need for NSAID regularly (every 3 months)

Figure 12.2 Use of NSAIDs in patients with cardiovascular risk. * Measure lipids, blood glucose and blood
pressure; it is useful to use a cardiovascular risk calculator. COX-2, COX-2-selective agent; NSAID, non-
selective anti-inflammatory drug; PPI, proton pump inhibitor.
02-PS Rheumatology-cpp:02-PS Rheumatology-ppp.QXD 18/3/08 14:29 Page 64

64 02 Osteoarthritis

(2.092.80) for diclofenac and 1.29 (1.161.43) for other NSAIDs. The risk of
death was related to dose of drug taken.
2 A nationwide Finnish study7 examined the use of NSAIDs in 33 309 patients
admitted with their first MI. These subjects were age- and gender-matched with
controls. The apparent risk with NSAIDS for first MI was 1.40 (95% confidence
interval 1.331.48). No difference was noted between non-selective agents, semi-
selective COX-2 inhibitors or selective COX-2 inhibitors. There was no
suggestion that any of the drugs was protective against MI.
3 A study to compare etoricoxib with diclofenac is under way.8 This study in
patients with rheumatoid arthritis or osteoarthritis should answer the question of
whether the risk associated with use of COX-2 inhibitors is any greater than that
for conventional NSAIDs. The study has enrolled nearly 35 000 patients.

Conclusion
Over 30 million people worldwide take NSAIDs regularly. Recent studies have shown
that use of these drugs is associated with increased risk of MI. If non-pharmacological
means and simple analgesia fail to control the above patients symptoms, then it is rea-
sonable to consider use of NSAIDs. It is probably best, initially, to use one of the conven-
tional agents and naproxen is the drug of choice, the dose and duration of exposure being
kept to a minimum. There is an argument for using higher-dose aspirin to overcome the
effects of COX-2 inhibition. A scheme for considering NSAID use is presented in Figure
12.2. There is considerable heterogeneity of cardiovascular risk amongst both COX-2-
selective and non-selective NSAIDs, although it seems likely that increased risk is related
to the ability of the individual drug to inhibit the COX-2 isoenzyme.

Further Reading
1 Hermann M, Ruschitzka F. Coxibs, non-steroidal anti-inflammatory drugs and cardiovascular
risk. Intern Med J 2006; 36: 30819.
2 Solomon SD, McMurray JJ, Pfeffer MA et al; Adenoma Prevention with Celecoxib (APC)
Study Investigators. Cardiovascular risk associated with celecoxib in a clinical trial for
colorectal adenoma prevention. New Engl J Med 2005; 352: 107180.
3 Lvesque L, Brophy J, Zhang B. The risk for myocardial infarction with cyclooxygenase-2
inhibitors: a population study of elderly adults. Ann Intern Med 2005; 142: 4819.
4 Kearney PM, Baigent C, Godwin J, Halls H, Emberson JR, Patrono C. Do selective cyclo-
oxygenase inhibitors and traditional non-steroidal anti-inflammatory drugs increase the risk
of atherothrombosis? Meta-analysis of randomised trials. BMJ 2006; 332: 13028.
5 Zhang J, Ding EL, Song Y. Adverse effects of cyclooxygenase 2 inhibitors on renal and
arrhythmia events: meta-analysis of randomised trials. JAMA 2006; 296: 161932.
6 Gislason GH, Jacobsen S, Rasmussen JN et al. Risk of death or reinfarction associated with the
use of selective cyclooxygenase-2 inhibitors and nonselective nonsteroidal anti-inflammatory
drugs after acute myocardial infarction. Circulation 2006; 113: 290613.
02-PS Rheumatology-cpp:02-PS Rheumatology-ppp.QXD 18/3/08 14:29 Page 65

13 Joint replacement surgery 65

7 Helin-Salmivaara A, Virtanen A, Vesalainen R et al. NSAID use and the risk of hospitalization
for first myocardial infarction in the general population: a nationwide case-control study from
Finland. Eur Heart J 2006; 27: 165763.
8 Cannon CP, Curtis SP, Bolognese JA, Laine L. Clinical trial design and patient demographics
of the Multinational Etoricoxib and Diclofenac Arthritis Long-term (MEDAL) study program:
cardiovascular outcomes with etoricoxib versus diclofenac in patients with osteoarthritis and
rheumatoid arthritis. Am Heart J 2006; 152: 23745.

P R O B L E M

13 Joint Replacement Surgery

Case History
Mary has been passing on your advice on preventing osteoarthritis (OA) to the members
of the local bowls club. The information may have come a little late for George who has
severe right hip OA. He is now aged 62 years, is moderately overweight at 89 kg (body
mass index 28.7 kg/m2) and has asked about a hip replacement.
Will weight reduction delay Georges need for surgery?
What are the indications for a knee or hip replacement?
What are the options for joint replacement and what is their expected lifespan?

Background
In the United States and United Kingdom (UK), the number and rate has doubled for
knee replacements and tripled for hip replacements over the past decade. Future projec-
tions are that >750 000 of these procedures will be performed per year by the year 2030. A
critical variable in these estimations is the low proportion of individuals who on the basis
of symptoms warrant joint replacement but remain unwilling to consider the procedure.

Weight reduction
Given that obesity is modifiable by conservative treatment such as weight loss, its impor-
tance in reducing the incidence of large joint OA is often advocated, yet determining the
impact prospectively of weight reduction is more problematic.1 The effect of adult weight
change on risk for total hip replacement resulting from primary OA was studied prospect-
ively in a large Norwegian cohort whose weight had been recorded over a mean 14-year

Atlas Medical Publishing Ltd


02-PS Rheumatology-cpp:02-PS Rheumatology-ppp.QXD 18/3/08 14:29 Page 66

66 02 Osteoarthritis

period from age mid-30s to late 40s. With an average follow-up of nine years, there was
no association of weight slope, absolute weight change or relative weight change between
screenings with later total hip replacement.2
A number of studies have shown that after large joint replacement weight increases,
from a small amount in the pre-operatively non-obese patient to an average 4.8 kg gain
in the pre-operatively obese.3,4 Post-operative weight gain has been documented for both
hip and knee replacement, with younger hip-replacement patients gaining a significant
amount of weight.5 Obesity therefore needs to be treated as an independent disease that
increases the risk of requiring major joint replacement, and is adversely impacted by
surgery that is aimed at reducing pain and increasing a persons exercise ability.

Exercise
Exercise is beneficial for people with arthritis, and pre-operative functional status is posi-
tively related to post-operative functional status. Inclusion of both pre-operative and
post-operative strength training in a total hip replacement programme improves phys-
ical function at 12 and 24 weeks post-operatively.6 The optimal type and the amount of
exercise plus the timing of the pre-operative intervention need to be established.

Indications for hip or knee replacement


These indications are summarized in Figure 13.1. Concerns about the timing of surgery
surface for two primary classes of patients: the younger, active candidate and the older
candidate. For the younger (less than 60 years of age), active patient, the long-term sur-
vival of the prosthesis is the major concern. While there are several studies on long-term
outcome, most of these describe prosthesis survival for the more typical older and less
active individual. Furthermore, as prosthesis design and materials improve, the newer
prostheses will have longer survival.
For the less active, older patient in whom long-term prosthesis survival is less of a con-
cern, some information may be provided by comparing outcomes in patients with poor
pre-operative symptoms versus those with better pre-operative symptoms as proxies for
late versus early surgery. When followed at 6 and 24 months post-operation, those with
poorer pre-operative status had poorer outcomes. Patients need to appreciate the small
but real risk of perioperative complications and, for younger patients, the potential for
revision surgery. However, they also should be appraised that likelihood of functional
benefit is higher if they enter into surgery with a better functional status.

Perioperative preparations and complications


Rheumatoid arthritis (RA) patients also benefit from joint replacement, and in these
individuals their immunosuppressant therapy needs to be considered. Weekly
methotrexate is the mainstay of most RA regimes, and can be continued up to the time of
surgery and recommenced upon return of baseline fluid balance and renal function. In a
prospective, randomized study of post-operative infection and surgical complications in
388 RA patients in the UK who underwent elective orthopaedic surgery, one group con-
tinued methotrexate and the other group ceased methotrexate two weeks pre-operatively
and recommenced treatment two weeks post-operatively.6 Complication rates were
compared between groups and with 228 RA patients not on methotrexate and under-
going elective orthopaedic surgery. No increase in the duration of early post-operative
02-PS Rheumatology-cpp:02-PS Rheumatology-ppp.QXD 18/3/08 14:29 Page 67

13 Joint replacement surgery 67

Indications: Pain limiting function despite conservative therapy


Nocturnal pain disrupting sleep
Understanding of potential complications

Contraindications: Severe neurological or cognitive impairment that will


impair rehabilitation
Vascular disease/unstable angina/anaesthetic risk
Chronic sepsis

Plain X-ray

Confirms significant No or mild X-ray


joint pathology changes

Review to exclude
periarticular source of pain

Orthopaedic
referral

Figure 13.1 Hip and knee joint replacement surgery.

infections or rate of complications was found. Patients who continued methotrexate had
significantly fewer adverse events than either of the other two groups (P <0.003).
Additionally, those who did cease methotrexate had a higher incidence of disease flares in
the six weeks following surgery than those who continued the medication.
The risk of death is approximately 1% in the 90 days following hip replacement and
slightly less for knee replacement. Pulmonary embolism and myocardial infarction occur
within 90 days in 1% of patients. Dislocation occurs in 3% of total hip replacements.
Deep joint infection occurs in about 0.4% of total knee replacements and 0.2% of total
hip replacements. While dislocation and infection remain potential complications over
the longer term, the risk is much higher in the first 90 days than subsequently.

How long will it last?


Long-term success of total hip arthroplasty is dependent on prosthetic component fix-
ation and the amount of wear and debris generated by the bearing surface. Advances in
02-PS Rheumatology-cpp:02-PS Rheumatology-ppp.QXD 18/3/08 14:29 Page 68

68 02 Osteoarthritis

both femoral cementing techniques and the design of cemented stems have resulted in
near-perfect (98%) survivorship at ten years and good survivorship (93%) at 25 years.7
Comparable survival rates have been reported using cementless techniques for the
femoral component. In the acetabular component, ten-year survival rates are similar for
cemented and cementless techniques (approximately 95%98%); however, at 15 years,
cementless technology supersedes cemented (85%95% versus 70%95%).
Cemented total knee arthroplasty (TKA) is the current gold standard, with consistent
long-term (1014 years) survival rates of 94%98%. Whilst some cementless TKA tech-
niques have demonstrated good long-term survival, the majority have not reliably resulted
in bone ingrowth. Long-term survival rates and functional abilities are comparable in cru-
ciate-retaining and cruciate-substituting prostheses. To improve patient satisfaction and
function, implants have an increased arc of flexion that may approach 150 degrees of knee
flexion. Recent literature searches and meta-analyses suggest that resurfacing the patella at
TKA likely improves outcomes and long-term, pain-free patella function.

Recent Developments
1 Ogonda et al.8 randomized patients to a standard 16 cm incision or a minimal
(10 cm) incision for their hip replacement. They found that although the
standard incision group had a higher estimated blood loss, there was no
difference in mean units of transfusion or haematocrit at discharge. There were
also no significant differences found in post-operative pain, timed 10 m walk on
the second post-operative day, length of hospital stay or functional outcome.
2 Although TKA is currently the gold standard for knee joint replacement,
unicompartmental arthroplasty has emerged as a suitable option for advanced
medial compartment osteoarthritis, with outcomes comparable with those for
TKA. The procedure is generally performed using a minimally invasive approach
and leads to more rapid recovery, minimal bone loss, less pain and early
discharge compared to TKA. The 1015-year survival rates for unicompartmental
knee arthroplasty are high and range from 95%98%.9

Conclusion
Total joint replacement should not be regarded as an intervention of last resort and
should be offered to patients when their quality of life is compromised. Pre-operative
functional status is the most significant predictor of post-operative functional status.
Surgery should be considered when symptoms warrant it and when the intervention will
contribute to a persons post-operative life experience. As procedures, total hip and knee
joint replacements (Figure 13.2) are costly, but total hip replacement is among the most
cost-effective interventions in medicine. Long-term follow-up is available on older pros-
theses and relief of pain and restoration of function can be expected in over 90% of
patients, with a complication rate that is rare and a mortality rate of <1%.
02-PS Rheumatology-cpp:02-PS Rheumatology-ppp.QXD 18/3/08 14:29 Page 69

13 Joint replacement surgery 69

(A)

(B)

Figure 13.2 Total hip and knee joint replacements. (A) An artificial hip joint with metallic femoral stem
and ultra-high-molecular-weight polyethylene acetabular cup. An artificial knee joint in situ, with metallic
femoral component and ultra-high-molecular-weight polyethylene bearing. (Both figures redrawn with
permission from Fisher 2004.10).
02-PS Rheumatology-cpp:02-PS Rheumatology-ppp.QXD 18/3/08 14:29 Page 70

70 02 Osteoarthritis

Further Reading
1 Powell A, Teichtahl AJ, Wluka AE, Cicuttini FM. Obesity: a preventable risk factor for
large joint osteoarthritis which may act through biomechanical factors. Br J Sports Med
2005; 39: 45.
2 Flugsrud GB, Nordsletten L, Espehaug B, Havelin LI, Meyer HE. Weight change and the
risk of total hip replacement. Epidemiology 2003; 14: 57884.
3 Donovan J, Dingwall I, McChesney S. Weight change 1 year following total knee or hip
arthroplasty. ANZ J Surg 2006; 76: 2225.
4 Aderinto J, Brenkel IJ, Chan P. Weight change following total hip replacement: a
comparison of obese and non-obese patients. Surgeon 2005; 3: 26972.
5 Heisel C, Silva M, dela Rosa MA, Schmalzried TP. The effects of lower-extremity total
joint replacement for arthritis on obesity. Orthopedics 2005; 28: 1579.
6 Grennan DM, Gray J, Loudon J, Fear S. Methotrexate and early postoperative
complications in patients with rheumatoid arthritis undergoing elective orthopaedic
surgery. Ann Rheum Dis 2001; 60: 21417.
7 Jones DL, Westby MD, Greidanus N et al. Update on hip and knee arthroplasty: current
state of evidence. Arthritis Rheum 2005; 53: 77280.
8 Ogonda L, Wilson R, Archbold P et al. A minimal-incision technique in total hip
arthroplasty does not improve early post-operative outcomes. A prospective, randomized,
controlled trial. J Bone Joint Surg Am 2005; 87: 70110.
9 Yang KY, Wang MC, Yeo SJ, Lon NN. Minimally invasive unicondylar versus total
condylar knee arthroplasty early results of a matched pair comparison. Singapore Med J
2003; 416: 55962.
10 Fisher J. Surgery for arthritis: total hip and knee joint replacement. ARC Topical Reviews
2004, Number 3.
03-PS Rheumatology-cpp:03-PS Rheumatology-ppp.QXD 18/3/08 15:26 Page 71

S E C T I O N T H R E E 03
Rheumatoid Arthritis
14 Causes
15 Laboratory and imaging investigations
16 Managing rheumatoid arthritis at onset
17 Evaluating the response to treatment
18 Pregnancy and rheumatic diseases
19 Diet and arthritis
20 Polyarthritis in the elderly

P R O B L E M

14 Causes

Case History
Steves rheumatoid arthritis (RA) has been managed for the past four years with a
combination of methotrexate, hydroxychloroquine and sulphasalazine. He is rheumatoid
factor (RF) positive and has significant joint damage. He wishes to explore with you why
he may have developed RA. His daughter, aged 24 years, has recently been told that she is
RF positive.
What genetic factors influence the development or severity of RA?
What environmental factors are known to be involved?
How does being RF positive influence the likelihood of developing RA?

Background
RA is a chronic inflammatory disease affecting the synovium and leading to joint damage
and absorption of adjacent bone. Peak age of onset is in the fifth decade and females are

Atlas Medical Publishing Ltd


03-PS Rheumatology-cpp:03-PS Rheumatology-ppp.QXD 18/3/08 15:26 Page 72

72 03 Rheumatoid Arthritis

two to three times more likely to be affected, although the sex distribution becomes less
apparent with increasing age. Apart from the disability caused by the disease, it also has a
significant impact on life expectancy, with RA patients living on average three to ten years
less than unaffected individuals. The overall incidence of the condition is 2050 per
100 000 per year. This may have decreased since the 1960s. The prevalence varies in dif-
ferent countries:1
0.5%1.1% in Northern Europe and North America
0.3%0.7% in Southern Europe and other warmer countries
0.5% in developing countries
In the pre-clinical phase of RA, immunoglobulin G (IgG) may be elevated and RF
(polyclonal antibodies directed at the Fc fragment of IgG) and anti-cyclic citrullinated
peptide (anti-CCP) antibodies may be detected. Citrullination is a post-translational
modification of arginine by peptidylarginine deaminase enzymes. Specific disease-
modifying treatment is not indicated at this early phase of the disease.

Genetics of RA
The importance of genetic factors is clear from family studies: prevalence is 2%12% in
first-degree relatives of RA sufferers i.e. approximately ten times that of the back-
ground population. Concordance rate is 10%30% in monozygotic twins and 5%10%
in same-sex dizygotic twins. The genetic component of RA has been studied by genome-
wide linkage analyses and by study of individual candidate genes.2 The major sus-
ceptibility gene for RA is the DRB1 gene located in the class II histocompatibility region
on chromosome 6p. Risk of RA is associated with the alleles HLA-DRB1*0401,
DRB1*0404, DRB1*0405, DRB1*0408, DRB1*0101 and DRB1*0102. These alleles have
similar or identical sequences at the third hypervariable region in the b chain between
amino acid positions 70 and 74. According to the shared epitope (SE) hypothesis, this
sequence is directly involved in the immunopathogenesis of RA, although the precise
mechanism is not clear.
The human leukocyte antigen (HLA) component accounts for around 30% of the
genetic risk. The susceptibility alleles linked to RA are present in around 70% of patients
but also in 40% of the general population. Thus genotyping is not currently useful in
clinical practice. Other genes associated with RA include:
Tumour necrosis factor (TNF)-a located in the class II part of the histocompatibility
complex on 6p. Polymorphisms influence the level of TNF-a expression
The TNF-a receptor type 2 gene (TNFR2) located at 1p36. The 196R allele of this
gene is linked with RA
Peptidylarginine deaminases (PADIs), also located at 1p36. These enzymes catalyse
the citrullination of arginine residues and, therefore, determine the level of citrulli-
nated peptides
SLC22A4 (solute carrier family 22 A4) at chromosome 5q31 also implicated in
Crohns disease
PTNP22 (tyrosine phosphate non-receptor type 22) at 1p13 also linked with type 1
diabetes and autoimmune thyroid disease
03-PS Rheumatology-cpp:03-PS Rheumatology-ppp.QXD 18/3/08 15:26 Page 73

14 Causes 73

Environmental factors
These triggers may begin to operate long before clinically apparent disease develops. The
pre-clinical phase of RA may last for up to 1520 years. During this phase the prevalence
of autoantibody markers increases amongst susceptible individuals as the onset of clin-
ical disease approaches. IgA rheumatoid factors are probably the first markers to appear
in most patients. Neither RF nor anti-CCP antibodies are entirely sensitive, but together
the markers are reasonably specific 85% to 90% for RF and 98% for anti-CCP. Markers
of inflammation including total IgG, erythrocyte sedimentation rate (ESR) and
C-reactive protein (CRP) may be increased during the clinical phase. The patient may
experience symptoms of malaise and fatigue and intermittent joint symptoms (palin-
dromic rheumatism).
Risk factors for RA are summarized in Figure 14.1. Environmental triggers may be
more important than genetic factors.3 Some studies have shown much higher association
than this in families but it is difficult to disentangle genetic influences from social and
other environmental influences.

Susceptibility Genetic Family history


Other autoimmune disease
HLA-DRB1 SE etc.
Female gender

Subclinical Environmental Smoking


Infections (EBV etc)
Low UV (vitamin D)

Diet Protective Aggravating


Omega-3 Red meat
Mediterranean Caffeine ?
diet
Fruit
Vitamin C
Disease onset Occupation Agriculture
Postal and print
Silica
Mineral oil

Smoking
Aging
Progression Oestrogen withdrawal
Decreased androgen levels
Periodontal disease

Figure 14.1 Development of rheumatoid arthritis. EBV, EpsteinBarr virus; UV, ultraviolet light.
03-PS Rheumatology-cpp:03-PS Rheumatology-ppp.QXD 18/3/08 15:26 Page 74

74 03 Rheumatoid Arthritis

Smoking is probably the most potent environmental trigger, with smokers at two- to
four-fold increased risk of RA. The risk is dose dependent with heavy smokers being at
higher risk than light smokers. A number of mechanisms could account for the link
between smoking and RA these include the immunomodulatory effect of tobacco com-
ponents, oxidative stress and oestrogenic effects, and the effect of smoke in promoting
peptide citrullination. Smoking also increases the risk for anti-double-stranded DNA
(anti-dsDNA)-positive systemic lupus erythematosus. Smoking is associated with more
severe rheumatoid disease: erosive arthropathy, rheumatoid nodules and vasculitis. The
Nurses Health Study was a large cohort study investigating the effect of aspirin and vita-
min E in preventing cardiac disease. In a prospective study of 103 818 women from this
cohort followed from 1976 to 2002,4 there were 680 new cases of RA. Women with a his-
tory of ten pack-years or more were at increased risk, and the risk of RA was related both
to the intensity and duration of smoking. The increased risk persisted for up to 20 years
after cessation of smoking.
Oestrogen exposure in women has a protective effect. This is consistent with increased
risk of the disease after the menopause and the slight protective effect of using hormone
replacement therapy. It is not clear why the prevalence of RA is so much higher in women
compared with men. As with other autoimmune diseases, pregnancy is protective, with
the incidence during pregnancy being 70% lower than that for age- and sex-matched
controls. However, there is a rebound increase after pregnancy with risk increased by as
much as five-fold in the months after delivery. Breast-feeding also appears to increase
risk. This may be due to the low-oestrogen state or to pro-inflammatory actions of pro-
lactin. The relationship between sex steroid status and susceptibility to autoimmune dis-
ease is complex.5
The influence of social, employment and educational status does not seem to be great
in RA. The disease is more prevalent in urban populations, but this could be partly due to
migration of patients with chronic RA to metropolitan centres. Certain occupations do
seem to have increased risk: men who work in agriculture and in the paper and transport
industries, and female printer and postal workers, appear to be at increased risk. Workers
exposed to silica dust (e.g. from drilling or crushing rock) are at up to three-fold
increased risk. Mineral oil exposure has been noted to increase risk of a number of
autoimmune diseases including RA.
Low intake of fruit and vitamin C relates to increased risk, while high intake of red
meat carries a high risk. Regular consumption of oily fish (a source of omega-3 fatty
acids) is protective, as is adherence to a Mediterranean diet. Low vitamin D status is
probably a risk factor for RA, but there is no evidence that vitamin D supplementation
protects against the disease. Some immune-mediated diseases, particularly type 1 dia-
betes and multiple sclerosis, are more common in latitudes where there is lower exposure
to ultraviolet light.6 While vitamin D status is the most plausible explanation for this
association, other metabolic factors may equally be involved.
The notion of an infectious trigger for RA is appealing. Evidence in favour of this
includes the decreasing incidence of the condition and the associations with previous
blood transfusion and pet ownership. There is no evidence for a seasonal incidence of the
condition. Amongst candidate organisms are EpsteinBarr virus (EBV), rubella, parvo-
virus and Borrelia burgdorferi. Of these, the strongest evidence is with EBV, the glyco-
protein of which has cross-reactivity with the HLA-DRB1 SE. Increased incidence and
03-PS Rheumatology-cpp:03-PS Rheumatology-ppp.QXD 18/3/08 15:26 Page 75

14 Causes 75

levels of EBV DNA have been reported in peripheral blood mononuclear cells and syn-
ovial fluid of RA patients and the virus is well documented to have immunomodulatory
properties.

Recent Developments
1 Two recent studies7,8 have pooled data from the United Kingdom and United States
to demonstrate that there are important susceptibility loci at chromosomes 6q and
16p. There appears to be an interaction between these loci and HLA determinants.
Further study of the genes at these loci may shed more light on the pathogenesis of
RA.
2 In a recent Swedish casecontrol study,9 positive immunostaining for citrullinated
peptides was found in the cells from bronchoalveolar lavage of smokers.
Furthermore, there was a strong association between smoking status and the
presence of anti-citrulline antibodies. There was a positive interaction between anti-
citrulline autoimmunty and the presence of SE in the HLA-DR region. Individuals
who smoked and had double copies of the HLA-DR SE had a 21-fold increased risk
of RA.
3 In a mouse model of RA, oestrogen deficiency by oophorectomy accelerated the
development of arthritis and increased levels of RF, anti-dsDNA and anti-collagen
antibodies.10
4 Periodontal disease is mainly caused by infection with Gram-negative anaerobes.
These lead to increased production of cytokines that damage gingival connective
tissue and increase resorption of dental alveolar bone. Marotte et al.11 have
shown that the presence of HLA-DR SE correlates with increased periodontal
bone destruction. Furthermore, periodontal bone destruction strongly correlated
with wrist bone resorption in patients with RA.

Conclusion
RA has a significant genetic component but perhaps not as strong as was once thought.
The most powerful genetic influence is the HLA-DRB1-related SE. Of known environ-
mental triggers, smoking is the most powerful. Women are more prone to the disease but
low oestrogen status appears to increase susceptibility. The presence of RF is not
absolutely specific for the disease but is generally highly predictive. The antibody does
occur in a proportion of the general population and may occur in other autoimmune
connective tissue disorders. A long pre-clinical phase with positive RF and anti-CCP
antibodies, sometimes with mild and variable symptoms, is common in RA.

Further Reading
1 Alamanos Y, Drosos AA. Epidemiology of adult rheumatoid arthritis. Autoimmun Rev 2005; 4:
1306.
03-PS Rheumatology-cpp:03-PS Rheumatology-ppp.QXD 18/3/08 15:26 Page 76

76 03 Rheumatoid Arthritis

2 Dieude P, Cornelis F. Genetic basis of rheumatoid arthritis. Joint Bone Spine 2005; 72: 5206.
3 Oliver JE, Silman AJ. Risk factors for the development of rheumatoid arthritis. Scand J
Rheumatol 2006; 35: 16974.
4 Costenbader KH, Feskanich D, Mandl LA, Karlson EW. Smoking intensity, duration, and
cessation, and the risk of rheumatoid arthritis in women. Am J Med 2006; 119: 503.e19.
5 Cutolo M, Lahita RG. Estrogens and arthritis. Rheum Dis Clin North Am 2005; 31: 1927.
6 Ponsonby AL, Lucas RM, van der Mei IAF. UVR, vitamin D and three autoimmune
diseases multiple sclerosis, type 1 diabetes, rheumatoid arthritis. Photochem Photobiol
2005; 81: 126775.
7 Etzel CJ, Chen WV, Shepard N et al. Genome-wide meta-analysis for rheumatoid arthritis.
Hum Genet 2006; 119: 63441.
8 John S, Amos C, Shepard N et al. Linkage analysis of rheumatoid arthritis in US and UK
families reveals interactions between HLA-DRB1 and loci on chromosomes 6q and 16p.
Arthritis Rheum 2006; 54: 148290.
9 Klareskog L, Stolt P, Lundberg K et al. A new model for an etiology of rheumatoid
arthritis: smoking may trigger HLA-DR (shared epitope)-restricted immune reactions to
autoantigens modified by citrullination. Arthritis Rheum 2006; 54: 3846.
10 Yoneda T, Ishimaru N, Arakaki R et al. Estrogen deficiency accelerates murine
autoimmune arthritis associated with receptor activator of nuclear factor-kB ligand-
mediated osteoclastogenesis. Endocrinology 2004; 145: 238491.
11 Marotte H, Farge P, Gaudin P, Alexandre C, Mougin B, Miossec P. The association
between periodontal disease and joint destruction in rheumatoid arthritis extends the link
between the HLA-DR shared epitope and severity of bone destruction. Ann Rheum Dis
2006; 65: 9059.
03-PS Rheumatology-cpp:03-PS Rheumatology-ppp.QXD 18/3/08 15:26 Page 77

15 Laboratory and imaging investigations 77

P R O B L E M

15 Laboratory and Imaging Investigations

Case History
Joan is a 52-year-old schoolteacher who you have diagnosed with rheumatoid arthritis
(RA). You have discussed a range of treatment options with her and also discussed with
her a number of investigations. She is keen to know about the monitoring and
investigation required for different medications and what information will be gained
from other scheduled investigations.
What investigations should be undertaken at the onset of RA?
What investigations help with monitoring the activity of the condition?
How should follow-up be planned?

Background
Making a diagnosis of RA
Joan has an inflammatory arthritis; it is established as a chronic problem, having
extended beyond eight to ten weeks, which makes a post-viral arthropathy unlikely. The
American College of Rheumatology classification for RA requires four of seven criteria to
be met, with the clinical features of arthritis being present for at least six weeks:
Morning stiffness of greater than one hour duration
Objective evidence of joint inflammation, such as soft tissue swelling or fluid in 3 of 14
defined joint areas (right or left proximal interphalangeal [PIP], metacarpophalangeal
[MCP], wrist, elbow, knee, ankle and metatarsophalangeal joints)
At least one of the joint areas demonstrating inflammation must be in the hands
Simultaneous involvement of the same joint area bilaterally in at least one pair
Rheumatoid nodules
A positive rheumatoid factor (RF) using a method that is positive in <5% of normal
subjects
Typical changes of RA in hand and wrist X-rays, which must include erosions or
unequivocal periarticular osteopenia
RA is one of the most common autoimmune diseases, affecting 1% of the population.
Other diagnoses, such as psoriatic arthritis or systemic lupus erythematosus (SLE), could
present with the same clinical scenario and the history and examination undertaken will
have considered these in the differential diagnosis.

Atlas Medical Publishing Ltd


03-PS Rheumatology-cpp:03-PS Rheumatology-ppp.QXD 18/3/08 15:26 Page 78

78 03 Rheumatoid Arthritis

Investigations for RA
RF is a polyclonal antibody predominantly of the immunoglobulin M (IgM) and IgG
classes, which targets the Fc region of the IgG. Whilst commonly thought to have diagnos-
tic significance by both doctors and patients, it has a very poor predictive value, being pres-
ent in only 70% of RA cohorts and also present in 5% of the normal population. RF
positivity increases with age and is positive in any circumstance with prolonged antigen
stimulation, so that chronic infections such as malaria and tuberculosis account for the
bulk of RF positivity globally. Similarly in individual patients, age and chronic sepsis need
to be considered in addition to the pattern of joint disease before attributing a positive RF
with a diagnosis of RA. RF positivity associates with increased joint erosions and vasculitis.
Radiographs of the hands and wrists, feet and cervical spine were historically under-
taken at baseline and then one- to two-yearly to monitor for erosive disease. Fortunately,
the model of therapy has changed so that the goal is suppression of inflammation as soon
as possible after diagnosis is established; this is combined with the knowledge that joint
erosions detected by magnetic resonance imaging (MRI) or musculoskeletal ultrasound
occur up to 18 months before their detection on plain X-ray. In Joans case, radiographs
will add little, demonstrating the soft tissue swelling that was observed on clinical examin-
ation and therefore reinforcing the clinical pattern noted. In later disease, different pat-
terns of involvement can be noted for psoriasis, SLE and RA. Psoriasis classically develops
pencil in cup erosions of both PIP and distal interphalangeal joints with the proximal
phalanx narrowing to the sharpened pencil, burrowing into the cup-like, widened distal
phalanx. SLE is almost always a non-erosive disease. X-ray changes in RA mirror the
joints clinically involved, primarily affecting the hands, the PIP, MCP and wrists. Soft tis-
sue swelling occurs early, with periarticular osteopenia noted, and subsequently there is
symmetrical joint space narrowing. Erosions when they occur begin at the site of synovial
lining of the joint capsule reflecting onto the cartilage/periosteum junctions, so that they
occur slightly distant from the joint surface.
Joans erythrocyte sedimentation rate (ESR) and serum C-reactive protein (CRP) will
both be increased and will act as a means to monitor therapy, with the goal being their
normalization. Both ESR and CRP are measures of acute phase proteins produced in the
liver in response to monocyte-derived cytokines. CRP testing is more expensive than ESR
but is the preferred measurement for monitoring success in suppressing joint inflamma-
tion, particularly in RA. The situation is more complex in SLE, with some individuals
having a cytokine pattern resulting in an elevated CRP, and others whose disease pattern
is not associated with a rise in CRP.1
The blood film commonly shows an inflammatory thrombocytosis, with more estab-
lished disease showing a normochromic normocytic anaemia; this can eventually mimic
the hypochromic microcytic picture of iron deficiency, due to the inflammatory process
inhibiting the utilization of iron stores. Concomitant therapies such as non-steroidal
anti-inflammatory drugs (NSAIDs) may cause genuine iron deficiency through gastro-
intestinal blood loss. Corticosteroids induce a neutrophil leukocytosis. The presence of
lymphopenia in Joan would have caused a reconsideration of the diagnosis of SLE.
Clinical examination should include urinalysis, with the presence of proteinuria or
leukocytes warranting further microscopy and evaluation. Proteinuria would suggest
glomerular inflammation. If SLE remains part of the differential diagnosis, an antinu-
clear antibody (ANA) test can be requested. A significant positive result is a titre of 1:160
03-PS Rheumatology-cpp:03-PS Rheumatology-ppp.QXD 18/3/08 15:26 Page 79

15 Laboratory and imaging investigations 79

or greater, and again in itself has poor diagnostic or prognostic value unless interpreted
as part of the clinical presentation. Evaluation of electrolytes, renal function and liver
function should be undertaken. NSAIDs may exacerbate hyperkalaemia and impair renal
function, with several of the disease-modifying agents notable for their potential to cause
hepatitis with elevation of transaminases.

Safety monitoring of RA medications


A guide to the safety monitoring for the agents used for treatment of RA is shown in
Table 15.1. Methotrexate, alone or in combination with other agents, is the basis of most
RA regimes. Biological agents that target specific cytokines such as tumour necrosis fac-
tor (TNF) and interleukin-1, or that block cell-to-cell signalling (such as cytotoxic lym-
phocyte-associated antigen linked to immunoglobulin [CTLA4-Ig; abatacept]) are
becoming increasingly used in those who do not respond quickly to standard therapy.
Screening for hepatitis B and C is recommended, although a positive result is not an
absolute contraindication to therapy. Recent studies have suggested an enhanced hepa-
titis C antiviral effect of interferon and ribavirin in the setting of concomitant etanercept.

Table 15.1 Guide to safety monitoring of commonly used antirheumatic medications

NSAIDs Methotrexate Sulphasalazine Leflunomide Anti-TNF therapy


+
Baseline BP, oedema, K , Chest X-ray, FBC, LFT Chest X-ray, Chest X-ray, hepatitis B
creatinine hepatitis B & C serology, hepatitis B & C & C serology; screen for
FBC, LFT, chest serology, FBC, LFT, latent TB as per local
auscultation chest auscultation guidelines
Periodic K+ and creatinine at FBC, LFT at four- to six- FBC, LFT monthly 3, FBC, LFT at four- to As clinically indicated
one week; BP and weekly intervals; then three-monthly six-weekly intervals;
oedema at each chest auscultation at chest auscultation at
clinical visit each clinical visit each clinical visit
BP, blood pressure; FBC, full blood count/screen; K+, serum potassium; LFT, liver function tests; TB, tuberculosis.

Liaison between specialist and primary care practitioner


A close collaboration and correspondence flow is required. The management plan will be
developed between Joan and her specialist, but much of its implementation will be with
her general practitioner. They will need to consider the impact that any new illness will
have on her RA management, and also the impact of her RA management on intercurrent
illness as well as preventative activities such as vaccinations. Common points of ambiguity
are who will undertake prescribing and who will monitor and communicate with Joan
about any abnormalities detected on periodic screening. Joan should be encouraged to
attend her general practitioner regularly and this eventually should allow alternating visits
between the two physicians, with specialist attendance four- to six-monthly.

Recent Developments
1 Detection of antibodies to cyclic citrullinated peptides (CCP) has a sensitivity similar
to RF, but specificity is much higher in the range of 95%99%.2,3 In patients with
early undifferentiated arthritis, the presence of anti-CCP antibodies has an odds
03-PS Rheumatology-cpp:03-PS Rheumatology-ppp.QXD 18/3/08 15:26 Page 80

80 03 Rheumatoid Arthritis

ratio of 38 for subsequent development of RA, and is an adverse prognostic marker


for both clinical disease and radiological progression. Repeated measurement
currently has no clinical utility, with therapeutic intervention leading to only modest
reduction in levels that do not correlate with clinical indicators.
2 Anti-TNF therapy appears to protect against joint damage but may have little
influence on systemic inflammatory markers such as ESR and CRP.4 High-sensitivity
CRP measurement can generate specific values lower than the previous threshold in
RA subjects with a normal CRP; subjects with high-sensitivity CRP values 28 mg/l
had higher disease activity scores, less remission rates and greater depression and
helplessness than those with values <2 mg/l. High-sensitivity CRP also performs
consistently better than ESR at predicting other disease activity variables.5 A patients
joint count is, not surprisingly, a good marker of activity and subsequent prognosis.
In subjects with early arthritis of less than one years duration, a joint count of
greater than seven actively diseased joints accurately predicted subsequent
performance and function.6
3 Detailed radiological imaging and scoring has been undertaken in trials of biological
therapy, which suggest that progression can be halted and that joint erosions can
mature with regrowth of cortical bone and infilling. Plain radiographs of the hands
and feet remain quick and affordable imaging modalities, but erosions can be
detected by more sensitive imaging techniques around 18 months before they are
detected on plain films. Imaging modalities that can be used include ultrasound,
MRI and computed tomography (CT). Most erosions at the wrist will be detected by
both CT and MRI, although CT will detect more erosions particularly at the
metacarpal bases, possibly because of its ability to delineate cortical bony margins.7
High-resolution ultrasound requires good apposition of the probe and an ability to
access the bone surface. As a consequence, wrist examination is suboptimal, with
MCP joints 2 and 5 exhibiting better correlation with MRI than ultrasound of
MCP 3 and 4, which can only be accessed in two planes.8

Conclusion
A clear and positive approach is needed as you discuss the investigations that will assist in
confirming the diagnosis, and in determining disease activity and its current impact on
function. The shared care between specialist and primary care physician is aimed at inhib-
ition of the inflammatory process and therefore halting the local and systemic impact of
the disease. Involving the patient in the goal setting at the start is helpful in gaining their
understanding of the strategies you are using to bring the disease under control. A clear
goal of the absolute minimum number of tender and swollen joints, lowering of inflam-
matory markers and improvement in a patients quality of life is critical. Antirheumatic
medications are highly effective in bringing disease under control, and that should always
be the opening when discussing any new medications: to clearly put the benefits so that
this can be weighed against the potential adverse events, which themselves are discussed
in the context of how to minimize them and how to detect them early. RA remains a
chronic disorder that, while not curable, is eminently treatable with an increasing range
of options and flexible delivery systems to maintain a high quality of life in nearly all
patients.
03-PS Rheumatology-cpp:03-PS Rheumatology-ppp.QXD 18/3/08 15:26 Page 81

15 Laboratory and imaging investigations 81

Further Reading
1 Liou LB. Different monocyte reaction patterns in newly diagnosed, untreated rheumatoid
arthritis and lupus patients probably confer disparate C-reactive protein levels. Clin Exp
Rheumatol 2003; 21: 43744.
2 Riedemann JP, Muoz S, Kavanaugh A. The use of second generation anti-CCP antibody
(anti-CCP2) testing in rheumatoid arthritis a systematic review. Clin Exp Rheumatol
2005; 23 (5 Suppl 39): S6976.
3 Zendman AJW, van Venrooij WJ, Pruijn GJM. Use and significance of anti-CCP
autoantibodies in rheumatoid arthritis. Rheumatology 2006; 45: 205.
4 Smolen JS, van der Heijde DM, St Clair EW et al. Predictors of joint damage in patients
with early rheumatoid arthritis treated with high-dose methotrexate with or without
concomitant infliximab: results from the ASPIRE trial. Arthritis Rheum 2006; 54: 70210.
5 Dessein PH, Joffe BI, Stanwix AE. High sensitivity C-reactive protein as a disease activity
marker in rheumatoid arthritis. J Rheumatol 2004; 31: 10957.
6 Gerber LH, Furst G, Yarboro C, el-Gabalawy H. Number of active joints, not diagnosis, is
the primary determinant of function and performance in early synovitis. Clin Exp
Rheumatol 2003; 21 (5 Suppl 31): S65-70.
7 Perry D, Stewart N, Benton N et al. Detection of erosions in the rheumatoid hand; a compara-
tive study of multidetector computerized tomography versus magnetic resonance scanning. J
Rheumatol 2005; 32: 25667.
8 Szkudlarek M, Klarlund M, Narvestad E et al. Ultrasonography of the metacarpophalangeal
and proximal interphalangeal joints in rheumatoid arthritis: a comparison with magnetic res-
onance imaging, conventional radiography and clinical examination. Arthritis Res Ther 2006;
8; R52.
03-PS Rheumatology-cpp:03-PS Rheumatology-ppp.QXD 18/3/08 15:26 Page 82

82 03 Rheumatoid Arthritis

P R O B L E M

16 Managing Rheumatoid Arthritis at


Onset

Case History
Jason, aged 48 years, has returned for his second visit with suspected rheumatoid arthritis
(RA). He has a three-month history of increasing pain involving his hands, wrists and feet.
He is a non-smoker. You note active synovitis affecting most of his proximal
interphalangeal joints and metacarpophalangeal joints, both wrists and his
metatarsophalangeal joints. Erythrocyte sedimentation rate (ESR) is 63 mm/h, serum
C-reactive protein (CRP) 26 mg/l. He has strongly positive rheumatoid factor (RF) and
elevated anti-cyclic citrullinated peptide (anti-CCP) antibodies.
What medications would you recommend to Jason?
Will providing Jason with educational material and support impact on his RA?

Background
To impact on the morbidity of RA, long-term management of patients is required, based
not only on drug treatment, but also on non-pharmacological approaches such as phys-
iotherapy and psychosocial support. The pharmacological treatment of RA is still prob-
lematical and as yet there are no reliably curative or disease-remitting therapies, although
considerable gains have been made recently with the advent of biological therapies.
However, RA often progresses to disability. It is hoped that the full application of the
older disease-modifying antirheumatic drugs (DMARDs) together with the newer
DMARDs and biological agents can improve this outlook. First-line pharmacological
therapy for all of the inflammatory arthritic diseases includes the non-steroidal anti-
inflammatory drugs (NSAIDs). These are useful symptomatically but have no clear effect
on the progression of RA, with virtually all patients requiring additional treatment with a
DMARD.

Omega-3 oil supplementation


Inflammation contributes to a range of acute and chronic human diseases and is charac-
terized by the production of inflammatory cytokines, arachidonic acid-derived
eicosanoids (prostaglandins, thromboxanes, leukotrienes), reactive oxygen species and
adhesion molecules. Long-chain omega-3 fatty acids eicosapentaenoic acid and

Atlas Medical Publishing Ltd


03-PS Rheumatology-cpp:03-PS Rheumatology-ppp.QXD 18/3/08 15:26 Page 83

16 Managing rheumatoid arthritis at onset 83

docosahexaenoic acid have anti-inflammatory properties when taken in doses greater


than 2.7 g daily and are of proven benefit in RA, providing reduced morning stiffness,
decreased tender joint count and a lessened need for NSAIDs.
At sufficiently high intakes, long-chain n-3 polyunsaturated fatty acids (PUFAs), as
found in oily fish and fish oils, decrease production of inflammatory eicosanoids,
cytokines and reactive oxygen species and decrease the expression of adhesion molecules.
Long-chain n-3 PUFAs act both directly (e.g. by replacing arachidonic acid as an
eicosanoid substrate and inhibiting arachidonic acid metabolism) and indirectly (e.g. by
altering the expression of inflammatory genes through effects on transcription factor
activation). Long-chain n-3 PUFAs also give rise to a family of anti-inflammatory medi-
ators termed resolvins. The required dose of omega-3 fish oil can be taken using a large
number of fish oil capsules (more than nine standard fish oil capsules containing oil from
eviscerated fish bodies [30% omega-3] daily); however, the cost can be prohibitive. A less
expensive alternative is to take about 1015 ml of bottled fish body oil daily. The palata-
bility and hence long-term acceptance of fish oil can be improved by using a two-glass
technique. This involves floating the dose of fish oil on 30 ml of fruit or vegetable juice
in a small glass. The contents are swallowed without contact with the lips, avoiding the
fish oil taste. This is followed by slowly sipping a further 40 ml of juice (from a separate
glass) to rinse the mouth. Reflux of the fish oil can be avoided by taking it immediately
before a solid meal and without any extra fluid to encourage mixing of the fish oil
with the food and its transport from the stomach into the small intestine. Passage of the
oil from the stomach can also be facilitated by lying on the left side for 15 minutes after
the dose, or by dividing the daily dose and having each half immediately before the
morning and evening meals.1

DMARDs
A recent change in the use of DMARDs is that they are generally commenced as soon as a
diagnosis of an inflammatory arthritis is made, in order to have the maximal effect on
slowing the progress of the disease.2 The rationale for early initiation of therapy is the
demonstration of rapid functional decline, and evidence that many of the deleterious
effects of RA, such as erosions of bone, occur within the first one to two years of disease.
This approach contrasts to the older disease-management regimens in which DMARDs
were only commenced on the appearance of bony erosions on joint X-rays.
Many patients with RA and other inflammatory arthritic diseases are also adminis-
tered intra-articular and/or low-dose oral corticosteroids. They are frequently adminis-
tered as bridging therapy until the effect of a DMARD is fully established. In some
patients they are used long term with a DMARD, in order to satisfactorily suppress the
disease. Corticosteroids may also be used alone for disease control in the elderly if this
can be achieved with a daily dose equivalent of 7.5 mg prednisolone or less.

Biological agents
These are inhibitors of either tumour necrosis factor (TNF) or interleukin-1. These drugs
produce rapid control of RA in a high percentage of cases, and joint destruction may be
halted and even reversed in some patients. There are, however, potential problems with
the biological agents, particularly the activation of infections, as well as very high finan-
cial costs.
03-PS Rheumatology-cpp:03-PS Rheumatology-ppp.QXD 18/3/08 15:26 Page 84

84 03 Rheumatoid Arthritis

The most recent strategy to optimize treatment of RA has been the introduction of ini-
tial combination therapy. This approach is based on the experience of oncologists, with
the combination of different mechanisms of action leading to additive or synergistic
effects without significantly increasing toxicity. Studies in which one arm receives inter-
mediate- to high-dose corticosteroids have demonstrated a clinical superiority that
becomes less noticeable, however, as the steroid dose reduces or is withdrawn.
Interestingly, in these studies a radiographic benefit does continue to be seen for several
years, suggesting that the initial, rapid clinical and laboratory improvement does retard
or halt the structural damage that is otherwise occurring.3
The first-line choices of a single agent in RA consist of methotrexate, leflunomide and
sulphasalazine. Methotrexate is currently the gold standard DMARD, with doses acceler-
ated to achieve 20 mg/week within 68 weeks of commencement, with folic acid supple-
mentation of 5 mg/week. Objective outcomes need to be recorded including swollen and
tender joint counts, inflammatory markers (ESR, CRP) and a quality of life/functional
index. Failure to achieve remission of disease activity after a maximum of 34 months of
therapy should result in an escalation or switching of therapy. An algorithm of treatment
is suggested in Figure 16.1.

Patient education
Patient education is thought to be beneficial in helping patients cope with their disease
and cooperate with its complex management. The focus is to teach patients to adjust
their daily activities as dictated by disease symptoms. In addition to teaching patients
what they should do, patients are also instructed on how to approach situations and to
make adjustments that are appropriate for each individual and their needs. However, it is
not clear which educational interventions are most effective in improving health status
for patients with chronic disease. A systematic review of randomized controlled trials
focused on the effects of patient education on pain, functional disability, joint counts,
patient and physician global assessment, affect scores and measures of acute phase reac-
tants.4 The quality of studies identified was not very high, with issues of randomization
and concealment impacting adversely on their quality. At first follow-up there is a small
but significant beneficial effect, particularly for functional disability. The magnitude of
the effect is small, however, similar to that gained from the use of the weaker DMARDs.
The duration of effect is also brief, being lost at follow-up 314 months later. Therefore
strategies to preserve and extend the initial beneficial effects of education over a longer
time need to be explored.

Recent Developments
1 The clinical response to treatment of RA can be assessed by the proportion of
patients who have a 70% or greater improvement according to seven clinical and
laboratory measures of disease activity; this is designated as having an American
College of Rheumatology (ACR)-70 response. ACR-70 responses were found in
19%21% of patients receiving methotrexate monotherapy in trials in early RA
but in 33%40% of those receiving TNF inhibitors with methotrexate. In
established RA, ACR-70 responses occurred in fewer than 5% of patients
receiving methotrexate monotherapy but in 10%27% of those receiving the
03-PS Rheumatology-cpp:03-PS Rheumatology-ppp.QXD 18/3/08 15:26 Page 85

16 Managing rheumatoid arthritis at onset 85

Establish RA diagnosis
or
RF 1 unclassified oligoarthritis/polyarthritis
or
RF 2 unclassified oligoarthritis/polyarthritis .3 months
duration

Document baseline disease activity

Patient education on disease and its therapy


Commence analgesia, NSAID as needed
Omega3 oil supplementation
Consider low-dose steroid
Start DMARD and safety monitoring

Methotrexate 2025 mg/wk


plus folate

Evaluate at 1012 weeks

Adequate response Inadequate response

Periodic review Combination therapy


Biological agents

Figure 16.1 Antirheumatic therapy in early RA. (Adapted with permission from the American College of
Rheumatology Subcommittee on Rheumatoid Arthritis Guidelines 20025 and Smolen et al. 2005.6)

combination. TNF inhibitors substantially reduce the erosive damage assessed on


X-ray and magnetic resonance imaging. They also decrease the self-assessed
disability measured using the Health Assessment Questionnaire and improve the
quality of life assessed using SF-36.7
2 Cigarette smoking remains the most important lifestyle risk factor and one that is
modifiable. Males are three times more likely to develop RA if they smoke, with the
risk increasing 13-fold in both sexes for heavy smokers (4150 pack-years). The
influence of smoking is substantially greater in those with a background HLA-DRB1
03-PS Rheumatology-cpp:03-PS Rheumatology-ppp.QXD 18/3/08 15:26 Page 86

86 03 Rheumatoid Arthritis

shared epitope genetic susceptibility. Stopping smoking can reduce this risk,
although the benefit is not seen until at least ten years after quitting, when the
increased risk halves.8

Conclusion
Early recognition of RA and prompt disease suppression with DMARDs and pred-
nisolone is the key to the initial improvement of patients quality of life and to minimiz-
ing subsequent progressive joint damage. Methotrexate retains its key role in the early
treatment of RA, and whilst there remains controversy over unequivocal demonstration
of the superiority of combination therapy, several studies do support the concept. Some
of the best evidence in support of combination therapy is that of TNF blockers when
combined with methotrexate, in which the combination is superior to monotherapy with
either agent with respect to clinical, functional and radiographic outcome. The strategy
of aiming for tight disease control with rapid switching of regimens if patients do not
improve when assessed against measurable and recorded outcome is also a key advance.
The literature also suggests that the addition of intermediate- to high-dose prednisolone
to DMARDs, whether as monotherapy or combination therapy, is beneficial if intro-
duced early in the course of RA.

Further Reading
1 Cleland LG, James MJ, Proudman SM. Fish oil: what the prescriber needs to know.
Arthritis Res Ther 2006; 8: 20210.
2 Nell VP, Machold KP, Eberl G, Stamm TA, Uffmann M, Smolen JS. Benefit of very early
referral and very early therapy with disease-modifying anti-rheumatic drugs in patients
with early rheumatoid arthritis. Rheumatology 2004; 43: 90614.
3 Smolen JS, Aletaha D, Keystone E. Superior efficacy of combination therapy for
rheumatoid arthritis: fact or fiction? Arthritis Rheum 2005; 52: 297583.
4 Riemsma RP, Taal E, Kirwan JR, Rasker JJ. Systematic review of rheumatoid arthritis
patient education. Arthritis Rheum 2004; 51: 104559.
5 American College of Rheumatology Subcommittee on Rheumatoid Arthritis Guidelines.
Guidelines for the management of rheumatoid arthritis. Arthritis Rheum 2002; 46: 32846.
6 Smolen JS, Aletaha D, Machold KP. Therapeutic strategies in early rheumatoid arthritis.
Best Pract Res Clin Rheumatol 2005; 19: 16377.
7 Scott DL, Kingsley GH. Tumor necrosis factor inhibitors for rheumatoid arthritis. N Engl J
Med 2006; 355: 70412.
8 Oliver JE, Silman AJ. Risk factors for the development of rheumatoid arthritis. Scand J
Rheumatol 2006; 35: 16974.
03-PS Rheumatology-cpp:03-PS Rheumatology-ppp.QXD 18/3/08 15:26 Page 87

17 Evaluating the response to treatment 87

P R O B L E M

17 Evaluating the Response to Treatment

Case History
Charles and Dianna are a married couple who have been under your care for some years.
Charles is aged 43 years with a 20-year history of ankylosing spondylitis (AS), and Dianna
is aged 48 years and has had rheumatoid arthritis (RA) for eight years. You have noted
that they are increasingly restricted in their activities. As part of your ongoing
commitment to care, you want to implement a method of recording the impact of your
treatment.
What measures are available for monitoring disease activity?
How can function and quality of life be quantified?

Background
The benefit of any therapeutic intervention must outweigh the cost, with cost most easily
measured in monetary terms. More difficult to measure is the cost to the individual in
terms of convenience of use, monitoring and adverse effects both actual and potential.
Similarly it can often be difficult to measure benefit, and this particularly applies to
rheumatic disease, where a hard end-point such as mortality is rarely the outcome mea-
sure. Patients with rheumatic disease experience pain and impaired function, but over
time they adapt to these circumstances and symptom-reporting alters, so that quietly and
without complaint function is lost. RA and AS are contrasting diseases. In Table 17.1 the
demographic and clinical characteristics are compared, providing insight into the differ-
ent outcome measures that need to be evaluated for each disease process.

Table 17.1 Characteristics of RA and AS

Rheumatoid arthritis Ankylosing spondylitis

Gender F>M M>F


Age Middle age onwards Late teens onwards
Peripheral Predominant with involvement of hand Unusual, with occasional involvement of hip and
disease and feet shoulder
Spinal disease Cervical inflammation Ascending spinal restriction with impaired posture
F, female; M, male.

Atlas Medical Publishing Ltd


03-PS Rheumatology-cpp:03-PS Rheumatology-ppp.QXD 18/3/08 15:26 Page 88

88 03 Rheumatoid Arthritis

Outcome measures vary according to the situation and purpose for which they are
being evaluated, often differing significantly when being used in routine clinical practice
and research trials. A pragmatic approach is required if they are to be utilized by the aver-
age practitioner who either does not have specialist knowledge or lacks specialist equip-
ment. In broad terms, the outcomes of interest in rheumatic disease are:
Laboratory evidence of inflammation recorded as erythrocyte sedimentation rate or
serum C-reactive protein
Clinical joint inflammation recorded as the number of tender or swollen joints on
examination
Clinical examination of spinal function
Patients self-reported experience of pain, fatigue or functional consequence of their
disease
Radiological progression

Laboratory markers of disease activity


Laboratory investigations, by themselves, can be misleading. It is not unusual for a per-
son to have normal inflammatory markers despite a small number of inflamed peripheral
joints; similarly, elevated markers may be caused by incidental illness such as urinary
tract infection. RA, with its distinct synovial inflammation, often leads to elevated
inflammatory markers, whereas the spondyloarthropathies (in which enthesial inflam-
mation at the tendonperiosteal interface predominates) have much lower and often
normal values.

Physical evaluation of disease activity


Counting a patients tender and swollen joints is a quick and easy way of both examining
the patient and evaluating the extent of their disease activity. While you could examine
every joint, examining the easy-to-access 18 joints of upper limbs and knees has been
shown to be a valid measure of disease activity (Figure 17.1). The joints examined are the
interphalangeal joint of the thumb, proximal interphalangeal joint of the fingers, the
metacarpophalangeal joints, wrists, elbows, shoulders and knees totalling 18 joints. The
method of examination is to palpate the joint in two directions applying sufficient pres-
sure to blanch the distal third of the nail bed of your thumb to determine separately the
presence of swelling and tenderness. The tender joint count (TJC) and swollen joint
count (SJC) can be recorded both within the clinical record and in correspondence to
other practitioners (i.e. SJC = 4/18, TJC = 6/18). This allows a quick analysis for trends
over time and also provides an indicator of absolute disease severity. A count of six to
eight indicates moderate severity, sufficient for entry into many trials of new therapeutic
agents.
Assessing spinal function requires a fuller examination and exposure of the patient but
does allow serial evaluations, which are valid in detecting progressive loss of function and
potentially act as patient motivation for their ongoing exercise programme. An index of
spinal movement the British Ankylosing Spondylitis Metrology Index (BASMI) pro-
vides a score of relative severity for any outcome and allows serial comparison. Tables
17.2 and 17.3 describe both the scoring system and method of measurement relating to
03-PS Rheumatology-cpp:03-PS Rheumatology-ppp.QXD 18/3/08 15:26 Page 89

17 Evaluating the response to treatment 89

Figure 17.1 Mannequin showing the 28 easily accessible joints for evaluation in routine practice
(highlighted).

Table 17.2 BASMI scoring system

Score 0 1 2

Tragus to wall (cm) <15 1530 >30


Lumbar flexion (cm) >4 24 <2
Cervical rotation (degrees) >70 2070 <20
Lumbar side flexion (cm) >10 510 <5
Intermalleolar distance (cm) >100 70100 <70
Adapted with permission from Irons and Jeffries 2004.1
03-PS Rheumatology-cpp:03-PS Rheumatology-ppp.QXD 18/3/08 15:26 Page 90

90 03 Rheumatoid Arthritis

Table 17.3 Obtaining BASMI measurements

Measure Starting position Method

Tragus to wall Standing with back to wall, outer edges of Patient draws chin in as far as possible. Examiner
feet 30 cm apart measures distance between tragus and the wall with a
rigid ruler
Lumbar flexion Outer edges of feet 30 cm apart. Examiner Patient flexes forward from waist with knees fully
(modified Schober) marks midpoint of line level with iliac crests; extended. Distance between upper and lower marks is
a second point is marked 10 cm above and a measured, and then 15 cm is subtracted
third 5 cm below
Cervical rotation Patient supine on bed Patient rotates head as far as possible. Angle between
(performed for neutral position and rotation is measured with
each side) goniometry
Lumbar side flexion As for tragus to wall Measure from tip of middle finger to floor with rigid 1 m
(performed for ruler. Patient side-flexes without forward flexion or
each side) knee flexion. Remeasure from tip of middle finger to floor.
Calculate difference between two measurements
Intermalleolar distance Patient lies supine on floor Keeping knees straight, patient moves legs as far apart
as possible. Distance between medial malleoli is
measured
Modified with permission from Irons and Jeffries 2004.1

AS. The ability to comment to a patient on the progress of their disease is important, and
can also be a motivating force for compliance with interventions, particularly exercise
and physiotherapy.

Evaluating quality of life


As medical practitioners, it is readily acknowledged that for many patients physicians are
not able to cure them of their disease but aim instead to improve their quality of life.
The latter has many domains, and in practice physicians are primarily aiming to influ-
ence those areas related to a persons health that impact on their global well-being so-
called health-related quality of life. Even within this more circumscribed area there are
instruments that measure this in a generic way, thus allowing the same instrument to be
used in different disease states providing comparisons that may be used to support health
resource distribution decisions. Disease-specific instruments also exist and often provide
greater insights into an individual disease and may respond quicker to interventions for
that illness. Some self-reported questionnaires available for RA and AS are listed in Table
17.4, and can be accessed via internet search.

Table 17.4 Self-reported health-related quality of life questionnaires

RA AS

Health Assessment Questionnaire (HAQ) AS Quality of Life (ASQOL)


RA Quality of Life (RAQOL) BAS Disease Activity Index (BASDAI)
BAS Functional Index (BASFI)
BAS, British Ankylosing Spondylitis.
03-PS Rheumatology-cpp:03-PS Rheumatology-ppp.QXD 18/3/08 15:26 Page 91

17 Evaluating the response to treatment 91

An important domain to consider in patients with RA and AS is that of fatigue. Unlike


normal tiredness, fatigue is chronic and typically is not related to over-exertion and is not
relieved by rest. Fatigue is a frequent complaint in many chronic diseases and can be
assessed via visual analogue scales and also by dedicated questionnaires. As an outcome it
has a strong relationship with the other joint symptoms in RA and AS, and is negatively
influenced by sleep disorders. Regular physical activity helps alleviate fatigue and
improves quality of life.

Recent Developments
1 There are conflicting data regarding survival in patients with AS. A broad
population-based study from Rochester, Minnesota, showed no difference in
mortality between men with AS and the general population. Other studies
indicate that mortality amongst AS patients seen in specialist referral centres is
higher standardized mortality ratio of 1.7 with a linear relationship between
mortality and disease severity. Furthermore, mortality in AS may be related to
disease duration. Mortality risk ratios relative to the general male population are
4.0 for gastrointestinal disease, 1.3 for circulatory diseases and 1.2 for
cerebrovascular disease. Smoking is a dose-dependent risk factor for the
development of atherosclerosis, but it is unknown whether or not AS patients
smoke more than the general population. However, smoking is associated with
worse clinical, functional and radiological outcomes in AS.2
2 The inflammatory processes in AS may affect various structures of the heart; the
most characteristic conditions are conduction defects and aortic insufficiency and,
less commonly, pericarditis, cardiomyopathy and mitral valve disease. Conduction
disturbances may occur in AS due to inflammation and fibrosis of the
interventricular septum thereby affecting the atrioventricular node. Aortic
insufficiency develops because the inflammatory process affects the aortic wall
directly behind and above the sinuses of Valsalva. This leads to scarred, fibrotic,
thickened and shortened aortic valve cusps and to a dilated aortic root, resulting in
aortic regurgitation. The occurrence of conduction disturbances in patients with AS
varies from 1% to 33%, and of aortic insufficiency from 1% to 10%, and increases
with age, disease duration and the presence of peripheral arthritis. Impaired
ventricular relaxation leading to diastolic dysfunction is being increasingly
recognized in AS, even in subjects with clinically mild disease. Yildirir and colleagues
examined 88 AS patients and 31 healthy controls and found that diastolic function
of the left ventricle was significantly disturbed in AS patients compared to controls. 3
3 It is now recommended that three domains be assessed in the clinic for
therapeutic responses in RA: patient-reported measures of physical function
and/or global disease activity; physician-reported measures of physical function
and/or global disease activity; and imaging of the hands and/or feet on a biannual
basis.4 In practice, a 10 cm visual analogue scale can be used by both patient and
physician for the global disease activity. Radiographic images should be compared
over time, and may require the assistance of your local radiologist. Measures of
improvement for individually relevant physical activities need to be defined for
each patient.
03-PS Rheumatology-cpp:03-PS Rheumatology-ppp.QXD 18/3/08 15:26 Page 92

92 03 Rheumatoid Arthritis

Conclusion
A range of practical and relevant measures exists for measuring meaningful outcomes in
patients with arthritis particularly AS and RA. These neatly combine laboratory, objec-
tive clinical and subjective physician and patients measurements to provide a multido-
main assessment that provides information on current disease activity and its impact on
the persons quality of life, and allows you to evaluate the utility of interventions that the
patient is receiving.

Further Reading
1 Irons K, Jeffries C. The Bath Indices. Outcome Measures for use with Ankylosing Spondylitis
Patients. National Ankylosing Spondylitis Society, East Sussex, United Kingdom, 2004.
http://www.nass.co.uk/bath_indices.htm [Accessed November 2007]
2 Doran MF, Brophy S, MacKay K, Taylor G, Calin A. Predictors of longterm outcome in
ankylosing spondylitis. J Rheumatol 2003; 30: 31620.
3 Yildirir A, Aksoyek S, Calguneri M, Oto A, Kes S. Echocardiographic evidence of cardiac
involvement in ankylosing spondylitis. Clin Rheumatol 2002; 21: 12934.
4 Zatarain E, Strand V. Monitoring disease activity of rheumatoid arthritis in clinical practice:
contributions from clinical trials. Nat Clin Pract Rheumatol 2006; 2: 61118.

P R O B L E M

18 Pregnancy and Rheumatic Diseases

Case History
Jane is a 32-year-old lawyer who has been treated for rheumatoid arthritis (RA) for the
past four years. Her symptoms are well controlled on methotrexate 20 mg/week and
diclofenac 50 mg bd as needed. She and her partner would like to start a family. She
wishes to discuss the effect a pregnancy would have on her arthritis and vice versa.
What are the beneficial and adverse effects of pregnancy on RA?
Does she need to change her treatment?
Will RA affect her chances of becoming pregnant?

Atlas Medical Publishing Ltd


03-PS Rheumatology-cpp:03-PS Rheumatology-ppp.QXD 18/3/08 15:26 Page 93

18 Pregnancy and rheumatic diseases 93

Background
RA tends to improve in pregnancy, with 80% of patients enjoying virtually complete
remission. The disease remains active in the remainder but only worsens in a small
minority. Most causes of inflammatory polyarthritis (IP) follow this pattern. A small
proportion of patients with systemic lupus erythematosus (SLE) experience flares during
pregnancy. These are now less common with more effective therapy before pregnancy
and with planning of pregnancy. Previously, pregnancy flares of SLE may have been
largely due to withdrawal of drugs in patients with disease that was still quite active.
Prolactin is a pro-inflammatory hormone and may contribute to some cases of IP wors-
ening in pregnancy and in the post-partum.
RA is an autoimmune inflammatory disease in which CD4+ T lymphocytes have a
prominent role in the underlying pathological process. CD4+ T cells can be divided into
two subsets: T helper 1 cells (Th1) and T helper 2 cells (Th2). Th1 cells produce inter-
feron-g, interleukin (IL)-2, tumour necrosis factor (TNF)-b and IL-12. Activation of this
cell type and dominance of the Th1 cytokine pattern facilitates cell-mediated immunity.
The Th2 cell pattern leads to secretion of IL-4, IL-10 and IL-13, which facilitates humoral
immunity. Of the two T-cell patterns, the Th1 cell-mediated immunity pattern is more
consistent with active autoimmune diseases. In general terms, pregnancy is associated
with a shift towards a Th2 humoral pattern of immunity.1,2 Furthermore, circulating
antagonists to Th1-type cytokines are increased, both during normal pregnancy and in
pregnancies of patients with autoimmune diseases. These antagonists include interleukin-
1 receptor antagonist (IL-1ra) and the soluble receptors for TNF and IL-6 (sTNFR and
sIL-6R). The cytokine pattern in pregnant patients with rheumatic diseases does not differ
markedly from that of normal pregnant women. Increased levels of oestrogen, androgens
and progesterone may all contribute to the shift away from the pre-pregnant Th1 pattern,
and this leads to decreased cell-mediated immunity. Impairment of antigen-presenting
function and neutrophil activity may also be relevant, as these are the initial steps that lead
to normal activation of lymphocytes. It is noteworthy that remission of inflammatory
conditions is more likely if the mother and foetus are dissimilar in HLA determinants,
suggesting that the more active pregnancy-related immunosuppression occurs in women
where there is the most marked HLA disparity between themselves and their foetus.

Fertility in rheumatic diseases


Women with RA are more likely to be nulliparous or to have a smaller family than their
non-RA counterparts. There is, however, no evidence of a general decrease in fertility
among patients with RA.3 Their general debility and drug treatment (including steroids)
may decrease sexual desire and the frequency of intercourse. They may also be concerned
about potential effects of disease-modifying antirheumatic drugs (DMARDs) on their
baby, and about their ability to deal with pregnancy and to care for a young child.
Concerns about worsening symptoms during pregnancy and afterwards may also play a
part. Paradoxically, while normal women often experience marked decrease in physical
function and increase in symptoms (including pain) during pregnancy, many women
with RA feel better and experience less pain. There is an increased rate of fetal loss related
either to the underlying disease states or to drug treatments, including non-steroidal
anti-inflammatory drugs (NSAIDs).
03-PS Rheumatology-cpp:03-PS Rheumatology-ppp.QXD 18/3/08 15:26 Page 94

94 03 Rheumatoid Arthritis

Cyclophosphamide is rarely needed in the management of RA but is used in the man-


agement of severe SLE, particularly for nephritis or vasculitis. Its use in women over the
age of 35 years generally renders them infertile, with its effects on younger women being
more variable. The effects of cyclophosphamide on the ovary can be diminished by use of
a high-oestrogen oral contraceptive preparation or with gonadotrophin-releasing hor-
mone agonists or antagonists. Alternatively, oocyte recovery with cryopreservation can
be used. In men, sperm count is decreased with methotrexate, cyclophosphamide or sul-
phasalazine. The count usually recovers when the drug is stopped but, if there is concern,
sperm can be stored. Fertility treatment is possible for women with RA and SLE care
should be taken with SLE as hormone treatments may increase disease activity.

Disease during pregnancy


The risk of increased activity of SLE may be less than was once thought. With modern
treatment, more women start pregnancy with the disease in remission. Also, we have
become comfortable with using some of the disease-modifying drugs in pregnancy so
that the disease can be effectively treated throughout. Flares of lupus during pregnancy
are generally mild. However, disease activity in pregnancy increases risk of intrauterine
death, fetal growth retardation, pre-eclampsia and premature rupture of membranes
leading to early delivery. Lupus flares in pregnancy are less likely if the disease is well con-
trolled at conception. Unplanned pregnancies, however, can lead to worsening of skin
rash, increased joint symptoms, mucosal ulceration, anaemia and thrombocytopenia,
hypertension and proteinuria, renal casts and increasing levels of anti-double-stranded
DNA (anti-dsDNA) antibodies with decreased complement (C3 and C4).
The risk of thromboses, particularly venous thromboses, is increased two- to four-
fold during normal pregnancy and in the six weeks post-partum, and is particularly high
after Caesarean section. The risk of both venous and arterial thrombosis is increased in
women with antiphospholipid syndrome (APS), and all women with lupus or a history of
pregnancy loss should be screened by testing for lupus anticoagulant and anti-cardiolipin
antibodies. APS is present in 37% of patients with SLE, but can occur in other auto-
immune disease and in the absence of autoimmune disease.4 APS also increases the risk
of pre-eclampsia, premature labour and HELLP (Haemolytic anaemia, Elevated Liver
enzymes, Low Platelets).
The effect of pregnancy on the spondyloarthropathies is more variable the periph-
eral manifestations (arthropathy and uveitis) tend to improve, while spinal symptoms
often worsen. Vasculitides are relatively rare and there are few systematic studies during
pregnancy. Like SLE, these conditions may flare up (particularly in the first and second
trimesters). Patients with Behets disease are at increased risk of thrombosis in preg-
nancy. Lupus nephritis and renal vasculitis can be difficult to distinguish from pre-
eclampsia. The latter is more likely to be associated with hypertension and improves
rapidly following delivery. Systemic sclerosis does not appear to be at particular risk of
progressing during pregnancy.

Effect on the foetus


Transplacental passage of immunoglobulin G (IgG) antibodies, including anti-SSA/Ro
and anti-SSB/La, can lead to neonatal lupus syndrome and congenital heart block (CHB).
The former can cause a transient photosensitive rash, hepatic dysfunction and bone
03-PS Rheumatology-cpp:03-PS Rheumatology-ppp.QXD 18/3/08 15:26 Page 95

18 Pregnancy and rheumatic diseases 95

marrow depression. CHB occurs in the offspring of mothers who are anti-SSA/Ro
positive, and most infants born with this complication require a pacemaker before the age
of one year. It is rare, occurring in only 2% of infants of such mothers, and in 1 in 22 000
of the general population. Ideally, all young women with an inflammatory arthropathy
such as SLE or RA should be screened for lupus anticoagulant, anti-cardiolipin antibodies
and anti-SSA/Ro prior to them considering a family, or they should be tested early in their
pregnancy. The rash of neonatal lupus resolves over the first three to five months of life, as
the maternally derived IgG is cleared from the babys circulation.
SLE disease activity at conception and the presence of antiphospholipid antibodies are
the major determinants of fetal outcome. Fetal loss occurs in up to 30% of patients with
active lupus. All inflammatory arthropathies increase the risk of premature delivery.
However, with modern obstetric monitoring and neonatal management, the risk to the
child has been markedly decreased in recent years.

Drugs
None of the drugs used in IP (except simple analgesics) is entirely safe during pregnancy.
Methotrexate, cyclophosphamide, azathioprine, gold and cyclosporine should all be
stopped at least three and preferably six months before pregnancy. Leflunomide has a
very prolonged functional half-life (several months) due to its enterohepatic recircula-
tion and is also very teratogenic. Prior to its prescription to a young woman, it is advis-
able to discuss the time frame of possible pregnancies and to clearly state that pregnancy
should not occur within two years of ceasing the medication, unless the drug has been
washed out using cholestyramine and a plasma drug level obtained that confirms
elimination.
NSAIDs should be stopped if possible before pregnancy as they may increase risk of
early pregnancy loss. Later adverse effects on the infant include premature closure of the
ductus arteriosus with resulting pulmonary hypertension, cutaneous and intracranial
bleeding and impaired renal function leading to oligohydramnios. If NSAIDs are
required in pregnancy, older agents with a short half-life are best indomethacin,
ibuprofen and diclofenac. Corticosteroids are relatively safe in pregnancy if the dose is
kept below the equivalent of 20 mg/day prednisolone. If it is considered necessary to treat
mother and foetus, then dexamethasone or betamethasone should be used. Otherwise,
hydrocortisone, cortisone or prednisolone should be used; less than 10% of these are
delivered to the foetus (the remainder that crosses the placenta is inactivated by 11b-
hydroxysteroid dehydrogenase). Side effects of steroid therapy mainly affect the mother
and include hypertension, oedema, pre-eclampsia and gestational diabetes.
The most extensively documented teratogenic effect is with methotrexate, which can
also induce miscarriage even with normal foetuses. The drug may cause a range of con-
genital abnormalities including limb anomalies, craniofacial anomalies and a range of
nervous system anomalies including anencephaly and hydrocephalus.
Sulphasalazine has been extensively used throughout pregnancy in patients with both
RA and inflammatory bowel disease and is considered safe. Where immunosuppression
is required, azathioprine is a reasonable choice. The foetus cannot convert it to its active
metabolite, 6-mercaptopurine, and it is therefore relatively free of teratogenic effect. It
has, however, been associated with growth retardation and premature rupture of
membranes.
03-PS Rheumatology-cpp:03-PS Rheumatology-ppp.QXD 18/3/08 15:26 Page 96

96 03 Rheumatoid Arthritis

Recent Developments
1 In a national (United States) study with access to data from over four million
deliveries, Chakravarty and colleagues5 showed that, compared with the background
pregnant population, women with RA or SLE were more likely to develop a
hypertensive disorder, spend more time in hospital and to have a Caesarean delivery.
The prevalence of hypertensive disorders was highest in patients with SLE and was
comparable in these patients to the prevalence of hypertensive disorders in
gestational diabetes (GDM). There was increased risk of premature rupture of
membranes and of intrauterine growth retardation in both RA and SLE. The
Caesarean section rate was not as high as in patients with GDM.
2 There are fewer data available on Sjgrens syndrome (SS) in pregnancy than for RA
or SLE. A recent casecontrol study6 investigated reproductive problems in patients
with SS. Complaints of vaginal dryness were found twice as commonly as in controls
and affected over 50% of SS patients. The previously documented association
between SS and endometriosis was confirmed. Fertility was not decreased in SS, but
women chose to have fewer children because of their disease.

Conclusion
The patient should stop methotrexate at least three months before pregnancy, which
should be carefully planned wherever possible. Careful monitoring of the condition in
the months before pregnancy should be achieved, which will determine the level of ther-
apy required. Clearly the latter should be kept to a minimum before and during preg-
nancy. One would not expect a major reduction in fertility in the above patient.

Further Reading
1 Gordon C. Pregnancy and autoimmune diseases. Best Pract Res Clin Rheumatol 2004; 18:
35979.
2 stensen M, Frger F, Villiger PM. Cytokines and pregnancy in rheumatic disease. Ann NY
Acad Sci 2006; 1069: 35363.
3 Katz PP. Childbearing decisions and family size among women with rheumatoid arthritis.
Arthritis Rheum 2006; 55: 21723.
4 Tincani A, Bompane D, Danieli E, Doria A. Pregnancy, lupus and antiphospholipid syndrome
(Hughes syndrome). Lupus 2006; 15: 15660.
5 Chakravarty EF, Nelson L, Krishnan E. Obstetric hospitalizations in the United States for
women with systemic lupus erythematosus and rheumatoid arthritis. Arthritis Rheum 2006;
54: 899907.
6 Haga HJ, Gjesdal CG, Irgens LM, stensen M. Reproduction and gynaecological
manifestations in women with primary Sjgrens syndrome: a case control study. Scand J
Rheumatol 2005; 34: 458.
03-PS Rheumatology-cpp:03-PS Rheumatology-ppp.QXD 18/3/08 15:26 Page 97

19 Diet and arthritis 97

P R O B L E M

19 Diet and Arthritis

Case History
Michelle is 37 years old and has had rheumatoid arthritis (RA) for four years. She is
compliant with her medication and has achieved near-remission but does not like taking
tablets. She has a body mass index of 27.6 kg/m2 and is seeking advice on whether there
are any effective diets or supplements.
What is the effect of being overweight or obese on rheumatic diseases?
Are there pro- or anti-inflammatory components of the diet?
What simple, risk-free advice might she be given?
Is there a place for food supplements?

Background
Pharmacological measures to improve symptoms and drugs to modify disease activity
have transformed the management of rheumatic diseases. Dietary measures have a much
less certain place but emerging evidence is beginning to point to a role in disease preven-
tion in patients who are at high risk of developing the diseases, and in controlling symp-
toms or preventing relapse in patients who have diagnosed disease. There is increasing
pressure on health professionals to be knowledgeable about dietary factors as patients are
increasingly informed. The increased predisposition to osteoarthritis amongst patients
who are either overweight or obese is now well documented, as is the importance of vari-
ous dietary components in RA.

Diet as a susceptibility factor for RA


The cause of RA is unknown but is likely to involve both genetic susceptibility and envir-
onmental factors such as diet, and may involve interplay of both factors. Available data
suggest that at least one-third of RA susceptibility is explained by environmental vari-
ation. The possibility that diet may play a role in both the onset and persistence of RA is a
frequently reported experience and there is a growing literature in support of this
hypothesis.1 A large prospective study was performed in the United Kingdom using RA
cases registered from the Norfolk Arthritis Register. A number of dietary factors emerged
as being of possible relevance.2 Low fruit and vitamin C intake was associated with a doub-
ling in the risk for RA, with the highest daily intake of the antioxidants b-cryptoxanthin
and zeaxanthin having the lowest risk for RA. It was suggested that one glass of freshly

Atlas Medical Publishing Ltd


03-PS Rheumatology-cpp:03-PS Rheumatology-ppp.QXD 18/3/08 15:26 Page 98

98 03 Rheumatoid Arthritis

squeezed orange juice per day would be sufficient to offer protection. A high intake of red
meat had a modest risk for RA, which is independent of any lowering of fruit and vege-
table intake, and may be explained by red meat providing a dietary source of omega-6
oils. The reported effects of decaffeinated/caffeinated coffee, tea or total caffeine con-
sumption are internally inconsistent and conflicting, making a real effect unlikely.2
Alcohol consumption and the risk of developing RA has only been studied in women.
The results are conflicting, at best suggesting a modest protective effect in women con-
suming the most alcohol.1

Fruit, vegetables and antioxidants


Monocytes, macrophages and granulocytes at the site of an inflammatory process gener-
ate reactive oxygen species. These, in turn, lead to increased production of inflammatory
mediators. This process, which is partly mediated by increased local nitric oxide produc-
tion, can potentially be inhibited by antioxidants, which scavenge reactive oxygen
species.3 Interestingly, in patients with RA, blood markers of antioxidant nutritional
status are lower than in normal controls. Fruit and vegetables are the main source of anti-
oxidants in the diet, including retinoids, vitamins C and E, carotenoids, selenium and
lutein. Lower intake of vitamin C is associated with increased risk of inflammatory
polyarthritis.4

Essential fatty acids and the Mediterranean diet


The Mediterranean diet is characterized by less red meat and more fish, with olive oil as
the primary fat source, an abundance of plant food and moderate wine consumption.
High levels of omega-3 fatty acids may protect against inflammation by substituting for
omega-6 fatty acids. Polyunsaturated fatty acids contain two or more double bonds, with
the omega-3 or omega-6 classification according to the site of the double bond proximal
to the methyl terminus. Mammals do not possess the enzyme necessary to introduce
double bonds at the 3 or 6 positions they are therefore essential fatty acids and must be
obtained from the diet. The Western diet is abundant in omega-6 fatty acids due to the
abundance of linoleic acid in soybean, safflower, sunflower and corn oil.5 Fish oil con-
tains the omega-3 fatty acids eicosapentaenoic acid (EPA) and docosahexaenoic acid
(DHA). These compete with arachidonic acid for cyclooxygenase (COX)-1, COX-2 and
lipoxygenase enzymes, and their metabolism leads to incorporation of anti-inflamma-
tory prostanoids and leukotrienes in cell membranes. Olive oil is rich in oleic acid, which
can be metabolized to eicosatrienoic acid, similar to omega-3 polyunsaturated fatty acid
from fish oil. Stamp et al.5 have extensively reviewed the role of dietary oils in RA. In
combination with non-steroidal anti-inflammatory drugs (NSAIDs)/aspirin, omega-3
oil produced a novel family of bioactive lipids known as resolvins, named for their anti-
inflammatory effect and tendency to resolve inflammation at the site of study. Fish oil
supplementation reduced expression of the antigen-presenting class II HLA-DR mol-
ecule on human monocytes, with a reduced ability to present antigen after culture with
EPA and DHA. The risk of RA has been documented to be lower in those who consume
higher amounts of olive oil or oily fish.6 These observations are consistent with several
randomized controlled trials showing modest antirheumatic benefit of omega-3 fatty
acid supplementation, including reduced early morning stiffness, tender joint count and
use of NSAIDs (Box 19.1).
03-PS Rheumatology-cpp:03-PS Rheumatology-ppp.QXD 18/3/08 15:26 Page 99

19 Diet and arthritis 99

Box 19.1 Effects of omega-3 oils on the manifestations of RA


Modest clinical improvement when used with standard pharmacotherapy
Deterioration in symptoms after discontinuation
Ability to decrease or cease NSAIDs and hence reduce associated gastrointestinal
and cardiovascular risk
Reduce sudden cardiac death

Dietary supplements and RA


In all studies of fish oils, the benefits were observed against a background of standard
therapy of NSAIDs, disease-modifying agents and immunomodulators. Collectively,
these studies have established that fish oil supplementation that delivers 2.6 g or more per
day of the long-chain fatty acids EPA plus DHA reduces symptoms after a latent period of
three months. The latency can be reduced with higher doses of fish oil. For anti-
inflammatory effect it is recommended to take 2.64.0 g of EPA and DHA per day. The
required dose of fish oil can be taken either as fish oil capsules or from bottled fish body
oil, with patient preference relating to cost and palatability. In addition to oil supple-
ments, patients should be encouraged to review their background diet so as to minimize
omega-6 oil intake and maximize omega-3 intake (Box 19.2).
Elemental diets provide food in the most basic forms i.e. free amino acids, medium
chain triglycerides and small sugars and are proposed as being hypoallergenic. General
benefit has not been evident in several trials. Vegetarian and vegan diets have been
reported to improve symptoms. Many of the studies commence with a fast with resultant
improvement in clinical and biochemical parameters, which is not always sustained in
the longer term when caloric intake in its modified form is reintroduced. Fasting has
been shown to reduce both clinical and laboratory markers of inflammation, possibly by
redirecting scarce resources away from inflammatory pathways and reducing disease
activity but compromising immune defences. It is important to ensure that sufficient
calories and nutrients are being provided to ensure well-being. Elimination diets seek to
remove foods that are putatively allergenic from the diet. Studies demonstrate that a pro-
portion of patients improve when putative allergenic foods are eliminated, with worsen-
ing of symptoms at some point when foods are sequentially reintroduced.

Box 19.2 Diet in rheumatoid arthritis


Diet as a risk factor
Low fruit, vitamin C, b-cryptoxanthin, zeaxanthin
High meat
Diet as a protective or therapeutic factor
High consumption of oily fish, olive oil, cooked vegetables
Conflicting data
Caffeine, decaffeinated coffee, tea
Vitamin D and dairy products
Doubtful benefit
Elemental diet
03-PS Rheumatology-cpp:03-PS Rheumatology-ppp.QXD 18/3/08 15:26 Page 100

100 03 Rheumatoid Arthritis

Recent Developments
1 Turmeric has been used for centuries in Ayurvedic medicine as a treatment for
inflammatory disorders including arthritis. In an animal model of arthritis, a
curcuminoid-containing turmeric extract demonstrated efficacy in preventing joint
swelling and destruction as determined clinically, histologically and by measurement
of bone mineral density.7 The putative mechanism of action was further elucidated
by analysis of turmerics effect on articular transcription factor activation and
microarray analysis of articular gene expression. Treatment in vivo prevented local
activation of nuclear factor-k-beta (NF-kB) and the subsequent expression of
NF-kB-regulated genes mediating joint inflammation and destruction, including
chemokines, COX-2 and receptor activator of NF-kB ligand (RANKL). Consistent
with these findings, inflammatory cell influx, joint levels of prostaglandin E2 and
periarticular osteoclast formation were all inhibited by turmeric extract. Other
researchers using curcumin have documented antiproliferative, anti-inflammatory
and immunosuppressive activities, as identified by inhibition of neutrophil
activation, synoviocyte proliferation and angiogenesis.8
2 Polyphenols are secondary metabolites of plants and are generally involved in the
defence mechanism against ultraviolet radiation and insects, but have anti-
inflammatory activity when consumed. Classified into four groups flavonoids,
stillbenes, lignas and phenolic acids these naturally occurring plant compounds
can have potent effects on reducing chronic disease in animal models.9 Flavanols
(one of the flavonoids) have been extensively studied, and there is an estimated daily
consumption of 2025 mg/day in the United States. The potential mechanisms of
their anti-inflammatory activities include inhibition of COX and lipoxygenase
enzymes, as well as nitric oxide synthase, and inhibition of the transcription factors
NF-kB and PPARs (peroxisomal proliferator-activated receptors). In collagen-
induced arthritis (a mouse model of RA, which is triggered by tumour necrosis
factor [TNF]-a) the oral administration of flavonoids improves the arthritis even
when the disease has become established.10 Dietary sources of flavonoids include tea,
red wine, fruits, vegetables and legumes. Flavanones are in citrus, isoflavones in soy
products, anthocyanidins in wine and bilberry, and flavans are in apples and tea.
Citrus flavonoids are found in citrus fruits, rutin in buckwheat, epigallocatechin
gallate in green tea and naringenin in grapefruit. Oligomeric proanthocyanidins are
found in grape seeds and skins. Quercetin is found in onions, tea and apples.
Polyphenols are the most abundant group of compounds in fresh tea leaves and are
found in green and black tea beverages as 30%42% and 3%10% of the total dry
matter, respectively. Many medicinal plants contain bioflavonoids, such as ginkgo
biloba, hawthorn and Chinese skullcap.

Conclusion
Does my diet affect my arthritis? is a question familiar to all rheumatologists and pri-
mary care practitioners. One-third to three-quarters of RA patients believe that food
plays an important role in their symptom severity, and up to half will have tried dietary
manipulation to improve their quality of life. The most consistent link between diet and
03-PS Rheumatology-cpp:03-PS Rheumatology-ppp.QXD 18/3/08 15:26 Page 101

19 Diet and arthritis 101

Table 19.1A Omega-3 friendly foods

FATS AND OILS


Use either canola, sunola or olive oil spread, mayonnaise and salad dressing instead of your usual oil products
Flaxseed oil can be used in salad dressings or mixed with sunola or olive oil and used in baking
BREADS AND CEREALS
Choose breads, crispbreads and cereals that have added linseed
NUTS
Macadamias, hazelnuts, cashews and almonds can be enjoyed every day in limited amounts (one handful)
Peanuts (and peanut paste) and pistachios one handful 23 times a week
MEAT
Use lean cuts of meat
FISH
Choose oily fish, i.e. mullet, flathead, snook, trevally, fresh tuna, canned pink salmon, canned sardines
FRUITS AND VEGETABLES
Eat freely
CONVENIENCE FOODS
Frozen chips (these are pre-cooked in canola oil)
McCain Healthy Choice; Straight Cut or Country Style
Birds Eye Steak House, Crinkle Cut, Home Style, French Fries, Flavour Crisp (Seasoned Crunchy Wedges only)
SNACK FOODS & CONFECTIONERY
Most potato crisps are cooked in omega-6 fats, which may prevent the omega-3 fats going into your body
Kettle Chip Company chips are cooked in sunola oil
FAST FOODS
Pizza Haven and Fasta Pasta use canola oil
Based on a pamphlet developed by Dr E Mantzioris, Dr MJ James and Prof LG Cleland, Royal Adelaide Hospital, Australia.

Table 19.1B Foods to avoid

FATS AND OILS


Avoid all other oils, cooking sprays, dressings, mayonnaise, lards, dairy blends, drippings, cooking margarines and spreads
NUTS & LEGUMES
Brazil nuts, pecan nuts, pine nuts, walnuts, sesame seeds (tahini paste, houmous), sunflower seeds
FISH
Packaged frozen fish such as fish fingers
CONVENIENCE FOODS
All other varieties of frozen chips. Avoid those fruits or vegetables that are canned in dressings (e.g. potato salad)
SNACK FOODS & CONFECTIONERY
Health food bars sesame bars, nut bars, muesli bars

arthritis to date remains that of polyunsaturated omega-3 oils. Additional potential


mechanisms are under exploration, and it is prudent advice to any patient with inflam-
matory polyarthritis that they aim to alter their omega-3/omega-6 balance (Table 19.1).
This should occur in parallel with the use of single and preferably combination disease-
modifying agents. There is limited evidence for benefit of restrictive or elimination diets
and any benefits are likely to be individual and not transferable to the next patient.
03-PS Rheumatology-cpp:03-PS Rheumatology-ppp.QXD 18/3/08 15:26 Page 102

102 03 Rheumatoid Arthritis

Research into the dietary polyphenols may open up new areas of dietary manipulation in
the treatment of chronic inflammatory disease.

Further Reading
1 Pattison DJ, Harrison RA, Symmons DPM. The role of diet in susceptibility to rheumatoid
arthritis: a systematic review. J Rheumatol 2004; 31: 131019.
2 Oliver JE, Silman AJ. Risk factors for the development of rheumatoid arthritis. Scand J
Rheumatol 2006; 35: 16974.
3 Pattison DJ, Symmons DPM, Young A. Does diet have a role in the aetiology of rheumatoid
arthritis? Proc Nutr Soc 2004; 63: 13743.
4 Pattison DJ, Silman AJ, Goodson NJ et al. Vitamin C and the risk of developing inflammatory
polyarthritis: prospective nested casecontrol study. Ann Rheum Dis 2004; 63: 8437.
5 Stamp LK, James MJ, Cleland LG. Diet and rheumatoid arthritis: a review of the literature.
Semin Arthritis Rheum 2005; 35: 7794.
6 Choi HK. Dietary risk factors for rheumatic diseases. Curr Opin Rheumatol 2005; 17: 1416.
7 Funk JL, Frye JB, Oyarzo JN et al. Efficacy and mechanism of action of turmeric supplements
in the treatment of experimental arthritis. Arthritis Rheum 2006; 54: 345264.
8 Jackson JK, Higo T, Hunter WL, Burt HM. The antioxidants curcumin and quercetin inhibit
inflammatory processes associated with arthritis. Inflamm Res 2006; 55: 16875.
9 Yoon J-H, Baek SJ. Molecular targets of dietary polyphenols with anti-inflammatory
properties. Yonsei Med J 2005; 46: 58596.
10 Kumazawa Y, Kawaguchi K, Takimoto H. Immunomodulating effects of flavonoids on acute
and chronic inflammatory responses caused by tumor necrosis factor alpha. Curr Pharm Des
2006; 12: 42719.
03-PS Rheumatology-cpp:03-PS Rheumatology-ppp.QXD 18/3/08 15:26 Page 103

20 Polyarthritis in the elderly 103

P R O B L E M

20 Polyarthritis in the Elderly

Case History
John is 69 years old and presents because of pain in his shoulders and hands. He is unable
to get out of bed with ease or dress himself. His symptoms started suddenly five weeks
ago and he thought they would get better with tablets he bought from the chemist.
What are the common forms of polyarthritis in the elderly?
Is there evidence that osteoarthritis can be modified by the use of medication?
What factors should be considered in the treatment of arthritis in the elderly?

Background
Arthritis and rheumatism are major physical and psychological burdens in the elderly
population. As most forms of arthritis are chronic, the combination of increasing life-
span, cumulative arthritis incidence and concomitant disease and treatments creates the
almost inevitable experience of arthritis. Approximately two-thirds of elderly people
experience sufficient symptoms each day to warrant regular use of non-steroidal anti-
inflammatory drugs (NSAIDs). Symptoms are sufficient to disrupt the sleep or leisure
activities of one-third of the elderly population, with nearly all these people using at least
one medical, complementary or self-care strategy. Arthritis is the most prevalent chronic
condition among adults over the age of 65 years (48.9 per 100 adults), followed by hyper-
tension (40.3 per 100 adults) and heart disease (28.6 per 100 adults). The psychological
burden of arthritis is often underestimated.

Osteoarthritis
Osteoarthritis (OA) is the commonest condition to affect synovial joints and the single
most important cause of locomotor disability. Although not an inevitable consequence
of aging, OA is strongly related to age, which may represent cumulative insult to the joint,
possibly aggravated by decline in neuromuscular function or senescence of homeostatic
repair mechanisms. OA is uncommon and multiple joint OA is rare in persons aged less
than 45 years. Prevalence of OA varies in different populations, but on average affects
60%70% of those aged over 45 years. The prevalence of radiographic OA exceeds that of
clinical OA, and is almost universal in the elderly population in distal interphalangeal
joints and knees. The prevalence of symptomatic OA also increases with age, 15% of
those aged 55 years having symptomatic knee OA. There is a pronounced female prepon-
derance after the age of 55 years for severe radiographic OA. This has suggested that

Atlas Medical Publishing Ltd


03-PS Rheumatology-cpp:03-PS Rheumatology-ppp.QXD 18/3/08 15:26 Page 104

104 03 Rheumatoid Arthritis

oestrogen deficiency may play a role in aetiology. Occupation predisposes to OA of spe-


cific joints (e.g. hip of farmers). The best-characterized OA risk factor is obesity, with OA
in the knee and to a lesser extent in the hip correlating with higher body weights.
OA is a condition of synovial joints characterized by focal cartilage loss and an accom-
panying reparative bone response. It is a slowly progressing inflammatory process and not
simply wear and tear. The final common pathway of OA is cartilage degradation. Brandt
and Mazzuca1 reviewed nine clinical trials purporting to demonstrate disease modifica-
tion in OA, with their focus on placebo-controlled randomized clinical trials in which
joint space narrowing (JSN) was used as an outcome. They also examined the effects of the
purported disease-modifying osteoarthritis drugs (DMOADs) on joint pain (Table 20.1).
Whilst some agents have shown possible efficacy, trial design has been poor with heteroge-
neous populations and insufficiently sensitive outcome measures. No therapy has yet been
approved as having either chondroprotective or structure-modifying properties in OA.

Table 20.1 Effect of disease-modifying osteoarthritis drugs (DMOADs)

Intervention Joint Study duration (months) JSN Sx control

GI. Sulp 1500 mg K 36 ?


GI. Sulp 1500 mg K 36
Ch. Sulp 800 mg K 24 ?
Ch. Sulp 800 mg K 12 ?
Doxycycline 200 mg K 30
Risedronate 5/15 mg K 12
IA Hyaluronan K 12 ? ?
Diacerein 100 mg H 36
Ch. Sulp, chondroitin sulphate; GI. Sulp, glucosamine sulphate; H, hip; IA, intra-articular; JSN, joint space narrowing; K, knee; Sx, symptom.

The Glucosamine/chondroitin Arthritis Intervention Trial (GAIT)2 involving 1500


overweight subjects with symptomatic and radiographic knee OA, found that neither
glucosamine nor chondroitin alone or in combination were better than placebo for
symptom control. The internal active control in this study was celecoxib and this was sig-
nificantly better than placebo for symptom control. A potential explanation for this lack
of clinical effect of glucosamine is the differences between the serum levels obtained in
vivo after standard doses of glucosamine and the doses used in the precursor loading of
cultured human chondrocytes studied in vitro. In clinical studies, a single dose of glu-
cosamine sulphate was detected in 17 of 18 subjects at 3045 minutes after administra-
tion, with peak concentrations occurring at 90180 minutes. The detected serum
concentration range of 211 mmol/l would contribute only 2% of galactosamine incorpor-
ation into chondroitin sulphate in the in vitro experiments in human chondrocytes.3

Rheumatoid arthritis
The onset of rheumatoid arthritis (RA) is often abrupt in the elderly, and although the
classical peripheral symmetrical small joint polyarthritis develops, shoulder symptoms
03-PS Rheumatology-cpp:03-PS Rheumatology-ppp.QXD 18/3/08 15:26 Page 105

20 Polyarthritis in the elderly 105

may predominate initially and prominent hand oedema may obscure the diagnosis. Late-
onset RA (LORA) has been reported to have a more equal gender distribution, an acute
onset and prominent elevation of erythrocyte sedimentation rate (ESR). It is also charac-
terized by disabling morning stiffness and marked pain, predominantly in the upper
extremities. The physical examination is of pronounced synovitis of the shoulders and
the wrists as well as the metacarpophalangeal joints and proximal interphalangeal joints,
with marked limitation of motion and severe soft tissue swelling. Conditions that may
mimic and cause diagnostic confusion with LORA are remitting seronegative symmetric
synovitis with pitting oedema (RS3PE) and polymyalgia rheumatica (PMR) (Table 20.2).
A diagnosis of PMR or LORA can be made in the same patient at different stages of the
same illness depending on the predominant clinical manifestation at the time. RS3PE is
an acute onset, bilateral symmetrical synovitis predominantly of the wrist, carpus, small
hand joints and flexor digitorum sheaths, associated with marked oedema of the dorsum
of the hand. Patients are persistently rheumatoid factor (RF) negative and respond
rapidly to low-dose steroids. Cantini et al.,4 in their prospective five-year study of PMR
and RS3PE, found no demographic, clinical or immunogenetic differences. No patient
developed RA, and they concluded that PMR and RS3PE constitute a spectrum within
the same disease.

Table 20.2 Differences and similarities between LORA, PMR and RS3PE

LORA PMR RS3PE

Age of onset Over 60 years Over 55 years Over 60 years


Sex ratio F:M 1:1 5:1 1:4
Mode of onset Acute or subtle Acute Acute
Predominant Peripheral joints in upper Shoulders, lower back, hips, Hands, wrists, shoulders, knees
joint pattern limbs and shoulders knees
Distal oedema May be present May be present Present
RF positivity 50% Negative Negative
HLA association DR4 ? B7
Course Severe in RF+ Long-term steroids 24 years Self-limited 12 years
Mild in RF Giant cell arteritis in 10%15%
Response to Poor in RF+ Good Good
low-dose steroid Good in RF
Modified with permission from Canoso JJ. Rheumatology in Primary Care. WB Saunders Co., Philadelphia, 1997.

In general, the therapeutic approach in LORA is similar to that for younger patients.
Consideration needs to be given, however, to concomitant illnesses and the elderly
patients reduced physiological reserve, and the ability of the liver and kidneys to metab-
olize medications. NSAIDs increase the risk of gastrointestinal bleeding and can impair
renal function, with the resultant salt and water retention impacting on hypertension and
cardiac function. The mortality for gastrointestinal bleeding increases rapidly in later life,
with risk factors for bleeding being age over 65 years, past history of peptic ulcers or
03-PS Rheumatology-cpp:03-PS Rheumatology-ppp.QXD 18/3/08 15:26 Page 106

106 03 Rheumatoid Arthritis

bleeds, concomitant steroids/anticoagulants/antiplatelet agents and general frailty. Risk


factors for renal failure include age over 65 years, hypertension, cardiac failure and con-
comitant diuretics or angiotensin-converting enzyme inhibitors. The evidence that select-
ive cyclooxygenase-2 inhibitors offer meaningful gastroprotection in an elderly cohort is
doubtful, and any positives are outweighed by adverse impact on renal and cardiac
function.
In the past, low-dose corticosteroids have been promoted for use in the elderly due to
their rapid onset of action. This strategy is acceptable if the goal is rapid suppression of
the inflammatory process with the steroids acting as a bridge to a longer-term alternative
agent allowing the withdrawal of steroids. The danger is if the alternative agent is not
introduced and/or the patients continue to receive steroids in the long term. Even low-
dose steroids (<7.5 mg prednisolone per day) are associated with reduction in bone min-
eral density, hyperglycaemia, weight gain and hypertension.

Gout
Prolonged hyperuricaemia is a necessary prerequisite for clinical gout, but fortunately
gout only develops in a small percentage of patients with chronic hyperuricaemia. In
men, uric acid accumulation begins after puberty, with the first attacks usually occurring
after the age of 3540 years. In women, oestrogen facilitates renal excretion of urate, such
that the age at which clinical gout occurs is older; occurrence is often the combination of
post-menopausal status and, frequently, thiazide diuretic use, and more recently the con-
comitant use of low-dose aspirin. In the elderly, joints previously damaged (mostly by
OA) facilitate seeding of uric acid crystals through exposed collagen and glycosaminogly-
cans. Age-related and concomitant illness impacts on renal function to further enhance
uric acid retention, leading to more frequent and more severe attacks of gout. Eventually,
the patient may have chronic polyarticular gout, which no longer resembles the acute,
self-limited illness, and is easily confused with a polyarthritis such as RA.
Colchicine is related to the cytotoxic agents vincristine and vinblastine, and is renally
excreted. Its use in the elderly is often associated with diarrhoea and risk of bone marrow
suppression. Oral, intramuscular and intra-articular steroids are well suited to suppress-
ing an acute attack of gout. In the longer term, allopurinol remains the agent of choice in
the elderly, as it does not rely on preserved renal function and does not increase the risk
of renal calculi. It is, however, renally excreted, with any dosage reduction either based
empirically on renal function, or more specifically based on the serum measurement of
the active allopurinol metabolite, oxypurinol.

Conclusion
OA, gout, pseudogout, RA and PMR are commonly encountered in the elderly popula-
tion. Diagnostic confusion may occur because of age-related effects on diagnostic tests
such as ESR, RF and antinuclear antibodies. The onset of scleroderma, systemic lupus
erythematosus or idiopathic polymyositis is uncommon in the elderly. Increasing life-
span is impacting both on overall prevalence and incidence of the rheumatic diseases in
the elderly community. The combination of aging, cumulative morbidities and
polypharmacy makes the elderly particularly susceptible to drug-related adverse events.
03-PS Rheumatology-cpp:03-PS Rheumatology-ppp.QXD 18/3/08 15:26 Page 107

20 Polyarthritis in the elderly 107

This, however, should not detract from the same overall goal of improving an individ-
uals quality of life and functioning.

Further Reading
1 Brandt KD, Mazzuca SA. Lessons learned from nine clinical trials of disease-modifying
osteoarthritis drugs. Arthritis Rheum 2005; 52: 334959.
2 Clegg DO, Reda DJ, Harris CL et al. Glucosamine, chondroitin sulfate, and the two in combi-
nation for painful knee osteoarthritis. N Engl J Med 2006; 354: 795808.
3 Biggee BA, Blinn CM, McAlindon TE, Nuite M, Silbert JE. Low levels of human serum
glucosamine after ingestion of glucosamine sulphate relative to capability for peripheral
effectiveness. Ann Rheum Dis 2006; 65: 2226.
4 Cantini F, Salvarini C, Olivieri I et al. Remitting seronegative symmetrical synovitis with pit-
ting oedema (RS3PE) syndrome: a prospective follow up and magnetic resonance imaging
study. Ann Rheum Dis 1999; 58: 2306.
04-PS Rheumatology-cpp :04-PS Rheumatology-ppp.QXD 18/3/08 14:30 Page 109

S E C T I O N F O U R 04
Systemic Lupus Erythematosus,
Sjgrens Syndrome and
Scleroderma
21 Antinuclear factor
22 SLE risk factors and diagnosis
23 Monitoring and managing SLE
24 Sjgrens syndrome
25 Raynauds phenomenon
26 Assessing and treating scleroderma
27 Immunosuppressive drugs

P R O B L E M

21 Antinuclear Factor

Case History
Mary is 32 years old and presents with chronic fatigue and general malaise. She has two
young children aged two and five years. Her pregnancies were uncomplicated. Her mother
suffers from autoimmune thyroid disease. Her erythrocyte sedimentation rate (ESR) is
slightly elevated at 25 mm/h and the immunology laboratory reports that her serum is
antinuclear antibody (ANA) positive at a titre of 1:320.
What is the clinical significance of the positive ANA?
What factors will aid the interpretation of this result?
Are there other tests that should be requested?

Atlas Medical Publishing Ltd


04-PS Rheumatology-cpp :04-PS Rheumatology-ppp.QXD 18/3/08 14:30 Page 110

110 04 Systemic Lupus Erythematosus, Sjgrens Syndrome and Scleroderma

Background
ANA is detected by indirect immunofluorescence on fixed and permeabilized Hep2 cells
derived from a human laryngeal carcinoma. ANA is positive in the majority of patients
with systemic lupus erythematosus (SLE) but is not specific for SLE, frequently being
positive in other connective tissue diseases. ANA may also be detected in autoimmune
thyroid and liver diseases. It may become positive in patients with infections and inflam-
matory disorders including inflammatory bowel disease. A small proportion of the nor-
mal population, particularly the elderly, are also ANA positive. ANA results are reported
as titres the maximum dilution at which the antibody can be detected by immunofluor-
escence.1 Beginning at 1:40 dilution, serial doubling dilutions are analysed until the
observer no longer detects the presence of antinuclear fluorescence (i.e. 1:40, 1:80, 1:160,
1:320, 1:640 dilutions and so on). Often reported as positive at a titre of 1:40, clinically
meaningful positives begin at 1:160. Although higher titres are more likely to indicate the
presence of an underlying disease, repeated measurements are generally not useful in
tracking the activity of underlying diseases. Different patterns of staining, indicating
which antibody may be present, have long been recognized but are now of less clinical
significance as specific markers for connective tissue diseases have been described:
Homogeneous: Due to anti-double-stranded DNA (anti-dsDNA), anti-histone (seen in
drug-induced lupus) or anti-nucleosome (histoneDNA complex).
Membranous: Antibodies to membrane components including lamins A, B and C.
Speckled: Antibodies directed at non-histone antigens. Coarse speckles anti-Sm (Smith
antigen) and anti-uracil-rich 1 ribonucleoprotein (anti-U1RNP); fine speckles anti-
SSA/Ro or anti-SSB/La.
Anti-centromere: Characteristic of limited form of systemic sclerosis (CREST syndrome).
Nucleolar: Anti-DNA topoisomerase I (Scl-70), a marker for systemic sclerosis.
The Hep2000 cell line is increasingly used for ANA testing as it contains a proportion
of Ro antigen-transfected cells and identifies both ANA and anti-Ro antibodies.
Sensitivities and specificities for various disease states are shown in Table 21.1. Positive
ANA should always be interpreted in the light of the clinical picture. Extractable nuclear
antigens are antibodies directed against small ribonuclear proteins and include anti-Sm,
anti-U1RNP, anti-SSA/Ro and anti-SSB/La. With modern diagnostic techniques, it is
debatable whether SLE can be diagnosed in the absence of antinuclear antigens although
older literature suggests that up to 5%10% of cases may be autoantibody negative. Of
263 patients referred with a presumptive diagnosis of SLE, 29% were ANA positive but
did not have autoimmune disease, and half of these had received treatment with cortico-
steroids at dosages as high as 60 mg/day.2

Anti-dsDNA
DNA was the first of the nuclear antigens to be identified (in 1957). High avidity
immunoglobulin G (IgG) antibodies directed at double-stranded DNA are highly
specific for SLE. The antigenicity of DNA is increased when it is complexed to histones or
other nucleosome proteins. The level of anti-dsDNA is a reasonable marker of disease
activity, particularly where there is renal involvement. Antibodies against the comple-
04-PS Rheumatology-cpp :04-PS Rheumatology-ppp.QXD 18/3/08 14:30 Page 111

21 Antinuclear factor 111

Table 21.1 ANA in connective tissue diseases

Disease Sensitivity (%) Specificity (%)

Systemic lupus erythematosus 96 57


Scleroderma 85 54
Secondary Raynauds 64 41
Polymyositis/dermatomyositis 61 63
Sjgrens syndrome 48 52
Rheumatoid arthritis 41 56
Adapted with permission from Habash-Bseiso et al. 2005.3

ment component C1q may also be useful. The antibodies are not only markers for the
disease but directly participate in the disease process through immune complex depos-
ition, cross reactivity with other antigens and promotion of immune reactions within tis-
sues. A variety of methods have been described for detection including the Farr assay,
Crithidia luciliae immunofluorescence and a range of enzyme-linked immunosorbent
assays (ELISAs).

Anti-SSA/Ro and anti-SSB/La


These were first described as antibodies against determinants in salivary glands of
patients with Sjgrens syndrome. They are directed against 52 kDa and 60 kDa Ro pro-
teins, La or to one of a number of RNA particles. Anti-Ro occurs in some patients with
SLE, mixed systemic sclerosis (SSc)/SLE syndrome, primary biliary cirrhosis and in
15%20% of patients with polymyositis/dermatomyositis (PM/DM). In SLE, it has been
associated with non-erosive (Jaccouds) arthropathy and with late onset of the disease (at
age >50 years). Anti-La is most commonly found in Sjgrens syndrome. In SLE, it gener-
ally occurs with anti-Ro and together they are a marker for low risk of renal disease. Anti-
La is also found in patients with hypergammaglobulinaemia or cryoglobulinaemia.
Anti-Ro is found in patients with complement C2 or C4 deficiency. Anti-Ro IgG crosses
the placenta and is responsible for congenital heart block in the offspring of mothers with
SLE. Heart block and tachyarrhythmias occur with increased frequency in mixed connect-
ive tissue disease (MCTD), SSc and PM/DM, where they are often associated with the
presence of anti-Ro.4

Anti-Sm and anti-RNP


Anti-Sm shows high specificity for SLE but is only found in 5%30% of patients, with fewer
Caucasian patients and up to 30% of black patients being positive. Anti-RNP is positive in
25%47% of SLE patients, and is a particularly good marker for MCTD. Although the two
autoantibodies are highly associated, they may have different clinical associations: anti-
RNP in patients with SLE and Raynauds phenomenon and typically with milder renal
involvement; anti-Sm, along with high anti-dsDNA, is a marker for more severe renal
involvement. Anti-Sm is directed at seven proteins that form the core of U1, U2, U4 and U5
small nuclear ribonucleoprotein particles. Anti-RNP is directed at 70 kDa, A and C pro-
teins in the core of U1RNA particles. Detection is by counterimmunoelectrophoresis,
04-PS Rheumatology-cpp :04-PS Rheumatology-ppp.QXD 18/3/08 14:30 Page 112

112 04 Systemic Lupus Erythematosus, Sjgrens Syndrome and Scleroderma

immunoblotting or ELISA. Anti-Sm is technically difficult to detect and more than one
method may be required. Anti-Sm may arise by molecular mimicry with an EpsteinBarr
virus, while cross-reactivity with influenza B has been suggested for anti-RNP.

Anti-Jo-1
Initially described in 1980 in a patient with interstitial pulmonary fibrosis (John P), these
antibodies are highly specific for inflammatory myopathies. Anti-Jo-1 is directed at the
histidyl-tRNA synthetase. Autoantibodies against six of the 20 aminoacyl-tRNA syn-
thetases have now been described. The antibody occurs in 20%30% of patients with
PM, but in 60%70% where there is concurrent pulmonary fibrosis. DM is more fre-
quently associated with malignancy than PM, but anti-Jo-1 is present in 5%10% of
cases. Anti-Jo-1 often occurs along with anti-Ro52. The association of inflammatory
myositis, interstitial pulmonary fibrosis and Raynauds phenomenon has been called
antisynthetase syndrome. Detection of anti-Jo-1 is by counterimmunoelectrophoresis,
immunoblotting or ELISA.

Anti-Scl-70
These are antibodies against the enzyme topoisomerase I. They are rarely found in
healthy individuals and are reasonably specific for SSc, particularly where there is diffuse
cutaneous involvement or interstitial pulmonary fibrosis. They are detectable in about
40% of patients with SSc, and have also been described in patients with localized sclero-
derma, eosinophilia myalgia syndrome and graft-versus-host disease.

Anti-hnRNP
Antibodies to heterogeneous nuclear ribonucleoproteins (hnRNPs) have only relatively
recently come to prominence and are positive in 35% of rheumatoid arthritis patients,
20%30% of patients with SLE and in 40% with MCTD. The hnRNPs are abundant
nuclear proteins involved in RNA processing in association with RNA polymerase II. The
most common anti-hnRNPs are antibodies against the A1 protein (a marker for
Raynauds phenomenon associated with SLE), the A2/B complex (associated with erosive
arthritis in SLE) and K protein. Anti-RA33 is thought to be a relatively specific marker for
rheumatoid disease.

The above autoantibodies should only be sought if ANA is positive and there is a strong
clinical suspicion of the related disease. The associations between autoantibodies and the
various disease states is summarized in Table 21.2.

Recent Developments
1 Environmental factors including drugs and infections are important triggers for the
development of ANA. In a recent study,5 ANA (measured by ELISA) was found in
10.9% of men and 12.2% of women. Records of infectious diseases in early life were
available. Participants who had mumps, rubella or a diarrhoeal illness later in life
were more likely to be ANA positive.
2 Smoking is another environmental trigger for rheumatoid arthritis, SLE,
autoimmune thyroid disease and multiple sclerosis. Generation of reactive oxygen
04-PS Rheumatology-cpp :04-PS Rheumatology-ppp.QXD 18/3/08 14:30 Page 113

21 Antinuclear factor 113

Table 21.2 Autoantibodies in connective tissue diseases

SLE MCTD SS SSc RA PM/DN Raynauds

ANA +++ ++ ++ +++ + ++ ++


dsDNA ++++ ++ +
Ro ++ ++ ++
La + ++
Sm ++ + +
RNP ++ +++ + ++
Jo-1 + + + +++ +
Scl-70 ++ +
hnRNP ++ ++ + ++
The higher number of + signs, the greater utility in diagnosis for each syndrome. This does not imply that the markers are useful for monitoring disease
activity. ANA, antinuclear antibody; dsDNA, double-stranded DNA; MCTD, mixed connective tissue disease; PM/DM, polymyositis/dermatomyositis; RA,
rheumatoid arthritis; RNP, ribonucleoprotein; SLE, systemic lupus erythematosus; SS, Sjgrens syndrome; SSc, systemic sclerosis.

species in smokers may lead to exposure of potential autoantigens or may alter


antigens rendering them more immunogenic. In the study by Freemer et al.,6 current
smokers had a higher risk for anti-dsDNA positivity than never smokers (odds ratio
[OR] 4.0; 95% confidence interval [CI] 1.610.4). Current smokers had a higher risk
than previous smokers (OR 3.0; 95% CI 1.37.1).
3 Recent technical advances both in immunofluorescence techniques7 and in
multiplex technologies8 are not only allowing for more standardized testing but are
increasing throughput and decreasing cost. Multiplex technologies include pre-
coating nitrocellulose strips with multiple antigens or using polystyrene beads coated
with antigen. In the latter case, the presence of autoantibodies to specific antigens
can be quantified by fluorescence-activated cell sorting. A line blot immunoassay has
been developed for the simultaneous detection of multiple antibodies to extractable
nuclear antigens.9
4 Endometriosis affects 5%15% of women and is characterized by growth of
endometrial tissue in extrauterine sites. An association with autoimmunity has long
been postulated. In a recent study, circulating ANA was found in 46 out of
112 women with laparoscopically proven endometriosis.10 Laminin-1 is a major
component of basement membranes. Autoantibodies to laminin-1 determinants
have been associated with infertility and recurrent abortion, and the incidence of
such antibodies is increased in endometriosis.11

Conclusion
ANA positivity does not necessarily signify that the patient has an underlying connective
tissue disorder. However, the above patient has a high titre of ANA and some symptoms
that would be consistent with an autoimmune disorder. History and examination should
focus on the presence or otherwise of joint symptoms, skin lesions, renal involvement
04-PS Rheumatology-cpp :04-PS Rheumatology-ppp.QXD 18/3/08 14:30 Page 114

114 04 Systemic Lupus Erythematosus, Sjgrens Syndrome and Scleroderma

Positive ANA

Signs and symptoms of


Arthropathy
Skin lesion
Lung involvement (fibrosis)
Nervous system involvement

Check renal and hepatic function

? Increased inflammatory markers (ESR and CRP)


? Normocytic anaemia

Titre ,1:80 Titre 1:801:320 Titre .1:320

? Clinical Possible clinical Likely to be


significance significance significant

Ix further only if If normal


dsDNA
strong clinical SLE unlikely
C3 and C4 levels
suspicion

Screen for ENA

Measure specific autoantibodies

Interpret results with clinical picture

Specific diagnosis and treatment

Figure 21.1 Investigation of a patient with positive ANA. ENA, extractable nuclear antigen; Ix,
investigation.
04-PS Rheumatology-cpp :04-PS Rheumatology-ppp.QXD 18/3/08 14:30 Page 115

21 Antinuclear factor 115

and central nervous system problems. Increased levels of inflammatory markers (ESR
and serum C-reactive protein [CRP]) would be consistent with an active autoimmune
condition. SLE should be excluded by measurement of anti-dsDNA and complement C3
and C4. Levels of the latter would be lower than normal in a patient with active SLE.
Management of the patient with positive ANA is summarized in Figure 21.1.

Further Reading
1 Muro Y. Antinuclear antibodies. Autoimmunity 2005; 38: 39.
2 Narain S, Richards HB, Satoh M et al. Diagnostic accuracy for lupus and other systemic
autoimmune diseases in the community setting. Arch Int Med 2004; 164: 243541.
3 Habash-Bseiso DE, Yale SH, Glurich I, Goldberg JW. Serologic testing in connective tissue
diseases. Clin Med Res 2005; 3: 1903.
4 Lazzerini PE, Capecchi PL, Guideri F, Acampa M, Galeazzi M, Laghi Pasini F. Connective
tissue diseases and cardiac rhythm disorders: an overview. Autoimm Rev 2006; 5: 30613.
5 Edwards CJ, Syddall H, Goswami R, Dennison EM, Cooper C. Infections in infancy and
the presence of antinuclear antibodies in adult life. Lupus 2006; 15: 21317.
6 Freemer MM, King TE, Criswell LA. Association of smoking with dsDNA autoantibody
production in systemic lupus erythematosus. Ann Rheum Dis 2006; 65: 5814.
7 Tsiakalou V, Tsangaridou E, Polioudaki H et al. Optimized detection of circulating anti-
nuclear envelope autoantibodies by immunofluorescence. BMC Immunol 2006; 7: 209.
8 Binder SR. Autoantibody detection using multiplex technologies. Lupus 2006; 15: 41221.
9 Damoiseaux J, Boesten K, Giesen J, Austen J, Tervaert JWC. Evaluation of a novel line-
blot immunoassay for the detection of antibodies to extractable nuclear antigens. Ann NY
Acad Sci 2005; 1050: 3407.
10 Dias JA, de Oliveira RM, Abrao MS. Antinuclear antibodies and endometriosis. Int J
Gynaecol Obstet 2006; 93: 2623.
11 Inagaki J, Kondo A, Lopez LR, Shoenfeld Y, Matsuura E. Pregnancy loss and
endometriosis. Pathogenic role of anti-laminin-1 autoantibodies. Ann NY Acad Sci 2005;
1051: 17484.
04-PS Rheumatology-cpp :04-PS Rheumatology-ppp.QXD 18/3/08 14:30 Page 116

116 04 Systemic Lupus Erythematosus, Sjgrens Syndrome and Scleroderma

P R O B L E M

22 SLE Risk Factors and Diagnosis

Case History
Jenny, aged 21 years, presented with fatigue, a photosensitive rash on her arms and face,
recurrent mouth ulcers and sore joints. She smokes 20 cigarettes per day and consumes
very little alcohol. Her medications are the oral contraceptive and minocycline for acne.
On what basis could you make a diagnosis of systemic lupus erythematosus (SLE)?
What further investigations should be undertaken?
What environmental or inherited risk factors predispose to SLE?

Background
Diagnosis of SLE
SLE is a disease characterized by the production of a variety of antibodies against nuclear
components that causes inflammation and injury to multiple organs. It primarily affects
women from the late teens onwards with a peak incidence between the ages of 15 and
45 years. In European populations the prevalence is 1264 cases per 100 000, and it is
three to five times more prevalent in African-American and Afro-Caribbean women.
Recent studies in human populations and animal models have associated elements of the
innate immune system and abnormalities in the immature B lymphocyte receptor reper-
toires with disease initiation. A variety of cytokines, most notably type 1 interferons
(IFN-a, IFN-b), play an important role in pathogenesis.
The American College of Rheumatology (ACR) criteria for the classification of SLE
(Table 22.1) act as an aide-memoire to the multisystem nature of SLE and to the investiga-
tions that should be undertaken. Patients with SLE almost always suffer from debilitating
fatigue, with the majority having associated weight loss and fever. Three-quarters of
patients with SLE have mucocutaneous disease in the form of butterfly rash, oral ulcers,
alopecia and Raynauds phenomenon; purpura, vasculitis or urticaria occur less often. The
majority (60%) experience arthralgia or arthritis in a pattern that may mimic rheumatoid
arthritis (RA). Renal disease and haematological problems occur in approximately one-
third of patients; cardiac, neuropsychiatric and gastrointestinal problems occur in 20%.
Lack of antinuclear antibody (ANA) at a titre of 1:160 or higher makes SLE very unlike-
ly. Remaining patients are often positive for anti-Ro antibodies. False-positive ANA
results increase with age, and up to 5% of healthy individuals may be ANA positive.
Specific ANA immunofluorescence patterns such as anti-centromere, anti-nucleolar or
proliferating cell nuclear antigen may be reported and require no further characteriza-

Atlas Medical Publishing Ltd


04-PS Rheumatology-cpp :04-PS Rheumatology-ppp.QXD 18/3/08 14:30 Page 117

22 SLE risk factors and diagnosis 117

Table 22.1 American College of Rheumatology (ACR) classification criteria for SLE1

1. Malar rash
Fixed erythema over the malar eminences, sparing the nasolabial folds
2. Discoid rash
Erythematous patch with keratotic scaling and follicular plugging
3. Photosensitive skin rash
4. Oral ulcers
5. Arthritis
Non-erosive joint inflammation in 2+ peripheral joints
6. Pleuritis or pericarditis
7. Renal disorder
Proteinuria >0.5 g/24 h, or cellular casts
8. Neurological disorder
Seizures or psychosis (not explained by drugs or metabolic changes)
9. Haematological disorder
Haemolytic anaemia, leukopenia, lymphopenia, thrombocytopenia
10. Immunological disorder
Anti-DNA antibody, anti-Sm antibody, antiphospholipid antibody
11. Antinuclear antibody

tion. Sera with either a homogeneous or rim pattern on ANA testing should undergo anti-
double-stranded DNA (anti-dsDNA) testing, and a speckled ANA pattern should undergo
testing for antibodies to extractable nuclear antigens. Anti-dsDNA antibodies are the most
specific autoantibody; however, they are not very sensitive, occurring in only one-half of
lupus patients during the course of their disease. Anti-dsDNA antibodies may also be
found in autoimmune hepatitis and chronic infections such as syphilis, subacute bacterial
endocarditis and parasitic infection.2 The presence of anti-Sm antibodies is specifically
identified in the ACR criteria, even though low titres have been reported in other diseases.
Anti-Sm is very rare in normal individuals and is virtually pathognomonic for SLE. It is
found in 5%30% of SLE patients and is more prevalent in black Americans.

Environmental and genetic interaction in the aetiology


Several genetic loci have been associated to SLE in multiplex families using genome-wide
linkage studies. Alleles within the major histocompatibility complex (Fcg receptors,
immunoglobulin receptor homologues, cell signalling molecules, cytokines/
chemokines, complement, opsonins etc.) have been found to be associated with SLE.
Concordance rate in monozygotic twins is <60%, invoking a significant environmental
contribution to the disease.
SLE, as already noted, is characterized by a wide range of autoantibodies that target
intracellular, cytoplasmic, cell-surface and plasma antigens. Autoantibodies to DNA and
histones individually and together (as the nucleosome complex) have been found in
serum of individuals years before the onset of disease, and progressive accumulation of
the autoantibodies precedes the onset of disease. An understanding of the mechanism
that allows B cells to be exposed to and subsequently to develop antibodies to nuclear
antigens that are shielded from immune surveillance is critical to our understanding of
the pathogenesis of SLE. A common link between many of the antigens that are targeted
in SLE is that they are translocated to the cell surface as nuclear blebs during apoptosis.
The current dogma is that clearance of intact dying cells prevents secondary necrosis of
apoptotic cells and hence release of nuclear blebs containing potential autoantigens.
04-PS Rheumatology-cpp :04-PS Rheumatology-ppp.QXD 18/3/08 14:30 Page 118

118 04 Systemic Lupus Erythematosus, Sjgrens Syndrome and Scleroderma

Table 22.2 Drugs implicated in drug-induced lupus3

Definitely capable of DIL


Hydralazine, procainamide, isoniazid, methyldopa, chlorpromazine, quinidine, minocycline
Possibly inducing lupus
Sulphasalazine, anticonvulsants, antithyroid drugs, statins, b-blockers, interferon-a, penicillamine, fluorouracil,
thiazide diuretics
Suggested to induce lupus
Multiple antibiotics, oestrogens and oral contraceptives, captopril, calcium channel blockers, etanercept,
infliximab

However, under special circumstances, apoptotic cells may trigger an immune response,
which may activate T and B cells and the formation of autoantibodies.

Drug-induced lupus
There are various kinds of lupus: (a) SLE; (b) discoid lupus (inflammatory scarring skin
lesions); (c) subacute cutaneous lupus (non-scarring, non-atrophy producing photo-
sensitive dermatosis); and (d) drug-induced lupus (DIL). Drugs responsible for develop-
ing DIL can be divided into three groups (Table 22.2).3
There are no symptoms pathognomonic for DIL; however, some features are com-
mon. Patients often present with mild lupus-like symptoms, which can develop as early
as one month after initiation of treatment or can be delayed as long as a decade. Men are
as frequently affected as women, and Caucasians are affected up to six times more fre-
quently than black patients and may have a more severe illness. Features common in DIL
are fever, arthralgia, pleuritis, pericarditis, mild cytopenia, anaemia and elevated erythro-
cyte sedimentation rate. Hypocomplementaemia and clinical features of malar or discoid
rash, photosensitivity, oral ulcers, alopecia and renal or neurological disorder are very
uncommon. Anti-histone antibodies, especially IgG anti-([H2A-H2B]-DNA) are posi-
tive in 75%95% of DIL patients, but also occur in 75% of SLE patients. There is usually
an absence of anti-dsDNA and anti-extractable nuclear antigen (anti-ENA) antibodies,
with a high frequency of anti-single-stranded DNA (anti-ssDNA) antibodies. Amongst
the medications definitely capable of inducing lupus, procainamide is currently the drug
most frequently associated with DIL. Between 2% and 21% of patients receiving
hydralazine also develop the disease, the greatest risk being with doses >200 mg/day.
The link between minocycline and DIL became evident when minocycline became used
for the treatment of acne and RA. A casecontrol prospective study found an eight-fold
increased risk for current users, and DIL has been individually confirmed by rechallenge.
Developing any time between three months and six years after initiation, minocycline DIL
(in contrast to classical DIL) has a female preponderance. Typical is a symmetrical poly-
arthritis with early morning stiffness, often with fever and myalgia. A wide range of cuta-
neous manifestations are reported: vasculitis, livedo reticularis, erythema nodosum and
subcutaneous nodules. Positive ANA is found in >80% of patients. Hepatic damage is often
associated but prognosis of minocycline DIL is generally good, with rapid resolution of all
aspects including chronic active hepatitis on withdrawal of the drug.
The high incidence of SLE in females during their reproductively capable years has
prompted the association with female sex hormones. However, it remains controversial
as to whether the addition of exogenous oestrogens on a background of endogenous
04-PS Rheumatology-cpp :04-PS Rheumatology-ppp.QXD 18/3/08 14:30 Page 119

22 SLE risk factors and diagnosis 119

hormones can induce or worsen lupus. It had been suggested that both oral contraceptive
pills (OCPs) and hormone replacement therapy (HRT) could be used without changing
disease activity if patients were normotensive, non-smoking, with only stable to moder-
ate disease and were antiphospholipid antibody negative.4 The Safety of Estrogens in
Lupus Erythematosus National Assessment (SELENA) trials comprised two separate ran-
domized controlled trials of low-dose synthetic oestrogen and progestin OCPs,5 and
HRT6 in women with inactive or stable SLE disease activity. The twelve-month rates of
severe flares and for mild or moderate flares were almost identical for the group receiving
OCPs and for the placebo group, demonstrating safe use of low-dose OCPs in SLE
patients with stable disease. In the HRT trial severe flares were rare, with a twelve-month
severe flare rate of 0.081 for the HRT group and 0.049 for the placebo group. Mild to
moderate flares were significantly increased in the HRT group: 1.14 flares/person-year
for HRT vs 0.86 flare/person-year for placebo (P <0.01), with an increased probability of
any type of flare by twelve months in the HRT group (P <0.01). Adding a short course of
HRT is associated with a small risk for increasing the natural flare rate of lupus.

Recent Developments
1 Whereas genetic information (with the exception of somatic mutations) is
homogeneous in an organism regardless of cell type, epigenetic modifications are
characteristic of different cell types and play a role in defining the transcriptome,
which determines the identity of each cell type.7 The epigenetic framework could
explain several disease characteristics, including age dependence and quantitative
nature, and the mechanism by which environment modulates genetic
predisposition to disease. Abnormalities in the DNA methylation system aberrantly
increase or decrease gene expression, which has been implicated in SLE. Support
for this concept is the global hypomethylation of T cells from patients with active
SLE and the hypomethylating effect of drugs used to induce SLE such as
procainamide, hydralazine and 5-azacytidine (Table 22.3). In the case of histone
modification, histone-deacetylating drugs skew gene expression of CD40L,
interleukin (IL)-10 and interferon-g. Alteration of T cellB cell interactions has
been proposed as the common pathogenic mechanism that leads to disease.
However, the recognition of dendritic cells (DCs) as efficient stimulators of B and
T cell lymphocytes, as well as key controllers of immunity and tolerance, has led
to the hypothesis that SLE may be driven by inappropriate DC activation. Two
mechanisms have been described: elevated functional levels of interferon,8 and
defective complement-mediated clearance of apoptotic cells (Figure 22.1).9
2 Monocytes from SLE patients have enhanced antigen-presenting ability and act like
myeloid dendritic cells; serum from SLE patients rapidly differentiates normal
monocytes into cells with DC morphology and function. A candidate serum cytokine
that may be responsible for this maturation is type 1 interferon (IFN-1). Elevated
levels of IFN-1 have been found in the serum of SLE patients. Administration of
exogenous IFN-1 as therapy for chronic viral infection or malignancy is associated
with autoantibody production in up to 34% of recipients and autoimmune
complications in 4%19%.10 Mature DCs activate cytotoxic T cells, leading to cell
death and the generation of nucleosomes that can be captured and presented by DCs
04-PS Rheumatology-cpp :04-PS Rheumatology-ppp.QXD 18/3/08 14:30 Page 120

120 04 Systemic Lupus Erythematosus, Sjgrens Syndrome and Scleroderma

Table 22.3 Epigenetic changes in SLE

Epigenetic change Consequence Disease association

Methylation of cytosine in Repression of transcription Autoreactivity via T cell response to sub-threshold


CpG dinucleotides promoters antigenic stimuli, and overexpression of adhesion
receptor LFA-1
Altered nuclear architecture CD70 overexpression leading to excess B cell
stimulation in SLE
Histone modification Repression of tumour suppressor genes leading to
Lysine acetylation Transcriptional activity malignant transformation
Lysine methylation, Variable transcriptional effect Hypomethylation causing chromosomal instability
serine phosphorylation Altered nuclear architecture Failed transcriptional regulation of imprinted genes
(e.g. PraderWilli syndrome)

Complement Viral infection IFN mutations


deficiency

Monocyte
Clearance of Type 1
apoptotic cells interferon

Apoptosis and Myeloid DC


nuclear blebs

Loss of tolerance and


autoantibody production

Figure 22.1 Decreased complement and increased interferon (IFN) predispose to autoantibody
production. DC, dendritic cell.

generated in the presence of IFN-1. Together with IL-6, IFN-1 promotes the
differentiation of mature B cells into plasma cells, which are long-lived autoantibody
producers. Thus, the effect of IFN-1 on DCs, B cells and T cells could explain the
breakdown of tolerance to nuclear antigens, and the subsequent autoantibody
secretion and immune complex formation characteristic of SLE.
3 In humans there is a very strong association between deficiencies of the complement
components C1 and C4 and the presence of SLE. It has been hypothesized9 that
complement deficiency leads to impaired clearance of dying cells and allows
prolonged exposure of nuclear blebs (the putative source of autoantigens). The same
authors expanded the theory to include any mechanism that impairs clearance of
apoptotic cells, with evidence for C1q, C4, IgM, macrophage phagocytosis and tissue
transglutaminase. In the presence of adequate complement levels, apoptotic cells are
rapidly taken up by immature DCs before the release of intracellular material can
04-PS Rheumatology-cpp :04-PS Rheumatology-ppp.QXD 18/3/08 14:30 Page 121

22 SLE risk factors and diagnosis 121

activate them, yet at the same time allowing for the nuclear material to be processed
by the DC in its immature form leading to tolerance.

Conclusion
SLE is one of the great masqueraders in medicine, with multiple varied presentations.
Despite its potential to be a devastating illness, in the majority of cases the presentation is
often a poorly defined illness with subjective complaints greater than any observed
abnormalities. Many of the classification criteria listed in Table 22.1 develop over time
and are not cumulative, such that a detailed previous history needs to be undertaken,
particularly as many of the features such as mouth ulcers or photosensitivity may not
subsequently be recalled spontaneously. It is in this setting that the clinical challenge is to
consider the diagnosis of SLE, to investigate appropriately, but then not to over-rely on
the presence of an antinuclear antibody to make the diagnosis.

Further Reading
1 Hochberg MC. Updating the American College of Rheumatology revised criteria for the
classification of systemic lupus erythematosus. Arthritis Rheum 1997; 40: 1725.
2 Kurien BT, Scofield RH. Autoantibody determination in the diagnosis of systemic lupus
erythematosus. Scand J Immunol 2006; 64: 22735.
3 Sarzi-Puttini P, Atzeni F, Capsoni F, Lubrano E, Doria A. Drug-induced lupus erythematosus.
Autoimmunity 2005; 38: 50718.
4 Kreidstein S, Urowitz MB, Gladman DD, Gough J. Hormone replacement therapy in systemic
lupus erythematosus. J Rheumatol 1997; 24: 214952.
5 Petri M, Kim MY, Kalunian KC et al. Combined oral contraceptives in women with systemic
lupus erythematosus. N Engl J Med 2005; 353: 25508.
6 Buyon JP, Petri MA, Kim MY et al. The effect of combined estrogen and progesterone
hormone replacement therapy on disease activity in systemic lupus erythematosus: a random-
ized trial. Ann Intern Med 2005; 142: 95362.
7 Ballestar E, Esteller M, Richardson BC. The epigenetic face of systemic lupus erythematosus. J
Immunol 2006; 176: 71437.
8 Pascual V, Farkas L, Banchereau J. Systemic lupus erythematosus: all roads lead to type 1
interferons. Curr Opin Immunol 2006; 18: 67682.
9 Cook HT, Botto M. Mechanisms of disease: the complement system and the pathogenesis of
systemic lupus erythematosus. Nat Clin Pract Rheumatol 2006; 2: 3307.
10 Borg FAY, Isenberg DA. Syndromes and complications of interferon therapy. Curr Opin
Rheumatol 2007; 19: 616.
04-PS Rheumatology-cpp :04-PS Rheumatology-ppp.QXD 18/3/08 14:30 Page 122

122 04 Systemic Lupus Erythematosus, Sjgrens Syndrome and Scleroderma

P R O B L E M

23 Monitoring and Managing SLE

Case History
Jane, having contacted the Arthritis Foundation and looked on the internet, realises that
systemic lupus erythematosus (SLE) is a multisystem disorder. She wishes to discuss the
potential problems that she may face in the future.
What are the most frequent manifestations of SLE?
What monitoring should she undergo?
What are the current treatments for SLE?

Background
The term lupus was first used in the early 1800s, and is derived from the Latin term for wolf
(Canis lupus). It was applied to the skin condition lupus vulgaris to imply an appearance of
the skin that looked to be torn off as a wolf might have done. Subsequently, the Latin term
for redness erythematosus was appended. The clinical features are represented in Figure
23.1. Almost ubiquitous is the presence of fatigue, weight loss and intermittent fevers. After
constitutional symptoms, arthralgia and mucocutaneous manifestations are the most com-
mon, and in various forms account for five of the eleven American College of
Rheumatology (ACR) criteria for the classification of SLE. Joint pain with or without object-
ive inflammation is the most common reason for patients to present to their doctor. Most
commonly, this presents as a small joint peripheral arthritis, which also involves wrists and
knees and in many respects resembles rheumatoid arthritis. Both can be multisystem in
manifestation, and nodules similar to rheumatoid nodules may be present in up to 10% of
patients. It is unusual for patients with SLE to develop any erosive changes.
The butterfly rash in a malar distribution with sparing of the nasolabial folds is the
pathognomonic rash of SLE. Photosensitivity occurs in around one-half of SLE patients,
and warrants reminding of the importance of sun avoidance and sunblock, with light in
the 360400 nm spectrum inducing both rash and systemic exacerbations. Other rashes
are less common. Alopecia varies in its pattern from more hair in my brush or shower
plug hole to distinct patches of hair loss, particularly if there have been scarring discoid
lesions of the scalp. Immunosuppressive agents may lead to hair thinning.
Progressive renal disease in SLE is often asymptomatic and should be sought by urin-
alysis for blood and protein and serial serum creatinine measurements. Any abnormality
warrants further evaluation including 24-hour urine collection, microscopy of urinary
sediment and a low threshold for review by a renal physician.

Atlas Medical Publishing Ltd


04-PS Rheumatology-cpp :04-PS Rheumatology-ppp.QXD 18/3/08 14:30 Page 123

23 Monitoring and managing SLE 123

Alopecia
Rash Depression
Oral ulcer Psychosis
75% 12%20%

Pericarditis
Pleurisy Cardiac disease
Lung disease 5%20%
15%30%

Renal
40%70%

Arthritis
5%75% Raynauds
35%50%

Gastrointestinal
15%25%

Figure 23.1 Clinical features of SLE.

Pleural manifestations have been reported in 30%60% of patients, and include


symptomatic pleurisy, effusions and pleural rubs. Pleurisy is usually not associated with
pleural rub, and the differential diagnosis includes primary pleural serositis and sec-
ondary serositis due to underlying infection, or pulmonary infarct (particularly in those
with thrombophilia). Pleural effusions may be found incidentally on chest X-ray and are
usually small. When sufficiently large to drain, an exudate is present with a normal glu-
cose level.

Monitoring SLE
SLE afflicts African-Americans three times more frequently than their European-
American counterparts, and they are considered to have a poorer prognosis. Some of the
04-PS Rheumatology-cpp :04-PS Rheumatology-ppp.QXD 18/3/08 14:30 Page 124

124 04 Systemic Lupus Erythematosus, Sjgrens Syndrome and Scleroderma

different socioeconomic, demographic, psychological and behavioural variables are


highly correlated. The LUMINA (Lupus in minorities: nature versus nurture) cohort
study resulted in a series of papers, with a recent report on predictors of disease activity
over time.1 The cohort of 554 subjects included 18% Texan Hispanics, 17% Puerto Rican
Hispanics, 36% African-Americans and 29% Caucasians. Univariate analysis confirmed
an odds ratio of 3 between Texan Hispanic or African-American ethnicity and
Caucasian, and higher disease activity over time. These characteristics remained signifi-
cant in a multivariate model. High disease activity was independently associated with
lack of health insurance, abnormal illness-related behaviour and poor social support, and
was negatively associated with age.

Laboratory monitoring
Early detection of renal disease is a major goal (Table 23.1), as the survival curves are
vastly different for those with and without renal disease. Specific symptoms are not
observed by the individual until there is advanced nephrotic syndrome or renal failure.
Urinalysis is an extremely cheap and highly sensitive approach to detect haematuria and
proteinuria, and can then be followed by a 24-hour urine collection and quantitative
microscopy. Regular serum creatinine measurement is recommended.
Routine haematology is undertaken for the detection of anaemia, leukopenia and
thrombocytopenia. Interpretation of results should take into account the effect of treat-
ment, with steroids elevating neutrophil counts and decreasing lymphocyte counts, and
immunosuppressants exerting cytotoxic effects.
In the LUMINA study, the immunological variable identified as being independently
associated with high levels of disease activity was the presence of anti-double-stranded
DNA (anti-dsDNA) antibodies.1 Previously these antibodies had been associated with
disease activity and with lupus nephritis but not concomitantly with flares, as it is pro-
posed that as the immune complexes are deposited in tissues their circulating levels
decrease. Increasing levels of anti-dsDNA antibodies, which then fall, may herald exacer-
bations of lupus nephritis or other organ involvement. Whilst this remains controversial,

Table 23.1 Monitoring of SLE patients

History Examination Laboratory investigations

Fatigue Vitals BP, pulse, temperature Urinalysis for proteinuria, haematuria


Mucocutaneous features (oral Cardiorespiratory rubs, effusions Urine microscopy for casts
ulcers, photosensitive rash) and fibrosis Full blood screen
Chest pain, SOBOE Arthritis number and severity Renal function
Arthritis Skin rash C-reactive protein
Urinary features
Preventative screening
Smoking cessation; body mass index goal <25 kg/m2
Monitor BP (target of <130/80 mmHg). Treat if >140/90 mmHg
Fasting lipids (target LDL-cholesterol <2.6 mmol/l. Treat if >34 mmol/l
Fasting glucose, and homocysteine
Anti-Ro, anticardiolipin and lupus anticoagulant prior to planned pregnancy
Anti-dsDNA antibodies
BP, blood pressure; LDL, low-density lipoprotein; SOBOE, shortness of breath on exertion.
04-PS Rheumatology-cpp :04-PS Rheumatology-ppp.QXD 18/3/08 14:30 Page 125

23 Monitoring and managing SLE 125

Hypertension

Hyperhomocysteinemia Hyperlipidaemia

Inflammation
Diabetes
CRP, TNFa

Cardiovascular disease
in SLE

Prothrombosis
Lipid oxidation
(anticardiolipin antibodies)

? SLE treatment Smoking


(steroid therapy)

Figure 23.2 Potential factors for cardiovascular disease in SLE. Adapted with permission from Frostegrd
2005.2

some authors have recommended corticosteroid therapy to prevent flares in SLE patients
manifesting this pattern. If an increasing titre of anti-dsDNA antibody is detected, careful
examination should be undertaken.

Preventative monitoring
With modern drugs, the survival rate for SLE has dramatically improved, as has survival
from acute disease flares. This has, however, revealed a later mortality due to cardiovas-
cular disease (CVD). The predisposition of younger women to SLE results in pre-
menopausal women with SLE having a 50-fold increased risk of myocardial infarction.
Autopsy and angiographic studies demonstrate increased prevalence of atherosclerotic
lesions in SLE patients. Measurements of intimamedia thickness (IMT) of the carotid
artery can be used as a surrogate measure of atherosclerosis; it is not clear from studies of
IMT in patients with SLE whether this marker is increased in the condition. Cross-
sectional studies reveal carotid plaques in one-third of women with lupus. There remains
uncertainty as to whether atherosclerosis in SLE is increased in a general distribution or
whether it is primarily an increased risk of localized plaque. Traditional risk factors for
CVD should be sought and monitored in patients with SLE, but research is highlighting a
number of other risk factors (Figure 23.2).2
The suppression of inflammation is the aim of SLE therapy, and this can be assessed by
both the erythrocyte sedimentation rate (ESR) and serum C-reactive protein (CRP). In
04-PS Rheumatology-cpp :04-PS Rheumatology-ppp.QXD 18/3/08 14:30 Page 126

126 04 Systemic Lupus Erythematosus, Sjgrens Syndrome and Scleroderma

SLE the profound hypergammaglobulinaemic response leads to chronic elevation of the


ESR, leaving CRP to be the better marker of inflammation. Systemic inflammation accel-
erates the atherosclerotic process. In the general population, increased levels of CRP are
associated with increased risk of CVD. In one of the LUMINA reports, CRP was a specific
risk factor for CVD: the median serum CRP in all patients with vascular events was
approximately three times that in those without vascular events. This finding, however,
also raises the conundrum of corticosteroids, which on the one hand are very potent
inhibitors of inflammation, yet in long-term high doses do contribute to adverse cardio-
vascular outcomes. It is possible that low doses of corticosteroids that optimize suppres-
sion of inflammation may have a net beneficial cardiovascular effect.3
Guidance and support in the cessation of smoking is advised, as smoking promotes
both atheroma and flares of lupus. A recent meta-analysis showed that smoking
increased the likelihood of SLE (odds ratio 1.5), most notably with antibodies to
dsDNA.4,5 The riskbenefit ratio of low-dose aspirin (60100 mg/day) favours its use in
all patients with SLE.6 All SLE patients who have defined risk for CVD should receive
low-dose aspirin, as should those patients with both SLE and either a positive
immunoglobulin G anticardiolipin antibody or lupus anticoagulant.

Treatment of SLE
Modifiable lifestyle factors include sun avoidance, stopping smoking, exercise, diet (with
emphasis on altering the omega-3:omega-6 balance and reducing salt intake if hyperten-
sive) and weight management. Specific interventions may be required for the treatment
of dyslipidaemia, diabetes and hypertension. The threshold for intervention is lower due
to the adverse effect of SLE itself.
In practice, sun avoidance needs to be constantly reinforced wide-brimmed hats,
clothing that provides sun protection and the regular application of quality sunblock.
The availability of a range of cosmetic-grade moisturisers and make-up containing sun-
block has aided the acceptability of this advice.
Non-steroidal anti-inflammatory drugs and selective cyclooxygenase-2 (COX-2)
inhibitors are frequently used for soft tissue symptoms and control of arthralgia and
arthritis in the general population, and have been used in SLE, although they are not usu-
ally specifically registered for this population group. They should, however, be used with
caution due to risks of induction of hypertension, peripheral oedema and reduced renal
function. Most regulatory agencies have stated that COX-2 inhibitors should not be used
in patients who have, or are at major risk of, CVD.
Hydroxychloroquine (200400 mg/day) is useful for skin rashes, joint pain and oral
ulcers. It also has a weak antithrombotic action. Doses of <6 mg/kg/day are associated
with a very low risk of retinopathy, and physicians are advised to discuss with a local oph-
thalmologist as to a monitoring regime. Higher doses can be used in the short term for
patients with ongoing photosensitivity. While its mechanism of action remains uncer-
tain, it does alter antigen processing by affecting the pH of the phagolysosome, and may
also regulate innate immunity via Toll-like receptors. A teratogenic effect was suspected
but seems to have been ruled out,7 and its use is safe and effective in pregnancy.
Dehydroepiandrosterone (DHEA) has been advocated for the treatment of SLE, pro-
viding a reduction in fatigue. DHEA is a sex-hormone precursor produced by the adren-
al glands and is converted to either oestrogen or androgen in different peripheral tissues.
04-PS Rheumatology-cpp :04-PS Rheumatology-ppp.QXD 18/3/08 14:30 Page 127

23 Monitoring and managing SLE 127

Randomized placebo-controlled trials support a modest beneficial effect when doses of


200 mg daily are used, but this is outweighed by side effects of hirsutism, acne, hyperten-
sion and adverse lipid profile.8
It is tempting to prescribe oral corticosteroids to the patient who is miserable with
symptoms of SLE. However, once commenced, it is very hard to keep good control on the
dose and to cease therapy. Corticosteroid toxicity includes hyperglycaemia, hypercholes-
terolaemia, hypertension, osteoporosis, glaucoma, weight gain, skin fragility, myopathy
and avascular necrosis. As an adjunct and steroid-sparing agent, azathioprine is one of
the immunosuppressants most commonly used in SLE (chapter 27). A purine analogue,
its effect is to inhibit DNA synthesis, and in doses of 1.52.5 mg/kg/day it has benefit in
preventing serious lupus complications. Drug hypersensitivity occurs in a small percent-
age of patients, resulting in systemic illness, rash, acute hepatitis and abdominal pain.
Monitoring of bone marrow, liver function and lipase is recommended at four- to six-
weekly intervals. As a cytotoxic agent it has been generally recommended that azathio-
prine not be used in pregnancy although the evidence for teratogenicity is limited and the
beneficial effects of disease control on fertility, fetal outcome and the health of the
mother may warrant its continuation during pregnancy.

Recent Developments
1 SLE nephritis is characterized by immune complex-mediated glomerular and
tubulointerstitial inflammation, which leads to chronic renal insufficiency in up to
30% of affected patients. Cyclophosphamide is the principal treatment of diffuse
glomerulonephritis (World Health Organization [WHO] class IV).
Cyclophosphamide given as 6 500 mg infusions followed by oral azathioprine was
as effective as the initial high-dose cyclophosphamide National Institutes of Health
(NIH) regime.9 The evolution of mycophenolate mofetil (MMF) from a transplant
rejection modifier to a broader immunosuppressant application has shown that
23 g/day is as effective as cyclophosphamide in inducing and maintaining remission
in those with mild to moderate renal disease. MMF has a better safety profile than
cyclophosphamide, avoiding haemorrhagic cystitis, ovarian failure and the longer-
term cancer risks. A systematic review and meta-analysis found that a complete or
partial (66%80%) response was more frequent with MMF than with
cyclophosphamide, with one additional complete or partial response for every eight
patients treated.10
2 Central nervous system involvement has been described in up to 50% of lupus
patients. Cognitive dysfunction and headaches are the most common features,
affecting 60%80% of patients. Aseptic meningitis, demyelination, myelopathy,
acute confusional states or psychosis are rare. The prevalence of depression is no
higher in SLE than in other chronic debilitating diseases.11 Psychosis in lupus is
characterized by delusions or hallucinations; it is uncommon in SLE and occurs in
only 2%5% of patients receiving high-dose (1 mg/kg/day) corticosteroids.
Secondary causes such as drug use or drug withdrawal, metabolic and electrolyte
derangements and sepsis should always be excluded before concluding that it is
lupus psychosis.
04-PS Rheumatology-cpp :04-PS Rheumatology-ppp.QXD 18/3/08 14:30 Page 128

128 04 Systemic Lupus Erythematosus, Sjgrens Syndrome and Scleroderma

Conclusion
SLE is challenging both to diagnose and to treat successfully. With its protean manifest-
ations, milder forms are being increasingly recognized or suspected due to the increasing
availability of autoantibody testing. There have been major advances in therapy.
Immunosuppressants such as MMF have led to significant gains in the treatment of
nephritis. Even newer therapies are on the horizon, with the monoclonal antibody rituxi-
mab providing dramatic and long-lasting remissions in patients who were previously
unresponsive. The remaining challenges are in improving overall quality of life, reducing
the impact of fatigue and protecting patients from cardiovascular deaths as they survive
the initial insult of SLE.

Further Reading
1 Alarcn GS, Calvo-Aln J, McGwin G et al; LUMINA study group. Systemic lupus
erythematosus in a multiethnic cohort: LUMINA XXXV. Predictive factors of high disease
activity over time. Ann Rheum Dis 2006; 65: 116874.
2 Frostegrd J. SLE, atherosclerosis and cardiovascular disease. J Intern Med 2005; 257: 48595.
3 Hall FC, Dalbeth N. Disease modification and cardiovascular risk reduction: two sides of the
same coin? Rheumatology 2005; 44: 147382.
4 Costenbader KH, Kim DJ, Peerzada J et al. Cigarette smoking and the risk of systemic lupus
erythematosus: a meta-analysis. Arthritis Rheum 2004; 50: 84957.
5 Freemer MM, King TE, Crisswell LA. Association of smoking with dsDNA autoantibody
production in systemic lupus erythematosus. Ann Rheum Dis 2006; 65: 5814.
6 Wahl DG, Bounameaux H, de Moerloose P, Sarasin FP. Prophylactic antithrombotic therapy
for patients with systemic lupus erythematosus with or without antiphospholipid antibodies:
do the benefits outweigh the risks? Arch Int Med 2000; 160: 20428.
7 Costedoat-Chalumeau N, Amoura Z, Duhaut P et al. Safety of hydroxychloroquine in preg-
nant patients with connective tissue diseases: a study of one hundred thirty-three cases com-
pared with a control group. Arthritis Rheum 2003; 48: 320711.
8 Sibilia J. Treatment of systemic lupus erythematosus in 2006. Joint Bone Spine 2006; 73: 5918.
9 Houssiau FA, Vasconcelos C, DCruz D et al. Early response to immunosuppressive therapy
predicts good renal outcome in lupus nephritis: lessons from long-term followup of patients
in the Euro-Lupus Nephritis Trial. Arthritis Rheum 2004; 50: 393440.
10 Moore RA, Derry S. Systematic review and meta-analysis of randomised trials and cohort
studies of mycophenolate mofetil in lupus nephritis. Arthritis Res Ther 2006; 8: R182.
11 Bruns A, Meyer O. Neuropsychiatric manifestations of systemic lupus erythematosus. Joint
Bone Spine 2006; 73: 63945.
04-PS Rheumatology-cpp :04-PS Rheumatology-ppp.QXD 18/3/08 14:30 Page 129

24 Sjgrens syndrome 129

P R O B L E M

24 Sjgrens Syndrome

Case History
You have been asked to see Wilma, aged 52 years, because she may have Sjgrens
syndrome (SS). Wilma has a teenage family of three and works part-time in the family dry
cleaning business. Recently she has become fatigued. Her optician recommended drops
for dry eyes and she complains of mouth dryness. Investigations have been normal apart
from a 1:160 positive antinuclear antibody (ANA).
What are the potential causes of sicca syndrome?
How would you investigate?
Assuming she has SS, how should she be followed-up and managed?

Background
After rheumatoid arthritis (RA), SS is the second most common autoimmune rheumatic
disease.13 It is named after Henrik Sjgren, who described a series of cases in 1933. SS
affects 0.5% of the population and is nine times more common in women. SS may affect
up to two million people in the United States. There are two peaks of onset one after
menarche and a second following menopause. The cardinal symptoms are dryness of the
eyes (keratoconjunctivitis sicca) and mouth (xerostomia) secondary to autoimmune dis-
ease of the exocrine glands (lacrimal and salivary). The international consensus criteria
for diagnosis of primary SS are summarized in Box 24.1. Diagnosis of secondary SS can
be made when there is an established connective tissue disease, at least one symptom of
sicca syndrome and at least two objective tests to confirm the diagnosis. SS associated
with RA may produce more lacrimal than oral symptoms. Secondary SS may occur in up
to 40%50% of patients with RA, 10%30% of those with SLE and may also occur in sys-
temic sclerosis, polymyositis and polychondritis. Tests to measure tear and saliva pro-
duction should be carried out when the patient is not taking anticholinergic drugs. At
least 50% of glandular tissue needs to be lost before patients begin to experience symp-
toms. The following should be considered in a differential diagnosis: radiation to the
head and neck, sarcoidosis, infection (human immunodeficiency virus [HIV], human
T-lymphotropic virus type 1 and hepatitis C) and graft-versus-host disease.
In SS, the exocrine glands are infiltrated by CD4+ helper T lymphocytes, which trigger
activation of cytotoxic T cells and B cells with consequent production of autoantibody
and destruction of glandular tissue. Anti-SSA/Ro or anti-SSB/La are present in a high
proportion of patients and may have a diagnostic sensitivity comparable with that of

Atlas Medical Publishing Ltd


04-PS Rheumatology-cpp :04-PS Rheumatology-ppp.QXD 18/3/08 14:30 Page 130

130 04 Systemic Lupus Erythematosus, Sjgrens Syndrome and Scleroderma

Box 24.1 International consensus criteria for diagnosis of SS


Ocular symptoms (at least one)
Persistent dry eyes every day for at least three months
Recurrent sensation of sand or gravel in the eyes
Use of a tear substitute more than three times per day
Oral symptoms (at least one)
Persistent dry mouth every day for at least three months
Recurrent feeling of swollen salivary glands
Need to drink liquid in order to swallow dry food
Objective evidence of dry eyes (at least one)
Schirmers test
Rose-Bengal
Lacrimal gland biopsy with focus score >1 (at least 50 lymphocytes per lobule with
four lobules assessed)
Objective evidence of salivary gland involvement (at least one)
Salivary gland scintigraphy
Parotid sialography
Unstimualted whole sialometry (1.5 ml per 15 minutes)
Laboratory abnormality (at least one)
Anti-SSA or anti-SSB
ANA
Immunoglobulin M (IgM) rheumatoid factor
Positive biopsy of a minor salivary gland
Focus score >1 (see above)
At least four of the above six criteria should be satisfied for a positive diagnosis of
primary SS

salivary gland biopsy.4 ANA is positive in the majority, but is not detectable in up to 25%
of patients. Other autoantibodies may also be detected, particularly in secondary SS.

Dry eyes
Over 85% of patients with SS present with sicca syndrome. Decreased tear production can
be demonstrated using Schirmers test, where a strip of filter paper is placed under the
lower eyelid. The degree of wetting of the paper over five minutes is measured. There is an
age-related decrease in tear formation, probably contributed to by androgen or oestrogen
deficiency. Subjects who habitually use computer screens may complain of dry eyes
because of decreased blinking, particularly if they work in a low-humidity environment.
Not only is the volume of tears decreased in sicca syndrome, but there is also decreased
tear content of lysozyme and lactoferrin. Management may include use of artificial tears,
topical immunosuppressives (corticosteroid, cyclosporin) and punctal occlusion.

Dry mouth
Patients with dry mouth are at risk of oral candidiasis, dental caries and fissuring of the
tongue and gums. Decreased saliva flow is easily demonstrated using sialometry the
04-PS Rheumatology-cpp :04-PS Rheumatology-ppp.QXD 18/3/08 14:30 Page 131

24 Sjgrens syndrome 131

patient discharges his/her saliva into a vessel for 15 minutes and the amount of saliva is
weighed. Flow rates of <0.1ml/min are abnormal. Differential diagnosis of dry mouth is
presented in Box 24.2. Salivary flow is stimulated by acetylcholine acting through mus-
carinic M1 and M2 receptors. Drugs with anticholinergic action (e.g. tricyclic antidepres-
sants) should be avoided where possible. Muscarinic secretagogues (pilocarpine or
cevimiline) may improve salivary flow. Patients should avoid alcohol and smoking if pos-
sible, and not use alcohol-containing mouthwashes. Careful attention should be paid to
oral and dental hygiene. Antibacterial mouthwashes (e.g. chlorhexidine) may be useful.
Chewing sugar-free gum or use of artificial saliva may provide temporary relief. Fluoride-
releasing preparations including dental varnishes are often used. A topical preparation of
interferon is being tried although, as with decreased lacrimation, the glands are largely
destroyed by the time symptoms develop and immunomodulatory treatment is likely to
be of limited use.

Extraglandular manifestations
These are extremely variable but can pose a considerable threat to health. Skin lesions
include palpable and non-palpable purpura, alopecia, vitiligo and an increased incidence
of cutaneous lymphoma. Arthralgia is typically symmetrical and non-erosive, although
erosive arthralgia can occur. Muscular pains can produce a picture like fibromyalgia or
polymyalgia, but inflammatory polymyositis can occur. Gastrointestinal manifestations
include swallowing problems related to decreased saliva, atrophic gastritis, abnormal
liver tests and autoimmune liver disease. The lungs may be affected by interstitial pneu-
monitis and there is an increased risk of pulmonary lymphoma. Pericarditis and pul-
monary hypertension can occur. Renal problems, including interstitial nephritis, may
occur with SS or as a result of immunosuppressives. Cranial and peripheral neuropathies

Box 24.2 Differential diagnosis of xerostomia


Sjgrens syndrome Drugs
primary Anticholinergics
secondary Tricyclic antidepressants
Antihistamines
Salivary duct aplasia or atresia
Benzhexol, benztropine
Salivary duct stones
Entacapone
Sialadenitis
Loperamide
bacterial (staphylococcal,
Diuretics
streptococcal)
viral (e.g. mumps)
Smoking
Irradiation or chemotherapy Alcohol
Marijuana
Dehydration, mouth breathing
Diabetes (mellitus or insipidus) Taste disturbances
Hypercalcaemia Head/neck trauma
Hyperlipoproteinaemia
Sarcoidosis
Autonomic neuropathy
Amyloidosis
LambertEaton syndrome
Cirrhosis
04-PS Rheumatology-cpp :04-PS Rheumatology-ppp.QXD 18/3/08 14:30 Page 132

132 04 Systemic Lupus Erythematosus, Sjgrens Syndrome and Scleroderma

are described. Autoimmune thyroiditis is very common. The chronic and disabling
symptoms may lead to anxiety, depression or other psychological problems. There is a
40-fold increase in the risk of lymphoma in SS. Manifestations may include enlarging
parotids, hypergammaglobulinaemia and hepatosplenomegaly.
The presence of systemic features may necessitate the prescription of disease-
modifying drugs. Hydroxychloroquine (68 mg/kg/day) is useful where there is myalgia
or arthralgia. Visceral manifestations may respond to low-dose corticosteroids (pred-
nisolone <15 mg/day). Other immunosuppressives used include azathioprine (12 mg/
kg/day), methotrexate (7.525 mg/week) and cyclophosphamide (0.51 g/m2/day).
Leflunomide or cyclosporin are useful in refractory cases.

Recent Developments
1 Both sarcoidosis and SS affect salivary and lacrimal glands and they share many
systemic features. The presence of autoantibodies obviously favours SS, while hilar
lymphadenopathy or hypercalcaemia would point towards sarcoidosis. A number of
cases have now been identified where the two conditions coexist.5 While this may be
coincidence, it could point to the presence of common aetiological factors.
2 Geographical distribution and seasonality of onset suggest that there could be an
infectious trigger. Amongst the leading candidates are Coxsackie viruses, which
appear to have a predilection for exocrine glands. Recently, Coxsackie virus B4
sequences have been identified in the salivary glands of SS patients.6 Subclinical
infection may trigger localization of helper lymphocytes in the salivary glands
leading to lymphocytic and dendritic cell activation.
3 Anti-centromere antibodies are characteristic of limited forms of scleroderma but
can also be detected in some patients with SS. The molecular determinants are a
group of centromere proteins (CENP-A, -B, -C, -D, -E, -F, -G and -H). Antibodies
to CENP-B or CENP-C have been described in about one-half of patients with
limited scleroderma and one-fifth of those with SS.7 By contrast, antibodies to
CENP-H may be more specific for SS, and may identify a distinct group separate
from those who are positive for anti-Ro and anti-La.8
4 The difficulty in defining SS as a clinical entity has led to a quest for more specific
serological markers. Antibodies to the protein a-fodrin were described about ten
years ago. a-fodrin is an intrinsic membrane protein that binds to calmodulin, actin
and microtubules, and is involved in regulation of exocytosis. A recent study9 has
confirmed that these antibodies are present in a small proportion of a large series of
SS patients, and may be positive in some who are negative for anti-Ro and anti-La.

Conclusion
Normally, up to 500 ml of saliva is produced daily. SS is an autoimmune condition, and
investigations should confirm diminished lacrimal and salivary secretion. Ideally,
autoimmune markers should be positive, and evidence of lymphocytic infiltration
should be demonstrated on a biopsy of lacrimal or salivary gland. SS may be primary or
occur in association with an underlying connective tissue disease. If the disease is thought
to be primary, careful inquiry should be made about systemic symptoms and involve-
04-PS Rheumatology-cpp :04-PS Rheumatology-ppp.QXD 18/3/08 14:30 Page 133

24 Sjgrens syndrome 133

Dry eyes
Decreased lacrimation (test)
Dry mouth
4 out of 6 positive Underlying CT disease
Decreased saliva (test)
Autoantibody (Ro, La, ANA)
Positive salivary/lacrimal biopsy

Primary SS Secondary SS

Dry eyes
Artificial tears
Punctal closure
Local immunosuppressives

Exclude other causes

Dry mouth
Dental/oral hygiene
Pilocarpine, cevimiline
Artificial saliva

Systemic/visceral involvement

Low-dose prednisolone

Azathioprine, cyclophosphamide or methotrexate

Leflunomide or cyclophosphamide

Figure 24.1 Investigation and management of SS. CT, connective tissue.

ment of organs other than the exocrine glands. Treatment is mainly symptomatic and
topical (Figure 24.1), with immunosuppressive treatment being reserved for patients
with involvement of other organs including the lung and kidney.

Further Reading
1 Derk CT, Vivino FB. A primary care approach to Sjgrens syndrome. Postgrad Med 2004; 116:
4959.
04-PS Rheumatology-cpp :04-PS Rheumatology-ppp.QXD 18/3/08 14:30 Page 134

134 04 Systemic Lupus Erythematosus, Sjgrens Syndrome and Scleroderma

2 Fox RI. Sjgrens syndrome. Lancet 2005; 366: 32131.


3 Fox RI, Liu AY. Sjgrens syndrome in dermatology. Clin Dermatol 2006; 24: 393413.
4 Kessel A, Toubi E, Rozenbaum M, Zisman D, Sabo E, Rosner I. Sjgrens syndrome in the
community: can serology replace salivary gland biopsy? Rheumatol Int 2006; 26: 3379.
5 Ramos-Casals M, Brito-Zeron P, Garcia-Carrasco M, Font J. Sarcoidosis or Sjgren
syndrome? Clues to defining mimicry or coexistence in 59 cases. Medicine 2004; 83: 8595.
6 Triantafyllopoulou A, Moutsopoulos HM. Autoimmunity and coxsackievirus infection in
primary Sjgrens syndrome. Ann NY Acad Sci 2005; 1050: 38996.
7 Gelber AC, Pillemer SR, Baum BJ et al. Distinct recognition of antibodies to centromere pro-
teins in primary Sjgrens syndrome compared with limited scleroderma. Ann Rheum Dis
2006; 65: 102832.
8 Hsu TC, Chang CH, Lin MC, Liu ST, Yen TJ, Tsay GJ. Anti-CENP-H antibodies in patients
with Sjgrens syndrome. Rheumatol Int 2006; 26: 298303.
9 Ruiz-Tscar JL, Lpez-Longo FJ, Snchez-Ramn S et al. Prevalence of IgG anti-a-fodrin anti-
bodies in Sjgrens syndrome. Ann NY Acad Sci 2005; 1050: 21016.

P R O B L E M

25 Raynauds Phenomenon

Case History
Rachel is aged 14 years and presents because she has poor circulation. Her mother
describes Rachels fingers as going white, blue and then red when the weather is cold or
when Rachel has been exercising. Her mother is concerned about this and wants to know
what it may mean for Rachels future health.
What is Raynauds phenomenon?
How would you determine whether it is a marker for future disease?
What treatment options exist for management?

Background
In 1862, a medical student Maurice Raynaud described the phenomenon that bears his
name; it is characterized by transient cessation of blood flow to the digits of the hands or

Atlas Medical Publishing Ltd


04-PS Rheumatology-cpp :04-PS Rheumatology-ppp.QXD 18/3/08 14:30 Page 135

25 Raynauds phenomenon 135

Box 25.1 Triphasic colour changes of Raynauds phenomenon


White: excessive vasoconstriction and cessation of regional blood flow
Blue: cyanosis due to residual blood desaturation
Red: hyperaemia as the attack subsides and blood flow is restored

feet (Box 25.1). Occasionally other structures are involved such as the tip of the nose, an
earlobe, nipple or even tongue.
Raynauds phenomenon occurs in 3%5% of the population, although symptoms have
been reported in 10% of women and 8% of men. The prevalence is higher in women, with
onset generally between menarche and menopause, and the severity is greater over this
period. Between 80% and 90% of Raynauds phenomenon is termed primary and has no
identifiable cause or associated disease. Secondary Raynauds phenomenon is most often
found in association with an underlying rheumatic disease (Box 25.2). There is some evi-
dence for a genetic predisposition, especially in those with early onset (age <40 years).

Box 25.2 Primary and secondary Raynauds phenomenon


Primary Raynauds phenomenon An uncomplicated and usually benign condition,
often with a long-standing history, beginning in adolescence, with no or minimal tissue
damage.
Secondary Raynauds phenomenon Associated with underlying connective tissue
disease (systemic sclerosis or mixed connective tissue disease), with potentially
disabling ulceration or tissue necrosis. Onset is usually after age 3040 years. Less
common causes are use of vibrating tools, paraproteinaemias, cryoglobulinaemia, large
blood vessel disease and drugs (b-blockers, bleomycin, cisplatin, fentanyl, nicotine).

The Practice Points (Box 25.3) identify a number of factors that can be evaluated in
predicting the likelihood of the Raynauds presentation being the first indicator of a more
serious disease. Of the four points listed, the absence of distorted nailfold capillaries is the
most predictive of a benign outcome. Even patients with isolated Raynauds with nailfold
capillaroscopy abnormalities and/or antinuclear antibody have only a 10%15% likeli-
hood of developing a connective tissue disease during long-term follow-up.

Box 25.3 Practice Points


Raynauds phenomenon as a precursor of an underlying connective tissue disease such
as scleroderma or systemic lupus erythematosus is very unlikely with:

Absence of tissue damage such as pulp scarring or atrophy


Normal erythrocyte sedimentation rate
Negative antinuclear antibody
Absence of distortion of nailfold capillaries observed with an ophthalmoscope set
at 40+ dioptre
04-PS Rheumatology-cpp :04-PS Rheumatology-ppp.QXD 18/3/08 14:30 Page 136

136 04 Systemic Lupus Erythematosus, Sjgrens Syndrome and Scleroderma

Table 25.1 Pathogenetic factors

Increased vasoconstriction Reduced vasodilation

Vascular factors Endothelin-1 NO and prostacyclin


Angiotensin II
Thromboxane A2
Neural factors Activation of a2-adrenoreceptors Calcitonin gene-related peptide
Circulating factors Platelet-derived thromboxane and serotonin
Oxidative stress damaging endothelium
Flow factors Reduced fibrinolysis predisposing to fibrin deposition and vascular obstruction
Increased viscosity and reduced red cell deformability
NO, nitric oxide.

Pathogenesis
The association with female gender suggests a hormonal influence. Increased vascular
reactivity also occurs in the pre-ovulatory part of the menstrual cycle. A large epidemio-
logical study found no association between Raynauds phenomenon and smoking,
although severity may be worse in smokers. The pathogenesis of Raynauds phenomenon
has been reviewed by Herrick1 and by Boin and Wigley.2 The overall model of aetiology is
an imbalance of vasoconstriction over vasodilation. This results from changes in the vas-
cular endothelium and smooth muscle, neural mediators of vascular tone and circulating
platelet-derived mediators. These factors are in turn moderated by the level of physical
activity, ambient temperature, emotional state and direct trauma or inflammation of the
vessels. Table 25.1 summarizes the multiple influences that underlie the pathogenesis.

Non-pharmacological therapy
Education and introduction of conservative management are needed for all patients
(Table 25.2). Avoidance of cold exposure is advocated, including frozen food sections of
supermarkets. Patients should maintain or try and increase the temperature of the chest
and abdomen to enhance peripheral vasodilation as a heat dissipation measure.
Thermal biofeedback (TBF) is a psychophysiological technique in which subjects are
trained to control peripheral vasoconstrictor responses and to acquire voluntary hand-warm-
ing skills. Initially taught with feedback such as peripheral temperature or blood flow, it is

Table 25.2 Treatment strategies

Non-pharmacological therapy Pharmacological therapy

Avoid cold, emotional stress, smoking Antiplatelet agents


Biofeedback
Vasodilation Vasoconstriction
Calcium channel blockers Angiotensin II receptor antagonists
Sildenafil/tadalafil ACE inhibitors
Intravenous/oral prostanoids Bosentan
L-arginine supplementation Serotonin reuptake inhibitors
a1/a2-adrenergic blockers
ACE, angiotensin-converting enzyme.
04-PS Rheumatology-cpp :04-PS Rheumatology-ppp.QXD 18/3/08 14:30 Page 137

25 Raynauds phenomenon 137

hypothesized that subsequent hand-warming can occur without the feedback instrument. A
systematic review of eight randomized controlled trials (RCTs) highlighted major shortcom-
ings in study design, particularly in the teaching of TBF and taking account of environmental
factors.3 However, the authors concluded that TBF treatment was efficacious.
Cervical spinal cord stimulation has been proposed for severe Raynauds phenom-
enon, but remains controversial and is not yet supported by good clinical trials. Low-
level laser therapy, applied to the fingers and dorsum of the hand for 30 minutes in five
sessions over three weeks, significantly decreased the frequency and severity of attacks in
a sham-controlled double-blind trial. The mechanism of action remains unexplained.

Pharmacological treatment
The sympathetic nervous system mediates a tonic vasoconstrictive effect on the vascular wall,
with noradrenaline-induced vasoconstriction mediated by a1 and a2 adrenoreceptors. The
a2 receptors are more important in the regulation of digital vascular tone. Trials of non-
selective sympathetic blockers have not been successful. Prazosin, an a1 receptor antagonist,
showed a modest benefit over placebo, but probably insufficient to balance side effects. An a2
receptor antagonist has been developed, and in an RCT reduced the frequency of Raynauds
attacks in scleroderma-associated Raynauds, but not in primary Raynauds patients.
Calcium channel blockers (CCBs), particularly of the dihydropyridine class, are wide-
ly prescribed for Raynauds phenomenon. A meta-analysis evaluated the effectiveness of
CCBs in primary Raynauds and noted an average reduction of three to five attacks per
week and a one-third reduction in severity.4 Nifedipine is the most studied and has bene-
fits over placebo and nicardipine. Adverse effects of CCBs include flushing, headache,
oedema and tachycardia.
Prostacyclin is a potent, short-lived vasodilator that also has antiproliferative effects
on smooth muscle and reduces platelet aggregation. Intravenous iloprost decreases the
number and duration of vasospastic episodes, but its use usually requires central-line
administration and is complicated by cost, hypotension and parotid and menstrual pain.
Serotonin is a potent platelet-derived vasoconstrictor, and it is therefore counter-
intuitive to use selective serotonin reuptake inhibitors (SSRIs) as they would be expected
to increase the plasma concentration of serotonin. Despite this, several case reports
describe the use of SSRIs in Raynauds phenomenon, with both beneficial and worsening
outcomes. Fluoxetine decreases the frequency and severity of vasospastic episodes and
increases the rate of rewarming after a cold challenge.
Angiotensin II receptor antagonists and angiotensin-converting enzyme inhibitors
have both shown effects on endothelial function and remodelling. On this basis, their use
in Raynauds phenomenon is being explored. The frequent findings of microthrombin in
secondary Raynauds and evidence of platelet activation suggest a potential benefit from
antiplatelet therapy or anticoagulation. Two controlled trials that studied aspirin and
dipyridamole in patients who had scleroderma failed to show a benefit; however, many
clinicians recommend low-dose aspirin unless there is a contraindication.

Recent Developments
1 Sildenafil is a selective inhibitor of cyclic guanosine monophosphate (cGMP)-specific
phosphodiesterase type V; its use leads to an increase in cGMP and thus it
04-PS Rheumatology-cpp :04-PS Rheumatology-ppp.QXD 18/3/08 14:30 Page 138

138 04 Systemic Lupus Erythematosus, Sjgrens Syndrome and Scleroderma

potentiates the effect of nitric oxide-mediated activation of guanylate cyclase. cGMP


causes a decrease in intracellular calcium, resulting in vascular smooth muscle
relaxation and dilatation. Lichtenstein was the first to successfully use sildenafil for
Raynauds phenomenon in a group of ten patients.5 Fries et al.6 reported a crossover
study of 16 patients with secondary Raynauds who had failed to respond to at least
two conventional vasodilatory agents. Active treatment was sildenafil 50 mg bd and
the frequency and duration of symptoms of Raynauds phenomenon decreased
significantly, leading to improved well-being of the patients. In this study, capillary
blood flow was severely impaired and sometimes hardly detectable, with sildenafil
leading to a >400% increase in flow velocity. Despite its short half-life (four hours),
the data on capillary flow and symptom control suggest a longer functional effect of
sildenafil. Further refinements in phosphodiesterase inhibition are on the horizon,
with the longer-acting tadalafil (half-life 17 hours) having been used in a patient with
secondary Raynauds who was resistant to sildenafil.7
2 Cilostazol (100 mg bd) is a phosphodiesterase type III inhibitor that in a crossover
trial showed no change in symptoms and only a very modest increase in flow-
mediated dilation of the brachial artery in patients with primary and scleroderma-
related Raynauds.8
3 Bosentan is a non-selective endothelin A/B antagonist. In a trial of scleroderma
patients with digital ulcers, bosentan almost halved the development of new ulcers
compared to placebo during the four-month trial.9 However, there was no benefit on
healing of existing ulcers, and self-assessed Raynauds severity was similar between
active and placebo therapy. In a case report of severe Raynauds phenomenon,
beneficial effects of bosentan in healing of gangrenous digits were observed. A
reduction of CD146 cells a population of mainly endothelial cells was noted,
suggestive of reduced vascular injury. A concomitant increase in CD34 cells which
are stem cells containing endothelial progenitors suggested increased vascular
repair.10

Conclusion
Cold-induced vasospasm is common in the teenage years and for the majority of patients
does not require medical review. For those patients presenting to a doctor, it is helpful to
have a scheme to identify those at risk of developing a connective tissue disease. The major-
ity of patients will fulfil the criteria for primary Raynauds phenomenon and require no fur-
ther investigation. Avoidance of cold and emotional triggers is important, as once
vasospasm has occurred its resolution can be protracted and painful. A range of treatments
is under investigation, although at this time the benefit:risk ratio is marginal for most.

Further Reading
1 Herrick AL. Pathogenesis of Raynauds phenomenon. Rheumatology 2005; 44: 58796.
2 Boin F, Wigley FM. Understanding, assessing and treating Raynauds phenomenon. Curr Opin
Rheumatol 2005; 17: 75260.
3 Karavidas MK, Tsai P-S, Yucha C, McGrady A, Lehrer PM. Thermal biofeedback for primary
Raynauds phenomenon: a review of the literature. Appl Psychophysiol Biofeedback 2006; 31:
20316.
04-PS Rheumatology-cpp :04-PS Rheumatology-ppp.QXD 18/3/08 14:30 Page 139

26 Assessing and treating scleroderma 139

4 Thompson AE, Pope JE. Calcium channel blockers for primary Raynauds phenomenon: a
meta-analysis. Rheumatology 2005; 44: 14550.
5 Lichtenstein JR. Use of sildenafil citrate in Raynauds phenomenon: comment on the article by
Thomson et al. Arthritis Rheum 2003; 48: 2823.
6 Fries R, Shariat K, von Wilmowsky H, Bhm M. Sildenafil in the treatment of Raynauds
phenomenon resistant to vasodilatory therapy. Circulation 2005; 112: 29805.
7 Baumhaekel M, Scheffler P, Boehm M. Use of tadalafil in a patient with secondary Raynauds
phenomenon not responding to sildenafil. Microvasc Res 2005; 69: 1789.
8 Rajagopalan S, Pfenninger D, Somers E et al. Effects of cilostazol in patients with Raynauds
syndrome. Am J Cardiol 2003; 92: 131015.
9 Korn JH, Mayes M, Matucci Cerinic M et al. Digital ulcers in systemic sclerosis: prevention by
treatment with bosentan, an oral endothelin receptor antagonist. Arthritis Rheum 2004; 50:
398593.
10 Dunne J, Dutz J, Shojania K, Ng B, van Eeden S. Treatment of severe Raynauds phenomenon
with bosentan in a patient with systemic sclerosis. Rheumatology 2006; 45: 91112.

P R O B L E M

26 Assessing and Treating Scleroderma

Case History
Mary is 42 years old and presents with increasing stiffness of her hands. Since her mid-
thirties she had noted painful blanching of her fingers in cold water or when handling
cold objects. Last winter she had ulcers on her fingertips and she had indigestion most of
this year. She has bilateral sclerodactyly. Her blood pressure (BP) is 145/93 mmHg and she
is easily fatigued.
What clinical and laboratory features will assist you in the diagnosis?
What treatments should be used for scleroderma?

Atlas Medical Publishing Ltd


04-PS Rheumatology-cpp :04-PS Rheumatology-ppp.QXD 18/3/08 14:30 Page 140

140 04 Systemic Lupus Erythematosus, Sjgrens Syndrome and Scleroderma

Background
Scleroderma is a rare condition with a prevalence of approximately 1 in 5000, and an
annual incidence of 1 in 50 000 to 1 in 300 000. Strictly, scleroderma denotes disease lim-
ited to the skin and subcutaneous tissues (localized scleroderma, or morphea), and dis-
ease involving the internal organs is systemic sclerosis (SSc). Morphea is characterized by
fibrosis of the skin and adjacent structures. Morphea can be differentiated from SSc by
the distribution of lesions (no sclerodactyly or perioral involvement), absence of
Raynauds phenomenon and/or periungual telangiectasia.1
The generalized form of disease can be divided into limited cutaneous SSc/sclero-
derma and diffuse cutaneous SSc/scleroderma; limited cutaneous disease does not extend
beyond the elbow and knee, although the face may be involved. In diffuse cutaneous dis-
ease, the skin lesions involve the more proximal limb and trunk. Diffuse scleroderma is
more common in African-Americans, black patients in Africa and the Choctaw Indian.
Genome-wide scanning has identified susceptibility loci on three different non-HLA
chromosomes: fibrillin on chromosome 15, SPARC (secreted protein acidic and rich in
cysteine) on chromosome 5 and topoisomerase 1 on chromosome 20.
The characteristics of diffuse and limited cutaneous scleroderma are presented in
Table 26.1. CREST is often but not always used as a synonym for limited cutaneous scle-
roderma, and dates from the early 1960s. CREST patients have three or more of the five
syndromic features:
Calcinosis
Raynauds phenomenon
oEsophageal dysmotility
Sclerodactyly
Telangiectasia
CREST can be considered with limited cutaneous SSc. Wollheim proposed that the
five CREST components were not meant to serve as criteria of a special subset of
scleroderma, and features of CREST occur with time irrespective of other disease
characteristics.2

Table 26.1 Characteristics of diffuse and limited scleroderma

Diffuse cutaneous scleroderma Limited cutaneous scleroderma

History of Raynauds phenomenon with onset within one year Skin involvement restricted to hands, face, forearm, feet
Skin sclerosis proximal to the elbow, and may involve trunk Prominence of calcinosis and telangiectasia
Tendon friction rubs may occur
Nailfold capillary dilation, aneurysm and dropout Nailfold capillary dilation, aneurysm and dropout
Early onset of pulmonary, renal and diffuse gastrointestinal Delayed but often severe pulmonary hypertension
involvement
Often anti-topoisomerase 1 antibodies (anti-Scl-70); rarely Anti-centromere antibodies common especially with
anti-centromere antibodies calcinosis and telangiectasia
Anti-RNA polymerase III
Ten-year survival: 54%62% Ten-year survival: 69%89%
04-PS Rheumatology-cpp :04-PS Rheumatology-ppp.QXD 18/3/08 14:30 Page 141

26 Assessing and treating scleroderma 141

Ninety-five per cent of patients with scleroderma have a positive antinuclear antibody
(ANA) test, but specific patterns are not reliably diagnostic. Both anti-centromere and
anti-topoisomerase 1 (anti-Scl-70) have a low sensitivity, and a diagnosis of scleroderma
is not excluded by a negative result. Anti-centromere antibodies favour a diagnosis of
limited cutaneous scleroderma. Anti-topoisomerase 1 antibodies are only 30% sensitive,
but are highly specific for SSc and correlate with interstitial lung disease.
The skin capillaries can be visualized at the nailfold using an ophthalmoscope or a der-
moscope, or in the research situation using video capture. A range of abnormalities have
been described including dilatation and aneurysm formation, dropout, haemorrhage
and budding. Three-quarters of scleroderma patients have abnormalities of their nailfold
capillaries, and the presence of normal capillaries is the strongest negative predictor of
scleroderma in patients with Raynauds phenomenon. There appears to be no nailfold
capillary difference between limited cutaneous and diffuse cutaneous scleroderma.
The initial complaint of limited cutaneous scleroderma is Raynauds phenomenon,
whereas patients with diffuse cutaneous scleroderma often present with generalized hand
swelling, skin thickening or arthralgias with or without Raynauds phenomenon. They
may initially complain of tight, puffy fingers that occurs in the morning but then lasts all
day. This is the oedematous stage, with oedema usually non-pitting and painless. This
usually progresses to a thickened, tight indurative stage over months to years, with the
skin becoming shiny and adherent to the underlying subcutis. Dermal thickening and
epidermal thinning lead to loss of dermal appendages, with hair loss and absent sweating.
Mottled hyper- and hypopigmentation may be noted. Facial involvement may lead to a
pinched nose, tightened lips and a mouse-like appearance. Finally, there is an atrophic
stage in which the dermis softens and thins but remains firmly bound down to the subcu-
taneous fat. With limited cutaneous scleroderma, clusters of dilated tortuous capillaries
and venules are noted. Joint pain, immobility and contractures are the result of fibrosis
around tendons and other periarticular structures. Contractures of the hand are most
common, but large joint contractures may occur. Some patients have palpable and/or
audible deep tendon friction rubs due to fibrinous tenosynovitis.
Skin ulcers are of two forms: digital tip ulcers secondary to ischaemia, and ulceration
over bony prominences where the skin is contracted and tight and susceptible to trauma.
Reduced vascularity perpetuates poor healing. Both limited and late diffuse cutaneous
scleroderma may develop calcinosis at areas of pressure such as the finger pads, olecra-
non bursa, extensor surface of forearm, buttocks and around the patella.
The gastrointestinal (GI) tract is the second most commonly affected organ, with
involvement in 75%90% of cases. The hallmark effect is smooth muscle fibrosis, leading
to dysmotility, with 80% of patients developing oesophageal dysfunction. Impaired gas-
tric emptying and decreased lower oesophageal sphincter tone allow reflux with heart-
burn and dysphagia. In the longer term, Barretts oesophagus and strictures may develop,
and the risk of adenocarcinoma of the oesophagus is increased. Dysmotility occurs
throughout the GI tract and can cause gastroparesis, small bowel bacterial overgrowth
and malabsorption, pseudo-obstruction, severe constipation and megacolon.
Pulmonary involvement occurs in more than 70% of cases and is the most common
cause of SSc-related death. Pulmonary fibrosis is common and can cause severe restrict-
ive lung disease. Chest X-ray shows lower lobe interstitial thickening in a reticular
nodular pattern. High-resolution computed tomography (HRCT) may show ground-glass
04-PS Rheumatology-cpp :04-PS Rheumatology-ppp.QXD 18/3/08 14:30 Page 142

142 04 Systemic Lupus Erythematosus, Sjgrens Syndrome and Scleroderma

appearance, representing alveolitis, or fibrotic honeycombing with traction bronchiecta-


sis. Pulmonary function testing can serially document changes in lung volumes with
restriction and increasing impairment of gas transfer. Readily detected by right heart
catheterization and echocardiograph, pulmonary artery hypertension (PAH) occurs in
20%40% of patients with SSc. The five-year cumulative survival is 10% in those with
PAH compared with 80% in those without. PAH can develop early, particularly in limit-
ed cutaneous SSc, and can occur without pulmonary fibrosis. In diffuse cutaneous SSc,
development of pulmonary fibrosis may lead to secondary PAH. The patient develops
dyspnoea disproportionate to any lung findings, and may have accentuation of the pul-
monary component of the second heart sound. A clue to PAH is disproportionate reduc-
tion in the carbon monoxide diffusion capacity relative to the forced vital capacity
(FVC).
Renal disease occurs almost only in those with diffuse cutaneous disease. Scleroderma
renal crisis, which could result in rapid death or dialysis, is essentially a thing of the past
due to the use of angiotensin-converting enzyme (ACE) inhibitors. The crisis occurs
most often in patients with early diffuse cutaneous SSc in the context of rapid skin pro-
gression. It is characterized by accelerated hypertension presenting as headache, short-
ness of breath, seizures, retinal haemorrhage, left heart failure and renal failure.

Prognosis
Italian and French-Canadian studies report increasing mortality with increasing skin
involvement using a diffuse, intermediate and limited classification. A five-year study
from the United Kingdom (UK) identified advanced age, diffuse cutaneous disease, high-
er skin score, elevated erythrocyte sedimentation rate (ESR) and signs of organ involve-
ment as poor prognostic factors in univariate analysis. However, logistic regression
analysis identified proteinuria, elevated ESR and reduced carbon monoxide diffusion
capacity as the most important factors.3,4 PAH has a devastating impact on survival and
outcome, because the progressive elevation of pulmonary artery pressure leads to right
ventricular failure, arrhythmias and death. The mean survival of untreated patients with
primary PAH is 2.8 years from diagnosis. In untreated and treated patients with SSc-
associated PAH, mean survival is less than one year in those with severe disease.
Several manifestations are associated with specific antibodies: interstitial lung disease
with anti-topoisomerase 1 (Scl-70), and anti-centromere antibodies with digital necrosis
and delayed pulmonary hypertension independent of pulmonary fibrosis. Anti-RNA
polymerase III antibodies in limited cutaneous scleroderma correlate with severe skin
and renal involvement. U3-RNP antibodies, including fibrillarin, occur in both limited
and diffuse disease and correlate with pulmonary hypertension, cardiac involvement and
myositis. In a multicentre study of 1012 Italian patients, ten-year actuarial survival was
72% in patients with and 81% in those without anti-centromere antibody, and 73% in
patients with ANA.5 Overall, ANA is of limited help in predicting prognosis.

Pathophysiology and treatment


Three abnormalities have been identified:
1 Fibroblast dysfunction leading to increased production and deposition of extracellu-
lar matrix, particularly collagen
04-PS Rheumatology-cpp :04-PS Rheumatology-ppp.QXD 18/3/08 14:30 Page 143

26 Assessing and treating scleroderma 143

2 Vascular abnormality leading to tissue hypoxia


3 An immune response manifest as altered T- and B-lymphocyte function and produc-
tion of autoantibodies
Endothelial damage and vascular dysfunction occur early, with activated lymphocytes
secreting transforming growth factor-b (TGF-b) causing endothelial injury and upregu-
lation of the major histocompatibility complex, adhesion molecules and connective tis-
sue growth factor (CTGF). TGF-b maintains fibroblasts in an activated state, promotes
collagen and matrix protein production and decreases synthesis of collagen-degrading
matrix metalloproteinases. CTGF is an important growth factor in fibrosis, triggering
angiogenesis and the structural organization of connective tissue. Endothelial damage
results in the increased production of endothelin (ET)-1 and impaired prostacyclin
release, resulting in imbalance between vasodilators and vasoconstrictors. ET combined
with platelet-derived factors changes the vascular smooth muscle cell into a myofibro-
blast, with proliferation of these narrowing the vessel lumen. Later in the disease process
the inflammatory vasculopathy subsides and fibrosis dominates, with skin thickening,
interstitial lung fibrosis, and GI and renal disease.
Skin: The natural history of skin involvement is quite variable, with striking improve-
ment occurring without intervention. Cyclophosphamide, mycophenolate mofetil and
methotrexate are all used, but only methotrexate has shown a statistical benefit in con-
trolled trials.
Kidney: Renal crises are treated with ACE inhibitors titrated to achieve a systolic BP
reduction of 1020 mmHg over 24 hours. Continuous low-dose prostacyclin improves
BP control and renal blood flow. Creatinine clearance is monitored and ACE inhibitors
maintained even during dialysis. There is a prolonged period of potential renal recovery.
Refractory hypertension may require calcium channel blockers or a-adrenergic blockers.
Pulmonary fibrosis: The greatest evidence of efficacy is available for cyclophos-
phamide. Recent trials include:
1 A UK study of 45 SSc patients who received low-dose prednisolone and six infusions
of cyclophosphamide monthly, followed by oral azathioprine or placebo. At one year
this did not demonstrate improvement in the primary endpoints (FVC and diffusing
capacity for carbon monoxide [DLCO]) or secondary endpoints (HRCT appearance
and dyspnoea scores).6
2 A United States study of 158 SSc patients with interstitial lung disease who received
oral cyclophosphamide (2 mg/kg/day) or placebo for one year and were followed
for one additional year.7 The mean difference in FVC at 12 months was 2.5% favour-
ing cyclophosphamide, which was maintained at 24 months. The clinical benefit was
modest, aiding skin scores, dyspnoea and health-related quality of life.
Pulmonary hypertension: Early diagnosis is essential to optimize outcome. Mortality
and morbidity of PAH has been improved by the use of long-term oxygen therapy,
diuretics and digoxin for symptom relief, and anticoagulation. Intravenous prostacyclin
leads to a rapid improvement in cardiac output, exercise capacity and survival but is
expensive and associated with adverse effects.
Gastrointestinal: Oesophageal dysmotility occurs in the majority and requires regular
use of proton pump inhibitors, sometimes with prokinetic agents. Monitoring for
04-PS Rheumatology-cpp :04-PS Rheumatology-ppp.QXD 18/3/08 14:30 Page 144

144 04 Systemic Lupus Erythematosus, Sjgrens Syndrome and Scleroderma

Barretts oesophagus is indicated. Broad-spectrum antibiotic and prokinetic agents may


aid midgut disease. Stool frequency can be improved with frequent, small, low-fibre
meals, with alternating aperients and antidiarrhoeals.

Recent Developments
1 ET is a key mediator in the pathology of PAH and also SSc, participating in
fibrotic, hypertrophic and inflammatory processes. The oral dual ET-A/ET-B
receptor antagonist bosentan has proven benefit in patients with severe PAH. In a
six-month open-label prospective study, eight patients with PAH and pulmonary
fibrosis related to SSc received bosentan 62.5 mg bd for four weeks followed by
maintenance dosing of 125 mg daily.8 Mean six-minute walking distance
increased from 72 m at baseline to 192 m after three months, and to 203 m after
six months. Six of eight patients responded with improvement in functional class.
2 The two pivotal trials of bosentan included 52 patients with SSc and other connective
tissue diseases. During a four-month trial, mean treatment effect on six-minute
walking distance was 37 m. In the long-term extension, survival was 82% after one
year and 67% after two years.9 Bosentan has also been used in connective tissue
disease patients with PAH who were non-responsive to prostanoid therapy.10
Thirteen patients (primarily with SSc) were treated for one year. A progressive
improvement in exercise capacity was noted, with half improving their six-minute
walking distance.
3 Scleroderma/SSc and chronic graft-versus-host disease (GVHD) resemble each
other in tissue distribution (skin, lung, oesophagus), lymphocytic infiltration in
affected tissues and tissue fibrosis. It has also been suggested that SSc may be a
form of GVHD.11 GVHD occurs when foreign immunocompetent cells in grafted
or transfused tissue react with the recipients cell-surface antigens. To induce
GVHD, the graft must contain immunocompetent cells, the host must appear
foreign to these cells and the host must be incapable of mounting an effective
response and destroying them. The hypothesis of microchimaerism underlying SSc
was postulated in 1989. As an aetiological factor, microchimaeric cells could be
from previous pregnancies, or from maternal origin in the case of nulliparous
women or male patients. The mere presence of these cells in peripheral blood is
therefore not sufficient to cause SSc and a second, unknown activation step is
postulated. Microchimaeric cells are absent from the skin of normal subjects, and
are present in higher numbers in the normal skin of SSc subjects compared with
involved skin.12
4 Statins have more widespread effects than just lowering cholesterol (Figure 26.1).
There appear to be substantial differences amongst the statins as to their impact on
these alternate pathways.13 The potential of this approach has been trialled in an
open-label study of atorvastatin 10 mg/day for twelve weeks.14 Raynauds
phenomenon improved, indicated by reduced subjective scoring. A two- to eight-
fold increase in circulating endothelial precursors was noted, although not to normal
levels. A reduction in the angiogenic factors vascular endothelial growth factor and
basic fibroblast growth factor was observed.
Statins

Block HMG-CoA reductase CEPs Leukocyteendothelial TGF- Collagen gene


adhesion expression

Mevalonate Endothelial repair Inflammatory chemokines


and neovascularization and cytokines Fibrosis
04-PS Rheumatology-cpp :04-PS Rheumatology-ppp.QXD

LDL Farnesyl PP
Geranylgeranyl PP
18/3/08

VEGF Endothelial Altered gene expression


apoptosis
14:30

Intimal
26 Assessing and treating scleroderma

eNOS ROS
Page 145

proliferation expression

Figure 26.1 Beneficial effects of statins in scleroderma. CEP, circulating endothelial precursors; eNOS, endothelial nitric oxide synthase; HMG-CoA, 3-hydroxy-3-
methylglutaryl coenzyme A; LDL, low-density lipoprotein; PP, pyrophosphate; ROS, reactive oxygen species; TGF-b, transforming growth factor-b; VEGF, vascular
145

endothelial growth factor.


04-PS Rheumatology-cpp :04-PS Rheumatology-ppp.QXD 18/3/08 14:30 Page 146

146 04 Systemic Lupus Erythematosus, Sjgrens Syndrome and Scleroderma

Conclusion
Scleroderma is a rare disease of unknown cause, which is diagnosed essentially on clinical
grounds. To date, most interventions have been of little clinical utility, except proton
pump inhibitors for reflux disease and ACE inhibitors for renal crisis. The challenges for
the future are in the early diagnosis of and effective intervention for pulmonary fibrosis
and pulmonary hypertension. Use of ET receptor antagonists has shown it is possible to
intervene with an oral medication, but the gains in either morbidity or mortality are
modest. A holistic approach is required, which includes a broad treatment strategy aimed
at reducing vascular complications of this disorder.

Further Reading
1 Chung L, Lin J, Furst DE, Fiorentino D. Systemic and localized scleroderma. Clin Dermatol
2006; 24: 37492.
2 Wollheim FA. Classification of systemic sclerosis. Visions and reality. Rheumatology 2005; 44:
121216.
3 Bryan C, Knight C, Black CM, Silman AJ. Prediction of five-year survival following
presentation with scleroderma: development of a simple model using three disease factors at
first visit. Arthritis Rheum 1999; 42: 26605.
4 Meyer O. Prognostic markers for systemic sclerosis. Joint Bone Spine 2006; 73: 4904.
5 Ferri C, Valentini G, Cozzi F et al. Systemic sclerosis: demographic, clinical, and serologic fea-
tures and survival in 1,012 Italian patients. Medicine 2002; 81: 13953.
6 Hoyles RK, Ellis RW, Wellsbury J et al. A multicentre, prospective, randomized, double-blind,
placebo-controlled trial of corticosteroids and intravenous cyclophosphamide followed by
oral azathioprine for the treatment of pulmonary fibrosis in scleroderma. Arthritis Rheum
2006; 54: 396270.
7 Tashkin DP, Elashoff R, Clements PJ et al. Cyclophosphamide versus placebo in scleroderma
lung disease. N Engl J Med 2006; 354: 265566.
8 Ahmadi-Simab K, Hellmich B, Gross WL. Bosentan for severe pulmonary arterial
hypertension related to systemic sclerosis with interstitial lung disease. Eur J Clin Invest 2006;
36 (Suppl 3): 448.
9 Denton C, Humbert M, Rubin L, Coghlan J, Black C. Dual endothelin receptor antagonism in
pulmonary arterial hypertension related to systemic sclerosis. Eur J Clin Invest 2005; 35 : I72.
10 Cozzi F, Montisci R, Marotta H et al. Bosentan therapy of pulmonary arterial hypertension in
connective tissue disease. Eur J Clin Invest 2006; 36 (Suppl 3): 4953.
11 Jimenez SA, Artlett CM. Mirochimerism and systemic sclerosis. Curr Opin Rheumatol 2005;
17: 8690.
12 Sawaya HHB, Jimenez SA, Artlett CM. Quantification of fetal microchimeric cells in clinically
affected and unaffected skin of patients with systemic sclerosis. Rheumatology 2004; 43: 9658.
13 Kuwana M. Potential benefits of statins for vascular disease in systemic sclerosis. Curr Opin
Rheumatol 2006; 18: 594600.
04-PS Rheumatology-cpp :04-PS Rheumatology-ppp.QXD 18/3/08 14:30 Page 147

27 Immunosuppressive drugs 147

14 Kuwana M, Kaburaki J, Okazaki Y, Yasuoka H, Kawakami Y, Ikeda Y. Increases in circulating


endothelial precursors by atorvastatin in patients with systemic sclerosis. Arthritis Rheum
2006; 54: 194651.

P R O B L E M

27 Immunosuppressive Drugs

Case History
Mrs TJ is 55 years old and developed sicca syndrome after the menopause at age 51 years.
Her serum is positive for antinuclear antibody (ANA), anti-Ro and anti-La. She has
symptoms of general malaise, arthralgia and myalgia, and a recent chest X-ray suggests
that she is developing pulmonary infiltrates. Inflammatory markers (erythrocyte
sedimentation rate and serum C-reactive protein) are elevated. She has been taking
prednisolone for the past six months but this has only partly relieved her symptoms.
What factors influence the choice of immunosuppressive agent?
How should she be monitored?
Is immunosuppression becoming safer and more effective?

Background
Immunosuppressive treatment is a large part of rheumatological practice. It is the cor-
nerstone for many of the immune-mediated connective tissue disorders including
rheumatoid arthritis (RA), systemic lupus erythematosus (SLE), systemic sclerosis, vas-
culitides and Sjgrens syndrome. Because of the widespread use of immunosuppressive
agents, the generalist and the general practitioner should understand current usage of
these drugs and how they are monitored.

Glucocorticoids
These bind to glucocorticoid response elements in the genome, thus regulating activity of
a broad range of genes. They downregulate expression of pro-inflammatory cytokines
including interleukin (IL)-1 and IL-6. Decreased IL-2 production from T cells is associ-
ated with decreased lymphocyte proliferation. Cytotoxic and phagocytic cells are
inhibited. Increased activity of the inhibitor of k-beta (Ikb) decreases activity of the key

Atlas Medical Publishing Ltd


04-PS Rheumatology-cpp :04-PS Rheumatology-ppp.QXD 18/3/08 14:30 Page 148

148 04 Systemic Lupus Erythematosus, Sjgrens Syndrome and Scleroderma

regulatory pathway involving nuclear factor-k-beta (NF-kb) with consequent down-


regulation of components of the immune response and increased lymphocyte apoptosis.
Between 1 in 400 and 1 in 200 of the population (higher in the elderly) is taking chron-
ic steroid medication, usually prednisolone. One of the major indications for use of other
immunosuppressive drugs is to minimize exposure to steroids and to protect the patient
from long-term side effects. The latter include hypertension, hyperglycaemia, osteo-
penia, cataracts, gastric ulceration, impaired wound healing and susceptibility to
infection. Topical agents (inhaled, rectal or skin) should be used where possible.

Antiproliferative and antimetabolic drugs


Azathioprine (Imuran) is a purine derivative that has been used as an antimetabolite
since the early 1960s. It is used in transplant patients and in treatment of RA. Side effects
include bone marrow suppression, susceptibility to infections (especially varicella and
herpes simplex viruses), hepatotoxicity, alopecia and increased risk of malignancies.
Methotrexate is used for psoriasis and RA as well as in treatment of malignancies. It
inhibits dihydrofolate reductase, therefore decreasing purine synthesis and progression
through the G1 and S phases of the cell cycle. This may explain its cytotoxic effects, with
its anti-inflammatory effects related to increasing adenosine levels. Side effects include
bone marrow suppression, interstitial pneumonitis and increased liver enzymes with
occasionally more severe hepatic impairment. A full blood count (FBC) and liver func-
tion tests (LFTs) should be monitored monthly and adequate contraception used.
Cyclophosphamide is used particularly in SLE and in vasculitic conditions including
Wegeners granulomatosis. Side effects include leukopenia, haemorrhagic cystitis and
increased risk of transitional cell carcinoma, infertility, alopecia and increased risk of
lymphoma.
Leflunomide is used in active RA, particularly in those unresponsive to methotrexate,
for whom leflunomide can be used alone or in combination with methotrexate. Side
effects are diarrhoea, alopecia, severe skin reactions, pneumonitis, hypertension and neu-
rological toxicity. FBC and LFTs should be monitored monthly for the first six months
and then two-monthly. Blood pressure should be monitored. Adequate contraception is
essential, with leflunomide having a long practical half-life due to enterohepatic
circulation.
Mycophenolate mofetil is hydrolysed to mycophenolic acid in the body, and inhibits
T- and B-cell proliferation through its inhibitory effect on the enzyme inosine
monophosphate dehydrogenase (IMPDH). It may cause bone marrow suppression,
pneumonitis and severe diarrhoea.
Sirolimus (Rapamune) binds to the 12 kDa FK506-binding protein (FKBP-12) and
inhibits the protein kinase mammalian target of rapamycin (MTOR), a key protein in
cellular proliferation. The drug is mainly used in transplant patients and in drug-eluting
stents for patients with coronary artery disease.

Calcineurin inhibitors
Calcineurin is a Ca2+/calmodulin-dependent protein phosphatase involved in T-cell dif-
ferentiation and proliferation. The enzyme catalyses dephosphorylation of the nuclear
factor of activated T lymphocytes (NFAT), which is subsequently translocated to the
nucleus and increases expression of pro-inflammatory cytokines including IL-2.
04-PS Rheumatology-cpp :04-PS Rheumatology-ppp.QXD 18/3/08 14:30 Page 149

27 Immunosuppressive drugs 149

Cyclosporin (by interacting with cyclophilin) and tacrolimus (by interacting with FKBP-
12) inhibit the phosphatase activity of calcineurin.
Cyclosporin is a cyclic peptide of eleven amino acids, which is secreted by the fungus
Beauveria nivea. Its predominant effect is inhibiting T-cell responses. It can be given
intravenously (Sandimmune) or orally (Neoral). Its main uses are in transplant, RA and
psoriasis. For transplant patients, it is usually combined with steroids and either azathio-
prine or mycophenolate mofetil. In RA, it is usually reserved for patients who do not
respond to first- or second-line disease-modifying antirheumatic drugs (DMARDs)
including methotrexate. With the introduction of anti-tumour necrosis factor (anti-
TNF) agents, cyclosporin is being used less. Nephrotoxicity is the most common severe
side effect. The drug may also worsen hypertension, hyperglycaemia and hyperuri-
caemia. Side effects also include tremor, hirsutism and gum hyperplasia. Monitoring to
keep the plasma concentration of the drug in the range 50100 ng/ml should be under-
taken.
Tacrolimus (Prograf) is a macrolide antibiotic secreted by Streptomyces tsukubaensis.
It can be given orally or systemically, and is mainly used for transplant patients.
Nephrotoxicity and neurotoxicity are relatively common with its use.

Antibody treatments
Lymphocyte immune globulin and antithymocyte globulin have been used in immuno-
suppression since the 1960s. To avoid the potential problems of using polyclonal sera,
monoclonal or monospecific antibody treatments have been developed. These include:

Anti-CD3 Directed at the e chain of the T-cell receptor


Anti-CD25 Daclizumab and basiliximab, directed at the IL-2 receptor
Anti-CD52 Alemtuzumab
Anti-TNF-a Infliximab, etanercept, used in refractory RA
Anti-lymphocyte function-associated antigen-1 (anti-LFA-1) Efalizumab
Anti-CD20 Rituximab, B-cell depleting antibody

Other drugs
Hydroxychloroquine (Plaquenil, 200400 mg/day) is an antimalarial drug used in RA
and SLE. It may be particularly useful for skin lesions in the latter. It is usually well toler-
ated but can cause macular damage. Patients should have an ophthalmological assess-
ment at baseline and six-monthly.
Sulphasalazine is used in RA and in seronegative spondyloarthropathies. It may cause
gastrointestinal symptoms, headache, rash (including photosensitivity) and discolour-
ation of soft contact lenses. Because of potential bone marrow toxicity, a FBC should be
measured every two weeks for the first three months, then three-monthly. LFTs should
be monitored monthly for the first three months then six-monthly.
Use of immunosuppressive drugs has transformed management of many non-
malignant diseases. They should only be used where they are clearly indicated (Box 27.1)
and with an appropriate programme of drug monitoring. An algorithm for monitoring
of the most commonly prescribed drugs is presented in Figure 27.1. Increasingly with
04-PS Rheumatology-cpp :04-PS Rheumatology-ppp.QXD 18/3/08 14:30 Page 150

150 04 Systemic Lupus Erythematosus, Sjgrens Syndrome and Scleroderma

newer drugs including cyclosporine, mycophenolate mofetil, sirolimus and tacrolimus


therapeutic monitoring of drug levels is being undertaken.1

Box 27.1 Use of immunosuppressive drugs


Underlying diagnosis well established
Less toxic alternatives have failed
The disease poses a severe threat to health
Patient is informed of the risks and benefits
Infection and malignancy excluded
No possibility that the patient will become pregnant
Arrangements for monitoring and treating complications in place

Recent Developments
1 The field of pharmacogenomics is developing rapidly. Expression of levels of key
enzymes varies considerably between individuals, and this may explain why some
patients fail to respond to drugs while others are very sensitive and more susceptible
to side effects. Levels of thiopurine methyltransferase may be useful in predicting
response to azathioprine.2 Those with high levels of the enzyme are at risk of being
underdosed, and patients with low enzyme levels may be more susceptible to side
effects including bone marrow suppression. In a similar fashion, variations in the
expression of methylene tetrahydrofolate reductase may be important determinants
of the response to methotrexate.3
2 It is not clear whether elderly onset RA behaves differently from that which has an
earlier onset. In the recent study by Tutuncu et al.,4 elderly patients with RA had
comparable duration and comparable indices of severity compared with younger
patients. They were, however, less likely to receive as aggressive treatment; this may
be partly due to under-recognition of the severity of the disease particularly if there
are comorbidities or to concern about potential side effects.
3 Much of the stimulus for development and clinical trials with newer agents has come
from the transplant field. Examples of the way practice is changing are the increasing
use of mycophenolate mofetil for lupus nephritis5 (where previously
cyclophosphamide was the drug of choice) and the increasing use of cyclosporin for
autoimmune diseases.6 Renal toxicity may limit the use of this drug by rheumatologists
but it certainly may prove very useful in life-threatening situations where other drugs
have failed. Amongst biological therapies, the TNF-a antagonists have found the
widest use in RA. The maximum effect of biological therapies in RA is likely to be seen
when they are combined with cytotoxic treatments such as methotrexate.7

Conclusion
The range of immunosuppressive drugs is increasing, as is trial evidence relating to
their use in individual diseases. The choice of agents depends on the clinical picture
04-PS Rheumatology-cpp :04-PS Rheumatology-ppp.QXD 18/3/08 14:30 Page 151

27 Immunosuppressive drugs 151

Baseline FBC, LFTs, U/E and CXR

FBC LFTs U/E

Weekly C*, A, M 2-weekly C, A, M Weekly C, A, M

Monthly M Monthly C*, A, M Monthly C*, A, M

3-monthly A 3-monthly A, M 2-monthly A, M

? Stop ? Stop ? Stop


WBC ,1.5 3 109/l Transaminases Creatinine
Pl ,100 3 109/l 3 3 upper limit >30% above
of normal baseline

Figure 27.1 Therapeutic monitoring of common immunosuppressives. Low white blood cell (WBC) count,
increased liver enzymes or increasing creatinine may necessitate temporarily or permanently stopping the
treatment a decision best made by the specialist responsible for starting it. * monitor at this frequency for
the duration of treatment; A, azathioprine; C, cyclophosphamide; CXR, chest X-ray; FBC, full blood count;
LFT, liver function text; M, methotrexate; Pl, platelet; U/E, urea and electrolytes.

and diagnosis, along with an assessment of the likely benefits and potential risks.
Corticosteroids are often used in the first instance but the tendency now is to use
lower doses, and thus they are often combined with other drugs. Methotrexate, aza-
thioprine and cyclophosphamide have been used for many years but still have a major
place in management. Monitoring of FBC, hepatic function and renal function is the
most important aspect generally. Use of newer drugs and biological agents is increas-
ing, although this is still limited by cost and clinical experience. With appropriate
monitoring and choice of regimens, the treatment of autoimmune connective tissue
disease with agents that suppress the immune system is becoming safer and more
effective.

Further Reading
1 Holt DW, Armstrong VW, Griesmacher A, Morris RG, Napoli KL, Shaw LM. International
Federation of Clinical Chemistry/International Association of Therapeutic Drug Monitoring
04-PS Rheumatology-cpp :04-PS Rheumatology-ppp.QXD 18/3/08 14:30 Page 152

152 04 Systemic Lupus Erythematosus, Sjgrens Syndrome and Scleroderma

and Clinical Toxicology working group on immunosuppressive drug monitoring. Ther Drug
Monit 2002; 24: 5967.
2 Patel AA, Swerlick RA, McCall CO. Azathioprine in dermatology: the past, the present, and
the future. J Am Acad Dermatol 2006; 55: 36989.
3 Hughes LB, Beasley TM, Patel H et al. Racial or ethnic differences in allele frequencies of sin-
gle-nucleotide polymorphisms in the methylenetetrahydrofolate reductase gene and their
influence on response to methotrexate in rheumatoid arthritis. Ann Rheum Dis 2006; 65:
121318.
4 Tutuncu Z, Reed G, Kremer J, Kavanaugh A. Do patients with older-onset rheumatoid
arthritis receive less aggressive treatment? Ann Rheum Dis 2006; 65: 12269.
5 Dooley MA. Mycophenolate mofetil: what role in the treatment of lupus? Lupus 2006; 15:
17982.
6 Ponticelli C. Cyclosporine: from renal transplantation to autoimmune diseases. Ann N Y Acad
Sci 2005; 1051: 5518.
7 Heiberg MS, Rdevand E, Mikkelsen K et al. Adalimumab and methotrexate is more effective
than adalimumab alone in patients with established rheumatoid arthritis: results from a
6-month longitudinal, observational, multicentre study. Ann Rheum Dis 2006; 65: 137983.
05-PS Rheumatology-cpp:05-PS Rheumatology-ppp.QXD 18/3/08 15:26 Page 153

S E C T I O N F I V E 05
Vasculitic Syndromes
28 Vasculitic disease
29 Giant cell arteritis and polymyalgia rheumatica
30 Behets syndrome

P R O B L E M

28 Vasculitic Disease

Case History
Mrs SR attends your clinic with a rash over her legs. She is aged 32 years and for the last
six months has been unwell, with intermittent fevers, loss of appetite and fatigue. Recent
blood tests show elevated erythrocyte sedimentation rate (ESR; 83 mm/h) and C-reactive
protein (CRP; 46 mg/dl). Today she has palpable purpura on her lower legs. Urinalysis is
positive for blood and protein.
What are the clinical clues to vasculitis?
What investigations will assist with a precise diagnosis?
How should the condition be treated and monitored?

Background
The vasculitides are an important group of disorders in which an immune reaction
affects the walls of the blood vessels.1,2 The immune dysfunction may be triggered by
drugs or by an underlying autoimmune disease, but often an underlying cause is not
identified. Manifestations vary between diseases and from person to person. The follow-
ing are common: general malaise, arthralgia, normocytic anaemia, skin lesions (palpable
purpura or necrotic ulcers), renal dysfunction and neurological dysfunction (particularly
wrist drop and ankle drop). Differential diagnosis includes drug reactions (antibiotics,
amphetamines, ergot derivatives), infections (human immunodeficiency virus [HIV],

Atlas Medical Publishing Ltd


05-PS Rheumatology-cpp:05-PS Rheumatology-ppp.QXD 18/3/08 15:26 Page 154

154 05 Vasculitic Syndromes

subacute bacterial endocarditis), malignant disease and antiphospholipid syndrome.


Classification of vasculitides, shown in Box 28.1, is based on the size of blood vessels
affected.
Individually, the diagnoses, with the exception of giant cell arteritis (GCA), are rela-
tively rare. Collectively (excepting GCA), there is an incidence of 2040 per million of the
population per year. There is considerable geographical variation and also a seasonal
variation in incidence. The latter suggests an infectious trigger and exposure to
Mycoplasma pneumoniae has been linked with risk of GCA. The latter is more common
in Caucasians than all the other diagnoses put together (around 170 per million of the
population per year). Cutaneous vasculitis may occur in isolation or as part of an under-
lying systemic disorder.3 In each case of suspected vasculitis, a careful search for multi-
organ involvement should be undertaken.

Small vessel vasculitis


The vessels involved are the venules, capillaries and arterioles of the skin, kidney and gas-
trointestinal (GI) tract. The hallmark lesion is palpable purpura, which is a slightly raised,
non-blanching lesion that typically affects the lower legs. Gut involvement causes colicky
abdominal pain and may lead to melaena.
HenochSchnlein purpura is common in children after viral or streptococcal infec-
tion. The blood vessels are affected by immunoglobulin A (IgA)-containing immune
complex deposition. Apart from the purpura, abdominal pain and arthralgia, renal
involvement develops in up to 40%, with progression to chronic nephritis in 10% of
cases. The condition is generally self-limiting and immunosuppressive treatment is not
usually required.
Cryoglobulins are antigenantibody complexes that precipitate in the cold. In add-
ition to the other common manifestations of small vessel vasculitis, they may be associ-
ated with serious liver and pulmonary disease. Cryoglobulinaemia occurs in one-third of
patients with hepatitis C infection. Cryoglobulins should be sought in all patients with a
clinical picture of small vessel vasculitis, and all positive patients should be tested for
hepatitis C.

Small to medium vessel vasculitis


These diseases generally affect the skin, upper respiratory tract, lungs and kidneys.
Wegeners granulomatosis often begins with apparently innocent symptoms including

Box 28.1 Classification of vasculitis


Small vessels Medium vessels
HenochSchnlein purpura Polyarteritis nodosa
Mixed cryoglobulinaemia Kawasaki syndrome

Small to medium vessels Large vessels


Wegeners granulomatosis Giant cell arteritis
ChurgStrauss syndrome Takayasus arteritis
Microscopic polyangiitis
05-PS Rheumatology-cpp:05-PS Rheumatology-ppp.QXD 18/3/08 15:26 Page 155

28 Vasculitic disease 155

sinusitis, nasal ulcers, otitis media and hearing loss (70% of cases). Pulmonary infiltrates
or nodules develop in 85% of cases, and 80% of patients have renal involvement with a
focal necrotizing glomerulonephritis that frequently progresses to renal failure.
Histologically, there is necrotizing vasculitis of medium-sized vessels with extravascular
granulomata. Cytoplasmic anti-neutrophil cytoplasmic antibody (c-ANCA) is present in
80% of patients and perinuclear ANCA (p-ANCA) in 20%, with at least 90% of patients
overall being ANCA positive. The disease is usually treated with prednisolone and
cyclophosphamide.
Microscopic polyangiitis is a variant of polyarteritis where patients present with pul-
monary haemorrhage and glomerulonephritis. ChurgStrauss syndrome presents with
allergic rhinitis and asthma. Eosinophilia, and infiltrates of the lungs and GI tract
account for the features of the disease. Up to 75% of patients have palpable purpura or
skin nodules. It affects the kidneys in up to 50% of cases. Immunosuppressives are fre-
quently required.

Medium vessel vasculitis


Patients with polyarteritis nodosa typically present with fever, myalgia and arthralgia.
Renal disease develops in up to 60% of cases, with cardiac and GI involvement in 40%
(may lead to bowel infarction), and central or peripheral nervous system involvement in
20%. Angiography reveals multiple microaneurysms of vessels, with beading and taper-
ing of the vessel lumen. Biopsy of affected tissues reveals focal necrosis of vessel walls.
p-ANCA is positive in 20% of cases and c-ANCA in 10%. The disease is associated with
hepatitis B and carries a worse prognosis in this circumstance. Treatment is with pred-
nisolone and cytotoxic agents, with plasma exchange used for extreme cases. Vidarabine
and interferon are useful where there is concurrent hepatitis B infection.
Kawasaki syndrome typically affects children presenting with fever, conjunctival
infection, red tongue, arthralgia and lymphadenopathy. Fatal coronary arteritis can
develop if the syndrome is untreated. Aspirin and intravenous immunoglobulin is the
mainstay of treatment.

Large vessel vasculitis


GCA almost exclusively occurs in patients over the age of 50 years and is frequently
accompanied by polymyalgia rheumatica. Visual loss may arise from central retinal
artery occlusion, anterior ischaemic optic retinitis or occipital cortex infarction.
Claudication or ischaemia of the tongue is a frequently overlooked but fairly common
symptom. Temporal artery biopsy should be carried out where possible and will reveal
lymphocytic and giant cell infiltration with fragmentation of the internal elastic lamina.
Treatment with prednisolone usually suffices, with doses tailored according to the ESR.
Occasionally, other immunosuppressive agents are required.
Takayasus arteritis (pulseless disease) usually affects young women who present with
fever, arthralgia and myalgia and have high ESR. Treatment is with prednisolone and
other immunosuppressive agents. Transluminal angioplasty or other mechanical revas-
cularization may be required where there is risk of infarction.
Patient demographics and the pattern of organ involvement provide clues to the
underlying diagnosis (Figure 28.1). A full blood count will often show a normocytic
05-PS Rheumatology-cpp:05-PS Rheumatology-ppp.QXD 18/3/08 15:26 Page 156

156 05 Vasculitic Syndromes

anaemia, particularly where there is systemic disease. Urea and creatinine should be care-
fully monitored in all cases so that renal involvement can be identified early. ESR and
CRP will almost invariably be elevated and are useful to monitor disease activity and
response to treatment. Liver tests should be monitored, not only because of potential
hepatic involvement in vasculitis, but also because of potential side effects of treatment.
Detection of antinuclear antibody and rheumatoid factor (RF) may indicate the presence
of an underlying connective tissue disorder. Apart from rheumatoid disease, RF may also
be positive in patients with Sjgrens syndrome, and is more likely to be positive in
patients with hepatitis B infection. Patients with both vasculitis and antiphospholipid
antibodies are recognized. Anti-neutrophil cytoplasmic antibodies (ANCA) are more
specific markers for certain vasculitic disorders (Table 28.1). c-ANCA are antibodies pri-
marily directed at the enzyme proteinase-3. The perinuclear pattern (p-ANCA) is much

Child: (Any HSP; Asian Kawasaki)

Demographics Young adult: (Middle Eastern Behets; Asian Takayasus)

Elderly: (GCA mainly Caucasian)

Palpable purpura: Small- to medium-sized vessels involved


Skin
Ulcers or ischaemia: PAN, CSS, Wegeners

Muscle Pain 1 stiffness: GCA

Joints Swollen/inflamed: PAN, CSS, Wegeners

Neuropathy PAN, CSS, Wegeners, cryoglobulinaemia

Ischaemia: PAN, Takayasus


Renal
Glomerulonephritis: MPA, Wegeners, cryoglobulinaemia, CSS, HSP

Figure 28.1 Clinical pointers as to underlying disease state. The pattern and nature of organ involvement
give clues as to the likely underlying diagnosis. CSS, ChurgStrauss syndrome; GCA, giant cell arteritis; HSP,
HenochSchnlein purpura; MPA, microscopic polyangiitis; PAN, polyarteritis nodosa.
05-PS Rheumatology-cpp:05-PS Rheumatology-ppp.QXD 18/3/08 15:26 Page 157

28 Vasculitic disease 157

less disease specific, and antibodies are directed at a number of determinants including
myeloperoxidase, lactoferrin, elastase, cathepsin G and lysozyme. c-ANCA is also posi-
tive in many patients with crescentic or necrotizing glomerulonephritis. ANCA (pre-
dominantly p-ANCA) is positive in a proportion of patients with antiglomerular
basement membrane disease, inflammatory bowel disease, autoimmune hepatitis, scler-
osing cholangitis and Felty syndrome. ANCA has traditionally been detected by indirect
immunofluorescence techniques but modern enzyme-linked immunosorbent assays
(ELISAs) with specific antigens provide a more sensitive test. Cryoglobulins should be
sought. Patients should be screened for hepatitis B and C, as well as HIV. Low levels of the
complement components C3 and C4 are consistent with systemic lupus erythematosus,
and may be increased in other inflammatory conditions. Chest X-ray will reveal the pres-
ence of pulmonary nodules or infiltrates. Biopsy of affected tissue may be required.
Punch biopsy is suitable for most skin lesions, but excision biopsy is required where large
vessel vasculitis is suspected. The characteristic change of leukocytoclastic angiitis is seen
in small to medium vessel disease.
Treatment depends on the diagnosis and severity of the disease. Simple measures for
skin lesions include avoiding contact with lesions (loose clothing, bedding etc.), elevating
the affected part, analgesia and non-steroidal anti-inflammatory drugs. The mainstay is
immunosuppression, usually initially with high-dose steroids with the later addition of
other immunosuppressive agents including cyclophosphamide, azathioprine and
methotrexate. Cyclosporine and mycophenolate mofetil may be used in more serious
cases, while plasmapheresis and intravenous immunoglobulin may also be useful in resist-
ant cases. Induction and maintenance phases of treatment require different approaches.
For some patients, colchicine or dapsone are useful to maintain remission. Finally, bio-
logical therapies may be required for the most serious cases.4 These include the anti-
CD20 (anti-B cell) antibody rituximab, and the antitumour necrosis factor (TNF)-a
agents: infliximab, adalimumab and etanercept.

Table 28.1 Major diseases associated with ANCA

Proteinase-3 (c-ANCA) Myeloperoxidase (p-ANCA)

Wegeners granulomatosis 80% 10%


Microscopic polyangiitis 30% 60%
ChurgStrauss syndrome 30% 30%
Figures show percentage of cases with each condition that has the ANCA subtype.

Recent Developments
1 Recent evidence5 suggests that ANCA antibodies are directly involved in the
vasculitic disease process. Their action helps promote neutrophil activation and
adhesion, facilitating interaction with cytokine-primed vascular endothelium. They
may arise from molecular complementarity the antibodies being directed against
peptides encoded by the antisense strands of key epitopes. This phenomenon has
05-PS Rheumatology-cpp:05-PS Rheumatology-ppp.QXD 18/3/08 15:26 Page 158

158 05 Vasculitic Syndromes

been invoked to explain the relationship between ANCA positivity and exposure to
infectious agents. Possible genetic determinants of ANCA-positive vasculitic
syndromes have also been identified: these include increased carrier frequency of the
Z allele of a1-antitrypsin (SERPINA1) and certain alleles of Fc receptors.
Environmental triggers include exposure to silica, solvents, asbestos and pesticides.
Drug triggers include the antithyroid drugs propylthiouracil and carbimazole,
phenytoin, cocaine and allopurinol.
2 Data to support the use of the antibody therapies (rituximab, infliximab and
etanercept) in vasculitic conditions, particularly Wegeners granulomatosis, are
accumulating.6 The role of TNF-a in many of these disease processes means that
infliximab and etanercept may find broader usage. Even in GCA, which is generally
very responsive to steroids, biological therapies may find a use where the individual
is susceptible to side effects of steroids. Interferon (a and b) has shown promise for
treatment of ChurgStrauss syndrome.
3 The recent interest in atherosclerosis as an inflammatory disease may give rise to
novel therapies for common vascular disorders including coronary heart disease and
stroke.7 Vascular inflammation is known to play a prominent role in the
development of atherosclerotic plaques and their subsequent rupture, giving rise to
acute vascular events. Therapeutic strategies to decrease this vascular inflammation
include use of statins, cyclooxygenase inhibition, blockade of the reninangiotensin
system and inhibition of pro-inflammatory cytokines. Proposed biological
approaches include immunization against the protein component of low-density
lipoproteins (apoB-100).

Conclusion
Vasculitic disease is extremely variable in its presentation. The most common cutaneous
manifestation is palpable purpura. The pattern of organ distribution and the nature of
the lesions give clues to the underlying disease process. In all cases, the potential for
involvement of major organ systems (lung, liver, heart, kidneys and nervous system)
should be considered. The lesions are accompanied by symptoms of systemic inflamma-
tion and inflammatory markers are increased. Once diagnosis is established, or before in
severe cases, treatment should be started. This is mainly with immunosuppressive agents.
The acute and maintenance phases of treatment require different approaches. Biological
treatments, including antibodies to B cells and TNF-a, are making a major impact on the
management of difficult and refractory cases.

Further Reading
1 Roane DW, Griger DR. An approach to diagnosis and initial management of systemic
vasculitis. Am Fam Physician 1999; 60: 142130.
2 Scott DGI, Watts RA. Systemic vasculitis: epidemiology, classification and environmental
factors. Ann Rheum Dis 2000; 59: 1613.
3 Carlson JA, Cavaliere LF, Grant-Kels JM. Cutaneous vasculitis: diagnosis and management.
Clin Dermatol 2006; 24: 41429.
05-PS Rheumatology-cpp:05-PS Rheumatology-ppp.QXD 18/3/08 15:26 Page 159

29 Giant cell arteritis and polymyalgia rheumatica 159

4 Foster R, Rosenthal E, Marques S, Vounotrypidis P, Sangle S, DCruz D. Primary systemic


vasculitis: treatment of difficult cases. Lupus 2006; 15: 1437.
5 Bosch X, Guilabert A, Font J. Antineutrophil cytoplasmic antibodies. Lancet 2006; 368:
400418.
6 Chan AT, Flossmann O, Mukhtyar C, Jayne DRW, Luqmani RA. The role of biologic therapies
in the management of systemic vasculitis. Autoimm Rev 2006; 5: 2738.
7 Stoll G, Bendszus M. Inflammation and atherosclerosis: novel insights into plaque formation
and destabilization. Stroke 2006; 37: 192332.

P R O B L E M

29 Giant Cell Arteritis and Polymyalgia


Rheumatica

Case History
Mrs FM is a 74-year-old woman who complains of headaches, general malaise and
stiffness of the upper arms and shoulders. The symptoms have gradually increased over
four weeks. She is generally healthy and takes no medication apart from paracetamol for
her mild osteoarthritis.
Is temporal artery biopsy required to diagnose giant cell arteritis (GCA)?
What is current thinking on the aetiology of GCA and polymyalgia rheumatica
(PMR)?
For how long is she likely to require steroid treatment?
How should the condition be monitored?

Background
GCA is the most common vasculitic disease and affects medium- to large-sized arteries.
GCA presents with headache, tenderness over the temporal artery or scalp, jaw claudica-
tion and sometimes visual loss. The latter develops quickly and is frequently permanent,
making urgent diagnosis and treatment mandatory. PMR is a closely related condition
that usually presents with pain and stiffness of the shoulder muscles, often accompanied
by general malaise, fever and weight loss. PMR occurs in about 50% of patients with

Atlas Medical Publishing Ltd


05-PS Rheumatology-cpp:05-PS Rheumatology-ppp.QXD 18/3/08 15:26 Page 160

160 05 Vasculitic Syndromes

GCA; of patients with PMR, around 10% will have GCA. The prevalence of PMR is
around 1015 per 1000 and women are affected twice as often as men. GCA is more com-
mon in colder climates, is less common than PMR and has an incidence of about 20 per
100 000 per year. Both GCA and PMR are very uncommon under the age of 50 years.1,2
GCA and PMR are much more common in Caucasians, particularly those of
Scandinavian descent. The fact that there is a racial predisposition and that familial clus-
tering has been described points to a genetic component to the aetiology. Alleles of HLA-
DRB1*04 have been commonly implicated. Amongst infectious triggers that have been
considered are Mycoplasma pneumoniae, parvovirus B19 and parainfluenza virus type 1.3
In GCA, inflammation in the vascular wall leads to structural changes including intimal
hyperplasia, fragmentation of the internal elastic laminae and luminal occlusion. The
disorder in the vessel wall is driven by activated T cells and macrophages. T cells enter the
vessel wall through the vasa vasorum, become activated through contact with antigen
and produce a range of pro-inflammatory cytokines including interferon-g. These
cytokines lead to macrophage homing and activation.
Shoulder pain and stiffness are the most common symptoms of PMR. Systemic symp-
toms include fever, mood changes, sweating, anorexia and weight loss. Erythrocyte sedi-
mentation rate (ESR) is usually greater than 50 mm/h and there is often mild
normochromic anaemia. Muscles are not strikingly tender. There is often some limita-
tion of movement due to muscle stiffness, and secondary increase in joint pains is com-
mon. Differential diagnosis includes fibromylagia, rheumatoid arthritis, polymyositis
and osteoarthritis. Some patients develop swelling of the distal digits, carpal tunnel syn-
drome and tenosynovitis.
GCA can affect any medium- to large-sized artery but most commonly affects the
internal and external carotid arteries. Headache has often been present for several weeks

Table 29.1 Features of GCA

Frequency (%)
Headache 85
Temporal artery thickened, pulseless or tender 73
Malaise, anorexia or weight loss 61
Jaw claudication 41
Polymyalgia rheumatica 40
Scalp tenderness 33
Retinal ischaemia 23
Visual loss 13
Fever 10
Dysphagia 5
Cerebrovascular accident 2
Recent onset of peripheral arterial disease 2
1
Adapted from Gonzalez-Gay et al. 2006.
05-PS Rheumatology-cpp:05-PS Rheumatology-ppp.QXD 18/3/08 15:26 Page 161

29 Giant cell arteritis and polymyalgia rheumatica 161

before the patient presents. Depression, malaise, weight loss and fever often accompany
the diagnosis, even in the absence of clinically apparent PMR. GCA is unusual when the
ESR is less than 50 mm/h. The temporal artery may be thickened, tender and lack pulsa-
tion. Eyes should be examined, looking for loss of vision, an afferent papillary defect and
ocular movement disorders (diplopia). Fundoscopy should be carried out to look for
signs of ischaemic retinopathy. Table 29.1 lists the common features of GCA.
The initial investigation should be ESR, which is seldom normal and frequently
greater than 100 mm/h. The normal upper limit for ESR is estimated as:
Men: Age (years) 2
Women: Age (years) + 10) 2
Up to 20% of patients with PMR have normal ESR. Serum C-reactive protein (CRP) is
also usually elevated. CRP and ESR values are generally very strongly correlated. Other
laboratory findings include normochromic anaemia, thrombocytopenia and elevated
liver enzymes. There may be a mild leukocytosis, and thrombocytosis is recognized.
Temporal artery biopsy is the cornerstone of diagnosis and should be considered even
where symptoms appear to strongly support the diagnosis. Before embarking on pro-
longed steroid therapy, a precise diagnosis is highly desirable. If the initial biopsy is nega-
tive, and symptoms are highly suggestive, biopsy of the contralateral temporal artery
should be considered. Positive biopsy shows inflammatory infiltrate with disruption of
the internal elastic lamina. The inflammation is focal and segmental and granulomatous
with multinucleate giant cells. The biopsy should be performed at the most symptomatic
site and should ideally be at least 2.5 cm long. Temporal artery biopsy is not usually car-
ried out in patients with what appears to be uncomplicated PMR. Newer diagnostic tech-
niques are colour-coded duplex ultrasonography and positron emission tomography
(PET). Both have a high sensitivity for detecting vascular disease, but changes are often
not specific. Magnetic resonance imaging (MRI) angiography is also emerging as a useful
tool. There are no documented associations with circulating autoantibodies.
Prednisolone is the first-line treatment. An initial dose of 1020 mg/day is usually suf-
ficient for patients with PMR. Response is usual within two weeks, and after one month
the dose can begin to be tapered down. The usual duration of therapy is two to three
years. GCA usually requires prednisolone 4060 mg/day. Initiation should not be
delayed while the results of investigations, including temporal artery biopsy, are awaited.
Visual symptoms seldom develop after steroid therapy has been commenced. If present,
or allowed to develop, visual symptoms usually do not recover. Severe, refractory or vis-
ual symptoms can be treated with intravenous methylprednisolone (1 g daily for three
consecutive days). Symptoms respond to prednisolone within days and the dose can be
tapered after a month. At six months, a dose in the range 7.510 mg/day is usual.
Treatment for two to three years is generally required. Alternate day steroid regimens or
adjuvant methotrexate are of uncertain benefit. Flare-up of disease activity may occur
with both PMR and GCA as the steroid dose is reduced below 7.5 mg/day. These flares
can be managed by temporarily increasing the steroid dose. Safely decreasing and stop-
ping prednisolone requires clinical judgement supported by the patients account of
symptoms and measurement of inflammatory markers. For refractory cases or those at
high risk from corticosteroid side effects, methotrexate, azathioprine and infliximab are
the agents with the best evidence for benefit.
05-PS Rheumatology-cpp:05-PS Rheumatology-ppp.QXD 18/3/08 15:26 Page 162

162 05 Vasculitic Syndromes

As well as anti-inflammatory therapy, antiplatelet therapy (e.g. low-dose aspirin)


should be considered. As steroid treatment is likely to be ongoing for at least two years,
baseline bone mineral density measurements should be carried out. Those with osteo-
penia or who are otherwise at high risk of osteoporosis should have prophylactic calcium
(1000 mg/day) and vitamin D (800 Units/day) supplements. Those with osteoporosis
should be considered for bisphosphonate treatment (chapter 44). Patients should be
screened periodically for impaired glucose tolerance/diabetes, and their blood pressure
should be monitored for the duration of treatment.

Recent Developments
1 Epidemiological studies are difficult because PMR and GCA are generally managed
in primary care. A recent study4 documented the incidence in the United Kingdom
between 1990 and 2001. During this period, the incidence of PMR increased from
6.9 to 9.3 per 10 000 person-years. There was no parallel increase in GCA, which
remained static at 2.2 per 10 000 person-years.
2 Beyond its association with HLA-DRB1*04, remarkably little is known about the
genetic basis of GCA. An association with alleles of the Fcg receptors has been
proposed.5 These are important in immune regulation. Improved knowledge of the
genetic factors in pathogenesis may help to identify susceptible individuals.
3 Study of the genes expressed in temporal artery biopsies may also give clues to disease
pathogenesis and help to identify likely responders to treatment. Cid et al.6 reported
increased expression of the chemokine monocyte chemoattractant protein-1 (MCP-1;
also called CCL2) in temporal artery biopsies of patients with GCA. Furthermore,
expression of CCL2 correlated with systemic measures of inflammation, and the
highest level of expression was seen in those with relapsing disease.
4 While ESR and CRP are highly correlated with each other and with a positive
temporal artery biopsy, this is not invariable.7 The sensitivity of ESR alone may be no
more than 76%, while positive CRP has a sensitivity of 97.5% alone and 99% when
combined with ESR. Neither is specific and values of ESR and CRP should be
interpreted in the light of the clinical picture and other investigations.
5 Lee et al.8 conducted a retrospective chart review and demonstrated that 16.2% of
GCA patients in the study who were taking antiplatelet drugs suffered an ischaemic
event compared with 48% of those who were not taking antiplatelet therapy. There is
no compelling evidence to start all GCA patients on aspirin or clopidogrel at present
but these interventions appear to pose little risk and may have considerable benefit.
6 Pulsed intravenous methylprednisolone is used for initial therapy in those at high risk
from an ischaemic event. Using this treatment initially may actually decrease steroid
requirements subsequently. In a recent small study,9 10 of 14 patients treated initially
with intravenous methylprednisolone were taking 5 mg/day prednisolone at 36 weeks,
compared with only 2 out of 13 patients treated conventionally (P = 0.003). Even
without intravenous steroid, most agree that high-dose steroid to bring the condition
under control quickly allows a decrease in steroid exposure over succeeding months.
7 It is not known why older people are uniquely susceptible to PMR and GCA.
Dehydroepiandrosterone (DHEA) is the most abundant circulating steroid hormone
and levels decline with age. This decline has been associated with a range of
05-PS Rheumatology-cpp:05-PS Rheumatology-ppp.QXD 18/3/08 15:26 Page 163

29 Giant cell arteritis and polymyalgia rheumatica 163

involutional changes including decreased cognitive function, decreased muscle bulk


and a decline in bone health. Low levels of DHEA have been reported in patients
with PMR or GCA plus PMR.10 There is currently no evidence to support routine
treatment with DHEA in PMR or GCA.

Conclusion
As the number of elderly people is increasing, it is likely that GCA and PMR will be
encountered more commonly in practice. Prompt diagnosis and effective treatment are

Muscle pain and stiffness

Consider other causes: High ESR and CRP


Statin treatment
Inflammatory myopathy
Osteoarthritis
Rheumatoid arthritis

Prednisolone 1020 mg/day*

Headache or temporal artery symptoms/signs


ESR .80 mm/hour
Constitutional symptoms
Poor response to low-dose steroids

Temporal artery biopsy

Negative Positive

? Contralateral biopsy Prednisolone 4060 mg/day*


? Another diagnosis

* Taper dose of prednisolone down after one month if there is satisfactory response
Monitor for: Hyperglycaemia
Hypertension
Osteoporosis (consider prophylaxis)

Figure 29.1 Diagnosis and management of PMR and GCA.


05-PS Rheumatology-cpp:05-PS Rheumatology-ppp.QXD 18/3/08 15:26 Page 164

164 05 Vasculitic Syndromes

essential not only to alleviate symptoms but also to prevent permanent disability from
blindness and other serious vasculitic complications. Many clinicians diagnose and man-
age GCA without recourse to temporal artery biopsy. However, most experts agree that
biopsy is indicated where there is a high suspicion of GCA. Given that the condition
requires prolonged steroid treatment, it seems prudent to make a definitive diagnosis
wherever possible. Diagnosis and management of PMR and GCA are summarized in
Figure 29.1. The aetiology remains unknown but there appears to be some genetic influ-
ence, and amongst environmental factors a variety of infectious triggers have been pro-
posed. Steroid treatment is usually required for at least two years. Careful monitoring of
the clinical response and inflammatory markers is required to minimize exposure to
steroids. Low-dose aspirin and osteoporosis prophylaxis should be considered, and all
patients should be monitored for potential side effects of corticosteroids.

Further Reading
1 Gonzalez-Gay MA, Garcia-Porrua C, Miranda-Filloy JA, Martin J. Giant cell arteritis and
polymyalgia rheumatica: pathophysiology and management. Drugs Aging 2006; 23: 62749.
2 Carroll SC, Gaskin BJ, Danesh-Meyer HV. Giant cell arteritis. Clin Experiment Ophthalmol
2006; 34: 15973.
3 Nordborg E, Nordborg C. Giant cell arteritis: epidemiological clues to its pathogenesis and an
update on its treatment. Rheumatology 2003; 42: 41321.
4 Smeeth L, Cook C, Hall AJ. Incidence of diagnosed polymyalgia rheumatica and temporal
arteritis in the United Kingdom, 19902001. Ann Rheum Dis 2006; 65: 10938.
5 Morgan AW, Robinson JI, Barrett JH et al. Association of FCGR2A and FCGR2A-FCGR3A
haplotypes with susceptibility to giant cell arteritis. Arthritis Res Ther 2006; 8: R109.
6 Cid MC, Hoffman MP, Hernandez-Rodriguez J et al. Association between increased CCL2
(MCP-1) expression in lesions and persistence of disease activity in giant-cell arteritis.
Rheumatology 2006; 45: 135663.
7 Parikh M, Miller NR, Lee AG et al. Prevalence of a normal C-reactive protein with an elevated
erythrocyte sedimentation rate in biopsy-proven giant cell arteritis. Ophthalmology 2006; 113:
18425.
8 Lee MS, Smith SD, Galor A, Hoffman GS. Antiplatelet and anticoagulant therapy in patients
with giant cell arteritis. Arthritis Rheum 2006; 54: 33069.
9 Mazlumzadeh M, Hunder GG, Easley KA et al. Treatment of giant cell arteritis using
induction therapy with high-dose glucocorticoids: a double-blind, placebo-controlled,
randomized prospective clinical trial. Arthritis Rheum 2006; 54: 331018.
10 Narvez J, Bernad B, Daz Torn C et al. Low serum levels of DHEAS in untreated polymyalgia
rheumatica/giant cell arteritis. J Rheumatol 2006; 33: 12938.
05-PS Rheumatology-cpp:05-PS Rheumatology-ppp.QXD 18/3/08 15:26 Page 165

30 Behets syndrome 165

P R O B L E M

30 Behets Syndrome

Case History
A 26-year-old woman presents with subacute onset of fever and arthralgia and has
developed oral and genital ulceration within the past two weeks. There is no family
history of note. She has recently returned from a holiday in a subtropical region but there
is no evidence of an infectious disease. Her previous health has been excellent and she
takes no medications apart from the oral contraceptive pill.
How can the diagnosis of Behets syndrome be established?
What treatment options are available?
What is her likely prognosis?

Background
Behets syndrome (AdamantiadesBehets disease) is a relapsing and remitting vas-
culitic condition of unknown aetiology, the manifestations and severity of which vary
widely from patient to patient. The condition typically runs a more severe course in men
and in those of either gender who develop it at an earlier age (<25 years). The condition
was first described by Hulusi Behet in 1937, and is most common in patients of
Mediterranean or Asian origin. The common features are summarized in Table 30.1.1,2

Table 30.1 Features of Behets syndrome

Oral aphthous ulceration 98%


Genital ulceration 80%
Anterior uveitis 50%
Acne Retinal vasculitis
Erythema nodosum
Skin ulceration
Arthralgia Non-destructive arthritis
Vasculitis: Arteritis Cerebral lesions
Thrombophlebitis Pulmonary lesions
Aneurysm formation
Gastrointestinal ulceration
Epididymitis

Atlas Medical Publishing Ltd


05-PS Rheumatology-cpp:05-PS Rheumatology-ppp.QXD 18/3/08 15:26 Page 166

166 05 Vasculitic Syndromes

The most common feature is oral ulceration. The ulcers vary in size but are typically
small and occur in crops on the gums, inside of the mouth and on the tongue. Genital
ulcers also occur in the majority, and frequently leave residual scarring when they have
healed. Uveitis causes blurred vision, discomfort (particularly when looking at bright
lights) and redness. Retinal vasculitis may lead to blindness if left untreated. Joint symp-
toms most commonly affect the knees, wrists, ankles and knees. Central nervous system
involvement may cause headache, neck stiffness (if the meninges are involved), features
of encephalitis (including fever, confusion, impaired consciousness, fits and coma) and
focal neurological signs and symptoms. The disease usually starts in the 20s and 30s. It is
more common in men in prevalent areas but in other countries, such as the United
States, it occurs slightly more commonly in women.
The aetiology appears to involve both genetic and environmental factors. The most
prominent pathological feature is involvement of the small blood vessels with either vas-
culitis or thrombosis. Abnormalities at the vascular endothelium appear to be responsible
for initiating the disease. Immunoglobulin M (IgM) anti-endothelial antibodies have been
described. These may be, in part, directed at the endothelial enzyme a-enolase, a compo-
nent of the glycolytic pathway. Cross-reaction of these antibodies with proteins of other
organisms has been proposed to account for some of the environmental component of the
aetiology. These organisms include Saccharomyces cerevisiae, Streptococcus pneumoniae and
Candida albicans. There is markedly increased expression of cytokines and their receptors
in affected tissues. The role of autoimmunity is uncertain there are no specific immune
markers. Diagnosis is made clinically and may be confirmed by biopsy of affected tissue.
Treatment depends on the severity and manifestations of the disease. There is no spe-
cific treatment. Milder cases only require symptomatic therapy (e.g. mouthwashes for
oral ulceration or non-steroidal anti-inflammatory drugs for arthralgia). Genital ulcer-
ation can be treated with azathioprine, cyclosporine, colchicine or thalidomide. The lat-
ter has to be used with care in female patients because of its teratogenicity. Biological
agents may be required in severe cases of orogenital ulceration. Agents with proven effect
include interferon-a, etanercept and infliximab. Uveitis or retinal vasculitis can be sight-
threatening and require prompt treatment. The usual first-line approach is cortico-
steroids plus one of the cytotoxic agents (azathioprine or cyclophosphamide).
Cyclosporine or interferon-a may be used for refractory cases. Anticoagulation should
be considered in those with severe thrombophlebitis. There is a risk of haemorrhage from
aneurismal or vasculitic lesions (particularly in the lungs).

Recent Developments
1 Behets syndrome sufferers have a high prevalence of psychological disorders and
quality of life is impaired. The latter particularly relates to fatigue, arthralgia and
mucocutaneous lesions.3,4 Effective management of the disorder requires that
psychological and psychosocial factors are taken into account.
2 The genetic contribution is not at all well understood. The best-characterized marker
is HLA-B51, but a wide variety of other genetic polymorphisms have been studied.
Recently, the tumour necrosis factor (TNF)-a-1031C allele has been linked with
susceptibility to Behets syndrome.5 Other workers have found no relationship with
05-PS Rheumatology-cpp:05-PS Rheumatology-ppp.QXD 18/3/08 15:26 Page 167

30 Behets syndrome 167

polymorphisms of the TNF-a gene.6 Other candidate genes that have been studied
include those for the solute carrier SLC11A1 and the interleukin-18 gene. No
association with polymorphisms of these genes was found, although levels of IL-18
are particularly high in patients with Behets syndrome. The disease may be a
disorder of the interaction between the innate immune system and bacterial
commensals or pathogens. A recent study found no link with polymorphisms for the
genes for Toll-like receptors and no link with leptin gene polymorphism.7,8 High
levels of leptin have been described in Behets syndrome. One positive link
described recently has been a relationship with the Val16 allele for the superoxide
dismutase gene in Japanese patients.9
3 Erdem et al.10 have reported increased insulin resistance in Behets syndrome
patients. This is not surprising since systemic inflammation and endothelial
dysfunction are both forerunners of insulin resistance. Other vascular risk factors in
Behets patients include impaired endothelium-dependent vasodilatation and
increased circulating levels of homocysteine.11 Recently, increased platelet activation
has been described and may contribute to the prothrombotic state.12

Conclusion
The aetiology of Behets syndrome is still relatively poorly understood. The diagnosis is
made on clinical grounds. There is no specific serological marker. Measures of systemic
inflammation including serum C-reactive protein and cytokine levels may be useful in
monitoring disease activity. Biopsy of affected tissue may help to establish the diagnosis.
There is no specific treatment. The cornerstone of its management is symptomatic mea-
sures along with judicious use of corticosteroids and other immunosuppressive agents.
The typical course is a relapsing and remitting one, but the disease does not usually lead
to premature mortality. Vascular disorders and neurological involvement are the major
threats but only occur in a minority of patients.

Further Reading
1 Kalayciyan A, Zouboulis C. An update on Behets disease. J Eur Acad Dermatol Venereol 2007;
21: 110.
2 Yazici H, Fresko I, Yurdakul S. Behets syndrome: disease manifestations, management, and
advances in treatment. Nat Clin Pract Rheumatol 2007; 3: 14855.
3 Bodur H, Borman P, Ozdemir Y, Atan C, Kural G. Quality of life and life satisfaction in
patients with Behets disease: relationship with disease activity. Clin Rheumatol 2006; 25:
32933.
4 Mumcu G, Inanc N, Ergun T et al. Oral health related quality of life is affected by disease activ-
ity in Behets disease. Oral Dis 2006; 12: 14551. [Erratum appears in Oral Dis 2006; 12: 356]
5 Akman A, Sallakci N, Coskun M et al. TNF-alpha gene 1031 T/C polymorphism in Turkish
patients with Behets disease. Br J Dermatol 2006; 155: 3506.
05-PS Rheumatology-cpp:05-PS Rheumatology-ppp.QXD 18/3/08 15:26 Page 168

168 05 Vasculitic Syndromes

6 Ates A, Kinikli G, Dzgn N, Duman M. Lack of association of tumor necrosis factor-alpha


gene polymorphisms with disease susceptibility and severity in Behets disease. Rheumatol Int
2006; 26: 34853.
7 Bacanli A, Sallakci N, Yavuzer U, Alpsoy E, Yegin O. Toll-like receptor 2 Arg753Gln gene
polymorphism in Turkish patients with Behets disease. Clin Exp Dermatol 2006; 31:
699701.
8 Aydin F, Kara N, Senturk N et al. Lack of association between leptin G2548A gene polymor-
phism and Behets disease. J Eur Acad Dermatol Venereol 2007; 21: 6871.
9 Nakao K, Isashiki Y, Sonoda S, Uchino E, Shimonagano Y, Sakamoto T. Nitric oxide synthase
and superoxide dismutase gene polymorphisms in Behet disease. Arch Ophthalmol 2007; 125:
24651.
10 Erdem H, Dinc A, Pay S, Simsek I, Turan M. Peripheral insulin resistance in patients with
Behets disease. J Eur Acad Dermatol Venereol 2006; 20: 3915.
11 Kayikioglu M, Aksu K, Hasdemir C et al. Endothelial functions in Behets disease.
Rheumatol Int 2006; 26: 3048.
12 Akar S, Ozcan MA, Ates H et al. Circulated activated platelets and increased platelet reactivity
in patients with Behets disease. Clin Appl Thromb Hemost 2006; 12: 4517.
06-PS Rheumatology-cpp:06-PS Rheumatology-ppp.QXD 18/3/08 14:31 Page 169

S E C T I O N S I X 06
Back and Specific Joint Problems
31 Acute back pain
32 Chronic back pain
33 Psoriatic arthritis
34 Asymptomatic hyperuricaemia
35 Gout acute attack and beyond
36 Pseudogout investigation and management
37 Joint and bone infections
38 Viral arthritis
39 Rheumatological complications of diabetes

P R O B L E M

31 Acute Back Pain

Case History
You have been asked to do a home visit to see Bob. He is a 46-year-old storeman who is
moderately overweight (body mass index 32 kg/m2) and who had a sudden onset of low
back pain yesterday when moving a box of parts in the storeroom.
What features enable you to plan investigation and management?
What are the important prognostic features?

Background
Low back pain (LBP) is common, with an annual incidence of 2%5%, and 70%90% of
the population will experience an episode of LBP at some point. Prevalence increases
with age until 65 years and decreases thereafter. LBP is the most common work-related
injury and is second only to the common cold as a cause of absence from work (Table

Atlas Medical Publishing Ltd


06-PS Rheumatology-cpp:06-PS Rheumatology-ppp.QXD 18/3/08 14:31 Page 170

170 06 Back and Specific Joint Problems

Table 31.1 Risk factors for LBP

Strongest: Previous history of back pain


Strong: Poor job satisfaction
Emotional distress
Manual labourer involving heavy lifting
Prolonged sitting or standing
Moderate: Vibration tool use
Smoking
Obesity
Poor physical fitness

31.1). Fortunately, 90% of individuals with acute LBP improve within four to eight
weeks, and only 5% of patients develop persistent or chronic LBP lasting longer than
three months. For practical purposes, LBP is bounded by the level of the 12th thoracic
vertebra and the gluteal folds, and pain may radiate into the leg.
The cause of LBP is unknown in at least 80% of cases (Table 31.2) LBP is classified as
non-specific LBP or simple backache, nerve-root (Box 31.1) or spinal nerve compromise,
or potentially serious spinal pathology (including infection, cancer, fracture, inflamma-
tory back pain and cauda equina syndrome). In the retrospective study of Deyo et al.,1 of

Table 31.2 Causes of LBP

97% Mechanical LBP


80% Idiopathic
20% Prolapsed intervertebral disc
Lumbar spondylosis
Spondylolisthesis
Spinal stenosis
Fracture
Scheuermanns disease
1% Non-mechanical spinal pain
Inflammatory spondyloarthritis
1%2% Visceral disease
Infection
Neoplasia

Table 31.3 Red flags of LBP

Weight loss, fever, night sweats


Nocturnal pain
History of malignancy
Recent or current infection
Acute onset <20 or >55 years
Constant or progressive pain
Bilateral or alternating symptoms
Neurological or sphincter disturbance
Morning stiffness
Claudication or peripheral ischaemia
06-PS Rheumatology-cpp:06-PS Rheumatology-ppp.QXD 18/3/08 14:31 Page 171

31 Acute back pain 171

Box 31.1 Indicators for nerve root problems3


Unilateral leg pain > low back pain
Radiates to foot or toes
Numbness and paraesthesia in same distribution
Straight leg-raising induces more leg pain
Localized neurology in one nerve root

subjects presenting with acute LBP, 4% had compression fractures and 1% had cancer,
infection, inflammatory disorders or cauda equina syndrome. Another cohort of patients
with acute LBP had an even smaller percentage of serious pathology.2 Sinister causes of
back pain are relatively rare and suspicious features are listed in Table 31.3.
Investigations are not usually indicated in simple backache. Where investigation is
required, it should be aimed at confirming a specific pathological process. Cauda equina
syndrome should be suspected when leg pain that includes several spinal nerve levels is
accompanied by widespread motor and/or sensory weakness, and when there is associ-
ated bladder or bowel dysfunction.

Specific pathology leading to acute LBP


The spine is commonly described as a three-joint complex, consisting of the interverte-
bral disc anteriorly and the two facet joints posterolaterally. These border the triangular-
shaped spinal canal (Figure 31.1). Deformity of the three-joint complex, anterior or
posterior subluxation (spondylolisthesis) of a vertebral body, as well as deformity of the
posterior longitudinal ligament and ligamentum flavum can lead to nerve-root impinge-
ment in the lower spine or spinal cord compression at higher levels. The spinal cord ends
at approximately the level of lumbar vertebrae L1/L2 so that neurological manifestations
in LBP syndromes are lower motor neurone lesions affecting peripheral nerve roots. The
most common site of disc prolapse is the L4/L5 disc where a posterolateral protrusion
impinges on the L5 nerve root. A centrally placed bulge may affect multiple lower nerve
roots bilaterally, causing a cauda equina syndrome. The combined pathologies of facet
joint hypertrophy/osteophyte formation, disc bulges and deformity of the long ligaments
stenoses the spinal canal and presents with pseudoclaudication, sciatica and dysfunction
of the cauda equina.

Intervertebral
disc

Normal intervertebral Central disc bulge and posterolateral bulge Facet joint hypertrophy
disc, facet joints and with ligamentum flavum hypertrophy
exiting nerve root

Figure 31.1 Nerve root pain.


06-PS Rheumatology-cpp:06-PS Rheumatology-ppp.QXD 18/3/08 14:31 Page 172

172 06 Back and Specific Joint Problems

Prolapsed intervertebral disc


The intervertebral disc comprises the inner nucleus pulposus and the outer annulus
fibrosus. The nucleus is a proteoglycan matrix that must remain hydrated to act as a
shock absorber. The overlapping zig-zag alignment of the layers of collagen fibres within
the annulus provides resistance to bulging when load is applied. The outer third of the
annulus is innervated such that mechanoreceptors respond to distortion.

Spinal fracture
In the older patient, particularly female, presenting with sudden-onset localized back
pain after minor trauma, osteoporotic compression fracture should be suspected.
Fracture in younger people without major trauma is a rare cause of LBP. Lateral spinal
views for compression fractures, or oblique views, will identify established or major
changes on X-ray. Isotope bone scan or single photon emission computed tomography
(SPECT) imaging may be more sensitive.

Cancer
Four clinical features have been proposed to be most predictive of malignancy as a cause
of LBP:
1 Previous history of cancer
2 Aged 50 years or older
3 Failure of conservative therapy
4 Weight loss
An erythrocyte sedimentation rate of >50 mm/h also leads to magnetic resonance
imaging (MRI) investigation.

Infection
A history of immunosuppression or risk factors for breaches in the normal barriers to
bacteraemia should be elicited, coupled with an examination looking for septic foci.
Culture and microbial analysis of relevant tissue should be undertaken. In 40% of cases
of spinal osteomyelitis there is haematogenous spread from an identifiable extraosseous
source, most commonly genitourinary, skin or respiratory. The most common organism
is Staphylococcus aureus, with Gram-negative organisms in the elderly or intravenous
drug users being common.

Inflammatory LBP
It has been proposed that amongst patients with chronic LBP, up to 5% have inflamma-
tory back pain due to spondyloarthropathy (i.e. ankylosing spondylitis, reactive arthritis,
psoriatic arthritis or inflammatory bowel disease).

Management of acute LBP


Patients benefit from non-steroidal anti-inflammatory drugs (NSAIDs) to relieve pain,
and should be encouraged to remain active (Box 31.2). Medications such as muscle relax-
ants may help but can cause drowsiness.
Although the contribution of psychosocial factors to chronic LBP is appreciated,
successful interventions aimed at patients with increased risk are yet to be identified. A
06-PS Rheumatology-cpp:06-PS Rheumatology-ppp.QXD 18/3/08 14:31 Page 173

31 Acute back pain 173

Box 31.2 Summary of treatment guidelines for acute LBP3


Reassure patient of favourable prognosis
Advise to remain active, and discourage bed rest
Do not prescribe specific back exercises
Prescribe regular medication consisting of:
Paracetamol
NSAIDs
Consider benefit of muscle relaxants

general practitioner-implemented intervention focused on the identification of psy-


chosocial prognostic factors and discussing these with the patient, setting specific goals
for reactivation and providing an educational booklet had no effect on any outcome
measure.4

Outcome of LBP
The prognosis of chronic LBP is poorly documented, with cohorts varying as to their
composition and adequacy for defining an underlying aetiology of pain. Socioeconomic
factors including litigation, unemployment benefits and educational opportunities also
impact on outcome. Poor prognostic factors include fear avoidance behaviour, leg pain
and low job satisfaction. However, it is consistently reported that over 90% of patients
improve by six to twelve weeks. Refshauge and Maher5 have explored this further; the
findings are a reminder that not all patients with LBP present for medical care (and that
this may not be influenced by severity of pain), and that others return to work despite
ongoing pain. Recurrences of LBP are common, with an estimated three-quarters of
patients having a recurrence within one year. The severity, however, is usually less and
does not always lead to a new visit to the doctor.

Recent Developments
1 In the majority of patients with ankylosing spondylitis there is a ten-year delay
between onset of symptoms and diagnosis. In about 95% of patients, the first
symptoms of spondyloarthritis develop before the age of 45 years and this should be
the target group for screening. To help reduce the delay between symptom onset and
diagnosis, it has been proposed that patients with LBP of duration greater than three
months should be screened for the presence of inflammatory back pain, the presence
of HLA-B27 and for evidence of sacroiliitis.6 If one or more of these parameters is
confirmed, patients should be referred to a rheumatologist.
2 New criteria have been proposed by Rudwaleit et al.7 to assist in defining
inflammatory LBP. The four parameters are (a) morning stiffness duration >30 min,
(b) improvement with exercise but not with rest, (c) awakening during the second
half of the night and (d) alternating buttock pain. For classification purposes, two or
more criteria have a sensitivity of 70% and specificity of 81%. As the number of
parameters increases, the likelihood ratio also increases, from 0.25 for no parameters
to 2.3 for two parameters and to 12.0 for three or more parameters.7
06-PS Rheumatology-cpp:06-PS Rheumatology-ppp.QXD 18/3/08 14:31 Page 174

174 06 Back and Specific Joint Problems

Conclusion
The majority of patients with LBP require either no medical intervention or only conser-
vative therapies consisting of advice, simple analgesics and continuation of daily activi-
ties. Considering, however, its prevalence and impact on health economy there are still a
large number of therapies offered but not of proven benefit. Critical appraisal of features
identified in history-taking and examination is required to provide a more accurate and
specific diagnosis, and hence provide guidance on investigation, treatment and prog-
nosis.

Further Reading
1 Deyo R, Rainville J, Kent D. What can the history and physical examination tell us about low
back pain? JAMA 1992; 268: 7605.
2 McGuirk B, King W, Govind J, Lowry J, Bogduk N. Safety, efficacy, and cost effectiveness of
evidence-based guidelines for the management of acute low back pain in primary care. Spine
2001; 26: 261522.
3 Koes BW, van Tulder MW, Thomas S. Diagnosis and treatment of low back pain. BMJ 2006;
332: 14304.
4 Jellema P, van der Windt DAWM, van der Horst HE, Twisk JWR, Stalman WAB, Bouter LM.
Should treatment of (sub)acute low back pain be aimed at psychosocial prognostic factors?
Cluster randomised clinical trial in general practice. BMJ 2005; 331: 8490.
5 Refshauge KM, Maher CG. Low back pain investigations and prognosis: a review. Br J Sports
Med 2006; 40: 4948.
6 Sieper J, Rudwaleit M. Early referral recommendations for ankylosing spondylitis (including
pre-radiographic and radiographic forms) in primary care. Ann Rheum Dis 2005; 64: 65963.
7 Rudwaleit M, Metter A, Listing J, Sieper J, Braun J. Inflammatory back pain in ankylosing
spondylitis: a reassessment of the clinical history for application as classification and
diagnostic criteria. Arthritis Rheum 2006; 54: 56978.
06-PS Rheumatology-cpp:06-PS Rheumatology-ppp.QXD 18/3/08 14:31 Page 175

32 Chronic back pain 175

P R O B L E M

32 Chronic Back Pain

Case History
Jolene is a 43-year-old woman who has become rather depressed and angry about her
painful back, which has been present for two years. She was a typist until her chair slipped
out from under her and she landed heavily on her sacrum. Despite the best efforts of
health practitioners she still experiences severe pain that no one can explain. She is
frustrated at the number of doctors who have seen her and left her with the impression
there is nothing abnormal to find.
Are there strategies for successful management of chronic low back pain (LBP)?
When should we investigate further?

Background
In patients with chronic LBP the terms non-specific or mechanical LBP are used with an
implication that an anatomical or pathological basis is understood. In reality, the aetiology
remains unknown for the majority of cases. Observation is used to determine alteration in
posture, muscle wasting, changes in the physiological lordosis and the effects of move-
ment on spinal alignment. Palpation can be undertaken to assist in evaluating movement
at a segmental level, and also to identify local tenderness. Dynamic procedures are used to
elicit alteration in symptoms to a manoeuvre that is aimed at identifying the cause of
symptoms. A systematic review analysed examination procedures used in the assessment

Table 32.1 Reliability of clinical examination

Observation
Judging lordosis: two quality studies indicating reliability, two against
Detecting lateral spinal shift: low reliability
Evaluating abnormal posture or movement: low reliability
Timed endurance testing: high reliability in low-quality studies
Palpation
Evaluating muscle tension or spasm: conflicting evidence
Existence of a fixation or manipulative lesion: low reliability
Identifying spinal level: low reliability
Instability testing: conflicting evidence
Symptom response
Pain response to repeated movement: conflicting to low reliability
Pain on palpation and trigger points: low reliability

Atlas Medical Publishing Ltd


06-PS Rheumatology-cpp:06-PS Rheumatology-ppp.QXD 18/3/08 14:31 Page 176

176 06 Back and Specific Joint Problems

of LBP.1 Methodological flaws were identified in the majority of studies, but the results
remain interesting. Palpation-based assessment was of low reliability, with moderate reli-
ability of some examination procedures based on symptom response (Table 32.1).
Abnormalities on X-ray and magnetic resonance imaging (MRI) are poorly associated
with the occurrence of non-specific LBP. Abnormalities found when imaging people with-
out back pain are just as prevalent as those found in patients with LBP, and it needs to be
remembered that radiological abnormalities of degeneration and spondylolysis have been
reported in 40%50% of people without LBP. With a prevalence of such magnitude, it has
been suggested that this background prevalence of abnormalities be included in radiologi-
cal reports.2 Computed tomography and MRI are equally effective in diagnosing lumbar
disc herniation and stenosis, but only have meaning if the presentation had suggested this
as the likely diagnosis and the imaging findings remain compatible with clinical findings.

Conservative treatment
Exercise and intensive multidisciplinary pain-treatment programmes are effective (Box
32.1). Evidence supports use of cognitive behaviour therapy, analgesics, antidepressants,
non-steroidal anti-inflammatory drugs (NSAIDs), back schools and spinal manipula-
tion. However, the effects are usually only small and short term. Unfortunately, many
commonly used interventions lack sufficient evidence of benefit. No evidence supports
using interventions such as steroid injections, lumbar supports and traction.

Box 32.1 Guidelines for the treatment of chronic LBP4,5


Recommend cognitive behavioural therapy, supervised exercise, brief educational
interventions and multidisciplinary (biopsychosocial) treatments
Advise patient to remain active
Short-term use of NSAIDs and weak opioids
Consider net benefit of muscle relaxants, back schools, antidepressants and
manipulation
Avoid passive treatment such as ultrasound and shortwave diathermy
Invasive treatments such as facet joint injections generally not recommended

Invasive treatment
A review of the efficacy of surgery and invasive interventions for the treatment of LBP
and sciatica indicated that facet joint, epidural, trigger point and sclerosant injection
have not clearly been shown to be effective.3 No sound evidence is available for the effi-
cacy of spinal stenosis surgery, although surgical discectomy may be considered in those
with sciatica due to lumbar disc prolapse who have failed to respond to conservative
management. Randomized controlled trials (RCTs) comparing fusion surgery with con-
servative treatment show conflicting results.

Recent Developments
1 Sciatica resulting from disc herniation resolves in the majority of patients within four
weeks. Of fifteen RCTs of epidural steroid injection, nine showed no benefit, with six
06-PS Rheumatology-cpp:06-PS Rheumatology-ppp.QXD 18/3/08 14:31 Page 177

32 Chronic back pain 177

showing short-term benefit that did not last longer than one month. Valat has
reviewed an additional three studies,6 in which patients received three epidural
injections at between two-daily (one study) and three-weekly (two studies) intervals.
Three-weekly methylprednisolone (80 mg) did not alter the primary outcome
measure (the Oswestry Disability Questionnaire). Alternate daily prednisolone
(50 mg) also had no effect. Three-weekly triamcinolone (80 mg) plus 10 ml
bupivacaine led to significant improvement in leg pain and self-reported function at
three weeks, which was not sustained to six weeks or at one year.
2 Martell and colleagues reviewed the prevalence of opioid use in chronic LBP, its
effectiveness and the prevalence of substance use disorders.7 Meta-analysis of four
studies assessing the efficacy of opioids compared with placebo or a non-opioid
control did not show reduced pain with opioids. Meta-analysis of five studies directly
comparing efficacy of different opioids showed there was no reduction in pain. On
the important end-point of substance use disorders, there was a lifetime prevalence
of substance use disorders of 1:2 to 1:3, with the estimate of the prevalence of current
substance use disorder as high as 43%. It has to be noted that none of the trials
extended beyond four months. While short-term use of opioids may be beneficial,
the value beyond four months is unclear, with substance use disorders common and
aberrant medication-taking behaviours documented in a quarter of cases.
3 In another recent meta-analysis, the efficacy of psychological interventions was
evaluated in patients with non-malignant chronic LBP.8 Outcomes included pain
intensity, emotional functioning, physical functioning, participant rating of global
improvement, healthcare use, pain medication and compensation status. Positive
effects of psychological interventions, contrasted with various control groups, were
noted for pain intensity, pain-related interference, health-related quality of life and
depression. Cognitive behavioural and self-regulatory treatments were found to be
efficacious. Multidisciplinary approaches that included a psychological component
were also noted to have short-term effects on pain interference and positive long-
term effects on return to work.

Conclusion
Chronic LBP is common and produces a significant burden on healthcare resources, the
individual and their caring practitioners. Treatment guidelines provide much-needed
information on what works and what does not, but the meta-analyses underpinning
these recommendations also identify the weaknesses in our studies to date. The apparent
benefit of psychological interventions should be borne in mind, and should be invoked in
the maintenance of an ongoing supportive and empathetic relationship with the patient
who has chronic LBP.

Further Reading
1 May S, Littlewood C, Bishop A. Reliability of procedures used in the physical examination of
non-specific low back pain: a systematic review. Aust J Physiother 2006; 52: 91102.
06-PS Rheumatology-cpp:06-PS Rheumatology-ppp.QXD 18/3/08 14:31 Page 178

178 06 Back and Specific Joint Problems

2 Roland M, van Tulder M. Should radiologists change the way they report plain radiography of
the spine? Lancet 1998; 352: 22930.
3 van Tulder MW, Koes B, Seitsalo S, Malmivaara A. Outcome of invasive treatment modalities
on back pain and sciatica: an evidence-based review. Eur Spine J 2006; 15: s8292.
4 Koes BW, van Tulder MW, Thomas S. Diagnosis and treatment of low back pain. BMJ 2006;
332: 14304.
5 Airaksinen O, Brox JI, Cedrashi C et al. European guidelines for the management of chronic
nonspecific low back pain. Eur Spine J 2006; 15 (Suppl 2): S192300.
6 Valat J-P. Epidural corticosteroid injections for sciatica: placebo effect, injection effect or anti-
inflammatory effect? Nat Clin Pract Rheumatol 2006; 2: 51819.
7 Martell BA, OConnor PG, Kerns RD et al. Systematic review: opioid treatment for chronic
back pain: prevalence, efficacy, and association with addiction. Ann Intern Med 2007; 146:
11627.
8 Hoffman BM, Papas RK, Chatkoff DK, Kerns RD. Meta-analysis of psychological
interventions for chronic low back pain. Health Psychol 2007; 26: 19.

P R O B L E M

33 Psoriatic Arthritis

Case History
Alice is 26 years old and has had psoriasis since age 18. This has been treated with topical
preparations and she currently has only mild skin disease. She presents with pain and
swelling in the small joints of her hands and reports pains in her back and right knee over
the past few months. Her mother, aged 56, has psoriasis with associated arthritis.
Is it likely that she is developing psoriatic arthritis (PsA)?
How relevant is the positive family history?
What steps should be taken to diagnose psoriatic arthritis?
What is the treatment and prognosis?

Atlas Medical Publishing Ltd


06-PS Rheumatology-cpp:06-PS Rheumatology-ppp.QXD 18/3/08 14:31 Page 179

33 Psoriatic arthritis 179

Background
Psoriasis is a common disorder of keratinocyte growth and differentiation affecting up to
2% of the population. The skin lesions are red, scaly, raised plaques typically found on
the scalp, elbows and knees. Onset is usually in adolescence or early adulthood although a
late-onset form is recognized. Prevalence estimates for psoriatic arthritis range from 10%
to 40% of all cases of psoriasis. Clinically significant arthritis affects about one in six
patients with psoriasis.1,2 There is a decrease in the granular layer of keratinocytes, which
are not fully differentiated and do not stack normally. Psoriasis is considered to be an
organ-specific autoimmune disease that is predominantly T-cell mediated. The lesions
are heavily infiltrated with lymphocytes, monocytes and neutrophils. There is activation
of antigen-presenting cells and endothelial cells, and blood vessels at the base of the
lesions are hyperplastic. The T-cell response is a type 1 reaction with interleukin (IL)-12
and IL-23 production leading to increased interferon-g and tumour necrosis factor
(TNF)-a, with consequent activation of inflammatory pathways through signal trans-
ducer and activator of transcription-1 (STAT1). There is decreased local expression of
the anti-inflammatory cytokine IL-10. Platelet-derived growth factor (PDGF) and vascu-
lar endothelial growth factor (VEGF) contribute to the intense inflammatory and vascu-
lar reaction. The normal keratinocyte maturation cycle spans 30 days, and this is
truncated to only four days in psoriatic lesions.
There is a strong genetic component in the aetiology of psoriasis. It is common in
Caucasians, much less common in those of Asian descent (prevalence 0.1%) and very
rare in individuals of African descent. Up to one-third of patients have a family history of
the condition and 8% have an affected first-degree relative. Concordance rate in dizy-
gotic twins is 15%30% and in monozygotic twins is around 65%. At least five suscepti-
bility loci have been identified for psoriasis by genome-wide scanning studies.3 The most
studied locus (PSORS1) is in the HLA region of chromosome 6p. Susceptibility is linked
with a haplotype that includes the HLA-Cw0602 allele. However, non-HLA determinants
in this region are also thought to be important (including polymorphisms of TNF-a),
and there has been particular interest in the gene for corneodesmin (CDSN), which codes
for a protein involved in keratinocyte maturation. The PSORS2 locus at chromosome
17q25 contains several candidate genes including SLC9A3R1, NAT9 and RAPTOR (the
p150 target of rapamycin). Other loci are PSORS3 (4q35), PSORS4 (1q21) and PSORS5
(3q21). PSORS2 and PSORS4 contain genes that are also involved in susceptibility to
atopic dermatitis. There is relatively little known about the genetics of PsA. The CARD15
gene at chromosome 16q has been suggested as a candidate, and is also involved in sus-
ceptibility to Crohns disease.
PsA characteristically begins after five to ten years of psoriasis, but may present at the
same time as the skin lesions or even precede them. PsA is an important determinant of
quality of life and disease burden associated with psoriasis. Studies have been hampered
by lack of a specific biomarker, thus placing reliance on clinical scoring systems. The sim-
plest of these is presented in Box 33.1. It should not be assumed that all arthritis in
patients with psoriasis is PsA other conditions should be carefully excluded. At onset,
there is pain, redness and stiffness which usually only affects a few joints in the first
instance, with development of the typical symmetrical polyarthropathy over time.
Involvement of the distal interphalangeal (DIP) joints is usual, and helps to distinguish
06-PS Rheumatology-cpp:06-PS Rheumatology-ppp.QXD 18/3/08 14:31 Page 180

180 06 Back and Specific Joint Problems

Box 33.1 Scoring system for diagnosis of PsA


PsA can be diagnosed when the patient has an inflammatory arthropathy (articular,
spine enthesis) plus 3 from the following:
1 Active psoriasis, a personal or family history of psoriasis; current active psoriasis
scores 2
2 Nail dystrophy pitting, onycholysis, hyperkeratosis
3 Rheumatoid factor (RF) negative
4 Active or previous dactylitis
5 Juxta-articular new bone formation on X-ray of the hands and feet (not osteophytes)
All features score 1 if present, except active psoriasis, which scores 2. Adapted from Taylor et al. 2006.4

Box 33.2 Comparison between PsA and RA


PsA RA
Small joint involvement ++ +++
DIP involvement +++ +
Spine, sacroiliac joint involvement ++ +
Enthesitis +++ -
Synovitis ++ +++
Subcutaneous nodules - ++
Skin lesions +++ +
RF positive + +++
Anti-cyclic citrullinated peptide (anti-CCP) antibodies +++
Inflammatory markers ++ +++
Family history +++ +
When fully expressed, both diseases can cause a symmetrical, deforming polyarthropathy, which predominantly affects small joints of the hands
and feet.

PsA from rheumatoid arthritis (RA) (Box 33.2). PsA is classified with the spondyloarthri-
tides and spine involvement occurs in up to 40% of cases. One important characteristic
of PsA is the involvement of entheses the points at which tendons, ligaments and joint
capsules insert into bone. Enthesitis most frequently affects the plantar fascia, Achilles
tendon and the ribs and pelvis. Another feature is dactylitis, where a whole digit is
inflamed because of a combination of enthesitis and synovitis. These changes are best
diagnosed on a fat-suppressed magnetic resonance imaging scan.
Five different forms of PsA are recognized (often reflecting the stage to which the dis-
ease has progressed): asymmetrical; symmetrical; predominantly DIP joint involvement;
spondylitis; and arthritis mutilans. The latter is the most severe form, affecting mainly
the small joints of the hands and feet and causing considerable deformity. It affects less
than 5% of patients with PsA. Spondylodiscitis is another serious, though fortunately
rare, complication. SAPHO syndrome (Synovitis, Acne, Pustulosis, Hyperostosis and
Osteitis) is an unusual overlapping syndrome. PsA is a serious diagnosis with the minor-
ity of cases running an entirely benign course.
06-PS Rheumatology-cpp:06-PS Rheumatology-ppp.QXD 18/3/08 14:31 Page 181

33 Psoriatic arthritis 181

Inflammatory polyarthritis

Active psoriasis Previous psoriasis Exclude RA


F/H psoriasis
Nodules
RF*
Anti-CCP
Predominantly DIP**
Spinal, SI joint
Dactylitis
Nail dystrophy

PsA

Symptomatic DMARDs
Physiotherapy
NSAIDs

Biological therapies

Figure 33.1 Diagnosis and management of PsA. * RF and anti-CCP are positive in a proportion of patients
with PsA. ** One of these is sufficient to make a diagnosis of PsA in the presence of active psoriasis; two
should be present to make the diagnosis when there is a previous or family history (F/H) of psoriasis.
anti-CCP, antibodies to cyclic citrullinated peptides; DIP, distal interphalangeal; DMARDs, disease-
modifying antirheumatic drugs; NSAID, non-steroidal anti-inflammatory drug; PsA, psoriatic arthritis; RF,
rheumatoid factor; SI, sacroiliac.

Diagnosis and treatment is summarized in Figure 33.1. Management of PsA overlaps


with that of the skin condition, particularly where systemic disease-modifying drugs are
used. For mild cases, physical treatments (exercises, splinting etc.) along with non-
steroidal anti-inflammatory drugs where needed suffice. Intra-articular steroid is useful
for large joint inflammation. Systemic steroids are generally avoided because of the risk
of flare-up of skin lesions when the steroid is stopped. Methotrexate is the most com-
monly used disease-modifying drug for the skin condition and is also useful for joint
involvement. As for RA, sulphasalazine, chloroquine, hydroxychloroquine, azathioprine
and leflunomide may be useful. Mycophenolate mofetil has also been used. The involve-
ment of T cells in the pathogenesis of PsA has stimulated the use of cyclosporine and
tacrolimus (both anti-T-cell agents), although their use is often limited by renal toxicity.
Finally, biological agents are increasingly used for patients with severe disease. These
06-PS Rheumatology-cpp:06-PS Rheumatology-ppp.QXD 18/3/08 14:31 Page 182

182 06 Back and Specific Joint Problems

include IL-2 receptor antagonists, anti-T-cell agents (alefacept and efalizumab) and
TNF-a antagonists (etanercept, infliximab and adalimumab).

Recent Developments
1 Disturbed angiogenesis is a key feature of psoriasis, with increased local
production of angiogenic peptides. Butt et al.5 have investigated polymorphisms
in the genes for VEGF, fibroblast growth factor (FGF)-1 and FGF-2, and
epidermal growth factor (EGF). Increased prevalence of a polymorphism of the
VEGF gene was demonstrated in patients with PsA.
2 High levels of RF in a patient with symmetrical polyarthritis supports the
diagnosis of RA. However, RF positivity is not uncommon in inflammatory or
immune disorders other than RA, and does occur in a minority of patients with
PsA. Anti-CCP antibodies are more specific for RA, but have also been described
in 5%10% of patients with clinical PsA.6
3 Self-assessed quality of life and health status are directly linked to disease activity
in PsA.7 The number of joints either involved or deformed has a direct bearing.
Morning stiffness, level of inflammatory markers and duration of disease are also
important. With increasing duration of disease, the activity of the disease
becomes a less important determinant of health status.
4 Evidence is accumulating that disease-modifying drugs for PsA, including the
newer biological agents, are not only clinically effective but that their use also
brings cost benefits.8 This is leading to earlier and more aggressive treatment. PsA
is a progressive condition and evidence of radiological joint damage begins to
appear as the number of joints involved increases.9 There are strong arguments
for making the diagnosis early and limiting the progression of a disease that can
be devastating and disabling.
5 Psoriasis and PsA are associated with deterioration in cardiovascular risk profile.10
This is probably due to the widespread inflammation that accompanies these
disorders. Thus dyslipidaemia, hyperglycaemia, endothelial dysfunction and
oxidative stress are all increased, as may be the impact of other conventional
cardiovascular risk factors such as smoking and obesity. Recently, patients with
PsA have been shown to have increased carotid intimamedia thickness a direct
correlate with increased risk of vascular disease.11

Conclusion
PsA is a fairly common accompaniment to the skin lesions of psoriasis. It appears likely
that the above patient is developing PsA, but other causes of arthritis should be consid-
ered. The family history is highly relevant. Several genetic determinants of psoriasis have
now been identified. Environmental triggers for the disease are relatively poorly under-
stood at present. It is not clear to what extent the genetic predisposition for PsA is separ-
ate to that of the skin disease. Diagnosis of PsA is largely clinical. There is no specific
biological marker. Initial treatment is usually symptomatic, including the use of anti-
inflammatory drugs. Systemic corticosteroids are not usually administered. There is
06-PS Rheumatology-cpp:06-PS Rheumatology-ppp.QXD 18/3/08 14:31 Page 183

33 Psoriatic arthritis 183

increasing evidence favouring the early use of disease-modifying drugs, including the
newer biological agents. PsA is a serious disease, often progressive, and associated with
impaired function and quality of life as well as with increased cardiovascular risk.

Further Reading
1 Myers WA, Gottlieb AB, Mease P. Psoriasis and psoriatic arthritis: clinical features and
disease mechanisms. Clin Dermatol 2006; 24: 43847.
2 Lowes MA, Bowcock AM, Krueger JG. Pathogenesis and therapy of psoriasis. Nature 2007;
445: 86673.
3 Bowcock AM, Cookson WOCM. The genetics of psoriasis, psoriatic arthritis and atopic
dermatitis. Hum Mol Genet 2004; 13: R4355.
4 Taylor W, Gladman D, Helliwell P, Marchesoni A, Mease P, Mielants H. Classification
criteria for psoriatic arthritis: development of new criteria from a large international study.
Arthritis Rheum 2006; 54: 266573.
5 Butt C, Lim S, Greenwood C, Rahman P. VEGF, FGF1, FGF2 and EGF gene
polymorphisms and psoriatic arthritis. BMC Musculoskelet Disord 2007; 8: 17.
6 Inanc N, Dalkilic E, Kamali S et al. Anti-CCP antibodies in rheumatoid arthritis and
psoriatic arthritis. Clin Rheumatol 2007; 26: 1723.
7 Husted JA, Tom BD, Farewell VT, Schentang CT, Gladman DD. A longitudinal study of
the effect of disease activity and clinical damage on physical function over the course of
psoriatic arthritis. Arthritis Rheum 2007; 56: 8409.
8 Bansback NJ, Ara R, Barkham N et al. Estimating the cost and health status consequences
of treatment with TNF antagonists in patients with psoriatic arthritis. Rheumatology 2006;
45: 102938.
9 Bond SJ, Farewell VT, Schentag CT, Gladman DD. Predictors for radiological damage in
psoriatic arthritis: results from a single centre. Ann Rheum Dis 2007; 66: 3706.
10 Wakkee M, Thio HB, Prens EP, Sijbrands EJG, Neumann HAM. Unfavorable
cardiovascular risk profiles in untreated and treated psoriasis patients. Atherosclerosis 2007;
190: 19.
11 Kimhi O, Caspi D, Bornstein NM et al. Prevalence and risk factors of atherosclerosis in
patients with psoriatic arthritis. Semin Arthritis Rheum 2007; 36: 2039.
06-PS Rheumatology-cpp:06-PS Rheumatology-ppp.QXD 18/3/08 14:31 Page 184

184 06 Back and Specific Joint Problems

P R O B L E M

34 Asymptomatic Hyperuricaemia

Case History
Geoffrey is a 43-year-old man who recently applied for life insurance. As part of the
application, a biochemical profile was undertaken which, from your pathology service,
includes a serum uric acid level. Geoffreys result was 0.62 mmol/l (normal range
0.150.50 mmol/l). He has neither gout nor renal stones and is generally very healthy.
Should his hyperuricaemia be treated?
Should he be worried about vascular disease or renal impairment?
What lifestyle measures should be recommended?

Background
Uric acid (UA) is the final breakdown product of purine nucleotides (adenine and gua-
nine) in the body. The final metabolic steps are hypoxanthine to xanthine, which in turn
forms UA. Both these steps are catalysed by the enzyme xanthine oxidase (XO). In most
animals, UA is further metabolized to allantoin by urate oxidase (uricase). Allantoin is
510 times more soluble that UA and is readily eliminated by the kidneys. Humans and
other primates have a non-sense mutation that results in defective uricase activity and
thus higher concentrations of serum urate are reached.
UA is a weak acid and in the extracellular fluid is predominantly ionized 98% as
monosodium urate. Plasma is saturated with UA at a concentration of 0.415 mmol/l
(6.8 mg/dl) and above. When the plasma is saturated, UA crystals are liable to form. The
precise threshold for this varies slightly because of other physicochemical and environ-
mental influences. Excretion of UA is almost entirely by the kidney, with urine at pH 5
being virtually saturated with UA. At higher pH, the solubility of UA increases exponen-
tially and urine can contain more UA without risk of crystallization. It is for this reason
that urinary alkalinization is used with uricosuric agents.
UA can only be formed in tissues that contain XO (liver and the small intestine).
About two-thirds of the daily purine load is generated endogenously from turnover of
cells, while one-third is derived from diet. Levels are generally higher in men, and
increase substantially at puberty. In women, plasma UA increases after the menopause to
levels comparable to that of men. Women are at lower risk of gout but, when the satur-
ation threshold for UA is reached, crystallization is equally likely to occur. Increased UA
is present in 5%8% of males in the United States, and is higher in some racial groups
with particular predisposition to obesity and vascular disease.

Atlas Medical Publishing Ltd


06-PS Rheumatology-cpp:06-PS Rheumatology-ppp.QXD 18/3/08 14:31 Page 185

34 Asymptomatic hyperuricaemia 185

Ribose-5-P 1 ATP

ATP PRPP GTP

AMP Inosine monophosphate GMP

Adenosine Inosine Guanosine

Adenine Hypoxanthine

XO

Xanthine Guanine

XO

Urate

Figure 34.1 Purine metabolism. Ethanol, fructose feeding, glycogen storage diseases and severe hypoxia
lead to ATP depletion and thus increased influx of AMP and adenosine into the UA synthetic pathway. The
last two steps in the above, simplified pathway are catalysed by xanthine oxidase (XO). AMP, adenosine
monophosphate; ATP, adenosine triphosphate; GMP, guanosine monophosphate; GTP, guanosine
triphosphate; PRPP, 5 phosphoribosyl 1-pyrophosphate; Ribose-5-P, ribose-5-phosphate.

A detailed knowledge of UA metabolism1 is not required by the practising clinician. A


simplified scheme is presented in Figure 34.1. This also serves as a reminder of how
closely interrelated UA metabolism is with other aspects of intermediary metabolism. UA
is almost entirely secreted by the kidneys. The UA transporter URAT1 (SLC22A12) is
expressed in the proximal tubule and is a target for drug treatment. In most individuals
with high UA, no underlying cause will be found. The clinician, however, should be
aware of the large number of clinical associations. When there is an underlying cause,
90% are due to disorders of UA excretion while the remaining 10% relate to increased
UA production. The causes of hyperuricaemia are summarized in Box 34.1.
The most common contributor to UA overproduction is excessive dietary intake,
including foods high in purines. Also, obesity and metabolic syndrome are associated
with a marked tendency to increased UA synthesis. Fructose intake increases UA: follow-
ing cellular uptake, fructose is phosphorylated, consuming adenosine triphosphate
(ATP) and generating adenosine diphosphate (ADP). As shown in Figure 34.1, ADP
feeds into the UA synthetic pathway, and this is favoured by intracellular phosphate
depletion, which can also accompany fructose ingestion. Alcohol ingestion also leads to
intracellular ATP depletion and consequent increases in both ADP and adenosine
06-PS Rheumatology-cpp:06-PS Rheumatology-ppp.QXD 18/3/08 14:31 Page 186

186 06 Back and Specific Joint Problems

Box 34.1 Causes of hyperuricaemia


A: Increased UA production (10%)
Obesity Myeloproliferative disease Pagets disease
Alcohol Polycythaemia vera Psoriasis
Haemolysis Rhabdomyolysis
Glycogen storage diseases (type III, V and VII)
Inborn errors of metabolism (including HPRT deficiency)

B: Decreased UA excretion (90%)


Renal impairment Starvation
Polycystic kidney disease Acidosis (ketoacidosis, lactic acidosis)
Toxaemia of pregnancy
Hypothyroidism
Hyperparathyroidism Bartters syndrome
Diabetes insipidus Downs syndrome
Sarcoidosis Lead poisoning
Drugs: Aspirin Diuretics
Cyclosporine Ethambutol
L-dopa Pyrazinamide

monophosphate (AMP). Beer and spirits are more likely to provoke increased UA than
wine. A number of rare inborn errors of metabolism increase UA synthesis. The most
common of these is deficiency of hypoxanthine phosphoribosyltransferase (HPRT),
complete deficiency of which causes LeschNyhan syndrome (self-mutilation, choreoa-
thetosis and mental retardation). The disorder is X-linked and carrier females are asymp-
tomatic. A partial deficiency of the enzyme causes KelleySeegmiller syndrome with the
metabolic consequences of increased UA but without central nervous system manifesta-
tions. Other inborn errors include activating mutations of the phosphoribosylpyrophos-
phate synthetase gene and deficiencies of adenine phosphoribosyltransferase,
adenylsuccinate lyase, myoadenylate deaminase, adenosine deaminase and purine nucle-
oside phosphorylase.
The association of high UA with metabolic syndrome and cardiovascular risk has
attracted a great deal of attention.2,3 Increased UA may be a useful marker for insulin
resistance. Even in subjects who have not yet developed metabolic syndrome there is a
correlation between UA levels and body mass, total and high-density lipoprotein (HDL)
cholesterol and triglycerides. As overweight individuals lose weight, UA levels decrease in
parallel with serum leptin. Twenty to forty per cent of patients with untreated hyperten-
sion have high UA, and this is increased where there is also renal impairment. UA levels
almost always increase in evolving renal failure. However, although increased UA is a
marker for cardiovascular risk, it is not certain that it is an independent risk factor and
one that we should be attempting to modify. To date, there is no direct evidence to sug-
gest that drug treatment to decrease UA protects against cardiovascular events. There is
suggestive evidence from the Greek Atorvastatin and Coronary Heart Disease Evaluation
06-PS Rheumatology-cpp:06-PS Rheumatology-ppp.QXD 18/3/08 14:31 Page 187

34 Asymptomatic hyperuricaemia 187

Box 34.2 Dietary recommendations for hyperuricaemia and gout


Reduce body weight if overweight
Avoid extremes of food intake both large, rich meals and fasting can increase UA.
Eat small, regular meals
Maintain fluid (water) intake aim for 23 litres per day, particularly during flare-
ups of gout
Limit alcohol intake to no more than one drink per day
Avoid fatty food fried foods, fatty meats and excessive use of butter or margarine
Limit protein intake and focus on proteins that are low in purines (dairy produce)
The ideal diet is relatively high in carbohydrate but some high-fibre and wholegrain
foods are relatively high in purines
The following are high in purines and best avoided:
Game meats and birds; liver; kidney; small oil fish (sardines, anchovies, mackerel);
seafood (mussels, scallops, prawns); meat extract; yeast supplements
The following are relatively high in purines and intake should be limited:
Oatmeal (< cup per day); wheat bran and wheat germ (< cup per day); peas, beans,
2
3
1
4

spinach, asparagus, cauliflower and mushrooms (< cup per day); poultry, meat, fish
1
2

and shellfish (12 servings per day); dried beans, lentils and other pulses (maximum 1
cup per day cooked)

study4 and from the Losartan Intervention for Endpoint Reduction (LIFE) study with
losartan that decreasing UA may parallel decreasing vascular risk in intervention studies.5
The vast majority of patients with hyperuricaemia do not require treatment. This
should be reserved for patients who are either symptomatic or who are perceived to be at
particularly high risk. The only widely used drug is allopurinol, which is an inhibitor of
XO. All patients with high UA should have the benefit of dietary advice (Box 34.2). A
purine-free diet may also be used to investigate the underlying cause of hyperuricaemia.
On such a diet, normal individuals will excrete less than 3.6 mmol/day (600 mg) of UA in
the urine. Those who continue to excrete greater than 4.2 mmol/day almost certainly
have a condition that leads to UA overproduction. Uric acid excretion can be increased
by alkalinizing the urine with either sodium bicarbonate or acetazolamide.

Recent Developments
1 Fructose feeding of rats leads to development of the metabolic syndrome and also
increases circulating UA. In a recent animal study,6 the XO inhibitor allopurinol
and the uricosuric agent benzbromarone decreased the effect of fructose feeding
on increasing insulin levels, systolic blood pressure and triglycerides, and also
prevented weight gain. Furthermore, direct effects of UA-lowering agents on
endothelial function were demonstrated by studying the response of isolated
aortic rings to acetylcholine.
2 The recommended adult daily allowance for protein is 0.8 g per kilogram ideal body
weight. High protein intake is associated with higher UA, which may contribute to
06-PS Rheumatology-cpp:06-PS Rheumatology-ppp.QXD 18/3/08 14:31 Page 188

188 06 Back and Specific Joint Problems

decline in renal function in patients with chronic kidney disease.7 There is debate
about the benefit of restricted protein diets in patients with declining renal function.
The balance of evidence slightly favours protein restriction as a measure for patients
with chronic kidney disease.
3 At present, allopurinol is the only agent in common use for lowering UA. The drug is
well tolerated and generally quite effective. Febuxostat is a non-purine XO inhibitor.
A recent trial8 with this drug in patients with gout demonstrated that the drug was
well tolerated and was more effective than allopurinol in lowering serum UA.

Conclusion
Increased UA predisposes to gout, nephrolithiasis and renal impairment. Current evi-
dence does not favour routinely treating hyperuricaemia in asymptomatic individuals.
The exception is for patients that were treated with cytolytic chemotherapy for malignant
disease. Increased cellular breakdown in these patients can cause marked hyperuri-
caemia. Most patients with increased UA do not develop gouty arthritis, and the latter
can be effectively treated when it arises. A causative role for UA in the genesis of vascular
and renal disease remains to be established. The above patient should be encouraged to
maintain a healthy body weight and may consider restricting foods that are known to be
high in purines.

Further Reading
1 Choi HK, Mount DB, Reginato AM. Pathogenesis of gout. Ann Intern Med 2005; 143:
499516.
2 Baker JF, Krishnan E, Chen L, Schumacher HR. Serum uric acid and cardiovascular disease:
recent developments, and where do they leave us? Am J Med 2005; 118: 81626.
3 Becker MA, Jolly M. Hyperuricaemia and associated diseases. Rheum Dis Clin North Am 2006;
32: 27593.
4 Athyros VG, Elisaf M, Papageorgiou AA et al. Effect of statins versus untreated dyslipidaemia
on serum uric acid in patients with coronary heart disease: a subgroup analysis of the GREek
Atorvastatin and Coronary-heart-disease Evaluation (GREACE) study. Am J Kidney Dis 2004;
43: 58999.
5 Alderman M, Aiyer KJV. Uric acid: role in cardiovascular disease and effects of losartan. Curr
Med Res Opin 2004; 20: 36979.
6 Nakagawa T, Hu H, Zharikov S et al. A causal role for uric acid in fructose-induced metabolic
syndrome. Am J Physiol Renal Physiol 2005; 290: F62531.
7 Mandayam S, Mitch WE. Dietary protein restriction benefits patients with chronic kidney
disease. Nephrology 2006; 11: 537.
8 Becker MA, Schumacher HR, Wortmann RL et al. Febuxostat compared with allopurinol in
patients with hyperuricaemia and gout. N Engl J Med 2005; 353: 245061.
06-PS Rheumatology-cpp:06-PS Rheumatology-ppp.QXD 18/3/08 14:31 Page 189

35 Gout acute attack and beyond 189

P R O B L E M

35 Gout Acute Attack and Beyond

Case History
Samuel is a 38-year-old mine worker who had an episode of pain and swelling in his left
great toe six months ago. He now presents with a one-week history of right ankle
swelling. His serum uric acid (UA) is elevated and aspiration of his right ankle joint
confirms that he has acute gout. His general health is good and he takes no medications.
What features of the clinical history are important?
What aspects of examination may help determine if there is an underlying cause?
How should he be treated and followed up?

Background
Gout is the most common inflammatory arthritis and usually presents with single joint
involvement, with the first metatarsophalangeal joint affected in over 70% of cases.1 It
has been recognized for centuries,2 affects about 1% of the population in developed
countries and is becoming more common because of the increasing prevalence of obesity
and metabolic syndrome. The prevalence increases with age and is around 4% in those
aged 65 years and over. Males are two to three times more commonly affected. Gout is
almost unique in being a disease state precipitated by a physicochemical reaction i.e.
crystallization of UA when the plasma and synovial fluid become saturated with UA at a
concentration of around 0.42 mmol/l. Dietary factors are extremely important and fre-
quently neglected. High-purine vegetable foods do not appear to affect UA levels so
much as meat-based foodstuffs.
The typical attack begins in the early hours of the morning and affects the great toe
(podagra), ankle, fingers, elbow or, occasionally, knee. The distribution of joints affected
relates to the prevailing temperature in the joints, with UA precipitation more likely to
occur in cool joints. A variety of nucleating agents in the joint space including collagen,
chondroitin sulphate and proteoglycans may initiate crystallization of UA. Gout is
therefore more likely to occur in joints that are already damaged, for example by
osteoarthritis. The pain is typically very severe and the affected joint is swollen, red and
tender. Differential diagnosis includes septic arthritis, pseudogout and reactive arthritis.
An acute attack usually resolves within 710 days, even without specific treatment.
However, there will be a recurrence within three years in at least 80% of cases. Tophi are
crystal deposits of UA in soft tissue and do not elicit an inflammatory response compar-
able to that found in joints. They typically occur on the ears, hands, feet or elbow.

Atlas Medical Publishing Ltd


06-PS Rheumatology-cpp:06-PS Rheumatology-ppp.QXD 18/3/08 14:31 Page 190

190 06 Back and Specific Joint Problems

The gold-standard investigation is the finding of UA crystals in a joint aspirate in asso-


ciation with elevated plasma UA. In practice, joint aspiration is frequently not under-
taken. Plasma UA may increase during an acute attack of gout, but one-third of patients
with acute gout have normal plasma UA levels. White cell count may be modestly ele-
vated, while a very high count might suggest septic arthritis. If the full blood count is
otherwise normal, a myeloproliferative disorder is unlikely (Box 35.1). Renal function
may be impaired in patients with chronic hyperuricaemia, and should be checked in all
cases. It may be useful to measure urinary urate concentration, since high levels would
preclude the use of uricosuric agents. Hyperglycaemia and dyslipidaemia are frequent
amongst patients with increased UA. Hypothyroidism is quite common, and may precip-
itate gout. X-ray of the affected joint usually only demonstrates the associated soft tissue
swelling, but may show erosions around the joint in patients with chronic gout.

Box 35.1 Common risk factors for gout


Age Male gender
Race (black > white) Acute illness with dehydration
Increased uric acid Overweight or obese
Diet high calorie and high purine Alcohol
Proliferative lesions psoriasis, haemolysis, trauma/burns, myeloproliferative disease
Drugs most commonly diuretics
Initiation of UA-lowering therapy

Management of the acute attack is usually straightforward.35 Conservative measures


include rest, elevation of the affected joint and cooling of the joint. It is important to
ensure that the patient has adequate analgesia. Non-steroidal anti-inflammatory drugs
(NSAIDs) are widely used and may be the only treatment required. Colchicine has been
used for many years, but should be regarded as a second-line drug. It acts by decreasing
phagocytosis of UA crystals by neutrophils, and thus limits the inflammatory response.
The drug has a narrow therapeutic window and toxic effects include diarrhoea, nausea
and vomiting. A dose of 0.5 mg tds is often sufficient. Some would precede this with a
loading dose of 1 mg. Up to 6 mg/day can be given, although side effects are almost uni-
versal at this dose. It can be given intravenously, but there is a high risk of toxicity. The
third line of therapy is corticosteroids, given either systemically or intra-articularly. For
the latter, methylprednisolone or triamcinolone are most commonly used. Systemic
steroids may be particularly useful where there is multiple joint involvement. A single
dose of 80120 mg methylprednisolone may be used, or a course of prednisolone starting
with 3060 mg/day. UA-lowering therapy should not be used during the acute attack.
Planning of chronic treatment for a patient with high UA is summarized in
Figure 35.1. The patient should be given appropriate dietary advice (chapter 34) and, if
overweight, advised to lose weight. The two major approaches to pharmacological ther-
apy are decreasing UA production by inhibiting xanthine oxidase (XO), or increasing UA
excretion using uricosuric agents. A third approach, not currently widely available, is to
06-PS Rheumatology-cpp:06-PS Rheumatology-ppp.QXD 18/3/08 14:31 Page 191

35 Gout acute attack and beyond 191

UA .0.42 mmol/l
Confirmed gout (clinically or by joint aspiration)

Treat acute attack


Wait for 4 weeks

? Overweight decrease weight


Dietary advice (low purine, low fat)
Limit alcohol intake
Water intake 23 litres/day

Renal function Plasma lipids


Thyroid function Blood glucose

First attack or .2 attacks per year


#2 attacks per year UA .0.6 mmol/l

Lifestyle management Failed


Allopurinol
Monitor UA at 3 months

Successful

Continue treatment UA ,0.36 mmol/l Check urinary urate


Monitor UA 1 renal function Decreased attacks
If not increased

Consider uricosuric agent (e.g. sulphinpyrazone)


Losartan if hypertensive
Fenofibrate if dyslipidaemic

Figure 35.1 Prophylaxis of gout.


06-PS Rheumatology-cpp:06-PS Rheumatology-ppp.QXD 18/3/08 14:31 Page 192

192 06 Back and Specific Joint Problems

use uricase preparations (bacterial or fungal) to increase UA breakdown in vivo. Humans


lack the enzyme uricase and cannot degrade UA.

Allopurinol
The mainstay of drug treatment for the past 50 years has been the XO inhibitor allopuri-
nol, itself a purine derivative. Generally, it should be reserved for patients who have two
or more attacks per year. Unless the UA level is very high, there is no justification for
using drug therapy in patients with asymptomatic hyperuricaemia. The drug is well toler-
ated but reactions may occur in up to 2% of patients. These range from mild urticaria
and pruritus to severe hypersensitivity. The latter causes severe (often ulcerating) skin
lesions, fever, neutrophilia and often renal impairment. Other rare side effects include
leukopenia, thrombocytopenia, peripheral neuropathy and GuillainBarr syndrome. In
trials, only as few as 20% of patients treated with allopurinol reach the target serum UA
value of 0.36 mmol/l. There are a number of potential drug interactions. Allopurinol
inhibits the breakdown of the purine drugs azathioprine and mercaptopurine, and will
increase risk of bone marrow toxicity from these drugs. Allopurinol may increase circu-
lating levels of theophylline and increase International Normalized Ratio (INR) in
patients taking warfarin. The dose of allopurinol should generally be kept to 300 mg/day
or less, and should be decreased in patients with renal impairment (Box 35.2).
Allopurinol is the pro-drug for the active metabolite oxypurinol, and many laboratories
are able to provide oxypurinol levels as a guide to compliance or for titration of a lower
dose in those with renal impairment.

Box 35.2 Allopurinol dose vs glomerular filtration rate


Estimated creatinine clearance (ml/min) Dose
>100 300 mg/day
60 200 mg/day
40 150 mg/day
20 100 mg/day
10 100 mg every other day
<10 100 mg every third day

Most rheumatologists concur with a costbenefit analysis indicating that allopurinol


only be commenced if >2 attacks of gout are occurring per year. After the acute attack, it
may be useful to commence prophylactic treatment with either a low-dose NSAID (e.g.
indomethacin 25 mg bd or diclofenac 25 mg bd) or colchicine 0.5 mg bd. This is contin-
ued throughout the introduction and adjustment of allopurinol and for three months
afterwards. After one month of prophylaxis and quiet joints, allopurinol 100 mg is com-
menced in addition to the colchicine/NSAID. At one month the serum UA is measured,
and if greater than 0.36 mmol/l, the dose of allopurinol is increased to 200 mg/day whilst
continuing on the colchicine/NSAID. This continues monthly until the serum UA is
below 0.36 mmol/l or 400 mg/day of allopurinol is reached. Patients are strongly advised
that in the event of an acute attack of gout, they must not discontinue or alter their allop-
urinol dose but rather reinstitute their acute management plan with either an NSAID or
06-PS Rheumatology-cpp:06-PS Rheumatology-ppp.QXD 18/3/08 14:31 Page 193

35 Gout acute attack and beyond 193

colchicine and seek medical review. If the aim is tophi dissolution, then the plasma UA
level needs to be lowered towards the lower limit of normal. The current recommenda-
tion is for lifelong therapy.

Uricosurics
For patients who either do not respond to allopurinol or are intolerant of the drug,
options are fairly limited.6 Currently, the second-line approach is to use uricosuric
agents. In order to work, uricosurics require near-normal renal function, a sufficient
urine volume and a suitable pH to prevent UA stone formation, and are contraindicated
in anyone with renal stones or in those with a high urinary excretion of UA. Uricosurics
will not work with concomitant salicylates. Probenecid and sulphinpyrazone are old-
fashioned drugs that are still available. The more potent uricosuric benzbromarone is not
licensed in most countries because of hepatic toxicity, but can be used on a named
patient basis in some countries. Losartan should be considered if the patient is hyperten-
sive, and fenofibrate if there is dyslipidaemia. Both of these drugs have uricosuric action,
as does the NSAID azapropazone. For patients with frequent attacks, or rebound attacks,
low-dose colchicine can be used to help prevent recurrences.

Recent Developments
1 Becker et al.7 studied 762 patients with gout and hyperuricaemia randomly
assigned to receive 300 mg/day of allopurinol or febuxostat at a dose of 80 mg or
120 mg. Of the latter two groups, 53% and 62%, respectively, reached the
primary endpoint a serum UA concentration of 0.36 mmol/l. No difference was
noted in the rate of flare-up of gout during the 52 weeks of the study. Decrease
in the size of gouty tophi was also demonstrated.
2 Martinon et al.8 have provided insight into the molecular basis for the intense
inflammation that arises as a consequence of UA crystal deposition in joints.
Activation of inflammatory cascades in phagocytic cells involves the formation of
a unit called an inflammasome. This is formed on activation of the cells by the
complexing of a member of the NALP family of proteins (pyrin domain-
containing proteins sharing structural homology with NODs [nucleotide-binding
and oligomerization domain proteins]) with the linking protein ASC (apoptosis-
associated speck-like protein) and the enzyme caspase-1. In this study, UA
crystals activated NALP3-containing inflammasomes. Colchicine, acting upstream
from inflammasome formation, decreased activation of the pro-inflammatory
cytokine interleukin-1b.

Conclusion
In the vast majority of patients with gout, there is no single underlying cause. A positive
family history could favour an underlying metabolic disturbance. Splenomegaly might
suggest an underlying myeloproliferative disorder. Associated conditions are much more
frequent than underlying causes. These include renal impairment, obesity, hypertension
06-PS Rheumatology-cpp:06-PS Rheumatology-ppp.QXD 18/3/08 14:31 Page 194

194 06 Back and Specific Joint Problems

and the metabolic syndrome. Management of the acute attack is with hydration and anal-
gesia, often with the addition of colchicine. In planning ongoing management, careful
attention should be paid to lifestyle factors. Allopurinol is the first-line agent to lower
UA, but does not need to be prescribed to all patients who either have high UA or have
had an attack of gout.

Further Reading
1 Choi H. Epidemiology of crystal arthropathy. Rheum Dis Clin North Am 2006; 32: 25573.
2 Nuki G. Treatment of crystal arthropathy history and advances. Rheum Dis Clin North Am
2006; 32: 33357.
3 Suresh E. Diagnosis and management of gout: a rational approach. Postgrad Med J 2005; 81:
5729.
4 Underwood M. Diagnosis and management of gout. BMJ 2006; 332: 131519.
5 Zhang W, Doherty M, Bardin T et al. EULAR evidence based recommendations for gout. Part
II: Management. Report of a task force of the EULAR Standing Committee for International
Clinical Studies Including Therapeutics (ESCISIT). Ann Rheum Dis 2006; 65: 131224.
6 Bardin T. Current management of gout in patients unresponsive or allergic to allopurinol.
Joint Bone Spine 2004; 71: 4815.
7 Becker MA, Schumacher HR, Wortmann RL et al. Febuxostat compared with allopurinol in
patients with hyperuricaemia and gout. N Engl J Med 2005; 353: 245061.
8 Martinon F, Ptrilli V, Mayor A, Tardivel A, Tschopp J. Gout-associated uric acid crystals
activate the NALP3 inflammasome. Nature 2006; 440: 23741.
06-PS Rheumatology-cpp:06-PS Rheumatology-ppp.QXD 18/3/08 14:31 Page 195

36 Pseudogout investigation and management 195

P R O B L E M

36 Pseudogout Investigation and


Management

Case History
Reg is aged 53 years and presented last week with a four-day history of an acutely
swollen and very painful left knee that had occurred without trauma. He was afebrile but
had a large knee effusion and Bakers cyst. Aspiration yielded 45 ml of non-viscous
blood-stained fluid that appeared turbid. The laboratory confirmed sterile synovial fluid
and a moderate presence of red cells, plus elevation of white cells (of which 60% are
neutrophils) and abundant intracellular calcium pyrophosphate crystals.
What are the known factors predisposing to pseudogout?
How should these be investigated?
What management strategies will you employ?

Background
The most frequent manifestation of calcium pyrophosphate dihydrate (CPPD) depos-
ition disease is chondrocalcinosis, the asymptomatic radiographic finding of calcification
of articular cartilage or fibrocartilage. Up to 5% of the population show radiographic evi-
dence of chondrocalcinosis, with the incidence rising with age to 15%40% of those over
60 years of age, and 30%60% of those >85 years old. Because over three-quarters of pre-
senting patients are >60 years old, and most have pre-existing joint damage, it is likely
that biochemical changes in aging cartilage favour crystal nucleation.
The acute symptomatic presentation of chondrocalcinosis is termed pseudogout. While
usually mild, it can lead to quite severe and rapidly destructive arthritis. The presentation is
of an inflammatory arthropathy with loss of function, early morning stiffness and improve-
ment with activity. Other manifestations include atypical forms of osteoarthritis (OA),
severe destruction mimicking neuropathic arthropathy, a symmetrical synovitis similar to
rheumatoid arthritis, and calcification of the intervertebral discs and longitudinal spinal
ligaments leading to restricted spinal mobility and hence resembling ankylosing spondylitis
but without sacroiliitis. Disordered calcification of cartilage and other skeletal tissues
occurs commonly among the elderly, yet the reasons for this are poorly understood.1
CPPD deposition is associated with acute attacks of pseudogout, characterized by
joint effusions with marked neutrophilia. The release of CPPD crystals into a joint space

Atlas Medical Publishing Ltd


06-PS Rheumatology-cpp:06-PS Rheumatology-ppp.QXD 18/3/08 14:31 Page 196

196 06 Back and Specific Joint Problems

is followed by neutrophilic phagocytosis and subsequent release of potent chemoattrac-


tant and inflammatory mediators. Pseudogout most commonly involves the knee fol-
lowed by the wrists, metacarpophalangeal joints, hips, shoulders, elbows and ankles. The
joint distribution may provide a clue to CPPD deposition disease, as primary OA rarely
involves the metacarpophalangeal joint, wrist, elbow, shoulder or ankle. CPPD crystals,
however, may also be found in the synovial fluids of patients with primary OA, either
alone or in association with basic calcium phosphate crystals. Acute attacks of pseudo-
gout may be precipitated by local events that induce crystal shedding from cartilage into
the synovial fluid, such as trauma, arthroscopy or intra-articular injection of high-
molecular-weight hyaluronic acid as a viscosupplement. Systemic changes affecting
calcium concentration such as rapid changes in fluid balance, medical illness, com-
mencement of thyroxine or parathyroid surgery can also induce an acute attack. A sys-
temic response to pseudogout is noted in half of patients, with fever, neutrophil
leukocytosis and raised inflammatory markers.
Definitive diagnosis of CPPD deposition disease would require unequivocal identifi-
cation of weakly positive birefringent rhomboid or rod-shaped CPPD crystals in joint
fluid or articular cartilage. Aspirated fluid is often turbid or blood-stained, with reduced
viscosity and a marked neutrophil leukocytosis. Articular cartilage at any site may
demonstrate chondrocalcinosis; the classical sites are the triangular ligament of the wrist,
the pubic symphysis and the menisci of the knee. It has been proposed that the presence
of CPPD crystals in synovial fluid combined with radiographic evidence of calcification
of the cartilage would make a definitive diagnosis, and either one would make a probable
diagnosis. Because joint aspiration or biopsy is impractical in population studies, pres-
ence of radiographic chondrocalcinosis is often used in epidemiological and clinical
studies.2 There are no population epidemiological studies of pseudogout, although most
case series suggest that the mean age at presentation is between the seventh and eighth
decades, with a female predominance of 23:1. The prevalence of radiographic chondro-
calcinosis increases with age, with the reported prevalence of radiographic chondrocal-
cinosis in the knee joints of elderly males increasing from <4% in those less than 70 years
of age to 27% in those aged over 85 years.3
Only a minority of patients with CPPD arthritis have metabolic or hereditary abnor-
malities. These include hyperparathyroidism, haemochromatosis, hypophosphatasia and

Table 36.1 Conditions associated with CPPD

Secondary to underlying medical condition


Hyperparathyroidism
Hypophosphatasia
Haemochromatosis
Hypomagnesaemia
Familial hypocalciuric hypercalcaemia
Possibly hypothyroidism
Chronic gout

Secondary to underlying cartilage alterations


Aging and OA
Post-meniscectomy
Epiphyseal dysplasia
06-PS Rheumatology-cpp:06-PS Rheumatology-ppp.QXD 18/3/08 14:31 Page 197

36 Pseudogout investigation and management 197

ATP AMP Hypomagnesaemia Iron Hypophosphatasia


Alkaline phosphatase

Pyrophosphatase
activity

Human ANKH Extracellular


mutation pyrophosphate

Iron, Fe31. Fe21

Crystal nucleation Hypercalcaemia

Hypomagnesaemia

CPPD Disease

Figure 36.1 Metabolic factors predisposing to CPPD disease.

hypomagnesaemia (Table 36.1 and Figure 36.1). Hypothyroidism is probably associated


with chondrocalcinosis, but not necessarily pseudogout. The presence of a CPPD
arthropathy in a patient younger than 50 years, or in those with florid polyarticular chon-
drocalcinosis, should lead to investigation for an underlying metabolic disorder.
Hereditary CPPD deposition disease has been reported in several ethnic populations,
usually with autosomal dominant inheritance, and has early onset and varying pene-
trance. Whether genetic risk factors are involved in later-life chondrocalcinosis has not
been established. Small studies suggest an increased recurrence risk in first-degree rela-
tives of 11%28%, and while this is likely to be greater than the prevalence in matched,
unrelated individuals, this has not formally been established. A comparison of the recur-
rence rate of chondrocalcinosis of the knee in siblings of index cases and unrelated com-
munity members found significantly increased familiarity for chondrocalcinosis (sibling
recurrence risk ratio [SRRR] 2.0); for pyrophosphate arthropathy (chondrocalcinosis
and OA combined), SRRR was 2.3.4 This showed that the familiarity of knee chondrocal-
cinosis in elderly patients is low, as would be expected for a common disease, and that
severely affected young cases may provide a more rewarding genetic focus.1

Treatment
The therapeutic options for pseudogout are more limited and less based on an under-
standing of the underlying metabolic derangement than is the treatment for gout.
Symptomatic therapy with non-steroidal anti-inflammatory drugs, colchicine, joint
aspirations, intra-articular steroids and non-pharmacologic support are the main
06-PS Rheumatology-cpp:06-PS Rheumatology-ppp.QXD 18/3/08 14:31 Page 198

198 06 Back and Specific Joint Problems

approaches to acute management of the acutely inflamed joint and are applied to
pseudogout. There are, however, few controlled trials.
There is currently no specific treatment to slow or prevent the gradual joint deterior-
ation due to chondrocalcinosis, or the progression of the crystal deposition, other than
treatment of any underlying biochemical or metabolic disorders.

Recent Developments
1 Toll-like receptors (TLRs) are a family of receptors with roles in host defence and
inflammation. They provide a critical step in the innate immune response and
are particularly adapted to recognize microbial components; for example,
lipopolysaccharide is recognized by TLR4, and peptidoglycan by TLR2. The TLRs
share a cytosolic Toll/interleukin-1 receptor domain that transduces upregulation
of pro-inflammatory genes through activation of nuclear factor-k-beta (NF-kB).
TLR2 is expressed constitutively in chondrocytes and is upregulated in cartilage
as a consequence of OA. Liu-Bryan et al. demonstrated that CPPD crystals could
function as a ligand for TLR2 and mediate signalling to initiate nitric oxide
production in chondrocytes.5
2 Two chromosome regions (on chromosomes 5 and 8) have been linked to
chondrocalcinosis. The chondrocalcinosis gene on chromosome 5p at the CCAL2
locus has been demonstrated to be the ANKH gene. ANKH is a transmembrane
pyrophosphate transporter and dysfunction of the gene causes elevation of
intracellular, and reduction in extracellular, pyrophosphates. Mutations of the ANKH
gene altering the amino terminal of the protein cause familial autosomal dominant
chondrocalcinosis; changes towards the carboxy terminal cause cranial metaphyseal
dysplasia.6,7 Cells transfected with ANKH variants have not shown significant effects
on pyrophosphate levels, leaving open the possibility that ANKH mutations cause
chondrocalcinosis through effects other than on pyrophosphate transport.7

Conclusion
Chondrocalcinosis and pseudogout remain enigmatic diseases. Currently, the vast
majority of cases are termed primary or idiopathic, reflecting our lack of knowledge
rather than providing a useful classification. Human genome mapping and lessons from
animal models have provided genetic clues in our understanding of pyrophosphate
metabolism and transport. It is hoped that further analysis of both populations and fam-
ilies with the various forms of CPPD deposition disease will identify both the basis of the
disease and effective therapies.

Further Reading
1 Zhang Y, Brown MA. Genetic studies of chondrocalcinosis. Curr Opin Rheumatol 2005; 17:
3305.
2 Choi H. Epidemiology of crystal arthropathy. Rheum Dis Clin North Am 2006; 32: 25573.
06-PS Rheumatology-cpp:06-PS Rheumatology-ppp.QXD 18/3/08 14:31 Page 199

37 Joint and bone infections 199

3 Felson DT, Anderson JJ, Naimark A, Kannel W, Meenan RF. The prevalence of
chondrocalcinosis in the elderly and its association with knee osteoarthritis: the Framingham
Study. J Rheumatol 1989; 16: 12415.
4 Zhang W, Neame R, Doherty S, Doherty M. Relative risk of knee chondrocalcinosis in siblings
of index cases with pyrophosphate arthropathy. Ann Rheum Dis 2004; 63: 96973.
5 Liu-Bryan R, Pritzker K, Firestein GS, Terkeltaub R. TLR2 signaling in chondrocytes drives
calcium pyrophosphate dihydrate and monosodium urate crystal-induced nitric oxide
generation. J Immunol 2005; 174: 501623.
6 Williams CJ, Zhang Y, Timms A et al. Autosomal dominant familial calcium pyrophosphate
dehydrate deposition disease is caused by mutation in the transmembrane protein ANKH. Am
J Hum Genet 2002; 71: 98591.
7 Pendleton A, Johnson MD, Hughes A et al. Mutations in ANKH cause chondrocalcinosis. Am
J Hum Genet 2002; 71: 93340.

P R O B L E M

37 Joint and Bone Infections

Case History
Mr DH is 32 years old and was diagnosed with type 1 diabetes at the age of 8. He presents
with a hot, swollen right knee with marked overlying cellulitis. The symptoms developed
rapidly over 24 hours. He had been working on his garden the day before. He is febrile
(38.3C) with tender lymphadenopathy in his right groin. He has maintained satisfactory
glycaemic control by increasing his insulin. His knee is aspirated and the aspirate is
purulent with large numbers of Gram-positive cocci.
What is the likely organism?
Is he likely to have a bone infection (osteomyelitis)?
What investigations and treatment are indicated?

Atlas Medical Publishing Ltd


06-PS Rheumatology-cpp:06-PS Rheumatology-ppp.QXD 18/3/08 14:31 Page 200

200 06 Back and Specific Joint Problems

Background
Septic arthritis
The diagnosis of joint infection is usually easy. The patient typically presents with a single
hot, swollen and tender joint with fever, rigors and systemic upset. The knee is affected in
around 50% of cases, the hip and ankle each in 15%, the elbow in 10% and the wrist and
shoulders each in 5%. Around 10% of cases are polyarticular, particularly where the
underlying infection is gonorrhoea, group B streptococcus, pneumococcus or Gram-
negative organisms. Risk factors are summarized in Box 37.1.
The incidence of septic arthritis is increasing, partly because of the aging population
structure and partly because of antibiotic resistance. Much of the increase is due to methi-
cillin-resistant Staphylococcus aureus (MRSA) and group B streptococci. The average age

Box 37.1 Risk factors for septic arthritis


Prior joint problems (osteoarthritis, rheumatoid arthritis, gout)
Joint surgery, prosthetic joints, injection or arthroscopy
Loss of skin integrity (ulcer, psoriasis, eczema)
Diabetes
Renal failure
Cirrhosis
Immunosuppression (including use of anti-tumour necrosis
factor [TNF]-a therapy)
Intravenous drug abuse
No underlying risk factor in 20% of cases

Box 37.2 Organisms responsible for septic arthritis


Organism % of cases
Staphylococcus aureus 44.3
Streptococcus pyogenes 7.6
Streptococcus pneumoniae 6.5
Haemophilus influenzae 4.3
Mycobacterium tuberculosis 4.2
Escherichia coli 3.8
Coagulase-negative staphylococci 3.5
Neisseria gonorrhoeae 3.2
Streptococcus agalactiae 2.9
Pseudomonas aeruginosa 1.5
Neisseria meningitidis 1.2
Salmonella 1.0
Other Gram-negative organism 4.6
Other b-haemolytic streptococci 4.3
Polymicrobial 1.4
Other (including fungi) 5.9
Adapted from Ross et al. 2003.1
06-PS Rheumatology-cpp:06-PS Rheumatology-ppp.QXD 18/3/08 14:31 Page 201

37 Joint and bone infections 201

at onset is 50 years. The elderly, those with comorbidities (including diabetes) and those
infected with b-haemolytic streptococci are at increased risk.2 S. aureus is by far the most
common organism and joint infection often follows a transient bacteraemia. The range of
organisms implicated is very wide (Box 37.2).
Unusual infections and at-risk groups include:

Brucellosis related to consumption of unpasteurised milk products. Presents with


mono- or oligoarthropathy, often affecting the sacroiliac joint. Diagnosed by blood
culture or serology and treated with doxycycline or streptomycin
Human and animal bites may present with mixed organisms (usually including
streptococci and staphylococci) or unusual organisms. Rat-bite fever is caused by
Streptococcus moniliformis and presents with arthralgia, systemic upset and rash of
the palms and soles
Melioidosis, caused by Burkholderia pseudomallei, is an endemic infection in rural
South-East Asia and tropical Australia. Prolonged treatment with Bactrim, doxy-
cycline or cephalosporin is required to prevent reactivation
Intravenous drug users. The most prominent organism (60% of cases) in this group
is Pseudomonas aeruginosa. The infection may affect joints not usually involved in
septic arthritis including the sternoclavicular joint, sacroiliac joints or the pubic
symphysis

Investigation and management is summarized in Figure 37.1. The patient is usually


febrile but peripheral white blood cell count is not always elevated. Inflammatory mark-
ers (erythrocyte sedimentation rate and, particularly, C-reactive protein [CRP]) are
almost always increased. Where possible, the joint should be aspirated; for hip infections,
this should be done in theatre. It is not clear whether arthroscopy with joint washout is
routinely superior to simple aspiration. Arthroscopy should be carried out if there is a
possibility of a foreign body in the joint and where there has been injury to the joint. The
white cell count on aspiration of a septic joint is usually greater than 50 000 cells/ml. This
level of white cell count can also be seen with crystal arthritis. Blood cultures can also be
taken, and the chances of a positive culture from a joint aspirate may be increased if a
sample of the aspirate is incubated in a blood-culture bottle. The choice of antibiotic
depends on the organism. An initial choice, unless Gram stain of the aspirate suggests
otherwise, should include cover against S. aureus. A combination of flucloxacillin 500 mg
qds and clindamycin 300 mg qds, both intravenously, is suitable. Generally, the patient
should stay on antibiotics for a total of six weeks.

Osteomyelitis
Bone infection is either blood-borne or arises from contiguous spread from a focus of
infection.3 Osteomyelitis may be further classified as acute or chronic. An anatomic clas-
sification is also used: medullary osteomyelitis is infection limited to medullary content
and endosteal surface (equivalent to early haematogenous spread); superficial implies
early change from local spread, and necrosis is limited to the exposed surface of bone;
localized implies full cortical thickness, but which could be removed without comprom-
ising the stability of the bone; diffuse implies a segment of bone is involved and debride-
ment would affect bone stability.
06-PS Rheumatology-cpp:06-PS Rheumatology-ppp.QXD 18/3/08 14:31 Page 202

202 06 Back and Specific Joint Problems

Hot, swollen, tender joint

FBC
Blood cultures
ESR 1 CRP

Arthroscopy 1
Aspirate
irrigation

Aspirate WBC .50 000 per ml


No crystals

Gram stain
Culture aspirate

Gram 1ve cocci e.g. Gram 2ve bacilli

Flucloxacillin 1 Other antibiotics


clindamycin e.g. vancomycin

Rest 1 elevation of joint


Analgesia

IV antibiotics until swelling subsided and CRP near normal

Oral antibiotics (6 weeks total antibiotics treatment)

Figure 37.1 Management of septic arthritis. +ve, positive; ve, negative; CRP, C-reactive protein; ESR,
erythrocyte sedimentation rate; FBC, full blood count; WBC, white blood cell count.
06-PS Rheumatology-cpp:06-PS Rheumatology-ppp.QXD 18/3/08 14:31 Page 203

37 Joint and bone infections 203

Haematogenous spread is the source of infection in about 20% of cases and S. aureus is
by far the most common organism. It usually presents with local pain but with relatively
little constitutional upset. The vertebral column is a common site, and the lumbar spine
is affected in 45% of cases, thoracic spine in 35% and cervical spine in 20%. Patients with
diabetes, renal failure or other chronic disease associated with a compromised immune
system are at particular risk, with men twice as likely to be affected.
Contiguous-focus osteomyelitis without vascular insufficiency may follow trauma or
surgical procedure (typically one month afterwards). Procedures include internal fix-
ation of fracture or insertion of a joint prosthesis. S. aureus is commonly implicated but
infections are often mixed and include Gram-negative bacilli and anaerobes.
Contiguous-focus osteomyelitis with vascular insufficiency is most commonly seen in
diabetic patients with poor vasculature, neuropathy and foot ulcers. It typically affects
the small bones of the foot and generally requires amputation.
The earliest X-ray signs of osteomyelitis in the long bones are periosteal thickening or
elevation. Lytic areas may not be apparent for some weeks, when up to 75% of the bone
matrix has been lost. Radionuclide scanning with technetium 99m is frequently helpful
in localizing osteomyelitis, although the presence of an area of increased uptake does not
necessarily equate with bone infection. A gallium scan may be helpful in difficult cases.
Indium-labelled leukocyte scans are seldom used in clinical practice and have little to
offer over other modalities. Magnetic resonance imaging (MRI) is very helpful, with
areas of infection showing as increased intensity on the T2-weighted image.
Management should include wound toilet and debridement where appropriate.
Antibiotics should be continued for at least six weeks. The antibiotic regimen will depend
on the clinical scenario and whether an organism has been isolated. Ciprofloxacin,
clindamycin, levofloxacin and cephalexin are frequently useful. Two separate anti-
staphylococcal agents are usually prescribed. Outpatient intravenous treatment using a
peripherally inserted central catheter is frequently used to avoid a prolonged hospital
admission. Hyperbaric oxygen (HBO2) therapy, though not universally available, is a
useful adjunct. The oxygen tension in infected bone is very effectively increased with
HBO2. Low oxygen tension decreases migration of cells involved in wound healing,
including fibroblasts, and also impairs the bacterial killing ability of phagocytic cells.4

Recent Developments
1 Culture-negative septic arthritis (5%20% of cases) may occur because of organisms
that are difficult to detect or because the patient has been partially treated with
antibiotics. A variety of markers in serum or synovial fluid have been proposed.
These include TNF-a, lactic acid, lactate dehydrogenase and procalcitonin. Recent
studies, both with children5 and adults,6 confirm that procalcitonin is a highly
specific marker for bone and joint infections. Sensitivity is improved by combining
the results with CRP level. An alternative is polymerase chain reaction (PCR)
screening for bacterial products. Availability of high-throughput techniques should
make this method of diagnosis a viable option in the very near future.
2 Gavet et al.7 have compared joint infections in elderly patients with those that occur
at a younger age. The range of causative organisms, the distribution of joint
involvement and the incidence of polyarticular disease were similar. There was a
06-PS Rheumatology-cpp:06-PS Rheumatology-ppp.QXD 18/3/08 14:31 Page 204

204 06 Back and Specific Joint Problems

marked increase in mortality with age: death occurred in 0.7% of patients aged under
60 years, in 4.8% of those aged between 60 and 79 and in 9.5% of those aged 80 and
older.
3 Spinal infections account for only between 2% and 4% of cases of osteomyelitis.8
Diagnosis is frequently delayed and should be suspected in patients who present with
back pain that is not clearly of mechanical origin. Inflammatory markers are usually
elevated and are extremely useful in following the response to treatment. Imaging is
extremely important in making the diagnosis and a combination of plain X-rays,
radionuclide scanning and MRI is frequently required. Definitive diagnosis
sometimes requires either open or percutaneous biopsy.
4 (18)-F-fluorodeoxyglucose-positron emission tomography (FDG-PET) is emerging
as a potentially useful tool in the diagnosis of musculoskeletal infections.9 It may be
even more accurate than conventional radionuclide scanning for the diagnosis of
osteomyelitis, and may be particularly useful where there are metal implants or
prostheses.
5 The increasing use of invasive radiological techniques for angiography and lesion
biopsy is placing an increasing number of patients at risk of staphylococcal
bacteraemia.10 Risk factors for bacteraemia include general debility, being on a
haemodialysis programme, having an indwelling vascular catheter and acquiring the
bacteraemia outside a hospital setting. The duration of symptoms is also important.
When undergoing a procedure, patients should be made aware of the need to report
symptoms suggestive of infection promptly.

Conclusion
By far the most likely causative organism in the above case is S. aureus. However, a wide
range of other organisms commonly causes septic arthritis. The diagnosis should be con-
firmed by joint aspiration where possible. The patients history of diabetes is highly rele-
vant and represents a major risk factor. It is unlikely that the patient has osteomyelitis.
Treatment should begin with intravenous antibiotics, which should be continued until
the infection is fully subsided and inflammatory markers have returned virtually to nor-
mal. In total, antibiotics should be given for at least four to six weeks. The risk of bone
and joint infections increases with age, and they are still associated with considerable
mortality, some of which is related to delayed diagnosis and inadequate antibiotic
therapy.

Further Reading
1 Ross JJ, Saltzman CL, Carling P, Shapiro DS. Pneumococcal septic arthritis: review of 190
cases. Clin Infect Dis 2003; 36: 31927.
2 Ross JJ. Septic arthritis. Infect Dis Clin North Am 2005;19: 799817.
3 Calhoun JH, Manring MM. Adult osteomyelitis. Infect Dis Clin North Am 2005; 19: 76586.
4 Kawashima M, Tamura H, Nagayoshi I, Takao K, Yoshida K, Yamaguchi T. Hyperbaric
oxygen therapy in orthopaedic conditions. Undersea Hyperb Med 2004; 31: 15562.
06-PS Rheumatology-cpp:06-PS Rheumatology-ppp.QXD 18/3/08 14:31 Page 205

38 Viral arthritis 205

5 Butbul-Aviel Y, Koren A, Halevy R, Sakran W. Procalcitonin as a diagnostic aid in


osteomyelitis and septic arthritis. Paediatr Emerg Care 2005; 21: 82832.
6 Martinot M, Sordet C, Soubrier M et al. Diagnostic value of serum and synovial procalcitonin
in acute arthritis: a prospective study of 42 patients. Clin Exp Rheumatol 2005; 23: 30310.
7 Gavet F, Tournadre A, Soubrier M, Ristori JM, Dubost JJ. Septic arthritis in patients aged 80
and older: a comparison with younger adults. J Am Geriatr Soc 2005; 53: 121013.
8 An HS, Seldomridge JA. Spinal infections: diagnostic tests and imaging studies. Clin Orthop
Relat Res 2006; 444: 2733.
9 Stumpe KDM, Strobel K. 18F FDG-PET imaging in musculoskeletal infection. Q J Nucl Med
Mol Imaging 2006; 50: 13142.
10 Fowler VG, Justice A, Moore C et al. Risk factors for hematogenous complications of intravas-
cular catheter-associated Staphylococcus aureus bacteremia. Clin Infect Dis 2005; 40: 695703.

P R O B L E M

38 Viral Arthritis

Case History
JG is a 45-year-old woman who presents with three days of pain and stiffness in her
knees, ankles and hands. She reports fever, mild upper respiratory tract symptoms and
accompanying myalgia. There is no past medical history of note. Her white blood cell
count is normal but erythrocyte sedimentation rate is elevated at 56 mm/h and
C-reactive protein is elevated at 80 mg/l.
What other clinical features should be sought?
Which viruses commonly give rise to this clinical picture?
How should she be investigated and managed?

Background
Viral arthritis is relatively uncommon, accounting for only around 3% of acute poly-
arthropathies.1,2 It is usually self-limiting. Persistent joint symptoms can occur, particu-
larly in patients who are immunocompromised or have persistent infection. In practice,
it is often difficult to be certain of a diagnosis of viral arthritis initially: evidence of recent
or past viral infection is very common; the range of viruses that can cause joint symptoms
Atlas Medical Publishing Ltd
06-PS Rheumatology-cpp:06-PS Rheumatology-ppp.QXD 18/3/08 14:31 Page 206

206 06 Back and Specific Joint Problems

is broad; joint tissue is very vascular, especially when inflamed, and even the finding of
viral particles or DNA in joint tissue cannot alone be taken as evidence for a viral aeti-
ology for the arthropathy. Joint inflammation following viral infection can occur by
direct damage due to viral replication in the joint (as in rubella) or alteration of the
humoral and cell-mediated immune systems (as with hepatitis B and C), and many
viruses can predispose to low-level autoimmunity. The latter can lead to autoantibodies
including those to double-stranded DNA (dsDNA), rheumatoid factor (RF), SSA (Ro),
SSB (La), neutrophil cytoplasmic antigen and cardiolipin. The appearance of low titres of
autoantibodies does not necessarily signify that an autoimmune disease has developed.
Conversely, many patients with autoimmune diseases have antiviral antibodies, but this
does not necessarily signify previous viral infection. An example is the frequency of anti-
bodies to human immunodeficiency virus (HIV) p24 core in patients with systemic lupus
erythematosus or Sjgrens syndrome (SS). The viruses that most commonly cause viral
arthritis are summarized in Table 38.1.

Parvovirus B19 (B19)


Until recently, B19 was thought to be a single species and the only parvovirus to infect
humans. In fact, three closely related genotypes (differing by less than 10%) have been
identified. Parvovirus is a single-stranded DNA virus encoding two capsid proteins VP1
and VP2 the latter being the major capsid protein. The virus replicates in erythroid pre-
cursors. Epidemics of infection occur in late winter and early spring. The virus is spread
by respiratory secretions. Symptoms develop after 718 days incubation and include
upper respiratory tract symptoms, fever, arthralgia, myalgia and skin lesions. The charac-
teristic skin lesion, erythema infectiosum (slapped cheek), occurs in most children but
is characteristically absent in adults. Joint symptoms develop in only 8% of infected chil-
dren but in 80% of adults. It typically causes symmetrical polyarthopathy, with affected

Table 38.1 Causes of viral arthritis

Parvovirus B19
Rubella
Hepatitis C
Alphaviruses
Chikungunya
Onyong-nyong
Ross River Virus
Barmah Forest Virus
Sindbis Virus, Pogosta Disease

Flaviviruses
Kunjin
Dengue
Kokobera
Retroviruses
Others Hepatitis B, EpsteinBarr, Varicella Zoster,
Coxsackie B4 (Bornholm Disease), Cytomegalovirus
06-PS Rheumatology-cpp:06-PS Rheumatology-ppp.QXD 18/3/08 14:31 Page 207

38 Viral arthritis 207

joints being tender and swollen but usually not erythematous and not deformed. Other
manifestations include a transient aplastic anaemia, which can also occur in developing
foetuses due to transplacental passage of the virus. Infection of a pregnant woman can
lead to spontaneous miscarriage or hydrops fetalis.
Development of immunoglobulin M (IgM) antibodies, followed by IgG antibodies, is
characteristic of acute infection. Isolation of viral DNA may be achieved by hybridization
or with polymerase chain reaction. As the pattern of joint involvement resembles that in
rheumatoid arthritis (RA), and B19 infection is frequently accompanied by low titre RF,
the virus has been considered as a potential aetiological agent for RA. Persistent joint
involvement following B19 infection may occur in patients who fail to develop neutraliz-
ing titres of antibody, and this may be difficult to distinguish from RA. Intravenous
immunoglobulin treatment should be considered for patients with evidence of contin-
ued infection.

Rubella
This is the only member of the Rubivirus genus of the Togaviridae family, and is an
enveloped, single-stranded, positive-sense RNA virus. Sporadic cases are now typical,
with the advent of vaccination programmes, while previously late-winter epidemics were
characteristic. After an incubation of 79 days, the patient develops fever, cervical and
suboccipital lymphadenopathy and a characteristic maculopapular rash that lasts up to
five days. Petechiae on the soft palate (Forchheimers spots) are characteristic. The vac-
cines used are live attenuated vaccines and may also cause joint symptoms. The arthralgia
of rubella has a typical rheumatoid distribution. Low-level titres of RF are quite com-
mon, and RF positivity may conversely give a false positive for rubella IgM. Joint mani-
festations are much more common in adults, and more likely in females.

Hepatitis C
This is an enveloped, single-stranded RNA flavivirus. More than 170 million people
worldwide have now been exposed to hepatitis C. The prevalence is increasing, largely
amongst intravenous drug users and prison populations. The infection now affects up to
3% of the United States population. Joint symptoms occur in up to 40% of hepatitis C-
infected patients, but arthritis occurs in only 2%. Again the distribution is usually like
that found in RA, but erosive changes are absent. Around 20% of patients have mixed
cryoglobulinaemia, and this is associated with an asymmetrical, pauciarticular, medium-
large joint arthritis. The virus has been associated with a range of autoimmune diseases
including RA, autoimmune hepatitis, glomerulonephritis and SS. Between 15% and 20%
of SS patients show evidence of hepatitis C, often with absent SSA or SSB antibodies. The
prevalence of RF positivity is greatly increased in hepatitis C-infected patients, but anti-
bodies to cyclic citrullinated peptides (anti-CCP; a more specific marker for RA) are usu-
ally not detected. The presence of severe joint involvement should lead to antiviral
therapy being considered. Up to 75% of patients with hepatitis C infection have extra-
hepatic manifestations, which usually improve with antiviral therapy. The usual treat-
ment is with weekly subcutaneous peginterferon-a for 48 months and twice-daily
ribavarin for 24 weeks. Corticosteroids may also be used for arthropathy, as may hydroxy-
chloroquine, and there is some evidence of benefit with anti-tumour necrosis factor
06-PS Rheumatology-cpp:06-PS Rheumatology-ppp.QXD 18/3/08 14:31 Page 208

208 06 Back and Specific Joint Problems

therapy. Hepatitis B infection is not commonly complicated by arthritis unless poly-


arteritis nodosa develops.

Alphaviruses
There are around 26 members of this genus of the Togaviridae family. The central RNA-
containing nucleoplasmid is surrounded by a lipid bilayer, into which is embedded
multiple copies of the two encoded glycoproteins. The viruses are transmitted by mos-
quitoes, and reservoirs include birds, mammals and marsupials. A careful travel history is
essential. Some of the common viruses of this group are listed in Table 38.1.
Chikungunya occurs in central Africa, South and East Asia and the West Pacific;
Onyong-nyong occurs in central East Africa; Ross River Virus is found in Australia and
the West Pacific; Barmah Forest Virus occurs only in Australia; Sindbis Virus occurs in
Scandinavia, Northern Russia, Africa and Australia; Pogosta Disease is largely confined
to Finland.
Symptoms are variable. After 712 days incubation, the patient may develop anorexia,
nausea and vomiting, abdominal pain, pharyngitis, headache and photophobia. Facial
flushing may give rise to a more generalized maculopapular rash. Joint symptoms a
symmetrical, non-deforming polyarthropathy may precede the rash and other symp-
toms. Diagnosis is made on paired sera 1014 days apart, with increased IgM antibody
indicating recent infection. Viral DNA can be persistent in synovial tissue, while
detection of viral DNA in blood indicates ongoing or chronic infection. Treatment is
symptomatic. Salicylates should be avoided if there is a possibility of dengue infection
because of the increased risk of haemorrhagic complications.

Retroviruses
Articular manifestations are common and include arthralgia, spondyloarthropathy, SS
(with CD8+ lymphocytic infiltration of salivary and lachrymal glands), myopathy, sys-
temic vasculitis and increased risk of septic arthritis. The latter may be caused by a variety
of organisms including Staphylococcus aureus, streptococcus, salmonella and atypical
mycobacteria. The presentation is often like reactive arthritis with uveitis and skin
lesions. Psoriasis is also more common, frequently with joint manifestations. With the
advent of highly active antiretroviral therapy in the 1990s, the risk of death from HIV
infection decreased, while improved prognosis has been associated with a higher inci-
dence of chronic complications, including those affecting the joints.

Recent Developments
1 A recent serological survey3 in the United Kingdom showed that 25% of young
children had been exposed to parvovirus B19, compared with up to 75% of the adult
population. Maternal infection could affect up to one in 500 pregnancies, and thus
be a significant contributor to fetal loss. Recent studies46 confirm increased B19
seropositivity in patients with RA. While this does not prove a causative relationship,
the finding that viral DNA is present in synovial tissue from a proportion of patients,
and that the presence of B19 DNA may precede the onset of clinical RA, adds weight
to the hypothesis that B19 infection may contribute to the risk of RA. Furthermore,
06-PS Rheumatology-cpp:06-PS Rheumatology-ppp.QXD 18/3/08 14:31 Page 209

38 Viral arthritis 209

cross-reactivity to the VP1 unique region has also been implicated in the
development of antiphospholipid antibodies.7
2 There has been recent interest in the RNA-induced silencing complex (RISC).8 It is
striking that the viruses that cause arthritis are RNA viruses, and that many of the
autoantibodies involved in connective tissue diseases are directed at proteins
involved in regulation of RNA replication and expression. RNA interference is an
important post-transcriptional regulatory mechanism whereby small RNA species of
up to 25 nucleotides direct the interaction between proteins and RNA. Nucleolytic
enzymes (Dicer and the Argonaute proteins) are responsible for the generation of the
small RNA species, which clearly could be of viral as well as of host origin.
3 Arthritis related to hepatitis C infection could become an increasing problem.9
Recent data confirm that the joint manifestations are not directly due to viral
replication, and that less-direct, immune-activating mechanisms must be
responsible.10 Understanding of the mechanism of hepatitis C-induced arthritis is
important since it may not be influenced by antiviral treatment but may be amenable
to other forms of immune modulation.

Travel history
Sexual history
Drug usage

NSAIDs
Other symptoms and signs
Range of motion
CRP, ESR
exercises

1st serum sample


IgM positive
4-fold increase in Ab titre
2nd serum sample (1014 days)

Viral arthritis

Serology negative
Persistent or worsening symptoms

Consider steroid treatment Reconsider diagnosis


Rheumatoid etc.
Lyme disease
Reactive arthritis

Figure 38.1 Diagnosis and treatment of viral arthritis. CRP, C-reactive protein; ESR, erythrocyte
sedimentation rate; NSAID, non-steroidal anti-inflammatory drug.
06-PS Rheumatology-cpp:06-PS Rheumatology-ppp.QXD 18/3/08 14:31 Page 210

210 06 Back and Specific Joint Problems

Conclusion
Given the range of viruses that cause clinically significant human infections, it is surpris-
ing how few have been associated with viral arthritis. Diagnosis and treatment are sum-
marized in Figure 38.1. The mainstay of diagnosis is paired serological tests with an
increase in antibody titre of four-fold above baseline or development of IgM antibodies
indicating recent infection. Treatment is symptomatic, and usually confined to non-
steroidal anti-inflammatory drugs and a range of motion exercises. Steroids may be use-
ful for patients who have a confirmed diagnosis and who have mounted a serological
response to the infecting agent.11 Prolonged arthritis may occur with parvovirus B19 and
with the alphaviruses. For patients with severe, atypical or persistent symptoms, other
diagnoses should be considered.

Further Reading
1 Calabrese LH, Naides SJ. Viral arthritis. Infect Dis Clin North Am 2005; 19: 96380.
2 Franssila R, Hedman K. Viral causes of arthritis. Best PractRes Clin Rheumatol 2006; 20:
113957.
3 Vyse AJ, Andrews NJ, Hesketh LM, Pebody R. The burden of parvovirus B19 infection in
women of childbearing age in England and Wales. Epidemiol Infect 2007; 135: 135462.
4 Caliskan R, Masatlioglu S, Aslan M et al. The relationship between arthritis and human par-
vovirus B19 infection. Rheumatol Int 2005; 26: 711.
5 Chen YS, Chou PH, Li SN et al. Parvovirus B19 infection in patients with rheumatoid
arthritis in Taiwan. J Rheumatol 2006; 33: 88791.
6 Baskan EB, Yilmaz E, Saricaoglu H et al. Detection of parvovirus B19 DNA in the lesional
skin of patients with Behets disease. Clin Exp Dermatol 2007; 32: 18690.
7 Tzang BS, Tsay GJ, Lee YJ, Li C, Sun YS, Hsu TC. The association of VP1 unique region
protein in acute parvovirus B19 infection and anti-phospholipid antibody production. Clin
Chim Acta 2007; 378: 5965.
8 Jakymiw A, Ikeda K, Fritzler MJ, Reeves WH, Satoh M, Chan EKL. Autoimmune targeting of
key components of RNA interference. Arthritis Res Ther 2006; 8: R8795.
9 Sanzone AM, Bgu RE. Hepatitis C and arthritis: an update. Infect Dis Clin North Am 2006;
20: 87789.
10 Tarantino G, Riccio A, Span A et al. HCV infection and chronic arthritis: Does viral replica-
tion matter? Hepatol Res 2006; 35: 23841.
11 Mylonas AD, Harley D, Purdie DM et al. Corticosteroid therapy in an alphaviral aarthritis.
J Clin Rheumatol 2004; 10: 32630.
06-PS Rheumatology-cpp:06-PS Rheumatology-ppp.QXD 18/3/08 14:31 Page 211

39 Rheumatological complications of diabetes 211

P R O B L E M

39 Rheumatological Complications of
Diabetes

Case History
Mr JT is a 62-year-old man who has had type 2 diabetes for ten years. Glycaemic control is
not perfect (glycosylated haemoglobin [HBA1C] 8.2%) on a combination of sulphonylurea
and metformin. He presents with stiffness and some pain around his left shoulder.
Shoulder movement is limited and he cannot place his hand behind his head.
Is his problem likely to be related to diabetes?
What might be the underlying mechanism?
Are musculoskeletal symptoms increased in patients with diabetes?

Background
The prevalence of diabetes is increasing globally. Currently, around 7% of the adult pop-
ulation is affected, and by 2025 the number of patients in the world with diabetes will
have increased to over 300 million. This increase is largely attributed to the aging popula-
tion structure and the rising prevalence of obesity. It is well established that the presence
of diabetes relates to impaired quality of life and functional ability. Much of this is due to
the direct metabolic consequences of diabetes, and also to its associated vascular compli-
cations. The development of diabetes also predisposes to a variety of musculoskeletal
complications. These are often under-recognized in clinical practice.
Rheumatological complications of diabetes have been classified according to whether
they are due to the metabolic consequences of the condition, to microvascular compli-
cations or to underlying aetiological mechanisms.1,2 The problems encountered will be
considered according to the affected tissue. Increased uric acid is frequent as part of the
metabolic syndrome, possibly making gout more common in patients with diabetes.
Osteoarthritis is much more common in obese individuals, who are also more likely to
develop diabetes. Nerve entrapment syndromes may also be more common amongst
patients with diabetes.

Muscle complications
Statin drugs are increasingly prescribed for the dyslipidaemia of diabetes. Muscle symp-
toms are relatively common with statin treatment, and severe muscle complications

Atlas Medical Publishing Ltd


06-PS Rheumatology-cpp:06-PS Rheumatology-ppp.QXD 18/3/08 14:31 Page 212

212 06 Back and Specific Joint Problems

including myalgia, myositis and rhabdomyolysis occur in around 1% of treated patients.


Diabetic muscular infarction is a rare disorder that usually occurs in patients with type 1
or type 2 diabetes of long standing and is associated with the presence of vascular compli-
cations. It presents as pain and swelling, most commonly in the thigh, but can occur in
the calf or other muscle groups and may cause symptoms in multiple muscle groups.
Conditions that should be excluded are venous thrombosis, abscess, haematoma, trau-
matic muscle tear and inflammatory myositis. The aetiology is not clear but may include
poor muscle perfusion from vascular disease, increased thrombotic tendency, metabolic
alterations in skeletal muscle and the mechanical demands made on large muscle groups
such as those in the thigh.

Ligament, tendon and capsular problems


Lower limb complications of diabetes osteomyelitis, foot ulceration and Charcots
arthropathy are more likely to be serious and life-threatening. However, upper limb
complications are more common and contribute considerably to the functional disability
associated with diabetes. The most common of these are adhesive capsulitis of the shoul-
der, Dupuytrens disease, limited joint mobility syndrome (cheiroarthropathy) and pal-
mar flexor tenosynovitis (trigger finger). These may occur in isolation or associated with
each other. They are more common in patients with long-standing diabetes, with poor
glycaemic control and with microvascular complications. Increased collagen glycosyla-
tion and cross-linking has been implicated in the pathogenesis of these complications.
Adhesive capsulitis causes pain and stiffness and limits the range of movement. It
occurs in 2%3% of the non-diabetic population but is up to five times more common in
patients with diabetes, who are also more likely to have bilateral involvement. Treatment
is with exercises, non-steroidal anti-inflammatory drugs (NSAIDs), injected steroids or
surgery. In a Finnish study,3 the prevalence of rotator cuff tendonitis and non-specific
shoulder pain was 2% and 12%, respectively, in the general population. The risk of symp-
toms was much higher amongst patients with insulin-requiring diabetes (odds ratio =
8.8). There was a strong association between shoulder symptoms and psychological dis-
turbances.
Dupuytrens disease (DD), due to fibrosis and nodules in the palmar fascia, leads to
flexion contractures of the digits and occurs particularly in patients with long-standing
and poorly controlled diabetes. DD primarily affects the middle and ring fingers. The
reported prevalence amongst patients with diabetes varies between 3% and 30%. It is
equally common in type 1 and type 2 diabetes. There is no strong relationship between
risk of DD and the level of glycaemic control. Generally DD is more common in men but
in diabetes the two genders are almost equally affected. Patients with diabetes are less
likely to require, or have, surgical treatment.
Cheiroarthropathy is usually manifest as an inability to extend the metacarpopha-
langeal joints fully. The prayer sign is usually positive the patient is unable to flatten the
hands together. The relationship between limited joint mobility (LJM) and poor diabetes
control is not established. LJM may contribute to the development of the diabetic foot,
including altering plantar pressures and predisposing to ulceration. The prevalence of
LJM may have decreased in recent years, perhaps because of improvements in manage-
ment.4 Nonetheless it still may occur in up to one in five patients, and is associated with
longer duration of diabetes.
06-PS Rheumatology-cpp:06-PS Rheumatology-ppp.QXD 18/3/08 14:31 Page 213

39 Rheumatological complications of diabetes 213

Bone complications
Hyperostosis may present as spondylosis, hyperostosis frontalis interna or calcification of
the joints and ligaments. All are much more common in patients with diabetes. Diffuse
idiopathic skeletal hyperostosis (DISH) is a condition with excessive new bone forma-
tion, particularly in the enthesal region. There is considerable interest in this condition
currently as it is associated not only with diabetes but also with other components of the
metabolic syndrome including hyperinsulinaemia and increased growth hormone.
The relationship between diabetes and low bone mineral density (BMD) has been
uncertain. Patients with type 1 diabetes may have decreased BMD from an early stage.
Low levels of vitamin D have been documented as a risk factor of diabetes, and this may
partly explain the association. Insulin is a growth factor for bone. Insulin deficiency in
type 1 diabetes has been suggested as a contributor, although patients with type 1 dia-
betes are generally not insulin deficient for long. Patients with type 2 diabetes are gener-
ally somewhat protected because of the associated obesity.

Joint disorders
Neuropathic arthropathy (Charcot joints) occurs in 0.1%2.0% of patients with dia-
betes. It most commonly affects the metatarsophalangeal, tarsometatarsal, ankle and
interphalangeal joints, and occurs when sensation is lost but motor function is preserved.
Initially affected joints are often swollen, red and tender. Charcot arthropathy is
described in a wide range of neurological conditions but diabetes is by far the most com-
mon disorder in which it occurs. It frequently occurs alongside diabetic foot ulceration
because of the altered foot pressure distribution, which occurs when the joints of the foot
are misaligned. The affected bones are usually osteoporotic. The major differential diag-
nosis is with osteomyelitis. Inflammatory markers (higher in bone infection) and mag-
netic resonance imaging help to distinguish the diagnoses. Treatment is with rest and
pressure relief. Bisphosphonate drugs are widely used most commonly a course of
intravenous pamidronate. Intranasal calcitonin is also used to decrease bone turnover.
Preliminary evidence supports the use of this treatment in patients with neuropathic
arthropathy.5

Recent Developments
1 Expression profiling of genes potentially involved in the pathogenesis of DD reveals
significant changes in the matrix-degrading matrix metalloproteinases (MMPs).6
These enzymes are also involved in vascular remodelling and in the pathogenesis of
the vascular complications. Expression of MMP1, MMP13 and MMP14 were all
increased in DD tissue. Another recent study has reported increased activity of
MMP2 in DD tissue.7 DD is due to increased fibroblast activity in affected tissue. As a
consequence, there is increased production of matrix and remodelling of
extracellular tissues.
2 It now seems certain that risk of osteoporotic fracture is increased amongst patients
with insulin-requiring diabetes. In the Troms study,8 over 27 000 patients were
followed up for six years. The relative risk (RR) of all non-vertebral fractures was 3.1
for men with type 1 diabetes and the RR for hip fracture was 17.8. For women, the
06-PS Rheumatology-cpp:06-PS Rheumatology-ppp.QXD 18/3/08 14:31 Page 214

214 06 Back and Specific Joint Problems

RR of hip fracture was 8.9 and 2.0 for type 1 and type 2 diabetes, respectively. The
Womens Health Initiative study9 followed more than 93 000 women for seven years.
Women with type 2 diabetes were at increased risk of fracture in spite of the fact that
BMD was at least as high in diabetic patients as in those without diabetes. There is
concern that thiazolidinedione drugs (rosiglitazone and pioglitazone), used in
treatment of type 2 diabetes, accelerate bone loss and thus predispose to fracture.10

MUSCLE

Statin-induced myopathy Diabetic muscle infarction

LIGAMENT
TENDON
CAPSULE

Shoulder Cheiroarthropathy Dupuytrens disease Trigger finger


Capsulitis (Limited Joint Mobility)
Tendonitis

BONE

Osteoporosis Hyperostosis
Local
JOINT Diffuse

Neuropathic arthropathy Association*


(Charcots) Rheumatoid
Psoriatic

Figure 39.1 Musculoskeletal complications of diabetes. * Rheumatoid disease and type 1 diabetes share
some genetic predisposition but it is uncertain whether the two are associated. There is emerging evidence
that psoriasis and its arthropathy are associated with diabetes, but the mechanism for this is not known.
06-PS Rheumatology-cpp:06-PS Rheumatology-ppp.QXD 18/3/08 14:31 Page 215

39 Rheumatological complications of diabetes 215

Conclusion
Musculoskeletal complications are common in patients with diabetes mellitus (Figure
39.1). Attention has focused on foot complications, including neuropathic arthropathy.
This is not unreasonable since these cause considerable disability and contribute greatly
to the cost of managing diabetes. However, diabetes is associated with a wide range of
musculoskeletal problems, the commonest of which involve the hand.11 Musculoskeletal
complications are not strongly related to the level of glycaemic control, or even to the
duration of diabetes. They do correlate with the presence of microvascular complications
(retinopathy and neuropathy) and it may well be that microvascular changes in connect-
ive tissue are important in their pathogenesis.

Further Reading
1 Crispin JC, Alcocer-Varela J. Rheumatologic manifestations of diabetes mellitus. Am J Med
2003; 114: 7537.
2 Arkkila PE, Gautier JF. Musculoskeletal disorders in diabetes mellitus: an update. Best Pract
Res Clin Rheumatol 2003; 17: 94570.
3 Miranda H, Viikari-Juntura E, Heistaro S, Helivaara M, Riihmki H. A population study on
differences in the determinants of a specific shoulder disorder versus nonspecific shoulder
pain without clinical findings. Am J Epidemiol 2005; 161: 84755.
4 Lindsay JR, Kennedy L, Atkinson AB et al. Reduced prevalence of limited joint mobility in
type 1 diabetes in a UK clinic population over a 20-year period. Diabetes Care 2005; 28:
65861.
5 Bem R, Jirkovsk A, Fejfarov V, Skibov J, Jude EB. Intranasal calcitonin in the treatment of
acute Charcot neuroosteoarthropathy: a randomized controlled trial. Diabetes Care 2006; 29:
13924.
6 Johnston P, Chojnowski AJ, Davidson RK, Riley GP, Donell ST, Clark IM. A complete
expression profile of matrix-degrading metalloproteinases in Dupuytrens disease. J Hand
Surg 2007; 32: 34351.
7 Augoff K, Ratajczak K, Gosk J, Tabola R, Rutowski R. Gelatinase A activity in Dupuytrens
disease. J Hand Surg 2006; 31: 16359.
8 Ahmed LA, Joakimsen RM, Berntsen GK, Fnneb V, Schirmer H. Diabetes mellitus and the
risk of non-vertebral fractures: the Troms study. Osteoporos Int 2006; 17: 495500.
9 Bonds DE, Larson JC, Schwartz AV et al. Risk of fracture in women with type 2 diabetes: the
Womens Health Initiative observational study. J Clin Endocrinol Metab 2006; 91: 340410.
10 Schwartz AV, Sellmeyer DE, Vittinghoff E et al. Thiazolidinedione use and bone loss in older
diabetic adults. J Clin Endocrinol Metab 2006; 91: 334954.
11 Ardic F, Soyupek F, Kahraman Y, Yorgancioglu R. The musculoskeletal complications seen in
type II diabetics: predominance of hand involvement. Clin Rheumatol 2003; 22: 22933.
07-PS Rheumatology-cpp :07-PS Rheumatology-ppp.QXD 18/3/08 14:32 Page 217

S E C T I O N S E V E N 07
Bone Diseases
40 Osteoporosis prevention and lifestyle management
41 Bisphosphonates for osteoporosis which agent and when?
42 Osteoporosis drugs other than bisphosphonates
43 Male osteoporosis
44 Glucocorticoid-induced osteoporosis
45 Pagets disease of bone
46 Bone complications of renal disease

P R O B L E M

40 Osteoporosis Prevention and


Lifestyle Management

Case History
Jane is a fit 51-year-old woman whose periods are becoming infrequent. She is concerned
about developing osteoporosis as she approaches the menopause. Her mother has
recently fractured her hip. Jane has recently had her bone mineral density (BMD)
measured, and was told that she has osteopenia.
What are the risk factors for osteoporosis?
What advice would you give her on preventing osteoporosis?
What is the role for calcium and vitamin D supplementation?

Background
Osteoporosis arises from loss of BMD with consequent disruption of bony microarchi-
tecture and increased fracture risk. Osteoporosis is, by definition, present when the
T score is 2.5 or less i.e. bone density is 2.5 standard deviations below the estimated

Atlas Medical Publishing Ltd


07-PS Rheumatology-cpp :07-PS Rheumatology-ppp.QXD 18/3/08 14:32 Page 218

218 07 Bone Diseases

peak BMD for the population. Osteopenia is defined as a T score between 1 and 2.5.
While both males and females are at risk of fracture in later life, the dramatic decrease in
oestrogen at menopause in women means that they are generally at greater risk from an
earlier age. BMD is most conveniently measured by dual-energy X-ray absorptiometry
(DEXA). Screening of the population with DEXA is not generally recommended but may
be justified in women aged over 65 years. Low BMD should always be interpreted in the
light of the overall clinical picture and estimated fracture risk. All patients with fragility
fractures should be screened for osteoporosis, and treatment should be considered where
indicated.
Fifty per cent of women and 20% of men will suffer a fragility fracture during their
lifetime. Osteoporotic fracture is uncommon below the age of 60 years, and 85% of frac-
tures occur in subjects over the age of 65. Peak bone density is attained in early adult life
(around age 30 years); there is a steady decline in BMD thereafter, and this accelerates
markedly after the menopause. Individuals with higher peak BMD are better able to
withstand the later decline in BMD. At least 50% of variance in peak BMD is genetically
determined. Polymorphisms in genes for the vitamin D receptor, collagen 1A1, low-
density lipoprotein (LDL) receptor-related protein-5 (LRP-5) and the oestrogen receptor
may all be determinants of peak BMD. The remainder of the variance in peak BMD is due
to environmental factors including nutrition in early life, calcium and vitamin D status
and exercise habits. These factors also determine the maintenance of BMD during mid-
dle life. At menopause, loss of oestrogen leads to activation of bone-resorbing cytokines
including interleukin-1 (IL-1) and tumour necrosis factor (TNF)-a. Osteoclasts are acti-
vated through the receptor activator of nuclear factor-k-beta (RANK). The ligand for
RANK (RANKL) is expressed on osteoblasts. Osteoprotegerin (OPG) a matrix protein
produced by osteoblasts and stromal cells functions as an orphan receptor for RANKL,
decreasing its ability to activate RANK on osteoclasts. Declining with age, OPG expres-
sion may contribute to development of osteoporosis.
Increasing availability of drug treatments over the past 20 years has revolutionized
management of patients with osteoporosis. Vitamin D and its analogues, oestrogen,
selective oestrogen receptor modulators (SERMs, e.g. raloxifene), bisphosphonates, teri-
paratide and strontium all increase both trabecular and cortical bone. Data with vitamin
D treatment suggest that it may reduce risk of fracture by up to 25%.1 Subclinical vitamin
D deficiency is common and the major impact of treatment is in patients with subopti-
mal vitamin D status. The place of oestrogen therapy has also altered, mainly as a result of
the Nurses Health Initiative.2 In this large group of healthy post-menopausal women,
oestrogen had the predicted benefits on bone health but risk of cardiovascular events in
women taking combined hormone replacement therapy (HRT) actually increased.
Stroke risk increased by 8 per 100 000 person-years; risk of breast cancer increased by a
similar amount. HRT is now only recommended for relatively short-term use in women
with vasomotor and other menopausal symptoms.3 Bisphosphonates are the first line of
treatment for patients with established osteoporosis. There is some doubt, however,
about how long they should be used for, and the typical three to five years of treatment
represents only a fraction of the time that many patients are exposed to risk of osteo-
porotic fracture.
Lifestyle factors and secondary causes of osteoporosis are summarized in Box 40.1.
Higher levels of activity, particularly weight-bearing exercise, are well documented to pro-
07-PS Rheumatology-cpp :07-PS Rheumatology-ppp.QXD 18/3/08 14:32 Page 219

40 Osteoporosis prevention and lifestyle management 219

Box 40.1 Risk factors for osteoporosis


Age Number of years since menopause
Family history Low body weight
Smoking Low calcium intake
Excessive alcohol intake Inadequate vitamin D
Prolonged immobility Prolonged amenorrhoea
Lack of exercise
Glucocorticoid exposure Hyperthyroidism
Hyperparathyroidism
Previous fragility fracture

tect against loss of BMD. Furthermore, exercise leads to improved muscle tone and func-
tion, and thus to lower risk of fall. Lock et al.4 have reviewed the literature on short- to
medium-term exercise interventions for patients at high risk. Studies were difficult to com-
pare because of differences in patient cohort, type of intervention and study design. At pres-
ent, the evidence that short-term exercise interventions protect against fracture is limited.
Up to 15% of the adult population, and up to 90% of very elderly subjects, have sub-
optimal vitamin D status.5,6 This can be checked by measuring plasma 25-hydroxy-
vitamin D (25[OH]D). A target range of 50100 nmol/l (2040 ng/ml) is generally
agreed, and 75 nmol/l (30 ng/ml) is a reasonable threshold below which supplements
should be considered. Toxicity is unlikely at levels below 250 nmol/l. The recommended
daily allowance of vitamin D is 400 International Units (IU) per day, increasing to 800
IU/day in those at high risk of osteoporosis. Cholecalciferol (vitamin D3) is produced in
the skin by the action of ultraviolet-B light in the wavelength range 290315 nm on the
precursor 7-dehydrocholesterol. Vitamin D3 is transported in the blood mainly bound to
vitamin D-binding protein. Sequential 25- and 1-hydroxylation in the liver and kidney,
respectively, lead to formation of active 1,25-dihydroxy-vitamin D. The actions of this
hormone include increasing intestinal and renal calcium absorption, as well as skeletal
actions. Maintaining plasma calcium in the optimal range suppresses parathyroid hor-
mone (PTH) secretion, therefore decreasing bone turnover. Low levels of vitamin D have
also been implicated in autoimmune diseases such as type 1 diabetes and multiple scler-
osis, and in malignancies including colon and breast cancers. For routine replacement,
cholecalciferol (vitamin D3) should be used. Activated vitamin D analogues (calcitriol
and alfacalcidol) should be reserved for use where there is a 1-hydroxylation defect i.e.
patients with hypoparathyroidism or renal impairment.

Recent Developments
1 Bisphosphonate treatment decreases the risk of fracture by up to 50% in high-risk
groups but only by around 20% in the age group 5059 years, which is at relatively
low risk of fracture. A recent costbenefit analysis7 confirms that it is not cost-
effective to routinely treat the younger age group by pharmacological means.
07-PS Rheumatology-cpp :07-PS Rheumatology-ppp.QXD 18/3/08 14:32 Page 220

220 07 Bone Diseases

2 In trials of bisphosphonates, patients are routinely supplemented with calcium and


vitamin D. Unfortunately, less attention is paid to nutritional factors in routine
clinical practice. The recent launch of a combination once-weekly treatment of
70 mg alendronate with 2800 IU (70 mg) cholecalciferol should streamline the
treatment of many patients with established osteoporosis.8

Perimenopausal and patient concerned


Family history of osteoporosis
Other risk factors

Avoid smoking
Limit alcohol intake
Exercise high intensity if possible
Dietary advice calcium and vitamin D intake

Measure BMD

Normal BMD Osteopenia or osteoporosis

Reassure Thyroid function


calcium, phosphate, PTH

Consider repeat DEXA


in 35 years

Consider HRT if post menopause Check 25(OH)D


and has menopausal symptoms

<50 nmol/l 5075 nmmol/l >75 nmol/l

800 IU D3/day Recheck at 6 months No supplement

Repeat DEXA at 1824 months

Further decrease in BMD or fracture

Consider drug treatment

Figure 40.1 Managing osteoporotic risk. 25(OH)D, 25-hydroxy-vitamin D; BMD, bone mineral density;
DEXA, dual-energy X-ray absorptiometry; HRT, hormone replacement therapy; IU, International Units; PTH,
parathyroid hormone.
07-PS Rheumatology-cpp :07-PS Rheumatology-ppp.QXD 18/3/08 14:32 Page 221

40 Osteoporosis prevention and lifestyle management 221

3 In a recent study,9 subclinical vitamin D deficiency was present in 75.4% of a cohort


of elderly women selected from acute hospital admissions. Of those with vitamin D
deficiency, 36.7% had secondary hyperparathyroidism. Even after supplementation,
35.3% of the cohort still had suboptimal vitamin D status, partly because of
imperfect compliance. This argues for more robust means of supplementation i.e.
larger doses given as infrequent boluses.
4 A better understanding of the relationship between physical activity and bone
physiology may lead to more effective exercise interventions. In a recent study of
pre-menopausal women,10 only relatively high-intensity exercise was associated with
favourable changes in BMD. Lower levels of exercise may, of course, be beneficial for
other aspects of health.

Conclusion
Osteoporosis will affect one in three women and one in eight men. Increasing numbers of
elderly people mean that the condition will become more prevalent over the next few
decades. An algorithm for management of osteoporotic risk in perimenopausal women is
shown in Figure 40.1. Adequate calcium and vitamin D status should be ensured. The
patient should receive advice on smoking, alcohol consumption and maintaining body
weight in a desirable range. There is a relatively limited role for drug treatment in the age
group of the above patient. HRT should be reserved for patients with menopausal symp-
toms, and the duration of therapy kept to a minimum.

Further Reading
1 Sambrook P, Cooper C. Osteporosis. Lancet 2006; 367: 201018.
2 Rossouw JE, Anderson GL, Prentice RL et al. Risks and benefits of estrogen plus progestin in
healthy postmenopausal women: principal results from the Womens Health Initiative
randomized controlled trial. JAMA 2002; 288: 32133.
3 Ettinger B, Harris ST, Kendler D, Kessel B, McClung MR. Management of osteoporosis in
postmenopausal women. Menopause 2006; 13: 34067.
4 Lock CA, Lecouturier J, Mason JM, Dickinson HO. Lifestyle interventions to prevent
osteoporotic fractures: a systematic review. Osteoporos Int 2006; 17: 2028.
5 Boonen S, Vanderschueren D, Haentjens P, Lips P. Calcium and vitamin D in the prevention
and treatment of osteoporosis a clinical update. J Intern Med 2006; 259: 53952.
6 Souberbielle JC, Friedlander G, Kahan A, Cormier C. Evaluating vitamin D status.
Implications for preventing and managing osteoporosis and other chronic diseases. Joint Bone
Spine 2006; 73: 24953.
7 Sanders KM, Nicholson GC, Watts JJ et al. Half the burden of fragility fractures in the
community occur in women without osteoporosis. When is fracture prevention cost-effective?
Bone 2006; 38: 694700.
8 Epstein S. The problem of low levels of vitamin D and osteoporosis: use of combination
therapy with alendronic acid and colecalciferol (vitamin D3). Drugs Aging 2006; 23: 61725.
07-PS Rheumatology-cpp :07-PS Rheumatology-ppp.QXD 18/3/08 14:32 Page 222

222 07 Bone Diseases

9 DeLappe E, McGreevy C, ni-Chadhain N, Grimes H, OBrien T, Mulkerrin E. Vitamin D


insufficiency in older female community-dwelling acute hospital admissions and the response
to supplementation. Eur J Clin Nutr 2006; 60: 100915.
10 Vainionp A, Korpelainen R, Vihril E, Rinta-Paavola A, Leppluoto J, Jms T. Intensity of
exercise is associated with bone density change in premenopausal women. Osteopor Int 2006;
17: 45563.

P R O B L E M

41 Bisphosphonates for Osteoporosis


Which Agent and When?

Case History
Mrs RC is a 66-year-old woman who has lost 3 cm in height over the past three years. She
has not had any other fractures. Her only medication is a diuretic that she takes for
hypertension. X-ray reveals a wedge fracture of the T12 vertebra and generalized
osteoporosis. A subsequent dual-energy X-ray absorptiometry scan shows the T score for
her lumbar vertebrae to be 2.8, and that for her hip to be 2.1.
Would you carry out any further investigations?
Should she have treatment with a bisphosphonate?
If so, which one would you choose and for how long should it be used?

Background
Bisphosphonates (BPs) have found wide usage in patients with metastatic bone disease,
myeloma, Pagets disease and osteoporosis. They are stable analogues of inorganic
pyrophosphate and bind to hydroxyapatite bone surfaces with high affinity, decreasing
bone turnover by inhibiting osteoclastic activity.1 The general structure of BPs is shown
in Figure 41.1. The R1 moiety is generally a hydroxyl group (except clodronate, where it
is a chloride), while the structure of the R2 side-chain moiety varies. Two classes of BPs
have been developed. The older group, which includes etidronate, tiludronate and clo-
dronate, has an R2 group that does not contain nitrogen. Within the osteoclast, BPs

Atlas Medical Publishing Ltd


07-PS Rheumatology-cpp :07-PS Rheumatology-ppp.QXD 18/3/08 14:32 Page 223

41 Bisphosphonates for osteoporosis which agent and when? 223

OH R1 OH

O P C P O

OH R2 OH

Figure 41.1 General structure of bisphosphonates.

become incorporated into non-hydrolysable adenosine triphosphate (ATP) analogues


and thus inhibit a number of intracellular processes. The second group of BPs have a
nitrogen-containing R2 group and include the more potent, modern drugs such as alen-
dronate, risedronate, pamidronate, ibandronate and zoledronate. These modify the syn-
thesis of isoprenoid compounds by inhibiting the enzyme farnesyl pyrophosphate
synthase. They thus modify activity of several guanosine triphosphate (GTP)-binding
proteins, inhibiting activity and promoting apoptosis of osteoclasts.
The first clinical use of BPs was in imaging because of the affinity of the drugs for areas of
bone with high turnover. BPs are now the mainstay of treatment for established osteopor-
osis. Over a typical three-year course of treatment, BPs increase spine bone mineral density
(BMD) by 40%50% and BMD at the hip by 20%40%. There is clear evidence from trials
that this increase in BMD translates into decreased fracture risk, both spinal and at other
sites including the hip.2 BPs are indicated for patients in the following categories:
Osteoporosis (T score <2.5) plus fracture(s)
Osteopenia (T score 1 to 2.5) plus fracture(s)
Osteoporosis without fracture but at high risk
At present, they are not indicated for patients with uncomplicated osteopenia. One of
the theoretical risks of bisphosphonate treatment was that, with prolonged use, they
might inhibit bone mineralization. The window between doses that inhibit osteoclast
activity and decrease mineralization is relatively narrow for etidronate but much wider
for more modern agents. It is also possible that the drugs could increase brittleness of
bone by progressively increasing bone mineral content. Patients from the major alen-
dronate trials have now been followed for up to ten years and there is no evidence of
long-term side effects or a rebound increase in fractures after the initial three years of
treatment. The agents currently recommended for treatment of patients with established
osteoporosis are:
Alendronate (Fosamax) can be given in a daily dose of 10 mg or in a weekly dose of
70 mg. The recent availability of a 70 mg/week preparation with 70 mg of cholecalcif-
erol (2800 International Units vitamin D; Fosamax Plus) should prove
advantageous, especially for older patients
Risedronate (Actonel) given in a daily dose of 5 mg or in a weekly dose of 35 mg
Ibandronate (Bonviva) given as a single dose of 150 mg once a month
BPs should all be taken on an empty stomach. The patient should swallow the tablet
with a full glass of water and remain upright for at least 30 minutes. The less frequent
07-PS Rheumatology-cpp :07-PS Rheumatology-ppp.QXD 18/3/08 14:32 Page 224

224 07 Bone Diseases

dosing schedules (weekly or monthly) have improved compliance since many patients
had difficulty adhering to the above routine on a daily basis. Intravenous infusion of
pamidronate or ibandronate can be used in patients who cannot tolerate oral BPs. Potent
drugs such as zoledronate may need to be given as infrequently as once a year.
The major uncertainty with bisphosphonate treatment is the optimal duration.
Following a three-year course of treatment, markers of bone turnover may be suppressed
for up to five years. Available evidence3 suggests that the drugs continue to be safe and
beneficial beyond three years. Long-term follow-up data with ibandronate are not avail-
able. Two non-placebo-controlled studies with alendronate following patients for up to
ten years, and two similar studies with risedronate, have been published. These are continu-
ation studies from the original three-year placebo-controlled trials. In these studies, con-
tinued fracture-prevention benefit was apparent. There was no evidence of major
gastrointestinal toxicity. Osteonecrosis is a very rare side effect and is often related to other
factors such as poor dentition, high doses of bisphosphonate or concurrent chemotherapy.
Alendronate, risedronate, ibandronate, raloxifine, calcitonin, strontium ranelate and
teriparatide have all been conclusively shown to prevent vertebral fracture. For non-
vertebral fracture, the available evidence shows that alendronate, risedronate, strontium
ranelate and teriparatide are effective. Based on currently available data, alendronate and
risedronate are the drugs of first choice for treatment of established osteoporosis in post-
menopausal women.4 The drugs are probably entirely comparable in terms of their effi-
cacy, and the safety profile of both is excellent.
The efficacy and safety of oral ibandronate given as a single monthly dose has been
established in animal, pre-clinical and clinical studies.5,6 The BONE study7 was a three-
year, randomized, double-blind, placebo-controlled trial involving 2946 post-
menopausal women with osteoporosis and at least one vertebral fracture. Daily (2.5 mg)
and alternate day (20 mg) doses were compared. The rate of new vertebral fractures in
the placebo group was 9.6% compared with 4.7% in the daily ibandronate group and
4.9% in the alternate day group. In a post hoc analysis, there was also a reduction in risk of
non-vertebral fracture. The MOBILE study8 was a two-year, randomized, parallel group
study, and was the first to examine the efficacy of a monthly dosing regimen. A total of
1609 women with post-menopausal osteoporosis were enrolled. The study confirmed
that monthly dosing regimens were superior to daily dosing schedules. The 150 mg
single-dose regimen gave the best results in terms of increased BMD.

Recent Developments
1 Osteonecrosis affecting either the mandible or maxilla is a recently described
complication of bisphosphonate therapy.9 It occurs particularly in patients who have
been exposed to high doses or multiple agents as used in metastatic disease. It
presents with swelling, tenderness and pain. Treatment is with analgesia,
withdrawing the bisphosphonate and using hyperbaric oxygen. It is thought that
exposure of bone in the periodontal space and rapid turnover of bone in patients
with dental problems or periodontal disease may predispose to osteonecrosis.
Patients starting BPs should have any anticipated dental work carried out before
starting bisphosphonate treatment, and be careful about dental hygiene while on
treatment.
07-PS Rheumatology-cpp :07-PS Rheumatology-ppp.QXD 18/3/08 14:32 Page 225

41 Bisphosphonates for osteoporosis which agent and when? 225

2 Siris et al.10 have examined compliance issues in a database of 35 537 women in the
United States who were prescribed BPs. Only 43% of patients were completely refill-
compliant and only 20% persisted with bisphosphonate therapy for the entire
24 months of the study. There was a decreased fracture rate in women who complied
with, and persisted with, bisphosphonate treatment.

BMD measured by DEXA


X-ray spine and other affected areas

Thyroid function, 25(OH)D


Deoxypyridinoline

Osteoporosis Osteopenia 1 Normal or


fracture osteopenia

Lifestyle
treatment
Raloxifene if osteoporosis predominantly Vitamin D
spinal; HRT if menopausal symptoms

Alendronate or risedronate

Deoxypyridinoline 3 months No response Oral ibandronate or


Repeat DEXA 1218 months IV bisphosphonate or
teriparatide
Response

Continue for 35 years


If osteoporosis
progressing

Stop if BMD restored and no further fracture


Monitor deoxypyridinoline 6 monthly
Check PTH*
DEXA every 1824 months

Figure 41.2 Bisphosphonate treatment of post-menopausal osteoporosis. * Increased parathyroid


hormone (PTH) may be a sign that bone turnover is increased and that further treatment is warranted.
25(OH)D, 25-hydroxy-vitamin D; DEXA, dual-energy X-ray absorptiometry; HRT, hormone replacement
therapy.
07-PS Rheumatology-cpp :07-PS Rheumatology-ppp.QXD 18/3/08 14:32 Page 226

226 07 Bone Diseases

Conclusion
The above patient has osteoporosis by definition. Baseline investigations might include
thyroid function, renal function, erythrocyte sedimentation rate and protein electro-
phoresis to exclude myeloma, and measurement of plasma 25-hydroxy-vitamin D, cal-
cium and parathyroid hormone. Calcium and vitamin D supplementation should be
considered if there are grounds for suspecting that the patient may be deficient. The first
line of treatment for this woman would be either alendronate or risedronate in a weekly
dosing regimen. Progress can be monitored as shown in Figure 41.2. If there is no
progress (determined by bone markers or BMD), changing to an alternative bisphosphon-
ate is suggested (e.g. monthly oral ibandronate). Teriparatide is useful in refractory cases.
There is considerable uncertainty about the optimal duration of bisphosphonate therapy.
Typically, three to five years is recommended but this may depend on demonstrated ben-
efit and perceived risk of stopping the treatment.

Further Reading
1 Russell RGR. Bisphosphonates: from bench to bedside. Ann N Y Acad Sci 2006; 1068: 367401.
2 Sambrook P, Cooper C. Osteoporosis. Lancet 2006; 367: 201018.
3 Liberman UA. Long-term safety of bisphosphonate therapy for osteoporosis: a review of the
evidence. Drugs Aging 2006; 23: 28998.
4 Iwamoto J, Takeda T, Sato Y. Efficacy and safety of alendronate and risedronate for
postmenopausal osteoporosis. Curr Med Res Opin 2006; 22: 91928.
5 Epstein S. Ibandronate treatment for osteoporosis: rationale, preclinical, and clinical develop-
ment of extended dosing regimens. Curr Osteoporos Rep 2006; 4: 1420.
6 Reginster J-Y, Felsenberg D, Cooper C et al. A new concept for bisphosphonate therapy: a
rationale for the development of monthly oral dosing of ibandronate. Osteoporos Int 2006; 17:
15966.
7 Chesnut CH, Skag A, Christiansen C et al. Effects of oral ibandronate administered daily or
intermittently on fracture risk in postmenopausal osteoporosis. J Bone Miner Res 2004; 19:
12419.
8 Miller PD, McClung MR, Macovei L et al. Monthly oral ibandronate therapy in
postmenopausal osteoporosis: 1-year results from the MOBILE study. J Bone Miner Res 2005;
20: 131522.
9 Farrugia MC, Summerlin DJ, Krowiak E et al. Osteonecrosis of the mandible or maxilla associ-
ated with the use of new generation bisphosphonates. Laryngoscope 2006; 116: 11520.
10 Siris ES, Harris ST, Rosen CJ et al. Adherence to bisphosphonate therapy and fracture rates in
osteoporotic women: relationship to vertebral and nonvertebral fractures from 2 US claims
databases. Mayo Clin Proc 2006; 81: 101322.
07-PS Rheumatology-cpp :07-PS Rheumatology-ppp.QXD 18/3/08 14:32 Page 227

42 Osteoporosis drugs other than bisphosphonates 227

P R O B L E M

42 Osteoporosis Drugs Other Than


Bisphosphonates

Case History
Mrs JM is a 60-year-old woman who developed a wedge fracture of her L1 vertebra two
years ago. She had low bone mineral density (BMD) (T score 2.6 for lumbar spine, 1.6 for
hip). At a recent follow-up dual-energy X-ray absorptiometry (DEXA) scan, her BMD had
deteriorated further (lumbar spine T score 2.9, hip 2.1). She insists that she has been
taking her bisphosphonate, and continues to suffer back pain.
Should she persist with bisphosphonate treatment?
What other treatments should be considered?
Is combination therapy an option?

Background
Bisphosphonate drugs have been the cornerstone of osteoporosis treatment in recent
years. However, a small proportion of patients do not appear to respond. While changing
to another, sometimes more potent, bisphosphonate may be the answer for some
patients, other treatment options are now available.

Raloxifene
Raloxifene is a benzothiophene derivative that acts as a selective oestrogen receptor mod-
ulator (SERM). It has protective, oestrogen-like effects on bone and breast. The Multiple
Outcome of Raloxifene Evaluation (MORE) trial, published in 1999, included 7705
women and confirmed that raloxifene increased spine and hip BMD. There was
decreased risk of vertebral fracture but no definite effect on non-vertebral fracture. A
recent meta-analysis1 has analysed seven studies in total, including the MORE trial.
Raloxifene consistently decreased risk of vertebral fracture by 40%49% in those with
previous vertebral fracture, and by 35% in those without. There is a suggestion of
decreased risk of non-vertebral fracture in patients who are at particularly high risk. The
issue of non-vertebral fracture is addressed in the Continuing Outcomes Relevant to
Evista (CORE) study, in which 4001 women from the MORE trial were followed for up to
eight years.2 Those originally taking placebo continued to take placebo. Those originally
randomized to 60 mg or 120 mg raloxifene continued with 60 mg. There was no signifi-
cant benefit of raloxifene in terms of non-vertebral fracture prevention.

Atlas Medical Publishing Ltd


07-PS Rheumatology-cpp :07-PS Rheumatology-ppp.QXD 18/3/08 14:32 Page 228

228 07 Bone Diseases

Tamoxifen is a first-generation SERM and is widely used in the treatment of breast can-
cer. It may also decrease the incidence of oestrogen receptor-positive breast cancer by up
to 48%. The MORE study also demonstrated that raloxifene prevented the development
of aggressive breast cancer. Tamoxifen slightly increases the risk of uterine cancer, while
raloxifene may be neutral in this regard. Oestrogen (in hormone replacement therapy
[HRT]) has beneficial effects on lipid profile. Recent trials, however, have not confirmed
that this translates into a decreased risk of cardiovascular disease. One of the hopes for
raloxifene, a second-generation SERM, has been that improved lipid profile may lead to
better cardiovascular outcomes. The Raloxifene Use for The Heart (RUTH) study
enrolled over 10 000 post-menopausal women who had, or were at high risk of, cardiovas-
cular disease. Prior to the results being published, the manufacturers disclosed an
increased risk of stroke. A more recent analysis3 of the patients enrolled in MORE and
CORE has shown neither benefit nor increased risk for cardiovascular disease. Raloxifene
is not recommended for cardiovascular protection and should not be administered to
women at high risk of stroke. Studies suggest that raloxifene may help protect against cog-
nitive decline with aging, but it remains to be established whether this is clinically relevant.
Raloxifene causes menopausal-type vasomotor symptoms in 10%25% of patients and leg
cramps in 7%, and increases the risk of venous thromboembolic disease by up to two-fold.

Strontium ranelate
Strontium is an alkaline earth metal with atomic number 38. For pharmacological use, it
is administered in a stable complex with an organic moiety (ranelate). Early in vitro stud-
ies confirmed that strontium ranelate had beneficial effects on bone formation (stimulat-
ing production of collagen and non-collagen proteins of the matrix) and on bone
absorption (inhibiting both differentiation and activation of osteoclasts). There is now
adequate clinical evidence to support the use of the drug in post-menopausal osteopor-
osis. It is well tolerated, but causes diarrhoea in up to 6% of patients. This usually disap-
pears within three months. Increases in muscle creatine kinase may also occur, but
seldom necessitate stopping the drug.
The Spinal Osteoporosis Therapeutic Intervention (SOTI) trial4 recruited 1649 post-
menopausal women with osteoporosis. Strontium ranelate (2 g/day) was associated with
a risk reduction of new vertebral fractures of 49% during the first year, and 41% over
three years. Vertebral BMD increased by 14.4% and hip BMD by 8.3%. The Treatment
Of Peripheral Osteoporosis Study (TROPOS)5 recruited 5091 women. Overall, stron-
tium treatment decreased risk of non-vertebral fracture by 16% (relative risk = 0.84),
while risk of hip fracture was decreased by 36% in high-risk individuals. A recent
Cochrane review has examined evidence from four trials6 and confirms that strontium
increases BMD (see Box 42.1) and prevents both vertebral and non-vertebral fracture.

Box 42.1 Strontium ranelate and BMD


Strontium has a higher atomic weight than calcium and thus has a disproportionate
effect on bone mineral content and BMD, which are therefore overestimated. A 1%
increase in strontium can increase BMD by as much as 10%. In patients treated with the
drug there is a relatively weak correlation between increases in BMD and fracture
protection.
07-PS Rheumatology-cpp :07-PS Rheumatology-ppp.QXD 18/3/08 14:32 Page 229

42 Osteoporosis drugs other than bisphosphonates 229

There was a suggestion of increased venous thrombosis and pulmonary embolism, and of
nervous system problems including headache, seizures and memory loss.

Teriparatide
Teriparatide (recombinant human parathyroid hormone [PTH] 134) differs from other
available agents in that it predominantly stimulates bone formation. Evidence supports
its use in both men and women with osteoporosis, in patients with corticosteroid-
induced osteoporosis and, particularly, in those at very high risk of fracture.7
Hyperparathyroidism classically causes increased bone turnover with hypercalcaemia.
The reasons why intermittent dosing with PTH stimulates net bone formation are
incompletely understood. There is evidence that intermittent PTH dosing stimulates
expression of a range of genes in the osteoblast including growth factors (transforming
growth factor [TGF]-b, epidermal growth factor [EGF], amphiregulin), cell-signalling
molecules and MCP-1 (monocyte chemoattractant protein-1).
Teriparatide increases osteoblast number and activity, increases the rate of bone
remodelling and increases both trabecular thickness and connectivity. The increase in
bone turnover is more marked in the first twelve months of treatment and tails off there-
after. The effect is more marked on trabecular bone than cortical bone. Typically, verte-
bral BMD increases by about 10% over 1218 months, while femoral neck BMD
increases by about 5%. Use of teriparatide decreases vertebral fracture rate by around
65% and non-vertebral fracture rate by 50%. Its effect on fracture reduction is clearly
related to increased BMD, although the two are imperfectly correlated.
The drug is given subcutaneously at a daily dose of 20 mg and is licensed to be given for
up to two years. Antibodies to the molecule develop in about 3% of cases but do not appear
to be clinically significant. Most patients have an increase in plasma calcium, usually within
the normal range, but hypercalcaemia has been noted in up to 3% of patients. Plasma cal-
cium, 25-hydroxy-vitamin D, PTH and renal function should be checked at baseline.
Calcium and renal function should be checked after a month of treatment. Teriparatide
increases uric acid levels and should only be given with caution in patients who are at risk of
gout. The most worrying potential side effect is the development of osteosarcoma. This has
been described with prolonged usage in animals, and occasional human cases have been
recognized. Teriparatide should not, therefore, be given to patients with an unexplained
high level of bone alkaline phosphatase, patients who have had previous bone irradiation or
to patients with Pagets disease. Recommended practice is to warn patients starting the
drug about this potential adverse reaction. There is now reasonable evidence that prior or
concurrent exposure to bisphosphonates blunts the effect of teriparatide. Since teriparatide
is often given to patients whose osteoporosis continues to progress in spite of treatment
with first-line drugs, many patients will have been exposed to bisphosphonates. It is sug-
gested to stop the latter drugs before treatment with teriparatide. It is justified to restart the
bisphosphonate once teriparatide treatment is complete. During treatment, calcium intake
(including supplements) should be kept to 1500 mg or less per day and vitamin D intake
should be no more than 100 International Units per day. Teriparatide is safe and effective in
the elderly and its benefits do not depend on baseline BMD.

Combination treatments
Whether combinations of the currently available treatments are useful or desirable is con-
troversial. There is considerable concern that a very marked decrease in bone turnover
07-PS Rheumatology-cpp :07-PS Rheumatology-ppp.QXD 18/3/08 14:32 Page 230

230 07 Bone Diseases

may lead to increased brittleness of bone. Short-term data (twelve months) show that
raloxifene combined with alendronate increases hip and spine BMD and decreases bone
turnover markers. It is not known whether these changes translate into decreased fracture
risk. It is clear that bisphosphonates should not be combined with teriparatide. Recent
data suggest that combination of raloxifene and teriparatide may increase BMD gain at the
hip,8 while teriparatide with HRT has also been reported to be synergistic.9 In general, data
on combination treatments have proved disappointing, in spite of the fact that combining
an anabolic agent with an inhibitor of bone turnover appears attractive. Sequential treat-
ments appear to be a much more viable option. One approach is to use teriparatide rela-
tively early to increase bone formation. The next phase is to use a bisphosphonate to
decrease bone turnover, following which the patient is left treatment-free for a period, and
then the process is repeated. This approach has been called ADFR (Activate, Decrease
osteoclast activity, Free of treatment and Repeat). Osteoporosis is a lifelong condition and
we need effective strategies for its long-term management.

Recent Developments
1 Third-generation SERMs are being developed. The ideal drug would protect bone
and the cardiovascular system without risk of thromboembolism. Amongst the
drugs being developed are lasofoxifene, arzofoxifene and bazedoxifene. Lasofoxifene
shows promise as a bone-protective agent, increasing bone density and bone
strength in animal studies.10
2 An analysis of patients treated with strontium ranelate in two previous multinational
studies has confirmed that the drug was effective in preventing fractures
independently of the baseline characteristics of the patient.11 Neither the baseline
BMD nor the presence or number of fractures appeared to influence the subsequent
response to strontium.
3 A costbenefit model has been used to assess the benefit of teriparatide.12 The
greatest costbenefit ratio was when the drug was used in more recent-onset disease.
The cost per quality-adjusted life year (QALY) gained was 20 000 Euros in patients
with a recent vertebral fracture, and 64 000 Euros in patients with previous vertebral
fracture. This study emphasizes the fact that many treatments for osteoporosis, and
other chronic diseases, are often used too late in the course of the disease.

Conclusion
The majority of osteoporotic patients respond well to bisphosphonates, but a significant
proportion shows no or little response. Other options for treating osteoporosis include
SERMs, strontium ranelate and teriparatide. These should be considered for patients
who do not tolerate or respond to bisphosphonates, and also for patients already treated
with bisphosphonate but who require ongoing treatment. The available options are sum-
marized in Figure 42.1. In general, combination treatments are often of limited benefit.
Bisphosphonates and teriparatide should not be given together. There is some limited
evidence favouring a combination of bisphosphonates with oestrogen or SERMs.
07-PS Rheumatology-cpp :07-PS Rheumatology-ppp.QXD 18/3/08 14:32 Page 231

42 Osteoporosis drugs other than bisphosphonates 231

Calcium and vitamin D


Perimenopausal to prevent OP
Elderly (many are vitamin D deficient)
No limit to duration
Can be used with other agents

Oestrogen
Only if menopausal symptoms
Early years after menopause
Maximum 5 years

Raloxifene
Generally ,65 years
Especially for vertebral OP

Strontium
Anabolic and antiresorptive
Established general OP (with or without fracture)
Use for 3 years (based on available trials)

Bisphosphonate
Established OP (especially with fracture)
Use up to 5 years
All ages

Teriparatide
Anabolic ( antiresorptive)
Use if high risk of fracture
Limit use to 18 months
Caution with other agents
? Use before bisphosphonate because of anabolic action

Figure 42.1 Drugs for the treatment of post-menopausal osteoporosis (OP).

Further Reading
1 Seeman E, Crans GG, Diez-Perez A, Pinette KV, Delmas PD. Anti-vertebral fracture efficacy of
raloxifene: a meta-analysis. Osteoporos Int 2006; 17: 31316.
2 Siris ES, Harris ST, Eastell R et al. Skeletal effects of raloxifene after 8 years: results from the
continuing outcomes relevant to Evista (CORE) study. J Bone Miner Res 2005; 20: 151424.
07-PS Rheumatology-cpp :07-PS Rheumatology-ppp.QXD 18/3/08 14:32 Page 232

232 07 Bone Diseases

3 Ensrud K, Genazzani AR, Geiger MJ et al. Effect of raloxifene on cardiovascular adverse


events in postmenopausal women with osteoporosis. Am J Cardiol 2006; 97: 5207.
4 Meunier PJ, Roux C, Seeman E et al. The effects of strontium ranelate on the risk of
vertebral fracture in women with postmenopausal osteoporosis. New Engl J Med 2004; 350:
45968.
5 Reginster JY, Seeman E, De Vernejoul MC et al. Strontium ranelate reduces the risk of
nonvertebral fractures in postmenopausal women with osteoporosis: treatment of
peripheral osteoporosis (TROPOS) study. J Clin Endocrinol Metab 2005; 90: 281622.
6 ODonnell S, Cranney A, Wells GA, Adachi JD, Reginster JY. Strontium ranelate for
preventing and treating postmenopausal osteoporosis. Cochrane Database Syst Rev 2006;
CD005326.
7 Hodsman AB, Bauer DC, Dempster DW et al. Parathyroid hormone and teriparatide for
the treatment of osteoporosis: a review of the evidence and suggested clinical guidelines for
its use. Endocr Rev 2005; 26: 688703.
8 Deal C, Omizo M, Schwartz EN et al. Combination teriparatide and raloxifene therapy for
postmenopausal osteoporosis: results from a 6-month double-blind placebo-controlled
trial. J Bone Miner Res 2005; 20: 190511.
9 Ste-Marie LG, Schwartz SL, Hossain A, Desaiah D, Gaich GA. Effect of teriparatide
(rhPTH(1-34)) on BMD when given to postmenopausal women receiving hormone
replacement therapy. J Bone Miner Res 2006; 21: 28391.
10 Ke HZ, Foley GR, Simmons HA, Shen V, Thompson DD. Long-term treatment of
lasofoxifene preserves bone mass and bone strength and does not adversely affect the
uterus in ovariectomized rats. Endocrinology 2004; 145: 19962005.
11 Roux C, Reginster JY, Fechtenbaum J et al. Vertebral fracture risk reduction with
strontium ranelate in women with postmenopausal osteoporosis is independent of baseline
risk factors. J Bone Miner Res 2006; 21: 53642.
12 Lundkvist J, Johnell O, Cooper C, Sykes D. Economic evaluation of parathyroid hormone
(PTH) in the treatment of osteoporosis in postmenopausal women. Osteoporos Int 2006;
17: 20111.
07-PS Rheumatology-cpp :07-PS Rheumatology-ppp.QXD 18/3/08 14:32 Page 233

43 Male osteoporosis 233

P R O B L E M

43 Male Osteoporosis

Case History
A 60-year-old man presents with back pain. X-ray of his spine shows a crush fracture of
L2 and a generalized decrease in bone mineral density (BMD). A subsequent dual-energy
X-ray absorptiometry (DEXA) scan confirms that he has osteoporosis with T scores of 3.2
for the lumbar spine and 2.6 for the hip. His serum testosterone is 7.2 nmol/l (normal
range 930 nmol/l).
What are the causative factors in male osteoporosis?
What investigations should routinely be carried out?
What treatment options are proven to work?

Background
Although osteoporosis is predominantly a disease affecting females, it should be remem-
bered that 20% of fractures, and 30% of hip fractures, occur in males. One in three men
over the age of 60 years will suffer a fracture. Osteoporosis in men is more likely to pre-
sent with a fragility fracture, while many cases in women are diagnosed from screening.
Morbidity and mortality following fracture is poorer in men than in women. For exam-
ple, 80% of men do not return to their pre-fracture physical function after hip fracture
and as many as 50% will not return to an independent existence. Underlying causes are
more commonly discovered in men, with glucocorticoid excess (mostly iatrogenic)
occurring in 20% of cases, heavy alcohol intake in 15%20% and hypogonadism in
15%20%. Other secondary causes such as hyperthyroidism, hyperparathyroidism, mal-
absorption, antiepileptic treatment and multiple myeloma should be considered as for
female osteoporosis (Box 43.1). Biological determinants of bone density in men include
activity of the growth hormone/insulin-like growth factor (IGF)-1 axis, decreasing
testosterone with age and local oestrogen production and action. With regard to the

Box 43.1 Indications for DEXA scanning in men


Low trauma fracture Undernutrition
Hypogonadism Hyperparathyroidism
Glucocorticoid treatment or excess Prolonged immobilization
Osteopenia on X-ray

Atlas Medical Publishing Ltd


07-PS Rheumatology-cpp :07-PS Rheumatology-ppp.QXD 18/3/08 14:32 Page 234

234 07 Bone Diseases

latter, polymorphisms of both the aromatase gene (responsible for peripheral conversion
of androgens to oestrogen) and the oestrogen receptor may be important. Vitamin D sta-
tus and parathyroid hormone (PTH) activity are also important, and change with age.
The data on benefits of calcium and vitamin D supplementation are variable, and may
relate to the background status of the population being studied.1 Patients who are defi-
cient in calcium and vitamin D are more likely to respond to supplementation. The
prevalence of vitamin D deficiency increases with age, partly because of decreased dietary
intake and reduced sunlight exposure. The recommended daily intake of calcium for
men under 65 years is 1000 mg, while for men over 65 years it is 1500 mg. Calorie intake
generally decreases with age, and the average diet contains less than 400 mg calcium per
1000 kcal. The recommended minimum vitamin D intake is 400 International Units (IU)
per day for men under 70 years and 600 IU/day for men over 70. Supplementation is
indicated for individuals whose calcium or vitamin D intakes fall below, or are in danger
of falling below, these thresholds. This will not necessarily recover lost bone density in
patients with established osteoporosis.
The mainstay of treatment for patients with established disease is bisphosphonates
(BPs). BPs should be used with calcium and vitamin D supplementation where neces-
sary. Currently, three oral BPs are used for male osteoporosis. Alendronate was the first
of the newer generation BPs to be studied. For example, in the study by Gonnelli et al.,2
alendronate treatment of osteoporotic men increased spine BMD by 4.2% at one year,
6.3% at two years and 8.8% at three years. Corresponding increases at the hip were 1.6%,
2.9% and 3.9%, respectively. There was a parallel increase in bone quality as assessed by
quantitative ultrasound at the heel. The drug protects against fractures at both vertebral
and non-vertebral sites. There is every reason to believe that two other BPs risedronate
and ibandronate are equally safe and effective. Alendronate (Fosamax) can be given in
daily (10 mg) or weekly (70 mg) dosing schedules and should, like the other bisphosphon-
ate drugs, be taken on an empty stomach with the patient upright, and the tablet should
be swallowed with a full glass of water. Alendronate has not only been shown to be clinic-
ally effective, but the cost-effectiveness of treatment has also been demonstrated.3
Risedronate (Actonel) may also be given daily (5 mg) or weekly (35 mg) and has
effects comparable to alendronate. The newer agent ibandronate (Bonviva) can be given
as a single monthly dose of 150 mg. For patients who have gastrointestinal side effects
from BPs, pamidronate or ibandronate can be administered parenterally. For patients
who either cannot tolerate or do not respond to BPs, treatment with PTH (teriparatide;
Forteo) may be indicated. This is given for up to 18 months as a daily subcutaneous injec-
tion. Patients should be monitored for hypercalcaemia. The long-term benefit and safety
of teriparatide have yet to be established. From animal studies there is a theoretical risk of
osteosarcoma.
Administration of testosterone to hypogonadal men with osteoporosis increases BMD
and protects against fracture. Testosterone can be administered orally, transdermally
(with either gel or patches), by depot intramuscular injection (every 24 weeks) or using
implants that are usually inserted every six months. Testosterone treatment of hypo-
gonadal men also improves muscle bulk and function, improves cardiovascular function
and protects against coronary artery disease, as well as generally improving quality of life.
Sex-steroid levels tend to decrease with age in normal men. This decrease is more marked
in some men than in others. The term partial androgen deficiency in aging men
07-PS Rheumatology-cpp :07-PS Rheumatology-ppp.QXD 18/3/08 14:32 Page 235

43 Male osteoporosis 235

(PADAM) has been proposed, and it is not clear whether this physiological change with
aging merits treatment, particularly in the face of markers of lower androgen status such
as decreasing BMD. There is certainly now some evidence that, in the short to medium
term, androgen replacement can reverse some of the biological changes that accompany
declining androgen status. Risks of androgen therapy include changes in liver function,
polycythaemia and predisposition to prostate and breast cancer. In the study by Gennari
et al.,4 bone density and sex-steroid status were monitored over four years in 200 aging
men. Androgen and oestrogen status both decreased with time, and those with the great-
est decrease also had the greatest loss of BMD. The ratio of testosterone to oestradiol, a
measure of aromatase activity, varied widely and patients with the lowest estimated levels
of aromatase lost bone faster. The role of oestrogen status in regulating bone loss in aging
men is controversial; Lormeau et al.5 found no relationship between oestrogen levels and
bone loss in aging men. They did, however, report that sex hormone binding globulin
(SHBG) levels were related to bone loss, with the highest levels being associated with the
fastest bone loss. SHBG tends to increase with age, and by binding free androgen and
oestrogen may decrease free, bioavailable levels of sex steroids.

Recent Developments
1 Assessment of vitamin D status is usually by measurement of total circulating
25-hydroxy-vitamin D3 (25[OH]D3). Levels of vitamin D-binding protein are vari-
able and may increase with age. This increase may effectively decrease the amount of
free and bioavailable vitamin D. In a recent study,6 25(OH)D3 levels were no differ-
ent in men with osteoporosis compared with controls. However, levels of vitamin
D-binding protein were increased in osteoporotic men and the concentration of
1,25-dihydroxy-vitamin D3 (1,25[OH]D3) was decreased.
2 Androgen deprivation therapy for patients with prostate cancer is now a leading
cause of osteoporosis in men.7,8 In recent years, this therapy has become much more
effective at decreasing prostatic exposure to androgen, but therefore places men at
risk of osteoporosis. BMD loss of up to 8% per year at the spine and 6% per year at
the hip is common, and up to 20% of men who survive five years or more will suffer
a fracture. Bone loss can be prevented with BPs but it is not clear whether this
translates into decreased fracture risk.

Conclusion
Osteoporosis in males is more likely to have an underlying cause. The most common of
these are excessive alcohol intake, use of glucocorticoids and hypogonadism. The diag-
nostic workup (Figure 43.1) should always include measurement of calcium and vitamin
D, serum testosterone, SHBG, luteinizing hormone (LH) and follicle-stimulating hor-
mone (FSH). Even mild calcium and vitamin D deficiency should be corrected. At pres-
ent, testosterone replacement should be reserved for patients with proven hypogonadism
and a precise diagnosis should be established in all cases. The mainstay of treatment is
bisphosphonate therapy. Alendronate, risedronate and ibandronate are widely used and
are probably of comparable efficacy and safety. Teriparatide should be considered for
patients with severe or unresponsive disease.
07-PS Rheumatology-cpp :07-PS Rheumatology-ppp.QXD 18/3/08 14:32 Page 236

236 07 Bone Diseases

Fragility facture
Osteopenia on X-ray
High clinical risk

Measure BMD

Osteoporosis confirmed

Measure calcium, 25(OH)D3, PTH


Urinary deoxypyridinolone*
Testosterone, SHBG, LH and FSH

Exclude secondary causes


Excessive alcohol
Glucocorticoids
Hypogonadism

Assess calcium and vitamin D status


Use supplements if necessary

Bisphosphonate treatment

Good response Poor response

Follow-up including Alternative bisphosphonate


annual BMD or teriparatide

Figure 43.1 Diagnosis and management of male osteoporosis. * Measurement of markers of bone turnover
may help to predict response to treatment, as they respond quicker than BMD. BMD, bone mineral density;
FSH, follicle-stimulating hormone; LH, luteinizing hormone; PTH, parathyroid hormone; SHBG, sex hormone
binding globulin.

Further Reading
1 Kamel HK. Male osteoporosis: new trends in diagnosis and therapy. Drugs Aging 2005; 22:
7418.
2 Gonnelli S, Cepollaro C, Montagnani A et al. Alendronate treatment in men with primary
osteoporosis: a three-year longitudinal study. Calcif Tissue Int 2003; 73: 1339.
3 Borgstrm F, Johnell O, Jnsson B, Zethraeus N, Sen SS. Cost effectiveness of alendronate for
the treatment of male osteoporosis in Sweden. Bone 2004; 34: 106471.
07-PS Rheumatology-cpp :07-PS Rheumatology-ppp.QXD 18/3/08 14:32 Page 237

44 Glucocorticoid-induced osteoporosis 237

4 Gennari L, Merlotti D, Martini G et al. Longitudinal association between sex hormone levels,
bone loss, and bone turnover in elderly men. J Clin Endocrinol Metab 2003; 88: 532733.
5 Lormeau C, Soudan B, dHerbomez M, Pigny P, Duquesnoy B, Cortet B. Sex hormone-
binding globulin, estradiol, and bone turnover markers in male osteoporosis. Bone 2004; 34:
9339.
6 Al-oanzi ZH, Tuck SP, Raj N et al. Assessment of vitamin D status in male osteoporosis. Clin
Chem 2006; 52: 24854.
7 Gilbert SM, McKiernan JM. Epidemiology of male osteoporosis and prostate cancer. Curr
Opin Urol 2005; 15: 237.
8 Holmes-Walker DJ, Woo H, Gurney H, Do VT, Chipps DR. Maintaining bone health in
patients with prostate cancer. Med J Aust 2006; 184: 1769.

P R O B L E M

44 Glucocorticoid-Induced Osteoporosis

Case History
Mrs SP is a 45-year-old woman who has suffered asthma since childhood. The asthma is
now stable, but she is concerned about osteoporosis since she has had an average of three
courses of steroids each year for the past ten years. She has regular periods but she is
approaching the menopause. Her mother developed osteoporosis in later life. A dual-
energy X-ray absorptiometry (DEXA) scan confirms that she has low bone mineral density
(BMD).
What are the causes of steroid-induced osteoporosis?
How might the condition be prevented?
What treatment options are available?

Background
Overall, 0.5%0.9% of the population requires intermittent or continuous treatment
with steroids. Up to 2.5% of the elderly population (age >70 years) take corticosteroids.
In addition to the risks imposed by steroids, some of the underlying conditions requiring
steroids also predispose to osteoporosis (e.g. rheumatoid arthritis and multiple

Atlas Medical Publishing Ltd


07-PS Rheumatology-cpp :07-PS Rheumatology-ppp.QXD 18/3/08 14:32 Page 238

238 07 Bone Diseases

myeloma). The pathogenesis of glucocorticoid-induced osteoporosis is complex,1,2


occurring in two phases with rapid bone loss of up to 12% of total BMD within the first
year followed by a slower rate of bone loss of typically up to 3% per year. Corticosteroids
decrease the formation of osteoblasts and markedly increase (by up to three-fold) apop-
tosis of osteoblasts. The early phase with increased bone loss is also characterized by
increased osteoclastic activity. Decreased osteoprotegerin (OPG) may be partly respon-
sible for this effect. OPG acts as a soluble decoy receptor for the receptor activator of
nuclear factor-k-beta ligand (RANKL), and decreased activity of OPG allows increased
interaction between RANKL and its native receptor (RANK) leading to activation of the
pathway that includes nuclear factor-k-beta (NF-kB). Other mechanisms involved
include decreased expression of bone morphogenetic proteins and a diversion of bone
marrow precursor cells into the adipocyte lineage.
Use of glucocorticoids is the most prevalent cause of secondary osteoporosis. In spite
of the widely known detrimental effect of steroids, perhaps only as few as 15% of patients
commencing steroids have investigations to exclude osteoporosis or treatment to prevent
loss of BMD. Patients with glucocorticoid-induced osteoporosis have a greater risk of
fracture than other subjects with comparable BMD. It has, therefore, been suggested that
the threshold for intervention should be lower and a T score of 1.5 is widely used.
Between 30% and 50% of patients taking long-term steroids will suffer a fragility frac-
ture. The equivalent of prednisolone 10 mg/day for six months or more increases the risk
of hip fracture seven-fold and the risk of vertebral fracture 17-fold. Investigation and
management is summarized in Figure 44.1. As with all osteoporosis patients, attention
should be paid to lifestyle factors, including calcium and vitamin D status, and other
potential secondary causes. The patient should be taking 1500 mg of calcium per day and
800 International Units (IU) per day of vitamin D. If it does not seem that these levels of
intake are being achieved, supplements should be prescribed. At all times efforts should
be made to minimize the exposure to glucocorticoids and to withdraw them completely
where possible. There is evidence for recovery of bone density when steroid treatment is
withdrawn.
Of the available treatments,3,4 the greatest evidence by far is with use of bisphosphon-
ates (BPs). Glucocorticoids impair calcium absorption from the gut and reabsorption in
the kidney. Calcium supplementation is therefore indicated in many cases but will not
prevent the rapid loss of BMD in the months after steroid therapy is initiated. Vitamin D
is more effective and may be particularly indicated in older people. Cholecalciferol at
doses between 300 IU/day and 100 000 IU/week helps to minimize the effect of glucocor-
ticoids; vitamin D analogues such as dihydrotachysterol, calcitriol and alfacalcidol are at
least as effective. However, no vitamin D analogue or dose is as effective as BPs. Sex-
steroid replacement is controversial and relatively short-term use only is recommended
for women with menopausal symptoms. Testosterone replacement should be prescribed
for men with proven hypogonadism. Current evidence does not favour the use of calci-
tonin for treatment of glucocorticoid-induced osteoporosis.
Alendronate and risedronate are now the most commonly used drugs for prevention
and treatment of osteoporosis caused by steroids. Both appear to be effective in at least
preserving hip and spine BMD, and there are reasonable data showing that they protect
against vertebral and non-vertebral fracture. Intravenous pamidronate may be used
where oral drugs are not tolerated or where compliance is poor. Etidronate is now sel-
07-PS Rheumatology-cpp :07-PS Rheumatology-ppp.QXD 18/3/08 14:32 Page 239

44 Glucocorticoid-induced osteoporosis 239

Is dose $5 mg prednisolone or equivalent?


Is treatment of 3 months or more likely?

Patient at risk

Lifestyle measures
Stop smoking
Weight-bearing exercise
Limit alcohol intake

Minimize steroid
Lowest possible dose
Topical steroids
Other immunosuppressives
Alternate day dose regimen

Check calcium and vitamin D status


(supplement if necessary)

DEXA scan

T score ,1.5 T score .1.5

Alendronate or Repeat DEXA in 12 months


risedronate

Repeat DEXA in 12 months

Figure 44.1 Investigation and management of glucocorticoid-induced osteoporosis. DEXA, dual-energy


X-ray absorptiometry.

dom used and is much less potent than second-generation BPs. Teriparatide (para-
thyroid hormone) increases BMD both at vertebral and other sites, but it is not known
whether it is effective in treatment of glucocorticoid-induced osteoporosis.
07-PS Rheumatology-cpp :07-PS Rheumatology-ppp.QXD 18/3/08 14:32 Page 240

240 07 Bone Diseases

Recent Developments
1 A number of new, potential therapeutic targets have been identified for primary and
glucocorticoid-induced osteoporosis.5 These include sclerostin, a bone
morphogenetic protein produced by osteocytes. Deficiency of this protein is
associated with high BMD. The RANKRANKLOPG pathway is central to osteoclast
recruitment. Attempts to manipulate activity of this pathway in animal models
include use of a monoclonal antibody to RANKL. CD40 and its ligand belong to the
tumour necrosis factor (TNF) superfamily and are involved in apoptosis of bone cells.
Inhibition of osteoblast apoptosis may increase bone formation.
2 Agents with improved anti-inflammatory and immunosuppressive activity without
steroid side effects would offer a major advantage.6,7 The anti-inflammatory effects
of steroids are mediated through inhibition of the NF-kB and activator protein-1
(AP-1) pathways, while side effects occur through transcriptional modification of a
variety of genes. The search for new drugs includes agents with selective
glucocorticoid receptor-modulator activity.
3 In a recent study,8 201 patients who were about to start steroids at doses equivalent
to prednisolone 7.5 mg/day or greater were randomly assigned to receive
alendronate 10 mg/day with an alfacalcidol placebo or alfacalcidol at a dose of
1 mg/day. Patients were followed for 18 months. BMD of the lumbar spine increased
by 2.1% in those treated with alendronate, while it decreased by 1.9% in the
alfacalcidol-treated group. There was a similar disparity in bone density at the hip.

Conclusion
Use of glucocorticoids is associated with a phase of rapid bone loss due to excessive bone
absorption over formation. This is followed by a slower phase of bone loss due mainly to
decreased bone formation resulting from decreased recruitment and increased apoptosis
of osteoblasts. Prevention should be by minimizing other risk factors for osteoporosis
and, where possible, eliminating other secondary causes. Exposure to steroids should be
minimized, and combination with other immunosuppressive agents should be consid-
ered where prolonged therapy is needed. Adequate calcium and vitamin D status should
be ensured. The bisphosphonates alendronate and risedronate are currently the mainstay
of treatment but are grossly underused in clinical practice.

Further Reading
1 Weinstein RS. Glucocorticoid-induced osteoporosis. Rev Endocr Metab Disord 2001; 2: 6573.
2 Canalis E, Bilezikian JP, Angeli A, Giustina A. Perspectives on glucocorticoid-induced osteo-
porosis. Bone 2004; 34: 5938.
3 Orcel P. Prevention and treatment of glucocorticoid-induced osteoporosis in 2005. Joint Bone
Spine 2005; 72: 4615.
4 Devogelaer J-P, Goemaere S, Boonen S et al. Evidence-based guidelines for the prevention of
glucocorticoid-induced osteoporosis: a consensus document of the Belgian Bone Club.
Osteoporos Int 2006; 17: 819.
07-PS Rheumatology-cpp :07-PS Rheumatology-ppp.QXD 18/3/08 14:32 Page 241

45 Pagets disease of bone 241

5 Mazziotti G, Angeli A, Bilezikian JP, Canalis E, Giustina A. Glucocorticoid-induced


osteoporosis: an update. Trends Endocrinol Metab 2006; 17: 1449.
6 Rhen T, Cidlowski JA. Antiinflammatory action of glucocorticoids new mechanisms for old
drugs. New Engl J Med 2005; 353: 171123.
7 Rosen J, Miner JN. The search for safer glucocorticoid receptor ligands. Endocr Rev 2005; 26:
45264.
8 de Nijs RNJ, Jacobs JWG, Lems WF et al. Alendronate or alfacalcidol in glucocorticoid-
induced osteporosis. New Engl J Med 2006; 355: 67584.

P R O B L E M

45 Pagets Disease of Bone

Case History
Mr JS is a generally fit 74-year-old man. He complains of increasing pain in his back and
in his left shin. The latter feels warm but is not deformed. A scintigraphic bone scan shows
increased uptake at several areas in the spine and in his left shin. There is no evidence of
malignant disease. His plasma calcium and phosphate levels are normal, but alkaline
phosphatase is increased at 420 units/l (normal = up to 120 units/l).
How should he be investigated and followed up?
What is the current thinking on the aetiology of Pagets disease?
When should treatment be considered, and what is the best treatment?

Background
First described as osteitis deformans by Sir James Paget in 1877, Pagets disease is a
chronic, focal disorder of bone where increased areas of marked bone turnover lead to
pain, local bone expansion, deformity and risk of fracture.1,2 It most commonly affects
the skull, clavicles, vertebrae, pelvis, femur and tibia. Pagets disease is relatively common
and may affect up to 1.5% of the population aged 40 years and over, 5% of those aged 55
years and over and as many as 20% of 85-year-olds. It is particularly common in subjects
of Northern European descent and thus relatively common in the United Kingdom,

Atlas Medical Publishing Ltd


07-PS Rheumatology-cpp :07-PS Rheumatology-ppp.QXD 18/3/08 14:32 Page 242

242 07 Bone Diseases

North America, Western Europe and Australia. The prevalence may have decreased by
up to 50% in the past 20 years. The vast majority of cases are asymptomatic.
The cause of Pagets disease is not known, but it is likely that both genetic and envir-
onmental factors are important. Up to 20% of patients have a history of Pagets disease in
one or more first-degree relatives. A high proportion of families with Pagets disease and
up to 15% of sporadic cases have mutations in the sequestosome 1/p62 gene, which is
involved in the activation of osteoclasts. Infectious agents have been implicated and
could account for some of the geographical variation in prevalence. Amongst candidate
agents are paramyxoviruses, measles, respiratory syncytial virus and canine distemper
virus. The disease typically presents with aching bone pain, which may be worse on
weight bearing. Occasionally it presents with deformity or pathological fracture. Many
cases are discovered incidentally because of radiological features or increased serum alka-
line phosphatase. Complications of Pagets disease are relatively uncommon but impor-
tant to recognize (Box 45.1). The most common complication is deafness due to
involvement of the petrous temporal bone.

Box 45.1 Complications of Pagets disease


Hearing loss involvement of petrous temporal bone
Osteogenic sarcoma <1% of cases
Pathological fracture
Cranial neuropathies
Entrapment neuropathies and radiculopathies
Spinal stenosis
Cardiac failure very rare
Hypercalcaemia with prolonged immobility

X-ray of painful or deformed sites should be undertaken, but systematic skeletal sur-
vey is not usually indicated. The initial changes are of focal lysis leading to areas with
mixed lysis and sclerosis, trabecular expansion, thickening of cortical bone and deform-
ity. Cortical fissure fractures may be identified and may coincide with sites of pain.
Metastatic disease may be considered in the differential diagnosis but it is unusual for
bone biopsy to be required. Skeletal scintigraphy is much more sensitive than plain radi-
ology but it is also less specific. Widespread disease may be apparent, even in patients
who are relatively asymptomatic. The most useful biochemical marker is serum alkaline
phosphatase, although it is only increased in 85% of patients. The level of alkaline phos-
phatase is directly related to activity and to extent of the disease. It is frequently normal in
patients with monostotic disease. Measurement of bone-specific alkaline phosphatase
may be useful where total alkaline phosphatase is normal or in the presence of liver dis-
ease. Other markers of bone turnover may also be useful, particularly in monitoring the
early response to therapy. Measurement of urinary deoxypyridinoline or the cross-linked
N-telopeptide of type 1 collagen is widely used.
Investigation and treatment are summarized in Figure 45.1. Often, supportive mea-
sures such as physiotherapy and use of aids such as a walking stick may suffice. For those
who require specific treatment, bisphosphonates (BPs) are the mainstay. Indications for
treatment include severe pain, extensive disease (particularly if major neurological or
07-PS Rheumatology-cpp :07-PS Rheumatology-ppp.QXD 18/3/08 14:32 Page 243

45 Pagets disease of bone 243

vascular structures are at risk), neurological complications, hypercalcaemia resulting


from prolonged immobility and where the disease is affecting the site of proposed
surgery (such as a hip or knee replacement). Available BPs are summarized in Box 45.2.
Etidronate and tiludronate are now seldom used. Alendronate at doses up to 40 mg/day
for up to six months or risedronate 30 mg/day for two to six months are equally effective.
Intravenous pamidronate can be given for patients who are at risk of gastrointestinal side
effects from BPs. A variety of regimens have been used, ranging from a single dose of
3060 mg, to three doses of 30 mg given on consecutive days, or 3060 mg given once per
week for up to six weeks. Serum alkaline phosphatase decreases with modern bisphos-
phonate treatment in the vast majority of cases. Patients are usually followed up at three-
monthly intervals and treatment is repeated after six months if there is recurrence of
symptoms or if the alkaline phosphatase increases again. BPs are contraindicated when
the glomerular filtration rate is less than 35 ml/min/1.73 m2. The newer BPs zoledronic
acid and ibandronate may be used. Salmon calcitonin is still useful in some cases,
particularly when BPs are either contraindicated or poorly tolerated, or as an add-on
therapy when there is severe pain. Calcitonin can be given subcutaneously at doses of
50100 IU/day or on alternate days for up to 18 months. The chemotherapeutic agent
plicamycin is now seldom used.

Box 45.2 Bisphosphonates for Pagets disease


Etidronate 200400 mg/day 6 months
Tiludronate 400 mg/day 6 months
Alendronate 1040 mg/day 36 months
Risedronate 30 mg/day 26 months
Pamidronate 3060 mg 16 doses

Recent Developments
1 Osteoclast differentiation is known to be regulated by members of the tumour
necrosis factor superfamily, including the receptor activator of nuclear factor-k-beta
ligand (RANKL) and osteoprotegerin (OPG). Deactivating mutations to the latter
gene (TNFRSF11B) are known to be responsible for most cases of juvenile Pagets
disease.3 Mutation of the sequestosome 1 (SQSTM1) gene on the long arm of
chromosome 5 is the most consistently identified genetic abnormality in adult
Pagets disease. An argument for genetic screening of patients who have or are at risk
of Pagets disease has been advanced.4
2 There appears to be very little to choose between the currently available potent BPs
in terms of efficacy.5,6 Alendronate, risedronate and pamidronate have very similar
effects in inducing remission, although use of a different agent may be justified if
there is no response or limited response to the first agent.
3 Zoledronic acid, a third-generation bisphosphonate, is widely used in the treatment
of hypercalcaemia of malignancy. Recent evidence suggests that a single infusion of
this agent is superior to current bisphosphonate regimens.7,8 However, as might be
expected, side effects may also be more common with more potent agents. These side
07-PS Rheumatology-cpp :07-PS Rheumatology-ppp.QXD 18/3/08 14:32 Page 244

244 07 Bone Diseases

Bone pain
Deformity
Increased AP

X-ray site of symptoms

Measure AP

Provisional diagnosis of Pagets


Bone-specific AP
PTH exclude hyperparathyroidism
Urinary deoxypyridinoline
25(OH)D exclude deficiency

Bone scintigraphy

Consider need for treatment

Analgesia (NSAIDs)
Aids (physiotherapy)

Alendronate or risedronate
(IV pamidronate if poorly compliant or GI intolerance)

Review with AP or other bone markers 3 monthly

Repeat treatment if recurrent symptoms


or increasing AP

Calcitonin if no response to or does not tolerate bisphosphonate

Figure 45.1 Investigation and management of Pagets disease. 25(OH)D, 25-hydroxy-vitamin D;


AP, alkaline phosphatase; GI, gastrointestinal; NSAIDs, non-steroidal anti-inflammatory drugs; PTH,
parathyroid hormone.
07-PS Rheumatology-cpp :07-PS Rheumatology-ppp.QXD 18/3/08 14:32 Page 245

45 Pagets disease of bone 245

effects include hypocalcaemia (often transient) and increases in serum creatinine.


Apart from potency in inducing remission, use of potent agents such as zoledronic
acid may increase compliance and decrease the cost of administering and monitoring
therapy.
4 Patients with Pagets disease have a higher rate of comorbidities than matched controls
and the disease is costly, with increased costs particularly relating to physician visits
and diagnostic tests.9 Compared with matched controls, patients with Pagets disease
had increased rates of pathological fracture (4.9% vs 0.4%), heart murmurs (3.3% vs
0.4%), low back pain (19.7% vs 8.6%) and hearing loss (13.5% vs 5.7%).

Conclusion
The diagnosis of Pagets disease generally relies on the finding of increased serum alkaline
phosphatase, along with radiological features and focal areas of increased uptake on iso-
tope bone scan. Other markers of bone turnover can be helpful. Bone biopsy is seldom
required. Environmental factors are poorly characterized at present, while genetic mark-
ers are recognized for both juvenile and adult forms of Pagets disease. Treatment is gen-
erally not required. The most common indication for treatment is bone pain.
Bisphosphonates are the mainstay of treatment and alkaline phosphatase is the most use-
ful marker for success of treatment.

Further Reading
1 Whyte MP. Pagets disease of bone. New Engl J Med 2006; 355: 593600.
2 Selby PL, Davie MWJ, Ralston SH, Stone MD. Guidelines on the management of Pagets
disease of bone. Bone 2002; 31: 36673.
3 Whyte MP. Pagets disease of bone and genetic disorders of RANKL/OPG/RANK/NF-kB
signaling. Ann N Y Acad Sci 2006; 1068: 14364.
4 Michou L, Collet C, Laplanche J-L, Orcel P, Cornlis F. Genetics of Pagets disease of bone.
Joint Bone Spine 2006; 73: 2438.
5 Walsh JP, Ward LC, Stewart GO et al. A randomized clinical trial comparing oral alendronate
and intravenous pamidronate for the treatment of Pagets disease of bone. Bone 2004; 34:
74754.
6 Rendina D, Mossetti G, Viceconti R, Sorrentino M, Nunziata V. Risedronate and pamidronate
in the clinical management of patients with severe Pagets disease of bone and acquired
resistance to bisphosphonates. Calcif Tissue Int 2004; 75: 18996.
7 Hosking D. Pharmacological therapy of Pagets and other metabolic bone diseases. Bone 2006;
38 (2 Suppl 2): S37.
8 Maricic M. The use of zoledronic acid for Pagets disease of bone. Curr Osteoporos Rep 2006; 4:
404.
9 Briesacher BA, Orwig D, Seton M, Omar M, Kahler KH. Medical care costs of Pagets disease
of bone in a privately insured population. Bone 2006; 38: 7317.
07-PS Rheumatology-cpp :07-PS Rheumatology-ppp.QXD 18/3/08 14:32 Page 246

246 07 Bone Diseases

P R O B L E M

46 Bone Complications of Renal Disease

Case History
JR is a 64-year-old man with type 2 diabetes. His glycaemic control is reasonable with
oral hypoglycaemic agents but blood pressure has been difficult to control. He currently
takes an angiotensin-converting enzyme inhibitor, a diuretic and a calcium channel
blocker for blood pressure control. Plasma creatinine is increased at 342 mmol/l. His
parathyroid hormone (PTH) is marginally high and bone mineral density (BMD), measured
using a dual-energy X-ray absorptiometry (DEXA) scan, is low (T score 2.4 for lumbar
spine, 1.8 for hip).
What other investigations would be useful?
Can we be certain that he is developing osteoporosis?
Is he a candidate for bisphosphonate therapy?

Background
Bone complications are common in chronic kidney disease (CKD). The bone complica-
tions of CKD are underdiagnosed and under-recognized. One of the reasons is that pre-
cise diagnosis has been based on bone biopsy findings. Bone biopsy is not carried out
frequently, except in specialist centres. The morphological changes in bone that occur
with renal failure have been known as renal osteodystrophy (ROD). The syndrome is
important because of the increasing prevalence of CKD, increasing recruitment of
patients into dialysis programmes with consequent increase in lifespan for CKD patients,
and also because it is accompanied by increased morbidity and premature mortality. The
latter relates to vascular changes including ectopic calcification. A broader term CKD-
mineral and bone disorder (CKD-MBD) has been proposed and encompasses the
many facets of this complication of CKD (Box 46.1).
ROD can be classified according to whether bone turnover is low, normal or high;
whether mineralization is normal or decreased; and whether bone volume is low, normal

Box 46.1 Definition of CKD-MBD


A constellation of bone disorders present or exacerbated by chronic kidney disease that
lead to bone fragility and fractures, abnormal mineral metabolism and extraskeletal
manifestations.

Atlas Medical Publishing Ltd


07-PS Rheumatology-cpp :07-PS Rheumatology-ppp.QXD 18/3/08 14:32 Page 247

46 Bone complications of renal disease 247

or high. Apart from the cellular component, bone comprises mainly organic matrix
(osteoid) and inorganic matrix (mineral). Around 15% of bone turns over each year.
Osteoid, secreted by the osteoblasts, consists mainly of type 1 collagen with proteogly-
cans and other, quantitatively, relatively minor proteins including fibronectin and
osteonectin. The inorganic component is mainly hydroxyapatite crystals. Apart from the
amount of mineralized bone (which determines BMD) and intact microarchitecture of
the bone, the correct balance of organic and inorganic components is essential for nor-
mal mechanical properties (strength, ability to absorb shock and deformability). PTH
and levels of active vitamin D (1,25-dihydroxy-vitamin D3 [D3]; calcitriol) are the most
important determinants of bone structure and calcium metabolism. PTH maintains
plasma calcium by increasing intestinal absorption, increasing renal reabsorption and
mobilizing calcium from bone. Excess PTH results in increased bone turnover. This
affects both trabecular and cortical bone. More than 98% of active D3 is synthesized in
the cells of the proximal renal tubule, and synthesis is decreased in patients with renal
impairment. Synthesis is normally increased by PTH, and there is a negative feedback
loop whereby D3 decreases PTH secretion. D3 increases calcium absorption from the gut
and calcium mobilization from bone. The key measurements made on bone biopsy are
bone volume, bone turnover (using double tetracycline labelling), osteoid tissue mineral-
ization and metal deposition (particularly aluminium). This allows for diagnosis of the
several forms of renal failure-related bone disease.1,2

1. High turnover bone disease (osteitis fibrosa cystica)


This is characteristic of PTH excess with increased osteoblast and osteoclast
activity. The balance may be tipped towards either excess bone formation or excess
breakdown. The former leads to excess osteoid and peritrabecular fibrosis, which may
impair bone marrow formation. Excess bone breakdown may lead to the characteristic
cystic change. Increased cortical porosity contributes to increased risk of fracture. The
disorder is usually asymptomatic, and routine biochemistry is normal apart from
increased PTH levels. Increased vitamin D (acting through the vitamin D receptor) and
increased calcium (acting through the calcium-sensing receptor) are both powerful
stimuli to decrease PTH secretion. Both responses are downregulated in secondary
hyperparathyroidism and PTH secretion is inappropriately high. The trigger for this
increase in PTH is probably low calcitriol levels. High levels of phosphate lead to cal-
cium binding, with a consequent decrease in ionized calcium and thus increased PTH
secretion. In addition, phosphate may directly increase PTH secretion and inhibits
renal 1a-hydroxylase.

2. Defective mineralization (osteomalacia)


This is a direct consequence of suboptimal vitamin D status, although it is not usually
severe enough to warrant a clinical diagnosis of osteomalacia. In the past, aluminium
exposure was a major risk factor, and because this is less prevalent, pure osteomalacia is
becoming less common in CKD.

3. Mixed ROD
Features of PTH excess are combined with defective mineralization due to the relative
deficiency of vitamin D.
07-PS Rheumatology-cpp :07-PS Rheumatology-ppp.QXD 18/3/08 14:32 Page 248

248 07 Bone Diseases

4. Adynamic bone disease (ABD; low turnover state)


This is the opposite end of the spectrum from the state that exists with PTH excess, and is
particularly common in the elderly and in those with diabetes. It occurs in up to 50% of
biopsy specimens. ABD is most common where calcium and vitamin D therapy is exces-
sive, following parathyroidectomy or with aluminium toxicity. There is decreased
osteoid and very little evidence of either osteoblast or osteoclast activity, and no tetracy-
cline labelling. ABD is associated with increased fracture risk, bone pain and ectopic cal-
cification. Bone mineralization is normal in ABD.
The relative rates of the different forms of bone disease are shown in Table 46.1.

Table 46.1 Distribution of ROD subtypes

Pre-dialysis (%) Dialysis (%)

High turnover 10 0
Mixed 20 65
Low turnover (osteomalacia + ABD) 30 35
Normal histology 40 0
Figures show the relative distribution of different types of ROD in pre-dialysis patients and in those undergoing
dialysis. Normal histology very rarely occurs in the dialysis population. Adapted from Schwarz et al. 2006.1

Evaluation should include measurement of serum PTH, calcium (corrected for albu-
min concentration or ionized), phosphate, alkaline phosphatase and plasma bicarbonate,
as well as imaging for soft tissue calcification. While precise diagnosis of ROD requires
bone biopsy, there is probably limited justification for more widespread use of this test as
it is invasive, ideally requires prolonged preparation of the patient with double tetracy-
cline labelling and can only be interpreted in specialized centres. PTH is the single most
useful biomarker and is increased where there is increased bone turnover. The place of
markers of bone turnover in diagnosis is not established, although they may be useful to
help follow the effects of treatment. Markers of bone formation include osteoprotegerin,
total and bone-specific alkaline phosphatase, osteocalcin and procollagen type 1 carboxy-
terminal extension peptide. Markers of bone resorption include pyridinoline and
deoxypyridinoline, tartrate-resistant acid phosphatase (TRAP) and procollagen type 1
cross-linked carboxy-terminal peptide.
BMD measurements do not correlate perfectly with fracture risk. Distal radius is the
preferred site for BMD measurement in CKD. The major concern is that low BMD does
not necessarily indicate osteoporosis and therefore the need for antiresorptive therapy.
Bone biopsy is indicated when biochemical features are difficult to interpret, when frac-
ture or bone pain are prominent and cannot be explained by non-invasive investigations,
where aluminium toxicity is suspected and where there is severe and progressive vascular
calcification. Bone histomorphometry uses measures of turnover, mineralization and
volume to classify ROD the TMV classification. Biopsy is usually undertaken at the iliac
crest. Use of two doses of tetracycline given four weeks apart prior to biopsy allows for
assessment of bone formation and mineralization rate.
07-PS Rheumatology-cpp :07-PS Rheumatology-ppp.QXD 18/3/08 14:32 Page 249

46 Bone complications of renal disease 249

In general, the metabolic abnormalities that lead to altered bone morphology and vas-
cular calcification do not occur until glomerular filtration rate (GFR) has decreased
below 60 ml/min/1.73 m2. However, these bone and vascular changes occur in children
and young adults with more modest decreases in GFR (to <90 ml/min/1.73 m2). Caution
should be exercised in diagnosing pure osteoporosis in a patient who has renal impair-
ment. For routine screening, vascular calcification is best detected on lateral abdominal
plain X-rays looking for aortic calcification. Computed tomography (CT) scans give

Low GFR

Measure:
Calcium (corrected total and ionized)
Phosphate
Vitamin D (25[OH]D and 1,25[OH]2D)
PTH

Assess vascular risk (BP, lipids)


Screen for glucose intolerance/diabetes

Lateral abdominal X-ray (vascular calcification)

Bone biopsy if:


Biochemistry does not explain clinical findings
Bone pain or fracture
Risk of aluminium toxicity

Decrease phosphate:
Vitamin D replacement Diet
Phosphate binders

Persistent high PTH:


Calcimimetic
Parathyroidectomy

Vigorous CV risk reduction

Figure 46.1 Diagnosis and management of bone complications of renal disease. 25(OH)D, 25-hydroxy-
vitamin D; 1,25(OH)2D, 1,25-dihydroxy-vitamin D; BP, blood pressure; CV, cardiovascular.
07-PS Rheumatology-cpp :07-PS Rheumatology-ppp.QXD 18/3/08 14:32 Page 250

250 07 Bone Diseases

more precise information and quantitative methods are described for research purposes,
but routine CT scanning is not currently justified.
High phosphate levels can be improved by effective renal replacement therapy (dialy-
sis or transplant), although response to dialysis is often incomplete. Dietary phosphate
should be restricted. For many patients, treatment with phosphate binders is required.
Alu-Caps swallowed whole before meals are very effective but, in the long term, there is a
risk of bone and brain toxicity. Calcichew, Titralac and Phos-Ex (calcium acetate) are
calcium-containing phosphate binders which, again, are taken 15 minutes before meals.
Sevelamer has been introduced recently and is a non-aluminium and non-calcium-
containing phosphate binder. Careful replacement of vitamin D should be undertaken.
As the problem is with renal generation of active 1,25-dihydroxy-vitamin D3, an active
analogue of vitamin D such as alfacalcidol is usually required. Cinacalcet is a cal-
cimimetic drug that acts at the calcium-sensing receptor on parathyroid gland cells and
leads to decreased PTH secretion. The drug is relatively expensive but effective. Surgery
to remove the parathyroid glands is required for patients with very high PTH levels.
Investigation and treatment are summarized in Figure 46.1.

Recent Developments
1 There is emerging evidence that many patients may harbour subtle abnormalities
from an early stage of renal decline, and that these abnormalities contribute to
prognosis. Levin et al.3 studied 1814 patients with varying degrees of renal function.
Suboptimal vitamin D status (<22 pg/ml) was present in 13% of patients with
estimated glomerular filtration rate (eGFR) of >80 ml/min and in over 60% of those
with eGFR <30 ml/min. High PTH was common in patients with even modest renal
impairment (eGFR >80 ml/min).
2 In patients with renal disease, vascular calcification leads to structural changes in the
vascular wall and accelerated atherogenesis. However, patients with asymptomatic
hyperparathyroidism may have subtle abnormalities in vascular function that
contribute to the development of macrovascular disorders. These abnormalities
include insulin resistance, endothelial activation and increased arterial stiffness. In
asymptomatic patients, the changes in glucose tolerance do not appear to progress
rapidly.4 Fallo et al.5 have demonstrated increased soluble E-selectin and von
Willebrand factor in patients with hyperparathyroidism. These markers provide
evidence of endothelial activation and may help to assess cardiovascular risk prior to
and following treatment of hyperparathyroidism.
3 Mineral abnormalities contribute to development of insulin resistance and type 2
diabetes. On the other hand, treatment of diabetes may influence bone and mineral
status. Thiazolidinediones (rosiglitazone and pioglitazone) are widely used in treatment
of type 2 diabetes. In bone, these drugs enhance adipogenesis while decreasing
osteoblast differentiation. There is concern, therefore, that they may predispose users to
osteoporosis. Certainly recent evidence suggests that use of glitazone drugs can decrease
BMD.6 It remains to be seen whether this translates into increased fracture risk.
4 Data from the third National Health and Nutrition Evaluation Survey (NHANES III)
confirmed that low vitamin D status was a risk factor for type 2 diabetes.7 This is not
only confirmed by recent data,8 but it also appears that patients with type 2 diabetes
07-PS Rheumatology-cpp :07-PS Rheumatology-ppp.QXD 18/3/08 14:32 Page 251

46 Bone complications of renal disease 251

and low vitamin D status are at greater risk of requiring insulin and are more likely
to develop microvascular complications. Consideration should be given to screening
for abnormalities of calcium and vitamin D status in patients with insulin-resistant
states, and studies are required to evaluate whether vitamin D supplementation
improves prognosis for patients with metabolic syndrome and type 2 diabetes.

Conclusion
The term renal osteodystrophy is of limited use in routine clinical practice as it can only
be diagnosed and classified with bone biopsy. At present, markers of bone turnover are
not able to specifically diagnose bone complications of CKD. Vascular calcification is an
important risk marker for patients with renal disease and is part of the metabolic disturb-
ance that also leads to bone disease. Biochemical changes in calcium, phosphate, vitamin
D and PTH metabolism appear when eGFR falls below 40 ml/min, but subtle changes
can occur with mild degrees of renal impairment. The above patient should have assess-
ment of his calcium and vitamin D status. His renal impairment means that bisphospho-
nate therapy is not indicated without more detailed knowledge of his underlying bone
status. Vitamin D supplements may decrease PTH levels and thus bone turnover. Low
BMD in this case confirms decreased bone mineral content but may reflect decreased vit-
amin D and increased PTH status and not simply primary osteoporosis.

Further Reading
1 Schwarz C, Sulzbacher I, Oberbauer R. Diagnosis of renal osteodystrophy. Eur J Clin Invest
2006; 36 (Suppl 2): 1322.
2 Ferreira A. Development of renal bone disease. Eur J Clin Invest 2006; 36 (Suppl 2): 212.
3 Levin A, Bakris GL, Molitch M et al. Prevalence of abnormal serum vitamin D, PTH, calcium,
and phosphorus in patients with chronic kidney disease: results of the study to evaluate early
kidney disease. Kidney Int 2007; 71: 318.
4 Ayturk S, Gursoy A, Bascil Tutuncu N, Ertugrul DT, Guvener Demirag N. Changes in insulin
sensitivity and glucose and bone metabolism over time in patients with asymptomatic primary
hyperparathyroidism. J Clin Endocrinol Metab 2006; 91: 42603.
5 Fallo F, Cella G, Casonato A et al. Biochemical markers of endothelial activation in primary
hyperparathyroidism. Horm Metab Res 2006; 38: 1259.
6 Schwartz AV, Sellmeyer DE, Vittinghoff E et al. Thiazolidinedione use and bone loss in older
diabetic adults. J Clin Endocrinol Metab 2006; 91: 334954.
7 Scragg R, Sowers M, Bell C; Third National Health and Nutrition Examination Survey. Serum
25-hydroxyvitamin D, diabetes, and ethnicity in the Third National Health and Nutrition
Examination Survey. Diabetes Care 2004; 27: 281318.
8 Suzuki A, Kotake M, Ono Y et al. Hypovitaminosis D in type 2 diabetes mellitus: Association
with microvascular complications and type of treatment. Endocr J 2006; 53: 50310.
08-PS Rheumatology-cpp :08-PS Rheumatology-ppp.QXD 18/3/08 14:32 Page 253

S E C T I O N E I G H T 08
Muscle Diseases
47 Steroid myopathy
48 Inflammatory myopathies
49 Muscle complications of statin therapy

P R O B L E M

47 Steroid Myopathy

Case History
JG is a 62-year-old man with severe obstructive pulmonary disease. He has been taking
steroids continuously for ten years generally 510 mg of prednisolone per day but more
with exacerbations. He complains of progressive weakness and muscle wasting, and notes
particular difficulty climbing stairs. He is a moderately heavy drinker. There is no history
of diabetes and no family history of muscle disease.
How can steroid myopathy be diagnosed?
What is his prognosis?
What treatment options are available for him?

Background
This chapter is a general background to muscle disease, with specific emphasis on
steroid-induced myopathy. While muscle symptoms are common, diseases of muscle are
relatively uncommon and many of the specific diagnoses are rare. Table 47.1 is a sum-
mary classification of disorders of muscle.
Congenital myopathies usually present in infancy with a floppy baby that has poor
muscular effort. Plasma creatine kinase (CK) is normal, and electromyography (EMG)
shows a myopathic pattern. Central core disease usually presents with mild, non-
progressive weakness in infancy leading to delay in walking and other physical develop-
ment milestones. It is inherited as an autosomal dominant condition. Nemaline rod

Atlas Medical Publishing Ltd


08-PS Rheumatology-cpp :08-PS Rheumatology-ppp.QXD 18/3/08 14:32 Page 254

254 08 Muscle Diseases

Table 47.1 Diseases of muscle

A: Congenital
Congenital myopathies Central core disease
Nemaline rod myopathy
Centronuclear (myotubular)
Muscular dystrophies
Channelopathies
Inherited metabolic diseases
Mitochondial myopathy syndromes

B: Acquired
Inflammatory Dermatomyositis
Inclusion body myositis
Polymyositis
Metabolic Hypothyroidism and hyperthyroidism
Vitamin D deficiency
Cushings syndrome
Conns syndrome and Addisons disease
Hypokalaemia
Critical care myopathy
Drugs and toxins Alcohol
Organophosphates
Snake venoms
Corticosteroids
Statins, clofibrate
Vincristine, cyclosporine
Amiodarone
Zidovudine
Paraneoplastic Carcinomas
Dermatomyositis

myopathy is more serious, presenting with weakness and hypotonia. The consequences
are feeding difficulties, delay in walking and sometimes respiratory muscle weakness. It is
slowly progressive and older children or adults with the condition characteristically have
decreased muscle bulk and an abnormally long face with protruding jaw. It can be inher-
ited in either a dominant or a recessive fashion. Centronuclear (myotubular) myopathy
again presents in infancy and is relentlessly progressive, usually leading to death at an
early age. Involvement of the eye muscles is common.
Muscular dystrophies. Onset and severity varies. Specific genetic tests are now available
for many of these conditions and all patients/families should receive genetic counselling.
Onset is usually in childhood, although some forms typically present later. The most
common forms are:
Myotonic dystrophy. This is the most common inherited disease of muscle.
Myotonic dystrophy type 1 (DM1) is inherited as an autosomal dominant disorder
and is due to an expanded cytosine-thymine-guanine (CTG) trinucleotide repeat in
the 3 untranslated region of the myotonic dystrophy protein kinase (DMPK) gene
on chromosome 19q13.3. In addition to myopathy, patients suffer cognitive
impairment, subcapsular cataracts, cardiac conduction abnormalities, sensorineural
08-PS Rheumatology-cpp :08-PS Rheumatology-ppp.QXD 18/3/08 14:32 Page 255

47 Steroid myopathy 255

deafness, frontal balding and hypogonadism. Myotonic dystrophy type 2 (DM2) is


also an autosomal dominant condition and is due to expansion of a cytosine-
cytosine-thymine-guanine (CCTG) repeat in intron 1 of the ZNF9 gene on
chromosome 3q. It causes a proximal muscle pattern of myopathy, sometimes with
pain and hypertrophy, but no cognitive impairment. Both DM1 and DM2 lead to
alternative splicing of the voltage-gated chloride channel (ClC-1) and are therefore
considered along with the channelopathies.
Duchenne muscular dystrophy. An X-linked condition, which therefore affects boys,
that is due to deletion in the dystrophin gene. It usually presents between the ages of
two and six years. The patient is usually wheelchair-bound by their early teens, and
survival beyond late teens or early adulthood is the exception. Weakness is proximal
and limb girdle and there may be pseudohypertrophy of the calves, cardiac
conduction abnormalities and scoliosis.
Becker muscular dystrophy. This is also X-linked, with a similar distribution of
muscle weakness to Duchenne muscular dystrophy. It is usually milder than
Duchenne, but is variable in severity. Symptoms may not begin until age ten years or
later, and patients can have a long lifespan, albeit with varying degrees of disability.
EmeryDreifuss muscular dystrophy. An X-linked disorder that is due to a mutation
in the emerin gene. Symptoms start around five years of age with upper arm and
lower leg weakness. Proximal muscle weakness develops later. Contractures and joint
problems may develop, and patients are at risk of sudden cardiac death because of
conduction abnormalities.
Limb girdle muscular dystrophy. There can be dominant (type 1) or recessive (type
2) inheritance. A number of gene abnormalities may cause the syndrome and
prognosis is, therefore, variable. Cardiac abnormalities may be present. Boys and
girls are equally affected, and symptoms typically begin in late childhood.
Facioscapulohumeral muscular dystrophy. This is an autosomal dominant condition
affecting males and females. Onset is usually in late childhood or early adulthood.
The symptoms can be quite mild, although may become more generalized, affecting
the lower limbs later.

Channelopathies are a recently recognized group of conditions where there is a genetic


defect in one of the ion channels involved in regulating normal muscle.1 A summary is
presented in Table 47.2. Myotonia arises from repetitive bursts of action potential when
muscle contraction is voluntarily activated. The result is inability to relax the muscle.
Symptoms usually improve during activity. By contrast, paramyotonia worsens with cold
and after exercise. Becker myotonia is the most common form while Thomsens disease,
although less common, usually presents with milder symptoms. Hyperkalaemic periodic
paralysis may be triggered by potassium intake and symptoms are ameliorated by glu-
cose. The reverse applies to hypokalaemic periodic paralysis. Andersen syndrome is an
autosomal dominant disorder where episodes of paralysis are provoked by prolonged
inactivity (including sleep), calorie deprivation and cold. It is associated with long QT
interval on the electrocardiogram, making patients prone to tachycardias. Malignant
hyperthermia can be provoked by volatile anaesthetics, depolarizing muscle relaxants or
extreme physical activity. A sustained increase in cellular calcium in skeletal muscle leads
08-PS Rheumatology-cpp :08-PS Rheumatology-ppp.QXD 18/3/08 14:32 Page 256

256 08 Muscle Diseases

Table 47.2 Channelopathies

Channel Disease Heritance

Sodium
(SCN4A, Ch 17q23.1) Paramyotonia congenita Dominant
Hypokalaemic periodic paralysis (type 2) Dominant
Hyperkalaemic periodic paralysis Dominant
Potassium
(KCNJ2, Ch 17q2324) Andersen syndrome Dominant
Calcium
(CACNA1S, Ch 1q3132) Hypokalaemic periodic paralysis (type 1) Dominant
Malignant hyperthermia Dominant
(RyR1, Ch 19q13.1) * Malignant hyperthermia Dominant
Chloride
(CLCN1, Ch 7q32) Becker myotonia Recessive
Thomsens disease Dominant
Acetylcholine receptor Congenital myasthenia Dom/Rec
* RyR1, calcium release channel. One of five genes affected, each encoding different subunits of the nicotinic acetylcholine receptor. Dom/Rec,
inheritance may be dominant or recessive.

to intense muscle contraction with hyperthermia, metabolic acidosis, hypoxia and


hyperkalaemia.
Mitochondrial myopathies are being increasingly recognized in clinical practice,
although they remain rare:
MELAS (Myopathy, Encephalopathy, Lactic Acidosis and Stroke) episodic
encephalopathy, stroke-like episodes; a progressive neurodegenerative disorder often
also causing diabetes
MERRF (Myoclonic Epilepsy with Ragged Red Fibres) optic atrophy, peripheral
neuropathy, dementia, myoclonic epilepsy, cerebellar ataxia and sensorineural
deafness
KearnsSayre syndrome progressive oculomotor symptoms including ptosis,
pigmentary retinal degeneration, sensorineural deafness, proximal myopathy and
cardiac conduction defects
CPEO (Chronic Progressive External Ophthalmoplegia) like KearnsSayre
syndrome but later onset and not associated with retinal degeneration
Congenital metabolic disorders should be considered in the differential diagnosis of
muscle disorders, particularly where they occur early in life or where there is a relevant
family history. Differential diagnosis includes the glycogen storage diseases (GSDs).
Disorders that lead to prominent muscle symptoms in adult patients include:
Pompe disease (Type II GSD) due to deficiency of the lysosomal enzyme alpha-1,4-
glucosidase (acid maltase) leading to unregulated accumulation of glycogen with
disruption of muscle structure and function
Cori disease (Type III GSD; limit dextrinosis) due to deficiency of the debrancher
enzyme leading to accumulation of abnormal glycogen, which cannot be broken
down to release glucose
08-PS Rheumatology-cpp :08-PS Rheumatology-ppp.QXD 18/3/08 14:32 Page 257

47 Steroid myopathy 257

McArdle disease (Type V GSD) due to deficiency of myophosphorylase which, again,


leads to defective glycogen breakdown. Muscle swelling and tenderness occurs, CK
levels are generally very high and there may be episodes of rhabdomyolysis
Tarui disease (Type VII GSD) leads to a clinical presentation similar to that of
McArdle disease and is due to deficiency of muscle phosphofructokinase
Carnitine-palmitoyl transferase deficiency causes episodes of muscle pain and
weakness, intermittent CK elevation and myoglobinuria
In adult practice, except in highly specialized centres, acquired muscle diseases are
much more common than congenital disorders. Amongst these, myopathy caused by
alcohol or drugs, including corticosteroids, is encountered not uncommonly.2
Acute alcoholic myopathy is relatively rare and leads to muscle necrosis, with variable
inflammatory infiltrate causing muscle weakness and pain. Plasma CK is markedly
increased, and there may be myoglobinuria and rhabdomyolysis with accompanying
renal impairment. Recovery generally occurs following alcohol withdrawal and support-
ive measures. Chronic alcoholic myopathy principally affects type II (fast twitch, anaero-
bic, glycolytic) fibres. It typically occurs after ten years of consuming greater than
100 grams (1012 units) of alcohol per day. The aetiology is not precisely known. Factors
include ethanol-induced impaired mitochondrial function, leading to defective adeno-
sine triphosphate generation and fatty acid utilization, acetaldehyde accumulation
inhibiting protein synthesis, defective protein synthesis because of decreased amino acid
availability and decreased activity of the growth hormone/insulin-like growth factor-1
(IGF-1) axis, and free radicals causing cell membrane damage.
Steroid myopathy does not always occur with either prolonged exposure or high
doses. It is more common with potent fluorinated steroids (dexamethasone, betametha-
sone and triamcinolone). As with alcoholic myopathy, acute and chronic forms are rec-
ognized. Acute steroid myopathy usually occurs after acute exposure to high-dose steroid
and can take many months to recover. A subacute, necrotizing form of myopathy with
steroids is described, and leads to severe symptoms with CK levels more than ten times
normal.3 Exposure of myocytes to steroid impairs protein synthesis and leads to loss of
the protective effects of IGF-I. Furthermore, increased cellular protease activity increases
muscle protein breakdown. Biopsy reveals variation in fibre diameters, loss of type II
fibres and necrotic and basophilic fibres throughout the muscle. As with other metabolic
myopathies, symptoms characteristically affect the proximal muscles although more gen-
eral involvement, including respiratory muscles, may occur in severe cases. In patients
exposed to steroids long-term, there are usually other clinical features of steroid excess
present by the time myopathy develops. Treatment (Figure 47.1) consists of minimizing
exposure by decreasing the dose, using topical preparations, using alternate day regimens
and avoiding fluorinated steroids. Progressive resistance-training exercises are useful in
restoring normal function and muscle bulk. Recovery is slow in chronic cases, and com-
plete recovery may not be achieved.
Critical illness myopathy (CIM), or acute quadriplegic myopathy, in its full-blown
form is a rare entity with acute onset of generalized weakness. Plasma CK levels are nor-
mal. It is similar to steroid myopathy indeed many patients have been treated with
steroids but is more severe and more generalized. Neuromuscular blocking drugs have
also been implicated in the aetiology. EMG shows low or normal action potentials.
08-PS Rheumatology-cpp :08-PS Rheumatology-ppp.QXD 18/3/08 14:32 Page 258

258 08 Muscle Diseases

Muscle weakness
Muscle wasting

Young age Markedly CK


1ve Family history 1ve Inflammatory markers
Increased CK Autoantibodies

? Congenital cause ? Inflammatory cause

Review steroid dose and duration

Alcohol history Muscle biopsy EMG


other drugs

Steroid
myopathy
diagnosed

Minimize exposure to steroids Resistance exercises


Avoid fluorinated steroids ? Nutritional supplements
Use topical agents
Use other immunosuppressive drugs
Minimize dose
Alternate day dose regimens

Figure 47.1 Diagnosis and management of steroid myopathy. +ve, positive.

Biopsy may show type II fibre atrophy or necrosis similar to that seen in steroid myo-
pathy. There is no specific treatment. Recovery is usually complete but can be slow.

Recent Developments
1 CIM is associated with increased hospital stay, increased risk of requiring mechanical
ventilation and increased mortality.4,5 Patients are at increased risk of developing
CIM if they have sepsis, are hyperglycaemic or require steroid treatment. Amongst
08-PS Rheumatology-cpp :08-PS Rheumatology-ppp.QXD 18/3/08 14:32 Page 259

47 Steroid myopathy 259

causative factors are systemic inflammation (particularly with sepsis), increased


proteolysis and oxidative and metabolic stresses. There are often neuropathic
features, and there is impairment of excitationcontraction coupling. Intensive
insulin therapy has been considered as a means of protecting patients from the
consequences of CIM.
2 Muscle loss with inactivity is exacerbated if it is accompanied by stress and this is
thought to be due to hypercortisolaemia. Essential amino acids formulated to
replicate the proportion found in muscle are a potent anabolic stimulus in the
myopathies of inactivity and steroid exposure.6 Attention should be paid to nutrition
in patients exposed to steroids, those who are critically ill and those who are likely to
suffer prolonged immobility.
3 Creatine supplementation diminishes the impaired exercise capacity that occurs
when experimental animals are administered supraphysiological doses of steroids.7,8
The supplement attenuates loss of muscle mass with steroids. Clinical trials in
patients taking steroids or admitted to critical care facilities are required to evaluate
this intervention, which may prove a safe prophylactic measure.

Conclusion
Steroid myopathy is diagnosed by excluding other causes of muscle weakness and wast-
ing. Except in the acute necrotizing form of steroid myopathy, there is not usually sys-
temic inflammatory activation or increased circulating muscle markers. Steroid dose
does not have to be high nor the duration of treatment long for the patient to develop
myopathy. Muscle biopsy may be required for definitive diagnosis. Prognosis is variable
and relates to severity. Improvement is usual if steroids can be decreased or withdrawn.
Other risk factors for muscle loss should be eliminated where possible. These include cer-
tain drugs and high intake of alcohol. There is no specific treatment. Resistance exercises
to rebuild muscle mass and nutritional supplements have been advocated but there is no
evidence to support this from randomized clinical trials.

Further Reading
1 Jurkat-Rott K, Lehmann-Horn F. Muscle channelopathies and critical points in functional
and genetic studies. J Clin Invest 2005; 115: 20009.
2 Owczarek J, Jasinska M, Orszulak-Michalak D. Drug-induced myopathies. An overview of the
possible mechanisms. Pharmacol Rep 2005; 57: 2334.
3 Bronner IM, Hoogendijk JE, Wintzen AR et al. Necrotising myopathy, an unusual presenta-
tion of a steroid-responsive myopathy. J Neurol 2003; 250: 4805.
4 Deem S. Intensive-care-unit-acquired muscle weakness. Respir Care 2006; 51: 104252;
discussion 10523.
5 Friedrich O. Critical illness myopathy: what is happening? Curr Opin Clin Nutr Metab Care
2006; 9: 4039.
08-PS Rheumatology-cpp :08-PS Rheumatology-ppp.QXD 18/3/08 14:32 Page 260

260 08 Muscle Diseases

6 Paddon-Jones D, Wolfe RR, Ferrando AA. Amino acid supplementation for reversing bed rest
and steroid myopathies. J Nutr 2005; 135: 1809S12S.
7 Campos AR, Serafini LN, Sobreira C, Menezes LG, Martinez JAB. Creatine intake attenuates
corticosteroid-induced impairment of voluntary running in hamsters. Appl Physiol Nutr
Metab 2006; 31: 4904.
8 Menezes LG, Sobreira C, Neder L, Rodrigues-Jnior AL, Martinez JAB. Creatine
supplementation attenuates corticosteroid-induced muscle wasting and impairment of
exercise performance in rats. J Appl Physiol 2007; 102: 698703.

P R O B L E M

48 Inflammatory Myopathies

Case History
A 56-year-old woman complains of difficulty in rising from a chair and climbing stairs.
She has an erythematous rash on the back of her hand. Symptoms have increased over the
past four weeks. Her previous health has been very good and she is not taking any
medications.
What other clinical features should be considered?
How should she be investigated?
What treatment is available and what is the prognosis?

Background
This group of disorders (Box 48.1) is quite rare but not infrequently needs to be consid-
ered in practice. This chapter considers only the primary inflammatory myopathies, of
which dermatomyositis (DM) and inclusion body myositis (IBM) are the most common.
Together, their incidence is 28 per million per year. They present classically with muscle
weakness. Pain and stiffness are less prominent symptoms.
DM and polymyositis (PM) present with similar distribution of involvement.
Proximal muscle weakness predominates over distal weakness. Pelvic girdle involvement
usually precedes and is more marked than shoulder girdle involvement. The typical ini-
tial symptoms are difficulty rising from a chair or climbing stairs. DM is a disorder of
humoral immunity where antibody binding/deposition or immune complexes lead to
capillary damage, resulting in localized atrophy or infarction. It may present at any age.

Atlas Medical Publishing Ltd


08-PS Rheumatology-cpp :08-PS Rheumatology-ppp.QXD 18/3/08 14:32 Page 261

48 Inflammatory myopathies 261

Box 48.1 Inflammatory myopathies


1. Primary
Dermatomyositis
Inclusion body myositis
Polymyositis
2. Connective tissue diseases
Mixed connective tissue disease (MCTD)
Systemic lupus erythematosus
Scleroderma
Sjgrens syndrome
Rheumatoid arthritis
3. Infections
Viral (Coxsackie, HIV etc.)
Bacterial
Parasitic
4. Miscellaneous
Graft-versus-host disease
Eosinophilic myositis
Macrophagic myofasciitis
Sarcoidosis
Associated with systemic vasculitis
HIV, human immunodeficiency virus. Adapted with permission from Hilton-Jones 2003.1

Twenty per cent of cases in later life occur with an underlying malignancy. There is no
association with any particular malignancy. In children, DM does not usually give rise to
profound weakness, but is more frequently associated with subcutaneous calcification
and facial flushing. However, the characteristic rash is usually absent and the condition is
seldom associated with malignancy. Presentation of DM is usually subacute but it may
develop very rapidly and present with severe symptoms. The characteristic rash, which
occurs in most patients, is a photosensitivity reaction on the cheeks, exposed anterior
chest or the knuckles. It can be similar to the rash of systemic lupus erythematosus (SLE).
The eyelids may show a purple discolouration and there may be a scaly eruption
(Gottrons sign). Interstitial lung disease occurs in up to 30% of cases and in over 60% of
those who have antibodies against aminoacyl-tRNA synthetases. Increased mortality in
DM relates to lung disease, underlying malignancy and, occasionally, associated
myocarditis. The serum of patients with lung disease is usually positive for anti-
aminoacyl-tRNA synthetase antibodies (particularly anti-Jo).
PM is much less common than the other two idiopathic inflammatory myopathies. Its
onset is usually more insidious than that of DM. It is not associated with skin lesions or
with underlying malignancy. Histologically, it is distinct from the other two disorders,
but muscle biopsy may not reveal characteristic changes in a patient with apparent PM.
PM is predominantly a disorder of cell-mediated immunity where CD8+ cells home to
muscle that is aberrantly expressing class I major histocompatibility antigens.
IBM classically involves the quadriceps and the long finger flexors, leading respectively
to unexplained falling or knees giving way and compromised hand grip.2,3 Unlike the
08-PS Rheumatology-cpp :08-PS Rheumatology-ppp.QXD 18/3/08 14:33 Page 262

262 08 Muscle Diseases

other two conditions, symptoms may be asymmetric with facial involvement or dyspha-
gia from an early stage. Foot drop is common. Microscopic features resemble those of
PM, and overlap with the immunocytochemical features of Alzheimers disease in the
brain has been suggested. The latter include vacuolar formation with foci of amyloid and
phosphorylated tau. The extent to which autoimmunity is involved in the pathogenesis is
uncertain. Hereditary forms are well described and, not surprisingly, may present at an
early age. IBM typically occurs in the sixth decade or beyond and is now the primary
inflammatory myopathy most commonly diagnosed in this age group. An association
with human immunodeficiency virus (HIV) and human T-lymphotropic virus type 1
(HTLV-1) has been described and it is associated with other autoimmune diseases in
about one-third of cases. IBM is often relentlessly progressive and response to treatments
is disappointing. There is emerging anecdotal evidence to support the use of anti-T-cell
therapies and stem-cell therapy has been considered.
Diagnosis and management of inflammatory myopathies is summarized in Figure
48.1. Measurement of serum creatine kinase (CK) is the simplest screening tool. Mild ele-
vations outside the normal range are common and may be due to exercise, trauma or
intramuscular injections. Men have higher values than women and values are higher in
patients of Afro-Caribbean origin. Electromyography characteristically shows fibrillation
patterns with sharp waves.
Ninety per cent of patients with inflammatory myopathy have autoantibodies to nuclear
or cytoplasmic antigens. Myositis-specific antibodies (MSA) are directed against cytoplas-
mic ribonucleoproteins and are present in 30% of cases. Antibodies against aminoacyl-
tRNA synthetases are particularly present in those with rapid onset, skin rash and interstitial
lung disease. Antibodies against six of the 20 aminoacyl-tRNA synthetases have been
described. The commonest of these, anti-Jo (directed against the histidyl-tRNA synthetase),
is present in around 10% of cases. Antibodies to signal recognition particle (SRP; involved in
endoplasmic reticulum transport of polypeptides) are found particularly in African-
American women, and are associated with acute onset, widespread features (including car-
diac) and poor prognosis. M1-2 is a nuclear antigen, antibodies to which are found in DM.
Muscle biopsy should be carried out where possible given the potential gravity of the
conditions and that there may be a need for immunosuppressive treatment. The deltoid
or quadriceps muscles are usually used. The best choice is an affected muscle but not one
that is severely affected, as the biopsy may then only show severe atrophy. Open biopsy or
needle biopsy may be used, and magnetic resonance imaging (MRI) or computed tomog-
raphy scanning may be used to guide the site of biopsy. MRI studies may be useful diag-
nostically and in following progress. Ultrasound may also be useful.
Exercise is important to ensure that residual muscle capacity is maximized. Also, pas-
sive exercises help to minimize the risk of contractures. All patients should have the help
of a qualified physiotherapist. Adequate calorie and protein intake should be ensured to
minimize muscle catabolism, particularly if the patient has dysphagia. The mainstay of
drug treatment is corticosteroids. Initially, intravenous methylprednisolone at a dose of
500 mg for five days may induce remission. This is followed by prednisolone 1 mg/kg
body weight/day, gradually reduced as the plasma CK decreases. Other immunosuppres-
sive drugs are also useful e.g. methotrexate (up to 30 mg/week) or azathioprine
(2.5 mg/kg/day). Cyclosporine and cyclophosphamide are also widely used. Intravenous
immunoglobulin is effective for DM or PM. IBM is much more refractory to steroids or
other immunosuppressive therapies.
08-PS Rheumatology-cpp :08-PS Rheumatology-ppp.QXD 18/3/08 14:33 Page 263

48 Inflammatory myopathies 263

Muscle weakness
1 stiffness
1 pain

Increased
CK
ESR and CRP

Possible inflammatory myopathy

Proximal muscle Quadriceps


involvement Hand muscles

? DM or PM ? IBM

? Skin lesions Face, chest, hands


? Lung involvement CXR, pulmonary function tests
? Internal malignancy CXR, CT scan, tumour markers

EMG Autoantibodies MRI or U/S Muscle biopsy

Diagnosis of DM or PM

Corticosteroid

Maintenance dose Second-line immunosuppressive

Biological agent

Figure 48.1 Diagnosis and management of inflammatory myopathies. CK, creatine kinase; CRP, C-reactive
protein; CT, computed tomography; CXR, chest X-ray; DM, dermatomyositis; EMG, electromyography; ESR,
erythrocyte sedimentation rate; IBM, inclusion body myositis; MRI, magnetic resonance imaging; PM,
polymyositis; U/S, ultrasound.
08-PS Rheumatology-cpp :08-PS Rheumatology-ppp.QXD 18/3/08 14:33 Page 264

264 08 Muscle Diseases

Recent Developments
1 There is increasing experience with use of biological agents,4,5 although the rarity of
the conditions means that controlled trials are not available. Rapamycin or
monoclonal antibodies have been used as anti-T-cell agents. There is some
experience with tumour necrosis factor (TNF)-a blockers (etanercept and
infliximab). The anti-B-cell (CD20) antibody rituximab has been used, as it has in
other connective tissue diseases. Eculizumab, a monoclonal antibody that inhibits
cleavage of the complement component C5, has also been used.
2 The pathogenesis of IBM is becoming more clearly understood. Although it is
frequently associated with immune disturbance, it may primarily be a degenerative
disease.2,6 Accumulation of amyloid-beta protein and its precursor, evidence of
oxidative stress, abnormal protein folding and disturbed proteasomal degradation of
proteins may all contribute to the pathological features.
3 Identification of further antigenic targets and other immune disturbances may
improve our understanding of pathogenesis but it is not clear whether these will
contribute to streamlining diagnosis and management. Recently, increased
interleukin-18 (IL-18) has been described in DM and PM, in keeping with a T helper
1 (Th1)-dominated immune response.7 Two novel nuclear antigens with molecular
masses of 140 kDa and 155 kDa have been described in Japanese patients.8 The
presence of antibodies to these antigens may be strongly associated with coexisting
malignancy.

Conclusion
Although rare, inflammatory myopathies are important to diagnose. They carry a signifi-
cant mortality but prognosis has been improved by use of modern diagnostic techniques
and immunosuppressive therapy.9 Patients with suspected myopathy should be carefully
and fully investigated and precise diagnosis relies on muscle biopsy. There should be
enquiry and investigation for cutaneous, pulmonary and cardiac involvement and a
search for malignancy in DM. The latter may not be apparent at diagnosis but typically
manifests itself within one year of diagnosis. Corticosteroids are the first line of treatment
and should be commenced in sufficiently high doses to control disease activity quickly.
Doses should also be decreased rapidly as the condition becomes less active. If this does
not occur, addition or substitution of other immunosuppressive agents should be con-
sidered. The outlook for patients with inflammatory myopathies has improved in recent
years because of more effective diagnosis and availability of a wider range of treatments.

Further Reading
1 Hilton-Jones D. Diagnosis and treatment of inflammatory muscle diseases. J Neurol Neurosurg
Psychiatry 2003; 74 (Suppl 2): ii25ii31.
2 Engel WK, Askanas V. Inclusion-body myositis: clinical, diagnostic, and pathologic aspects.
Neurology 2006; 66 (2 Suppl 1): S209.
08-PS Rheumatology-cpp :08-PS Rheumatology-ppp.QXD 18/3/08 14:33 Page 265

49 Muscle complications of statin therapy 265

3 Dalakas MC. Inflammatory, immune, and viral aspects of inclusion-body myositis. Neurology
2006; 66 (2 Suppl 1): S338.
4 Cordeiro AC, Isenberg DA. Treatment of inflammatory myopathies. Postgrad Med J 2006; 82:
41724.
5 Dalakas MC. Therapeutic targets in patients with inflammatory myopathies: present
approaches and a look to the future. Neuromuscul Disord 2006; 16: 22336.
6 Askanas V, Engel WK. Inclusion-body myositis: a myodegenerative conformational disorder
associated with Abeta, protein misfolding, and proteasome inhibition. Neurology 2006; 66
(2 Suppl 1): S3948.
7 Tucci M, Quatraro C, Dammacco F, Silvestris F. Interleukin-18 overexpression as a hallmark
of the activity of autoimmune inflammatory myopathies. Clin Exp Immunol 2006; 146: 2131.
8 Kaji K, Fujimoto M, Hasegawa M et al. Identification of a novel autoantibody reactive with
155 and 140 kDa nuclear proteins in patients with dermatomyositis: an association with
malignancy. Rheumatology 2007; 46: 258.
9 Briani C, Doria A, Sarzi-Puttini P, Dalakas MC. Update on idiopathic inflammatory
myopathies. Autoimmunity 2006; 39: 16170.

P R O B L E M

49 Muscle Complications of Statin Therapy

Case History
MO is a 52-year-old man with type 2 diabetes. Glycaemic control is reasonable with diet
alone. Six months ago, he was started on a statin because of his cholesterol level. Over the
past three months, he has noted muscle aches and pains. These have become much more
severe in the past two weeks and his calves are tender bilaterally.
What is the risk of muscle complications in patients treated with statins?
Does the risk differ with different agents?
How should statin-induced muscle problems be prevented, diagnosed and treated?

Atlas Medical Publishing Ltd


08-PS Rheumatology-cpp :08-PS Rheumatology-ppp.QXD 18/3/08 14:33 Page 266

266 08 Muscle Diseases

Background
Statins 3-hydroxy-3-methylglutaryl-CoA reductase inhibitors decrease endogenous
cholesterol synthesis. Statins are now the most commonly prescribed group of drugs.
They lower total and low-density lipoprotein (LDL) cholesterol, as well as slightly lower-
ing triglycerides and increasing high-density lipoprotein (HDL) cholesterol. For sec-
ondary prevention of cardiovascular disease, total and LDL cholesterol levels of
4.0 mmol/l and 2.0 mmol/l, respectively, are now recommended. Statins typically
decrease LDL cholesterol by 30%40%, while combination treatments or high-dose
statins can decrease LDL by up to 55%. Lipid-lowering therapy is also recommended for
primary prevention where the estimated risk of a cardiovascular event is greater than
20% in ten years. This includes many patients with type 2 diabetes. While statins are very
effective and safe, the number of patients receiving these drugs means that even unusual
side effects are encountered routinely in practice. Setting tighter cholesterol targets has
led to higher doses being prescribed and increasing use of more potent agents, which also
increases the risk of side effects. For similar cholesterol-lowering activity, different doses
and drugs are comparable in their fringe benefits (levels of triglycerides and HDL choles-
terol, anti-inflammatory effects) and their risk of side effects.
The risk of muscle disorders became widely recognized when cerivastatin (Baycol) was
withdrawn five years ago.1 Muscle problems occurred particularly following high doses
and when the drug was combined with the fibrate gemfibrozil. Increased hepatic
transaminases occurs in 0.5%2.0% of cases, although progression to liver failure is rare.
Muscle side effects are now commonly reported but it is not clear to what extent these are
greater than in non-treated patients. Myopathy usually implies a condition that leads to
muscle weakness. However, the term is used in the statin literature to represent any mus-
cle disorder. Myalgia is defined as muscle symptoms without elevation of serum creatine
kinase (CK); myositis signifies muscle aches and weakness with CK elevation; rhabdomy-
olysis is muscle necrosis usually associated with severe symptoms and CK greater than
ten times normal. Rhabdomyolysis with renal impairment can be fatal but occurs in less
than one per million patients prescribed statins. Available statins appear to have similar
likelihood of causing myopathy (0.2%0.5%). Rhabdomyolysis occurs in 0.02%0.04%
of patients. Fibrate treatment alone carries a similar risk of muscle side effects. When
fibrates and statins are used together, risk of myopathy increases to around 1%. Patients
should be warned to report muscle side effects. There is no role for routine monitoring of
CK. The benefits of statin therapy are clear. Muscle symptoms are common in the general
population. If muscle symptoms occur with normal or less than three times normal CK,
it is usually reasonable to follow the symptoms and CK levels at one- to two-weekly inter-
vals without discontinuing the drug. For patients with mild symptoms and modest or no
CK elevation, the drug should be discontinued for a few weeks and cautiously reintro-
duced with careful monitoring. Risk of statin-induced myopathy is increased in the fol-
lowing situations:
Advanced age (>80 years), especially in women
Low body weight
Strenuous exercise
Hypothyroidism
08-PS Rheumatology-cpp :08-PS Rheumatology-ppp.QXD 18/3/08 14:33 Page 267

49 Muscle complications of statin therapy 267

Multisystem disease, particularly renal failure


Perioperative or other catabolic states
With other medications:
Nicotinic acid
Fibrates
Cyclosporine
Macrolide antibiotics
Antifungal agents
Verapamil
Amiodarone
Protease inhibitors for human immunodeficiency virus (HIV) infection
Alcohol abuse
The prevalence of muscle side effects may have been underestimated in the major
statin trials since high-risk individuals were excluded (e.g. the elderly, those with renal
impairment). Muscle symptoms are often overlooked in routine clinical practice. The
PRIMO (Prediction of Muscular Risk in Observational conditions) survey2 investigated
the prevalence of muscular symptoms in patients receiving high-dose statins. Of the 7924
patients surveyed, muscular symptoms developed in 10.5%. Amongst patients who
reported symptoms, 38% stated that the exertion required for everyday activities was pre-
vented, while 4% described themselves as being severely limited. Fluvastatin XL had a
rate of muscular symptoms (5.1%) that was only half of that with other statins. Drugs
that inhibit the cytochrome P450 enzyme CYP3A4 increase risk of statin-induced
myopathy since most statins are metabolized by this enzyme. Fluvastatin is metabolized
by CYP2C9, and this may explain the lower incidence of muscle symptoms. While the
PRIMO study was only observational, it does underline the fact that muscular side effects
are common, particularly when the patient is taking higher doses of statins.
Rhabdomyolysis is the most severe muscular complication.3 It is important to appre-
ciate that it is more likely to occur with a fibrate alone than with a statin alone. Adding a
fibrate to statin therapy leads to a twelve-fold increase in risk of muscle complications.
These are most common within a month of starting or escalating therapy.
Rhabdomyolysis causes myoglobinaemia, myoglobinuria and renal impairment. It may
present with back pain or proximal muscle pain, but many cases have a non-specific pre-
sentation (e.g. fatigue or flu-like symptoms). Risk of rhabdomyolysis does not appear to
relate to serum levels of the drugs. However, high serum levels of rosuvastatin have been
noted in Asian patients, for whom a lower dose of the drug is recommended.
Rhabdomyolysis is diagnosed when muscle symptoms are accompanied by a CK rise to
greater than ten times normal. Enquiry should be made about agents that may provoke
statin toxicity (including alcohol), and for other causes of CK increase such as exercise,
fall and intramuscular injections. If there is doubt as to whether symptoms are related to
statin treatment, it is reasonable to stop the statin for two weeks to see whether symptoms
improve and CK levels decrease. Plasma levels of thyroid-stimulating hormone (TSH),
antinuclear factor (ANF) and anti-Jo antibodies should be checked. One of the postu-
lated mechanisms for muscle toxicity is depletion of coenzyme Q10. Supplementation at
doses of up to 1200 mg/day has been advocated. Acute management includes careful
monitoring of renal function, rehydration, alkalinizing the urine to prevent myoglobin
08-PS Rheumatology-cpp :08-PS Rheumatology-ppp.QXD 18/3/08 14:33 Page 268

268 08 Muscle Diseases

Muscle pain or weakness


Fatigue
Decreased exercise tolerance

Review drug and alcohol history*

CK, TSH, ANF, Anti-Jo

Toxicity possible Toxicity probable Rhabdomyolysis

Drug holiday Drug holiday U/E, myoglobin

Stop the statin


Rehydrate
Lower dose of statin Alkalinize urine
Less toxic statin Muscle biopsy
Pravastatin
Rosuvastatin
Fluvastatin
Ezetimibe Diet 1 resin
Ezetimibe

Regular follow-up
Muscle symptoms
Lipid control
Renal function and CK

Figure 49.1 Management of statin-induced muscle disease. * Check for drugs that may precipitate muscle
toxicity. Plasma and urine myoglobin should be measured. ANF, antinuclear factor; CK, creatine kinase;
TSH, thyroid-stimulating hormone; U/E, urea and electrolytes.

precipitation and discontinuing the statin. The optimal approach to ongoing lipid-low-
ering therapy is unclear. Changing to a more hydrophilic statin (pravastatin or rosuvas-
tatin) should be considered. Fluvastatin has been associated with lower risk of muscle
problems. The risk of recurrence is high if the statin is reinstituted. Ezetimibe may be
used alone or with a decreased dose of statin but, in either case, there appears to be some
risk of recurrent muscle symptoms. Niacin and fibrates may cause muscle symptoms in
patients who have previously had problems with statins. Resins such as cholestyramine
08-PS Rheumatology-cpp :08-PS Rheumatology-ppp.QXD 18/3/08 14:33 Page 269

49 Muscle complications of statin therapy 269

have a low risk of toxicity but patients may not reach current cholesterol targets.
Management of statin-induced muscle disease is summarized in Figure 49.1.

Recent Developments
1 Kaufmann et al.4 have demonstrated mitochondrial toxicity whereby exposure to
statins dissipated the mitochondrial membrane potential with consequent decreased
beta oxidation and disruption in mitochondrial structure. The latter effect may be
important in promoting apoptosis of myocytes.5 Effects were seen with the lipophilic
statins (cerivastatin, fluvastatin, simvastatin and atorvastatin) but not with the
hydrophilic pravastatin. Another recent study6 has reported inhibition of the human
monocarboxylate transporter 4, which is responsible for lactate efflux from the
myocyte. Again, inhibition was seen to a major degree only with the lipophilic statins.
2 Potential benefits of statins beyond their ability to lower cholesterol include anti-
inflammatory properties and favourable effects on bone remodelling.7 Annual sales of
statins are worth over $12.5 billion (United States). Their increasing use includes many
elderly patients. It is reassuring that elderly patients do not appear to be particularly
susceptible to side effects.8 A meta-analysis of 119 trials including a total of 86 000
patients9 concluded that statins were very safe with an odds ratio for rhabdomyolysis of
1.59 and myositis of 2.56. There was a very low withdrawal rate because of adverse
events. It must be borne in mind, however, that high-risk individuals are excluded
from clinical trials but frequently require treatment in routine practice.
3 Ezetimibe inhibits intestinal absorption of cholesterol. It is useful in patients who do
not achieve target cholesterol values with statins alone, or in patients who are at risk
of adverse effects of statins and where it is, therefore, desirable to keep the dose of
statin to a minimum. Vytorin is a combination of ezetimibe and simvastatin at
respective doses of 10/10, 10/20, 10/40 and 10/80 mg. An analysis of 17 trials with
this drug combination10 reports that the incidence of muscle side effects is no greater
than with the statin used alone.
4 Rosuvastatin (Crestor) is the first of a new generation of potent statins. In view of
their increased potency, the safety of these newer drugs is obviously a matter of
concern. Recent studies are reassuring. Using a database of over two million Dutch
patients, more than 45 000 statin users were identified.11 Overall, the incidence of
adverse events was less than 1 per 3000 person-years and there was no difference in
adverse events comparing rosuvastatin with other available drugs of this class.
Shepherd et al.,12 using a large database of nearly 17 000 patients taking statins, again
showed a side-effect profile that was very little different to that of patients taking
placebo. Rosuvastatin was similar to other statins in the incidence of side effects,
including those affecting muscle.

Conclusion
Muscle side effects occur in a minority of patients taking statins but can, at worst, lead to
fatality. The risk of clinically apparent muscle side effects has been estimated at around
1 per 3000 patient-years. This may be minimized by avoiding the drugs in very high-risk
08-PS Rheumatology-cpp :08-PS Rheumatology-ppp.QXD 18/3/08 14:33 Page 270

270 08 Muscle Diseases

patients, by minimizing the dose and being aware of potential drug interactions. The
available statins appear to be similar in terms of risk of muscle complications. In vitro
studies suggest that the more hydrophilic compounds may be safer, as may fluvastatin
because its route of metabolism differs from that of other statins. Management of muscle
complications depends on the severity. The drug should be withdrawn, but may be cau-
tiously reintroduced after symptoms have abated if the adverse event is mild. Otherwise,
management of hydration and renal impairment are usually the most important aspects.
For patients whose CK reaches greater than ten times the upper limit of normal, statin
should not be reintroduced and management of hyperlipidaemia in this scenario pre-
sents a challenge.

Further Reading
1 Pasternak RC, Smith SC, Bairey-Merz CN, Grundy SM, Cleeman JI, Lenfant C.
ACC/AHA/NHLBI Clinical Advisory on the use and safety of statins. J Am Coll Cardiol 2002;
40: 56772.
2 Bruckert E, Hayem G, Dejager S, Yau C, Bgaud B. Mild to moderate muscular symptoms
with high-dosage statin therapy in hyperlipidemic patients the PRIMO study. Cardiovasc
Drugs Ther 2005; 19: 40314.
3 Antons KA, Williams CD, Baker SK, Phillips PS. Clinical perspectives of statin-induced
rhabdomyolysis. Am J Med 2006; 119: 4009.
4 Kaufmann P, Trk M, Zahno A, Waldhauser KM, Brecht K, Krhenbhl S. Toxicity of statins
on rat skeletal muscle mitochondria. Cell Mol Life Sci 2006; 63: 241525.
5 Dirks AJ, Jones KM. Statin-induced apoptosis and skeletal myopathy. Am J Physiol Cell Physiol
2006; 291: C120812.
6 Kobayashi M, Otsuka Y, Itagaki S, Hirano T, Iseki K. Inhibitory effects of statins on human
monocarboxylate transporter 4. Int J Pharm 2006; 317: 1925.
7 Almuti K, Rimawi R, Spevack D, Ostfeld RJ. Effects of stains beyond lipid lowering: potential
for clinical benefits. Int J Cardiol 2006; 109: 715.
8 Agostini JV, Tinetti ME, Han L, McAvay G, Foody JM, Concato J. Effects of statin use on
muscle strength, cognition, and depressive symptoms in older adults. J Am Geriatr Soc 2007;
55: 4205.
9 McClure DL, Valuck RJ, Glanz M, Hokanson JE. Systemic review and meta-analysis of
clinically relevant adverse events from HMG CoA reductase inhibitor trials worldwide from
1982 to present. Pharmacoepidemiol Drug Saf 2007; 16: 13243.
10 Davidson MH, Maccubbin D, Stepanavage M, Strony J, Musliner T. Striated muscle safety of
ezetimibe/simvastatin (Vytorin). Am J Cardiol 2006; 97: 2238.
11 Goettsch WG, Heintjes EM, Kastelein JJP, Rabelink TJ, Johansson S, Herings RMC. Results of
a rosuvastatin historical cohort study in more than 45,000 Dutch statin users, a PHARMO
study. Pharmacoepidemiol Drug Saf 2006; 15: 43543.
12 Shepherd J, Vidt DG, Miller E, Harris S, Blasetto J. Safety of rosuvastatin: update on 16,876
rosuvastatin-treated patients in a multinational clinical trial program. Cardiology 2007; 107:
43343.
09-PS Rheumatology-Index-ppp:Index 18/3/08 14:33 Page 271

Index

ACR-70 response, rheumatoid arthritis 84 American College of Rheumatology (ACR),


activator protein-1 (AP-1) pathway 240 classification criteria for SLE 117
acupuncture, value in fibromyalgia syndrome 31 aminoacyl-tRNA synthetase antibodies 261, 262
acute quadriplegic myopathy 2579 Andersen syndrome 255
adalimumab androgen deprivation therapy, osteoporosis risk
use in psoriatic arthritis 182 235
use in vasculitides 157 androgen therapy, male osteoporosis 2345
AdamantiadesBehets disease see Behets angiogenesis, abnormalities in psoriasis 182
syndrome angiotensin II receptor antagonists, use in
ADAMTS-5 gene 41 Raynauds phenomenon 137
adenine phosphoribosyltransferase deficiency 186 angiotensin-converting enzyme (ACE) inhibitors
adenosine deaminase deficiency 186 use in Raynauds phenomenon 137
adenylsuccinate lyase deficiency 186 use in scleroderma renal disease 142, 143
ADFR (Activate, Decrease osteoclast activity, Free animal bites 201
of treatment and Repeat) 230 ANKH gene 198
adhesive capsulitis (frozen shoulder) 1416, 212 ankylosing spondylitis 4, 87
adiponectin 43 BASMI 8890
Adsons test 22 diagnosis 173
adynamic bone disease 248 effects on heart 91
alcohol consumption mortality risk 91
as cause of male osteoporosis 233 anterior compartment syndrome 24
effect on uric acid synthesis 1856 anterior cruciate ligament tears 6
relationship to RA risk 98 antibiotic treatment
alcoholic myopathy 257 of osteomyelitis 203
alefacept, use in psoriatic arthritis 182 of septic arthritis 201, 204
alemtuzumab 149 antibody treatments 149
alendronate 223 use in Behets syndrome 166
combination with cholecalciferol 220 use in inflammatory myopathies 264
combination with raloxifene 230 use in psoriatic arthritis 1812
fracture prevention 224 use in vasculitides 157, 158
long-term use 224 see also anti-TNF therapy
use during corticosteroid therapy 238, 240 anti-CCP antibodies 5, 72, 73, 7980
use in male osteoporosis 234 association with smoking 75
use in Pagets disease 243 in psoriatic arthritis 182
alfacalcidol 219, 240, 250 anti-centromere antibodies 110
alkaline phosphatase, levels in Pagets disease 242 scleroderma 141, 142
allantoin 184 Sjgrens syndrome 132
allopurinol 106, 187, 188, 1923, 194 anticonvulsants, use in fibromyalgia syndrome 30
alopecia, in SLE 122 antidepressants, use in fibromyalgia syndrome 30
alpha2-adrenergic agonists, use in fibromyalgia anti-dsDNA antibodies 11011, 117
syndrome 30 in monitoring of SLE 1245
alpha-adrenergic antagonists, value in Raynauds anti-histone antibodies 117, 118
phenomenon 137 anti-hnRNP antibodies 112
alphaviruses 208 anti-inflammatory properties, statins 269
Alu-Caps 250 anti-Jo-1 antibodies 112, 261, 262
Alzheimers disease, similarity to inclusion body anti-neutrophil cytoplasmic antibodies (ANCA)
myositis 262 155, 1567, 1578
09-PS Rheumatology-Index-ppp:Index 18/3/08 14:33 Page 272

272 Index

anti-nuclear antibodies (ANAs) 789, 110, in SLE 122


11315 see also osteoarthritis; psoriatic arthritis;
anti-dsDNA 11011 rheumatoid arthritis; septic arthritis; viral
anti-hnRNP 112 arthritis
anti-Jo-1 112, 261, 262 arthritis mutilans 180
anti-Scl-70 112, 141, 142 arthroscopic release, frozen shoulder 16
anti-Sm and anti-RNP 11112 arthroscopy, in septic arthritis 201
anti-SSA/Ro and anti-SSB/La 111, 116, 129 arylalkanoic acids 61
in endometriosis 113 2-arylpropionic acids (profens) 61
environmental factors 112 arzofoxifene 230
in scleroderma 141, 142 ASC protein, role in gout 193
in Sjgrens syndrome 130 aspiration of joints 4
in SLE 11618 diagnosis of septic arthritis 7, 201
antioxidant intake, relationship to RA risk 97, in gout 190
98 knee 89
antiphospholipid antibodies 209 in pseudogout 196
antiphospholipid syndrome (APS), risks during aspirin, low dose
pregnancy 94 concurrent non-selective NSAID therapy 61
antiplatelet therapy use in Raynauds phenomenon 137
in giant cell arteritis 162 use in SLE 126
use in Raynauds phenomenon 137 atherosclerosis, therapeutic strategies 158
anti-RA33 antibodies 112 athletes, risk of osteoarthritis 43
anti-RNA polymerase III antibodies, scleroderma atrophic stage, scleroderma 141
142 autoantibodies, viral infection as trigger 206
anti-RNP antibodies 11112 azapropazone 193
anti-Scl-70 antibodies 112, 141, 142 azathioprine 148, 151
anti-Sm antibodies 11112, 117 interaction with allopurinol 192
anti-SSA/Ro antibodies 111, 116, 129 prediction of response 150
anti-SSB/La antibodies 111, 129 use in Behets syndrome 166
antisynthetase syndrome 112 use in GCA and PMR 161
anti-TNF therapy 51, 80, 83, 85 use in inflammatory myopathies 262
in hepatitis C 207 use during pregnancy 95, 127
in inflammatory myopathies 264 use in psoriatic arthritis 181
in psoriatic arthritis 182 use in Sjgrens syndrome 132
safety monitoring 79 use in SLE 127
in vasculitides 157 use in vasculitides 157
see also etanercept; infliximab
anti-topoisomerase (anti-Scl-70) antibodies 112,
141, 142 B19 (parvovirus) 2067
antiviral antibodies 206 back pain see low back pain
anxiety disorders, in fibromyalgia syndrome 28 bacteraemia, risk factors 204
aortic calcification 24950 Barmah Forest Virus 208
aortic insufficiency, AS 91 basiliximab 149
aplastic anaemia, parvovirus B19 as cause 207 bazedoxifene 230
apoB-100 immunization 158 Becker muscular dystrophy 255
APPROVe (Adenomatous Polyp Prevention on Becker myotonia 255
Vioxx) study 61 Behets syndrome 1656
aromatase activity, relationship to bone loss 235 effect of pregnancy 94
arthritis genetic factors 1667
acute monoarthritis, differential diagnosis 68 insulin resistance 167
new onset benzbromarone 193
examination 3 betamethasone, intra-articular injection 51
history-taking 23 bicipital tendinitis 13
investigations 34 biological agents
patterns of 4 use in Behets syndrome 166
09-PS Rheumatology-Index-ppp:Index 18/3/08 14:33 Page 273

Index 273

use in inflammatory myopathies 264 in septic arthritis 9, 201


use in psoriatic arthritis 1812 in SLE 1256
use in vasculitides 157, 158 caffeine intake, relationship to RA risk 98
see also anti-TNF therapy Calcichew 250
biomarkers of osteoarthritis 43 calcineurin inhibitors 1489
biopsy, in vasculitis 157 calcinosis, in scleroderma 141
see also bone biopsy; muscle biopsy calcitonin
bisphosphonates 218, 21920, 2234, 226, 231 fracture prevention 224
in combination treatment 230 use in neuropathic arthropathy 213
interaction with teriparatide 229 use in Pagets disease 243
osteonecrosis risk 224 calcitriol (1, 25-dihydroxy-vitamin D3) 219,
patient compliance 225 247
structure and function 2223 calcium channel blockers, value in Raynauds
use during corticosteroid therapy 2389, 240 phenomenon 137
use in male osteoporosis 234 calcium levels, in teriparatide therapy 229
use in neuropathic arthropathy 213 calcium pyrophosphate dihydrate (CPPD)
use in osteoarthritis 51 deposition disease (pseudogout) 8, 9, 1956
use in Pagets disease 2423, 245 associated conditions 1967
use in post-menopausal osteoporosis 225 diagnosis 196
blood cultures, septic arthritis 201 hereditary 197
blood film appearance, rheumatoid arthritis 78 role of ANKH mutations 198
body mass index (BMI), relationship to toll-like receptors 198
osteoarthritis risk 45 treatment 1978
bone biopsy 247, 248 calcium supplementation 231
bone histomorphometry 248 in corticosteroid therapy 238
bone mineral density (BMD) 21718 men 234
in diabetes 213 cancers
effect of strontium ranelate 228 association with dermatomyositis 2601
effect of teriparatide 229 breast cancer risk, effect of SERMS 228
relationship to osteophyte formation 43 as cause of lower back pain 172
in renal osteodystrophy 248 vitamin D levels 219
bone mineralization, effect of bisphosphonates capsaicin cream, value in osteoarthritis 47
223 carbon monoxide diffusion capacity, scleroderma
bone remodelling, effects of statins 269 142
BONE study 224 CARD15 gene 179
bone turnover 247 cardiac complications, ankylosing spondylitis 91
bone turnover markers, Pagets disease 242 cardiovascular disease risk
bosentan 144 effect of NSAIDs 614
value in Raynauds phenomenon 138 effect of raloxifene 228
botulinum toxin injection, fibromyalgia in hyperuricaemia 1867
syndrome 31 in psoriasis 182
breast cancer risk, effect of SERMS 228 in SLE 1256
breast-feeding, effect on rheumatoid arthritis risk carnitine-palmitoyl transferase deficiency 257
74 carpal tunnel, anatomy 22
British Ankylosing Spondylitis Metrology Index carpal tunnel release 245
(BASMI) 8890 endoscopic 26
brucellosis 201 carpal tunnel syndrome (CTS) 21
Burkholderia pseudomallei 201 causes 23
butterfly rash 122 clinical signs 23
imaging techniques 256
management 245
c-ANCA 156, 157 cartilage, in osteoarthritis 41
C-reactive protein (CRP) 4, 161 cartilage oligomeric matrix protein 43
in giant cell arteritis 162 caspase-1 role in gout 193
in rheumatoid arthritis 78, 80 cauda equina syndrome 171
09-PS Rheumatology-Index-ppp:Index 18/3/08 14:33 Page 274

274 Index

CCL2 162 cognitive dysfunction, in SLE 127


CD4* T lymphocytes 93 colchicine 9, 106
in Sjgrens syndrome 129 use in Behets syndrome 166
celecoxib, cardiovascular risk 612, 63 use in gout 190, 1923
CENP antibodies 132 use in osteoarthritis 51
central core disease 253 use in pseudogout 197
central nervous system involvement use in vasculitides 157
Behets syndrome 166 collagen genes 41
SLE 127 combination therapy
centronuclear (myotubular) myopathy 254 in osteoporosis 22930
cerivastatin 266 in rheumatoid arthritis 84
cervical rib syndrome 22 compartment syndromes 24
cervical rotation measurement 89, 90 complement deficiencies
cervical spinal cord stimulation, Raynauds antinuclear antibodies 111
phenomenon 137 association with SLE 120, 157
channelopathies 2556 compliance, bisphosphonate treatment 225
Charcot joints (neuropathic arthropathy) 213 computer use, dry eyes 130
cheiroarthropathy 15, 212 conduction disturbances, AS 91
Chikungunya 208 congenital heart block 94, 95, 111
children, investigation of acute monoarthritis congenital myopathies 2534
910 connective tissue growth factor (CTGF), role in
chloroquine, value in psoriatic arthritis 181 scleroderma 143
cholecalciferol 219 continuous-focus osteomyelitis 203
combination with alendronate 220 contractures, in scleroderma 141
cholestyramine 2689 coracoacromial arch anomalies 1314
chondrocalcinosis 195 CORE (Continuing Outcomes Relevant to Evista)
see also pseudogout study 227
chondroitin, value in osteoarthritis 46, 49, 104 Cori disease 256
chronic kidney disease corticosteroid-induced osteoporosis, investigation
protein restriction 1878 and management 2389
use of allopurinol 192 corticosteroid therapy 1478, 151
chronic kidney disease-mineral and bone as cause of male osteoporosis 233
disorder (CKD-MBD) 2467, 251 in carpal tunnel syndrome 24
adynamic bone disease 248 in frozen shoulder 16
diagnosis and management 24850 in GCA and PMR 161
in early renal disease 250 in gout 106
high turnover bone disease (osteitis fibrosa in hepatitis C 207
cystica) 247 in inflammatory myopathies 262
mixed 247 injection 9
osteomalacia 247 in carpal tunnel syndrome 24
ChurgStrauss syndrome 155 epidural 1767
ANCA 157 in fibromyalgia syndrome 31
interferon therapy 158 in frozen shoulder 16
cilostazol, value in Raynauds phenomenon 138 intravenous, in giant cell arteritis 161, 162
Cinacalcet 250 in osteoarthritis 51
citrullination 72 in pseudogout 197
citrulline residues 5 in psoriatic arthritis 181
CLASS (Celecoxib Long-term Arthritis Safety in rotator cuff disease 14
Study) 57 in tennis elbow 19, 20
clodronate, structure and function 2223 long-term 162
clonidine, use in fibromyalgia syndrome 30 during pregnancy 95
coenzyme Q10 depletion 267 in rheumatoid arthritis 83, 84, 106
cognitive behavioural therapy, chronic low back in Sjgrens syndrome 132
pain 177 in SLE 126, 127
cognitive decline, effect of raloxifene 228 steroid myopathy 257, 258, 259
09-PS Rheumatology-Index-ppp:Index 18/3/08 14:33 Page 275

Index 275

in vasculitides 157 use in SLE 1267


in viral arthritis 210 dental hygiene, Sjgrens syndrome 131
costoclavicular syndrome 22 depression
COX isoenzymes, role in ulcer healing 57 in Sjgrens syndrome 132
COX-2 inhibitors 56, 57 in SLE 127
cardiovascular risk 614 dermatomyositis (DM) 2601
use in SLE 126 antinuclear antibodies 111, 112, 113
coxibs 61 pathogenesis 264
cardiovascular side effects 613 dextropropoxyphene 51
Coxsackie virus, as trigger for Sjgrens diabetes
syndrome 132 bone complications 213
CPEO (Chronic Progressive External cheiroarthropathy 15, 212
Ophthalmoplegia) 256 joint disorders 213
creatine kinase (CK) 262 ligament, tendon and capsular problems 212
monitoring in statin therapy 266, 270 musculoskeletal complications 21112, 21415
creatinine supplementation, value during steroid osteoporosis risk 21314
therapy 259 type 2, risk from low vitamin D status 2501
CREST syndrome 110, 140 vitamin D levels 219
critical illness myopathy (CIM) 2579 diacerein 51
Crohns disease 179 value in osteoarthritis 104
CRTL1 gene 41 diastolic dysfunction, AS 91
CRTM gene 41 diclofenac
cryoglobulinaemia 154 cardiovascular risk 64
antinuclear antibodies 111 side effects 57
crystal arthritis 78, 9 diet 1002
see also gout; pseudogout in inflammatory myopathies 262
crystals, intra-articular 7 recommendations in hyperuricaemia and gout
culture-negative septic arthritis 203 187
curcumin, effect on inflammation 100 dietary risk factors, rheumatoid arthritis 74,
cyclooxygenase (COX) inhibitors 567 978
COX-2 selective drugs 56, 57 dietary supplements 99100
cardiovascular risk 614 see also vitamin D treatment
use in SLE 126 diffuse cutaneous scleroderma 140
cyclooxygenase (COX) isoenzymes, role in ulcer diffuse idiopathic skeletal hyperostosis (DISH)
healing 57 213
cyclophosphamide 148, 151 diffuse osteomyelitis 201
effect on fertility 94 disc prolapse 171, 172
use in inflammatory myopathies 262 discectomy 176
use in scleroderma 143 discoid lupus 118
use in Sjgrens syndrome 132 disease-modifying antirheumatic drugs
use in SLE nephritis 127 (DMARDs) 834
use in vasculitides 157 effect on fertility 934
cyclosporin 149 use during pregnancy 95
use in Behets syndrome 166 disease-modifying drugs, benefits in psoriatic
use in inflammatory myopathies 262 arthritis 182
use in psoriatic arthritis 181 disease-modifying osteoarthritis drugs
use in vasculitides 157 (DMOADs) 512, 104
cytokine patterns in pregnancy 93 dislocation, risk after hip replacement surgery 67
diuretics, use in carpal tunnel syndrome 24
DNA methylation abnormalities 119
daclizumab 149 docosahexaenoic acid 98
dactylitis, in psoriatic arthritis 180 benefit in RA 823
dapsone, use in vasculitides 157 doxycycline, value in osteoarthritis 51, 53, 104
dehydroepiandrosterone (DHEA) DRB1 gene 72
levels in GCA and PMR 1623 drug abuse, septic arthritis 201
09-PS Rheumatology-Index-ppp:Index 18/3/08 14:33 Page 276

276 Index

drug-induced lupus 11819 in septic arthritis 201


drug interactions, with allopurinol 192 in SLE 1256
dry eyes, Sjgrens syndrome 130 etanercept 149
dry mouth, Sjgrens syndrome 1301 use in Behets syndrome 166
dual-energy X-ray absorptiometry (DEXA) 218 use in inflammatory myopathies 264
indications in men 233 use in psoriatic arthritis 182
Duchenne muscular dystrophy 255 use in vasculitides 157, 158
duloxetine, use in fibromyalgia syndrome 30 ethnic differences
Dupuytrens disease (DD) 212 in GCA and PMR 160
matrix metalloproteinases 213 in gout 184
in psoriasis 179
in scleroderma 140
eccentric loading exercises, rotator cuff disease in SLE 1234
14 in use of statins 267
eculizumab, use in inflammatory myopathies etidronate 2389
264 structure and function 2223
efalizumab 149 use in Pagets disease 243
use in psoriatic arthritis 182 etoricoxib, risk comparison with diclofenac 64
eicosapentaenoic acid 98 exercise
benefit in RA 823 protection against fractures 21819, 221
elbow pain value in inflammatory myopathies 262
golfers elbow 19 value in osteoarthritis 46
tennis elbow 1819, 20 exercise programmes
elderly people in frozen shoulder 1516
polyarthritis 103 in joint replacement surgery 66
gout 106 in plantar fasciitis 36
osteoarthritis 1034 in rotator cuff disease 14
rheumatoid arthritis 1046, 150 extractable nuclear antigens 110
recurrent haemorrhagic shoulder 14 ezetimibe 268, 269
elemental diets 99
elimination diets 99
EmeryDreifuss muscular dystrophy 255 facet joint hypertrophy 171
endometriosis 113 facet joint injections 176
endoscopic carpal tunnel surgery 26 facial pain, in fibromyalgia syndrome 28
endothelial dysfunction, scleroderma 143 facioscapulohumeral muscular dystrophy 255
endothelin (ET), role in scleroderma 143, 144 family history, in new-onset joint pain 3
enthesitis, in psoriatic arthritis 180 fasting, effect on inflammation 99
entrapment neuropathies 214, 256 fatigue 91
association with diabetes 211 in fibromyalgia syndrome 28
carpal tunnel syndrome 245 FDG-PET, in diagnosis of infections 204
as cause of heel pain 35 febuxostat 188, 193
eosinophilia myalgia syndrome 112 fenofibrate, use in gout 191, 193
epidermal growth factor (EGF), polymorphisms fertility 934
in psoriasis 182 in Sjgrens syndrome 96
epidural steroid injections, value in low back fetus, effects of maternal rheumatic disease 945
pain 1767 fever, significance in monoarthritis 7
epigenetic modifications, SLE 119, 120 fibrates, muscle side effects 266, 267, 268
EpsteinBarr virus (EBV), as trigger for fibrillin 140
rheumatoid arthritis 745 fibroblast dysfunction, scleroderma 142, 143
erythema infectiosum (slapped cheek) 206 fibroblast growth factors, polymorphisms in
erythrocyte sedimentation rate (ESR) 4, 161 psoriasis 182
in giant cell arteritis 162 fibromyalgia syndrome (FMS) 278
in polymyalgia rheumatica 160 epidemiology and aetiology 29
in rheumatoid arthritis 78 investigation 29, 30
in scleroderma 142 outlook and management 2931
09-PS Rheumatology-Index-ppp:Index 18/3/08 14:33 Page 277

Index 277

fingernails, in psoriasis 3 DHEA levels 1623


fish oil, benefit in RA 823, 989 epidemiology 162
flavonoids 100 genetic basis 162
fluorinated steroids, risk of myopathy 257 investigation 161, 163
fluoxetine, value in Raynauds phenomenon 137 treatment 1612, 163
fluvastatin, risk of muscle side effects 267, 268 ginger extracts, value in osteoarthritis 52
a-fodrin antibodies 132 glenohumoral osteoarthritis 12
footwear choice, osteoarthritis 47 glitazones, osteoporosis risk 214, 250
fracture prevention glucocorticoid-induced osteoporosis 2378, 240
raloxifene 227 investigation and management 2389
strontium ranelate 228, 230 glucocorticoid therapy see corticosteroid therapy
teriparatide 229 glucosamine, value in osteoarthritis 46, 489,
fractures, osteoporotic 218 104
in men 2335 glycogen storage diseases 2567
risk in glucocorticoid-induced osteoporosis golfers elbow 19
238 gonococcal arthritis 7
see also osteoporosis Gottrons sign 261
frozen shoulder 13, 1416, 17 gout 78, 9, 10, 106, 189, 1934
fructose, effect on uric acid synthesis 185, 187 acute management 190
fruit intake, relationship to RA risk 97, 98 association with diabetes 211
dietary recommendations 187
gender and ethnic differences 184
gabapentin, use in fibromyalgia syndrome 30 investigations 190
GAIT (Glucosamine/chondroitin Arthritis molecular basis for inflammation 193
Intervention Trial) 104 prophylaxis 1903
gastric side effects, NSAIDs 55, 56, 59, 61 risk factors 190
protection strategies 567 serum urate levels 7
risk evaluation 58 see also hyperuricaemia; uric acid
gastro-oesophageal reflux, in fibromyalgia graft-versus-host disease 112
syndrome 28 similarity to scleroderma 144
gastrointestinal problems Greek Atorvastatin and Coronary Heart Disease
in scleroderma 141, 1434 Evaluation study 1867
Sjgrens syndrome 131 green tea, value in osteoarthritis 52
gemfibrozil, combination with cerivastatin 266
gender differences
in gout 184 headaches
in osteoarthritis 103 in fibromyalgia syndrome 28
in osteoporosis risk 218 in SLE 127
in Raynauds phenomenon 135 health status, in psoriatic arthritis 182
in Sjgrens syndrome 129 heart, effects of ankylosing spondylitis 91
in SLE 11819 Heberdens nodes, genetic predisposition 41
gene therapy, osteoarthritis 512 heel pain
genetic factors common causes 34
in Behets syndrome 1667 plantar fasciitis 337
in osteoarthritis 41, 43 heel spurs 34
in pseudogout 197, 198 Helicobacter pylori infection, interaction with
in psoriasis 179 NSAID therapy 57, 59
in rheumatoid arthritis 72, 75 HenochSchnlein purpura 154
in systemic lupus erythematosus 117 Hep2000 cell line 110
genetic screening, Pagets disease 243 hepatitis, viral arthritis 206
giant cell arteritis (GCA) 154, 155, 15960, hepatitis B 208
1634 hepatitis C 2078, 209
antiplatelet therapy 162 cryoglobulinaemia 154
biological therapies 158 hereditary neuropathy with liability to the
clinical features 1601 pressure palsies (HNPP) 2
09-PS Rheumatology-Index-ppp:Index 18/3/08 14:33 Page 278

278 Index

hexosamine biosynthetic pathway 489 hypogonadism 233


high turnover bone disease (osteitis fibrosa testosterone treatment 2345
cystica) 247 hypokalaemic periodic paralysis 255
hip, septic arthritis 9 hypomethylation, association with SLE 119
hip fracture, morbidity 233 hypothyroidism
hip fracture prevention, strontium ranelate 228 chondrocalcinosis 197
hip replacement surgery 65, 69 gout 190
complications 67 hypoxanthine phosphoribosyltransferase (HPRT)
exercise programmes 66 deficiency 186
indications 66, 67
minimal incision 68
prosthesis survival 678 ibandronate 223, 224
value of weight reduction 656 use in male osteoporosis 234
histamine H2 receptor antagonists, use with use in Pagets disease 243
NSAIDs 56 ibuprofen
histone modification 119 cardiovascular risk 63
HIV infection 208 interaction with aspirin 61
association with inclusion body myositis 262 imaging
HLA-B51 166 chronic low back pain 176
HLA-Cw0602 allele 179 rheumatoid arthritis 78
HLA-DRB1*04 160, 162 see also magnetic resonance imaging; positron
homocysteine levels, Behets syndrome 167 emission tomography (PET);
homogeneous staining, ANA 110, 117 radiography
hormone replacement therapy (HRT) 218, 231 immobility, muscle loss 259
combination with teriparatide 230 immunoglobulin, use in inflammatory
in osteoarthritis 43, 51 myopathies 262
use in SLE 119 immunosuppressive drugs 147
HTLV (human T-lymphotropic virus)-1 antibody treatments 149
infection, association with inclusion body antiproliferative and antimetabolic drugs 148
myositis 262 see also azathioprine; cyclophosphamide;
human bites 201 leflunomide; methotrexate;
hyaluronan, serum levels 43 mycophenolate mofetil
hyaluronic acid, intra-articular injection 47, 49, calcineurin inhibitors 1489
104 glucocorticoids 1478
hydralazine, association with DIL 118 see also corticosteroid therapy
hydroxychloroquine 149 hydroxychloroquine 149
use in hepatitis C 207 indications for use 150
use in psoriatic arthritis 181 sulphasalazine 149
use in Sjgrens syndrome 132 therapeutic monitoring 14950, 151
use in SLE 126 use in vasculitides 157
hyperbaric oxygen therapy, osteomyelitis 203 impingement theory, rotator cuff disease 1314
hypergammaglobulinaemia, antinuclear incision length, hip replacement 68
antibodies 111 inclusion body myositis (IBM) 2612
hyperglycaemia, association with osteoarthritis pathogenesis 264
412 indurative stage, scleroderma 141
hyperkalaemic periodic paralysis 255 infection, risk after joint replacement surgery 67
hyperostosis 213 infections
hyperparathyroidism, vascular dysfunction 250 association with arthritis 3
hypertensive disorders of pregnancy 96 as cause of lower back pain 172
hyperuricaemia 188 as cause of Pagets disease 242
cardiovascular disease risk 1867 as trigger for giant cell arteritis 154, 160
causes 1856 as trigger for rheumatoid arthritis 745
dietary recommendations 187 as trigger for Sjgrens syndrome 132
treatment 187 see also septic arthritis
see also gout inflammation assessment, SLE 1256
09-PS Rheumatology-Index-ppp:Index 18/3/08 14:33 Page 279

Index 279

inflammatory lower back pain 172, 173 unicompartmental knee arthroplasty 68


inflammatory markers, as outcome measures 88 value of weight reduction 656
inflammatory myopathies 2602 juvenile Pagets disease 243
diagnosis and management 2623
use of biological agents 264
infliximab 149 Kawasaki syndrome 155
use in Behets syndrome 166 KearnsSayre syndrome 256
use in GCA and PMR 161 KelleySeegmiller syndrome 186
use in inflammatory myopathies 264 knee joint
use in psoriatic arthritis 182 aspiration 89
use in vasculitides 157, 158 magnetic resonance imaging 10
insulin resistance monoarthritis, differential diagnosis 68
in Behets syndrome 167 osteoarthritis, corticosteroid injection 51
possible effect of glucosamine supplementation knee replacement surgery 65, 69
49 complications 67
interferon therapy indications 66, 67
in Behets syndrome 166 post-operative weight gain 66
in ChurgStrauss syndrome 158 prosthesis survival 68
interferon 1, possible role in SLE 11920 unicompartmental 68
topical use in Sjgrens syndrome 131
interleukin-1 inhibitors 83
interleukin-18 laminin-1 113
association with Behets syndrome 167 lansoprazole, gastric protection in NSAID
levels in DM and PM 164 therapy 59
intermalleolar distance measurement 89, 90 large vessel vasculitis 155
interstitial pulmonary fibrosis 112 laser therapy, Raynauds phenomenon 137
intervertebral disc prolapse 171, 172 lasofoxifene 230
discectomy 176 late-onset RA (LORA) 1056, 150
intravenous drug users, septic arthritis 201 lateral cutaneous nerve of the thigh, meralgia
intravenous immunoglobulin, use in vasculitides paraesthetica 24
157 lateral epicondylitis (tennis elbow) 1819
irritable bowel syndrome, in fibromyalgia RCT of treatments 20
syndrome 28 leflunomide 148
avoidance during pregnancy 95
safety monitoring 79
joint aspiration 4 use in psoriatic arthritis 181
diagnosis of septic arthritis 7, 201 use in Sjgrens syndrome 132
in gout 190 leptin levels, Behets syndrome 167
knee 89 LeschNyhan syndrome 186
in pseudogout 196 LIFE (Losartan Intervention for Endpoint
joint count 88, 89 Reduction) study 187
prognostic value in RA 80 lifestyle modification, SLE 126
joint pain ligamentum flavum hypertrophy 171
acute monoarthritis, differential diagnosis 68 limb girdle muscular dystrophy 255
new onset limited cutaneous scleroderma 140
examination 3 limited joint mobility (LJM), diabetes 212
history-taking 23 linoleic acid 46
investigations 34 a-linolenic acid 46
joint replacement surgery 65, 69 local anaesthetic injection, fibromyalgia
complications 67 syndrome 31
exercise programmes 66 losartan, use in gout 191, 193
indications 66, 67 low back pain
minimal incision 68 acute 16970, 174
perioperative methotrexate therapy 667 ankylosing spondylitis 173
prosthesis survival 678 causes 1701
09-PS Rheumatology-Index-ppp:Index 18/3/08 14:33 Page 280

280 Index

low back pain contd MELAS (Myopathy, Encephalopathy, Lactic


acute contd Acidosis and Stroke) 256
management 1723 melioidosis 201
outcome 173 meloxicam, cardiovascular risk 62
specific pathology 1712 membranous staining, ANA 110
chronic menopause
clinical examination 1756 effect on BMD 218
imaging 176 effect on osteoarthritis risk 43
opioid use 177 meralgia paraesthetica 24
treatment 1767 mercaptopurine, interaction with allopurinol 192
indicators for nerve root problems 171 MERRF (Myoclonic Epilepsy with Ragged Red
inflammatory 172, 173 Fibres) 256
red flags 170 metabolic syndrome, association with
risk factors 170 hyperuricaemia 186
Low-Dye taping, plantar fasciitis 35 methotrexate therapy 148, 151
lumbar flexion measurement 89, 90 effect on fertility 94
LUMINA (lupus in minorities: nature versus in GCA and PMR 161
nurture) study 124, 126 in inflammatory myopathies 262
lupus nephritis 122, 124, 127, 150 joint replacement surgery 667
Lyme disease 7 prediction of response 150
lymphoma, risk in Sjgrens syndrome 132 in psoriasis 181
teratogenicity 95
in rheumatoid arthritis 84, 86
M1-2 antibodies 262 safety monitoring 79
McArdle disease 257 in scleroderma 143
magnetic resonance imaging (MRI) in Sjgrens syndrome 132
in carpal tunnel syndrome 245 in vasculitides 157
in giant cell arteritis 161 methylene tetrahydrofolate reductase levels 150
in inflammatory myopathies 262 methylprednisolone
of knee joint 10 epidural injection 177
in low back pain 176 intra-articular injection 51
in osteomyelitis 203 intravenous, in inflammatory myopathies 262
in psoriatic arthritis 180 use in GCA 161, 162
male osteoporosis 2335 microchimaerism hypothesis, scleroderma 144
diagnosis and management 236 microscopic polyangiitis 155
malignancy ANCA 157
association with dermatomyositis 2601 minimal incision, hip replacement surgery 68
breast cancer risk, effect of SERMS 228 minocycline 51
as cause of lower back pain 172 association with DIL 118
vitamin D levels 219 misoprostol 56
malignant bone pain 2 mitochondrial myopathies 256
malignant hyperthermia 2556 mitochondrial toxicity, statins 269
mammalian target of rapamycin (MTOR) 148 mixed connective tissue disease (MCTD),
manipulation under anaesthesia (MUA), value in antinuclear antibodies 111, 112, 113
frozen shoulder 16 MMP-3 gene 41
matrix metalloproteinases (MMPs) MOBILE study 224
in Dupuytrens disease 213 monitoring
as therapeutic targetes 51 of immunosuppressive treatment 14950, 151
medial calcaneal nerve entrapment 35 of systemic lupus erythematosus (SLE) 1235
medial compartment syndrome 24 monoarthritis 4
medial epicondylitis (golfers elbow) 19 diagnostic errors 7
median neuritis 23 differential diagnosis 68
Mediterranean diet 98 see also gout; septic arthritis
medium vessel vasculitis 1545 monocarboxylate transporter 4 inhibition, statins
medullary osteomyelitis 201 269
09-PS Rheumatology-Index-ppp:Index 18/3/08 14:33 Page 281

Index 281

monoclonal antibodies 149 N-arylanthranilic acids 61


monocyte chemoattractant protein-1 (MCP-1) nailfold capillaries, Raynauds phenomenon 135,
162 140, 141
MORE (Multiple Outcome of Raloxifene NALP proteins, role in gout 193
Evaluation) trial 227, 228 naproxen, interaction with aspirin 61
morphea 140 NAT9 gene 179
see also scleroderma nemaline rod myopathy 2534
movement disorders, in fibromyalgia syndrome neonatal lupus syndrome 945
28 Neoral see cyclosporin
multidisciplinary management, low back pain nerve root problems 171
177 neural theory of tendinopathy 16
multiple sclerosis, vitamin D levels 219 neuropathic arthropathy (Charcot joints) 213
multiplex technologies 113 neuropathies, Sjgrens syndrome 1312
muscle biopsy 262 niacin, muscle symptoms 268
muscle complications of statins 21112, 2667, nifedipine, value in Raynauds phenomenon 137
26970 night pain, shoulder 1112
diagnosis and management 2679 nitric oxide synthase inhibitors, value in
pathogenesis 269 osteoarthritis 51
rhabdomyolysis 267 nitric oxide synthases, role in ulcer healing 57
muscular dystrophies 2545 non-steroidal anti-inflammatory drugs (NSAIDs)
muscular infarction, diabetic 212 54-5
muscular pains, Sjgrens syndrome 131 classification 61
myalgia 266 gastric side effects 569
mycophenolate mofetil (MMF) 148, 150 mechanism of action 556
use in psoriatic arthritis 181 safety monitoring 79
use in scleroderma 143 use in carpal tunnel syndrome 24
use in SLE nephritis 127 use in elderly people 1056
use in vasculitides 157 use in gout 190, 192
Mycoplasma pneumoniae, as trigger for giant cell use in osteoarthritis 501
arteritis 154, 160 use in patients with cardiovascular risk 63
myoadenylate deaminase deficiency 186 use during pregnancy 95
myocardial infarction risk use in pseudogout 197
after joint replacement surgery 67 use in SLE 126
NSAIDs 57, 614 nuclear factor-k-beta (NF-kB) pathway 53, 240
myopathies nucleolar staining, ANA 110
acquired Nurses Health Initiative 218
alcoholic myopathy 257 Nurses Health Study, smoking, relationship to
critical illness myopathy 2579 RA risk 74
steroid myopathy 258, 259 nutritional supplements, value in osteoarthritis
channelopathies 2556 467, 52
classification 254
congenital 2534
in congenital metabolic disorders 2567 obesity
inflammatory 2602 association with osteoarthritis 412, 43, 45
diagnosis and management 2624 association with plantar fasciitis 34
mitochondrial 256 and joint replacement surgery 656
muscular dystrophies 2545 as risk factor for osteoarthritis 104
myositis-specific antibodies (MSA) 262 occupational risk factors
myotonia 255 osteoarthritis 43
myotonic dystrophy 2545 rheumatoid arthritis 74
myotubular (centronuclear) myopathy 254 oedematous stage, scleroderma 141
oesophageal dysfunction, scleroderma 141,
1434
n-3 fatty acid supplementation, value in oestrogen
osteoarthritis 467 effect on osteoarthritis risk 43
09-PS Rheumatology-Index-ppp:Index 18/3/08 14:33 Page 282

282 Index

oestrogen contd prevention in long-term steroid therapy 162


effect on rheumatoid arthritis risk 74, 75 risk factors 21819
oestrogen therapy risk in insulin-requiring diabetes 21314
in osteoarthritis 51 risk from thiazolidinediones 250
in osteoporosis 218, 231 risk management 220
see also hormone replacement therapy (HRT) spinal fracture 172
oligoarthritis 3 osteoprotegerin (OPG) 218, 238, 243, 248
olive oil 98 osteosarcoma, as side effect of teriparatide 229
omega-3 friendly foods 101 outcome measures, RA and AS 8892
omega-3 oil 98 oxicams 61
benefits in rheumatoid arthritis 823, 99
Onyong-nyong 208
opioids, use in chronic low back pain 177 p-ANCA 1567
oral contraceptive pills, use in SLE 119 Pagets disease 2412
orlistat 45 avoidance of teriparatide 229
orthoses comorbidities 245
value in osteoarthritis 47 complications 242
value in plantar fasciitis 35 genetic factors 243
osteitis fibrosa cystica (high turnover bone investigation and management 2423, 244
disease) 247 pain 2
osteoarthritis 3940, 1034 pain management
aetiology 41 in fibomyalgia syndrome 30
association with diabetes 211 in low back pain 176
comparison with pseudogout 196 painful arc 13
drug treatment 503 painful joints
non-pharmacological treatment 459 acute monoarthritis, differential diagnosis 68
radiological severity scoring 40 new onset
risk factors 413 examination 3
secondary causes 40 history-taking 23
osteoblast apoptosis inhibition 240 investigations 34
osteoblasts, effect of corticosteroids 238 pamidronate 224
osteoclast differentiation 243 structure and function 223
osteoclasts, activation at menopause 218 use during corticosteroid therapy 238
osteoid 247 use in male osteoporosis 234
osteomalacia 247 use in neuropathic arthropathy 213
osteomyelitis 201, 203 paracetamol 61
plasma procalcitonin 10 value in osteoarthritis 501
spinal 172, 204 parainfluenza virus 160
osteonecrosis, as side effect of bisphosphonates paramyotonia 255
224 parathyroid hormone (PTH) 247
osteopenia, indications for bisphosphonates 223 in evaluation of renal osteodystrophy 248
osteoporosis 21718, 221 recombinant see teriparatide
drug treatments 218, 231, 234 parathyroidectomy, in renal osteodystrophy 249,
bisphosphonates 21920, 2234 250
combination treatment 22930 parecoxib, cardiovascular side effects 62
raloxifene 2278 partial androgen deficiency in aging men
sequential treatments 230 (PADAM) 2345
strontium ranelate 2289, 230 parvovirus B19 160, 2067, 2089
teriparatide 229, 230 patient education, rheumatoid arthritis 84
third generation SERMS 230 peak bone mineral density 218
glucocorticoid-induced 2389 peginterferon-a therapy, hepatitis C 207
investigations 226 pentazocine 51
male 1335 peptidylarginine deaminases (PADIs) 72
diagnosis and management 236 periodontal disease, effect on rheumatoid
management 225, 226 arthritis 75
09-PS Rheumatology-Index-ppp:Index 18/3/08 14:33 Page 283

Index 283

Phalens sign 23 preventive screening, SLE 124, 1256


pharmocogenomics 150 primary biliary cirrhosis, antinuclear antibodies
Phos-Ex (calcium acetate) 250 111
phosphate binders, use in chronic kidney disease primary Raynauds phenomenon 135
250 PRIMO (Prediction of Muscle Risk in
phosphoribosylpyrophosphate synthetase gene Observational conditions) survey 267
mutations 186 probenecid 193
photosensitivity, SLE 122, 126 procainamide, association with DIL 118
pioglitazone, osteoporosis risk 214, 250 procalcitonin (PCT) 10
piroxicam, side effects 57 as marker for infection 203
plantar fascia stretching 36 prolactin 93
plantar fasciitis 335 prolapsed intervertebral discs 171, 172
management 357 discectomy 176
plasmapheresis, use in vasculitides 157 prostacyclin
pleural disease, SLE 123 effect of NSAIDs 612
Pogosta Disease 208 value in Raynauds phenomenon 137
polyarteritis nodosa 155 value in scleroderma 143
polyarthritis 3 prostaglandin generation, COX-1 and COX-2 55
in elderly people 103 prostate cancer, androgen deprivation therapy 235
gout 106 prosthesis survival, joint replacement 66, 678
osteoarthritis 1034 protein restriction, chronic kidney disease 1878
rheumatoid arthritis 1046 proteinuria
polymerase chain reaction (PCR) screening, in in rheumatoid arthritis 789
diagnosis of infection 203 in scleroderma 142
polymyalgia rheumatica (PMR) 105, 155, proton pump inhibitors, use with NSAIDs 56,
15960, 1634 57, 59
DHEA levels 1623 pseudoclaudication 171
epidemiology 162 pseudogout 8, 9, 1956
investigation 161, 163 associated conditions 1967
treatment 1612, 163 diagnosis 196
polymyositis (PM) 260, 261 hereditary 197
antinuclear antibodies 111, 112, 113 role of ANKH mutations 198
pathogenesis 264 toll-like receptors 198
polyphenols 100 treatment 1978
value in osteoarthritis 52 Pseudomonas aeruginosa joint infections 201
Pompe disease 256 psoriasis 3, 179
positron emission tomography (PET) cardiovascular disease risk 182
in diagnosis of giant cell arteritis 161 in retrovirus infection 208
in diagnosis of infections 204 psoriatic arthritis 4, 17980, 1823
posterior compartment syndrome 24 comparison with rheumatoid arthritis 180
prayer sign 212 diagnosis 180, 181
prazosin, value in Raynauds phenomenon 137 management 1812
prednisolone X-ray changes 78
in GCA and PMR 161 PSORS loci 179
in inflammatory myopathies 262 psychological disorders
intra-articular injection 51 in Behets syndrome 166
risk of fracture 238 in fibromyalgia syndrome 28
see also corticosteroid therapy in Sjgrens syndrome 132
pregabalin, use in fibromyalgia syndrome 30 psychological interventions, value in low back
pregnancy pain 177
effect on rheumatoid arthritis risk 74 psychosis, in SLE 127
effect on rheumatoid diseases 93, 94 PTNP22 gene 72
parvovirus B19 infection 208 pulmonary artery hypertension (PAH)
use of azathioprine 127 bosentan therapy 144
use of hydroxychloroquine 126 scleroderma 142, 143
09-PS Rheumatology-Index-ppp:Index 18/3/08 14:33 Page 284

284 Index

pulmonary embolism, risk after joint replacement Sjgrens syndrome 131


surgery 67 in SLE 122, 124, 127, 150
pulmonary fibrosis, scleroderma 1412, 143 renal osteodystrophy (ROD) 2467, 251
pulmonary manifestations adynamic bone disease 248
dermatomyositis 261 diagnosis and management 248
Sjgrens syndrome 131 in early stage disease 250
pulseless disease (Takayasus arteritis) 155 high turnover bone disease (osteitis fibrosa
purine metabolism 1845 cystica) 247
purine nucleoside phosphorylase deficiency 186 mixed 247
purine-rich foods 187 osteomalacia 247
purpura 154, 158 renal side effects, NSAIDs 57
pyrazolidine derivatives 61 rofecoxib 63
resolvins 98
retinal vasculitis, Behets syndrome 166
quality of life, in psoriatic arthritis 182 retroviruses 208
quality of life evaluation 901 reverse Phalens sign 23
quercetin 100 rhabdomyolysis, statin therapy 266, 267
rheumatoid arthritis 712
anti-CCP antibodies 7980
radial neuritis 23 antinuclear antibodies 111, 113
radiography comparison with psoriatic arthritis 180
in gout 190 diagnosis 45, 77
in low back pain 176 investigations 789
in osteomyelitis 203 diet, susceptibility factors 978
in Pagets disease 242 dietary supplements 99100
in pseudogout 196 disease-modifying antirheumatic drugs 834,
in rheumatoid arthritis 78 86
vascular calcification 249 effect on fetus 945
radionuclide scanning, in osteomyelitis 203 effect of pregnancy 93
raloxifene 2278, 231 effect on pregnancy 96
in combination treatment 230 effect of smoking 5, 856
fracture prevention 224 in elderly people 1046, 150
RANK-RANKL pathway 218, 238, 240, 243 environmental factors 735
rapamycin, use in inflammatory myopathies 264 fertility 934
RAPTOR gene 179 genetic factors 72, 75
rashes omega-3 oil supplementation 823
butterfly 122 parvovirus B19 seropositivity 2089
in dermatomyositis 261 patient education 84
rat-bite fever 201 primary care and specialist liaison 79
Raynauds phenomenon 1345 prognostic markers 80
autoantibodies 111, 113 safety monitoring of medications 79
benefits of statins 144 treatment algorithm 85
pathogenesis 136 treatment response 845
in scleroderma 141 treatment response evaluation 878, 902
treatment 1368 rheumatoid factor (RF) 4, 72, 73, 77, 78, 156
reactive arthritis 4, 7 in hepatitis C 207
recurrent haemorrhagic shoulder of the elderly in parvovirus B19 infection 207
14 in psoriatic arthritis 182
red meat intake, relationship to RA risk 98 in rubella 207
referred pain 3 ribavarin therapy, hepatitis C 207
around shoulder 11, 12 rimonabant 45
remitting seronegative symmetric synovitis with risedronate 223
pitting oedema (RS3PE) 105 fracture prevention 224
renal disease long-term use 224
in scleroderma 142, 143 use during corticosteroid therapy 238
09-PS Rheumatology-Index-ppp:Index 18/3/08 14:33 Page 285

Index 285

use in male osteoporosis 234 investigation and management 201, 202


use in osteoarthritis 104 mortality, relationship to age 204
use in Pagets disease 243 plasma procalcitonin 10
rituximab 149 in retrovirus infection 208
use in inflammatory myopathies 264 treatment 9
use in SLE 128 sequential treatments, osteoporosis 230
use in vasculitides 157 sequestosome 1 (SQSTM1) gene mutations 243
RNA-induced silencing complex (RISC) 209 Sevelamer 250
rofecoxib sex hormone binding globulin (SHBG),
cardiovascular risk 61, 62, 63 relationship to bone loss 235
renal toxicity 63 shoulder pain 11, 212
rosiglitazone, osteoporosis risk 214, 250 causes and characteristics 12
Ross River Virus 208 clinical assessment 1113
rosuvastatin 269 frozen shoulder 1416, 17
dosage in Asian patients 267 imaging 16
rotator cuff disease 11, 1314, 212 rotator cuff disease 1314
clinical assessment 1113 sialometry 1301
imaging 16 sibutramine 45
rubella 206, 207 sicca syndrome 130
RUTH (Raloxifene Use for The Heart) study side effects
228 of allopurinol 192
of androgen therapy 235
of azathioprine 148
sacroiliitis 4 of bisphosphonates 224
salicylates 61 of colchicine 190
see also aspirin, low dose of corticosteroids 106, 127, 148
Sandimmune (cyclosporin) 149 of cyclophosphamide 148
SAPHO (Synovitis, Acne, Pustulosis, of cyclosporin 149
Hyperostosis and Osteitis) syndrome 180 of hydroxychloroquine 149
sarcoidosis 132 of leflunomide 148
Schirmers test 130 of methotrexate 148
sciatica, value of epidural steroid injection 1767 of mycophenolate mofetil 148
scleroderma 1402, 146 of NSAIDs 57
antinuclear antibodies 111, 112 cardiac 614
pathophysiology 1423 gastrointestinal 55, 569
prognosis 142 of raloxifene 228
similarity to graft-versus-host disease 144 of statins 21112, 26670
treatment 1434 of strontium ranelate 228
sclerosant injections, low back pain 176 of sulphasalazine 149
sclerostin 240 of teriparatide 229
secondary Raynauds phenomenon 135 signal recognition particle (SRP) antibodies 262
selective oestrogen receptor modulators (SERMS) sildenafil, value in Raynauds phenomenon
227 1378
third generation 230 simvastatin, combination with ezetimibe 269
see also raloxifene Sindbis Virus 208
selective serotonin reuptake inhibitors sirolimus (Rapamune) 148
use in fibromyalgia syndrome 30 Sjgrens syndrome 12930
use in Raynauds phenomenon 137 anti-centromere antibodies 132
SELENA (Safety of Estrogens in Lupus antiviral antibodies 206
Erythematosus National Assessment) trials association with hepatitis C 207
119 autoantibodies 111, 113
septic arthritis 2001, 204 dry eyes 130
culture-negative 203 dry mouth 1301
of hip, diagnosis 9 extraglandular manifestations 1312
investigation 4, 7 fertility 96
09-PS Rheumatology-Index-ppp:Index 18/3/08 14:33 Page 286

286 Index

Sjgrens syndrome contd use in psoriatic arthritis 181


infections as trigger 132 sulphate intake, relationship to osteoarthritis risk
international consensus criteria for diagnosis 49
130 sulphinpyrazone 191, 193
investigation and management algorithm 133 superficial osteomyelitis 201
skin lesions, Sjgrens syndrome 131 superoxide dismutase gene, Val16 allele 167
slapped cheek disease (erythema infectiosum) suprascapsular neuritis 22
206 swollen joint count (SJC) 88
SLC9A3R1 gene 179 synovial fluid characteristics 89
SLC11A1 gene 167 systemic lupus erythematosus (SLE) 3, 4, 78,
SLC22A4 gene 72 121, 128
SLC22A12 (URAT1) 185 ACR classification criteria 117
sleep abnormalities, in fibromyalgia syndrome aetiology 11718, 11920
28, 29 antiviral antibodies 206
small vessel vasculitis 1545 autoantibodies 110, 111, 113
smoking 11213 cardiovascular disease risk 1256
effect on ankylosing spondylitis 91 central nervous system involvement 127
effect on osteoarthritis 42 clinical features 1223
effect on Raynauds phenomenon 136 diagnosis 11617
effect on rheumatoid arthritis 5, 74, 75, 856 drug-induced 118
effect on SLE 126 effect on fetus 945
SOTI (Spinal Osteoporosis Therapeutic effect of pregnancy 93
Intervention) trial 228 effect on pregnancy 94, 96
SPARC 140 fertility 94
speckled staining, ANA 110, 117 monitoring 1235
spinal canal stenosis 171 nephritis 127
surgery 176 preventive screening 124, 1256
spinal cord stimulation, Raynauds phenomenon risk from smoking 74
137 SELENA trials 119
spinal fracture 172 treatment 1267
spinal function assessment 8890 systemic sclerosis 140
spinal fusion surgery 176 autoantibodies 111, 112, 113
spinal infections 172, 203, 204 effect of pregnancy 94
spondyloarthropathies, effect of pregnancy 94
spondylodiscitis 180
spondylolisthesis 171 T score, definition of osteoporosis 21718
statins tacrolimus 149
benefits in scleroderma 1445 use in psoriatic arthritis 181
cholesterol-lowering activity 266 Takayasus arteritis (pulseless disease) 155
muscle symptoms 212, 2667, 26970 tamoxifen 228
diagnosis and management 2679 taping
pathogenesis 269 value in osteoarthritis 47
rhabdomyolysis 267 value in plantar fasciitis 35
steroid myopathy 257, 258, 259 Tarui disease 257
steroids see corticosteroid therapy temporal artery biopsy 155, 161, 162, 164
Streptococcus moniliformis 201 tender joint count (TJC) 88
stress reduction, fibromyalgia syndrome 31 tendinopathy, neural aetiology 16
stroke risk, raloxifene 228 tennis elbow 1819
strontium ranelate 2289, 230, 231 RCT of treatments 20
fracture prevention 224 teriparatide 224, 226, 229, 231
subacute cutaneous lupus 118 in combination treatment 230
sulphasalazine 149 cost-benefit analysis 230
effect on fertility 94 use during corticosteroid therapy 239
safety monitoring 79 use in male osteoporosis 234
use during pregnancy 95 testosterone, use in male osteoporosis 2345
09-PS Rheumatology-Index-ppp:Index 18/3/08 14:33 Page 287

Index 287

tetracycline labelling, bone biopsy 248 tumour necrosis factor (TNF)-a polymorphisms,
tetracyclines, value in osteoarthritis 51, 53, 104 association with RA 72
Th1 and Th2 cell patterns 93 turmeric, effect on inflammation 100
thalidomide, use in Behets syndrome 166 twin studies
therapeutic monitoring, immunosuppressives osteoarthritis 41
14950, 151 rheumatoid arthritis 72
thermal biofeedback (TBF) 1367
thiazolidinediones, osteoporosis risk 214, 250
thiopurine methyltransferase levels 150 U3-RNP antibodies, scleroderma 142
Thomsens disease 255 ulcer healing, role of COX isoenzymes 57
thoracic outlet syndromes 22 ulcers
three-joint complex, spine 171 in Behcets syndrome 166
thrombosis, risk during pregnancy 94 in scleroderma 141
thromboxane A2, effect of NSAIDs 612 ulnar neuritis 223
thyroiditis, in Sjgrens syndrome 132 ultrasound
tiludronate in carpal tunnel syndrome 245
structure and function 2223 in giant cell arteritis 161
use in Pagets disease 243 unicompartmental knee arthroplasty 68
Tinels sign 23 URAT1 (SLC22A12) 185
tissue-engineering, osteoarthritis 52 urate levels, in diagnosis of gout 7
Titralac 250 uric acid metabolism 1845
tizanidine, use in fibromyalgia syndrome 30 see also gout; hyperuricaemia
TNFR2 gene 72 uricase 184
TNFRSF11B mutations 243 uricase preparations, use in gout prophylaxis
toll-like receptors (TLRs) 198 192
tongue, claudication 155 uricosuric agents 190, 191, 193
tophi 189 urinary C-terminal peptide of collagen type II
dissolution 193 43
topical NSAIDs, value in osteoarthritis 51 urine alkalinization 184, 187
topoisomerase 1 140 uveitis, Behets syndrome 166
tragus to wall measurement 89, 90
tramadol 30
use in osteoarthritis 51 vascular calcification, in renal disease 24950,
transforming growth factor-b (TGF-b), role in 251
scleroderma 143 vascular dysfunction
transplant patients, cyclosporin therapy 149 in hyperparathyroidism 250
treatment response evaluation, AS and RA in scleroderma 143
8792 vascular endothelial growth factor (VEGF)
triamcinolone polymorphisms, psoriasis 182
epidural injection 177 vascular theory, rotator cuff disease 14
intra-articular injection 51 vasculitides 1534
tricyclic antidepressants, use in fibromyalgia ANCA 1578
syndrome 30 antibody therapies 158
trigger point injections classification 154
fibromyalgia syndrome 31 diagnosis 1557
low back pain 176 effect of pregnancy 94
trigger points, fibromyalgia syndrome 28, 29 large vessel 155
triphasic colour changes, Raynauds 135 medium vessel 155
TROPOS (Treatment of Peripheral Osteoporosis small vessel 154
Study) 228 treatment 157, 158
tumour necrosis factor (TNF)-a-1031C allele see also Behets syndrome; giant cell arteritis
166 (GCA)
tumour necrosis factor (TNF)-a antagonists 51, VDR gene 41
80, 83, 85, 149, 150 vegetarian/ vegan diets 99
safety monitoring 79 vertebral fracture prevention 224
09-PS Rheumatology-Index-ppp:Index 18/3/08 14:33 Page 288

288 Index

vertebral fracture prevention contd in corticosteroid therapy 238


raloxifene 227 men 234
strontium ranelate 228 Vytorin 269
teriparatide 229
VIGOR (Vioxx Gastrointestinal Outcomes
Research) 57, 61 walking sticks, use in osteoarthritis 47
viral arthritis 2056 warfarin, interaction with allopurinol 192
alphaviruses 208 Wegeners granulomatosis 1545
causes 206 ANCA 157
diagnosis and treatment 20910 weight reduction
hepatitis C 2078, 209 value before joint replacement surgery 656
parvovirus B19 2067, 2089 value in fibromyalgia syndrome 31
retroviruses 208 women, rheumatoid arthritis risk 74
rubella 207 wrist splints, carpal tunnel syndrome 24
viscosupplementation 47, 49
visual symptoms, GCA 161
vitamin C intake, relationship to RA risk 97, 98 X-rays see radiography
vitamin C status, relationship to risk of xanthine oxidase (XO) 184
osteoarthritis 46 xanthine oxidase inhibitors 188
vitamin D 219 see also allopurinol
calcitriol (1, 25-dihydroxy-vitamin D3) 247 xerostomia
vitamin D deficiency 221 differential diagnosis 131
vitamin D receptor gene 41 Sjgrens syndrome 1301
vitamin D status
in male osteoporosis 235
relationship to rheumatoid arthritis risk 74 zoledronate
relationship to risk of type 2 diabetes 2501 dosage 224
vitamin D treatment 218, 231 structure and function 223
in chronic kidney disease 250 use in Pagets disease 243, 245

Você também pode gostar